UPL, MDP and MJP (Defining What Lawyers Do and McGuireWoods LLP Where They Can Do It): Part II T. Spahn (1/1/19) Hypotheticals and Analyses

UPL, MDP AND MJP (DEFINING WHAT LAWYERS DO AND WHERE THEY CAN DO IT): PART II

Hypotheticals and Analyses*

Thomas E. Spahn McGuireWoods LLP

* These analyses primarily rely on the ABA Model Rules, which represent a voluntary organization's suggested guidelines. Every state has adopted its own unique set of mandatory ethics rules, and you should check those when seeking ethics guidance. For ease of use, these analyses and citations use the generic term "legal ethics opinion" rather than the formal categories of the ABA's and state authorities' opinions -- including advisory, formal and informal. ______© 2019 McGuireWoods LLP. McGuireWoods LLP grants you the right to download and/or reproduce this work for personal, educational use within your organization only, provided that you give proper attribution and do not alter the work. You are not permitted to re-publish or re-distribute the work to third parties without permission. Please email Thomas E. Spahn ([email protected]) with any questions or requests.

35951124_10 UPL, MDP and MJP (Defining What Lawyers Do and McGuireWoods LLP Where They Can Do It): Part II T. Spahn (1/1/19) Hypotheticals and Analyses

TABLE OF CONTENTS

Hypo No. Subject Page

Multijurisdictional Practice -- Governmental Authority and Consequences

1 Governmental Authority to Regulate Lawyers ...... 1 2 Jurisdiction Over Out-of-State Lawyers ...... 12 3 Status of Pleadings Signed by Out-of-State Lawyers ...... 16 4 Out-of State Lawyers' Ability to Collect Fees ...... 23

Lawyers "Practicing" in Another State Where They Are Not Physically Present

5 Providing Advice about Another State's Law ...... 32 6 Establishing a "Virtual" Presence in Another State ...... 41

Temporary Practice in Another State: Litigators

7 Pro Hac Admission in Another State's Court ...... 75 8 Involvement in Pre-Litigation Activities ...... 104 9 Involvement in Discovery ...... 121 10 Participation in Non-Court Proceedings ...... 126 11 Lawyers Representing Their Clients in Arbitrations Where the Lawyers Are Not Licensed ...... 131

i

35951124_10 UPL, MDP and MJP (Defining What Lawyers Do and McGuireWoods LLP Where They Can Do It): Part II T. Spahn (1/1/19) Hypotheticals and Analyses

Hypo No. Subject Page

Temporary Practice in Another State: Transactional Lawyers

12 Non-Litigation Work in Another State -- Definition of "Temporary" Presence ...... 142 13 Requirement of a Relationship with the Lawyer's Home State: Narrow View ...... 148 14 Requirement of a Relationship with the Lawyer's Home State: State Variations ...... 156 15 Requirement of a Relationship with the Lawyer's Home State: Broad View ...... 172 16 Non-Litigation Work Without a Relationship with the Lawyer's Home State, but Based on Nationally-Uniform Law ...... 176

Lawyers Moving Permanently to Another State

17 Lawyers Systematically and Continuously Practicing Law in a State Where They Are Not Licensed ...... 182 18 Lawyers "Waiving in" to Another State's Bar ...... 197 19 Permissible Activities While Waiting to be Admitted by Motion in a New State ...... 204

Effect of the Supremacy Clause on Lawyers Practicing Continuously in Another State Where They Are Not Licensed

20 Practicing Before Federal Agencies and Specialized Tribunals ...... 218 21 Practicing in Federal Courts ...... 225 22 Practicing "Federal" Law ...... 240

Serving as a Nonlawyer in Another State

23 Lawyers Acting as Paralegals in a State Where They Are Not Licensed ...... 258

ii

35951124_10 UPL, MDP and MJP (Defining What Lawyers Do and McGuireWoods LLP Where They Can Do It): Part II T. Spahn (1/1/19) Hypotheticals and Analyses

Hypo No. Subject Page

In-House Lawyers

24 In-House Lawyers Practicing in States Where They Are Not Licensed: General Rule ...... 262 25 In-House Lawyers Practicing in States Where They Are Not Licensed: Limitations ...... 273

Foreign Lawyers

26 Foreign Lawyers ...... 283

iii

35951124_10 UPL, MDP and MJP (Defining What Lawyers Do and McGuireWoods LLP Where They Can Do It): Part II T. Spahn (1/1/19) Hypotheticals and Analyses

Governmental Authority to Regulate Lawyers

Hypothetical 1

You are interviewing for a law school professorship, and over lunch you and a current professor start discussing governmental power to regulate the legal profession. The professor claims that the federal government has the ultimate power to regulate the legal profession, because more than any other profession lawyers affect interstate commerce. You don't know whether to politely agree with the professor, or push back against what might be a deliberately incorrect view.

Does the federal government have the ultimate power to regulate the legal profession?

NO

Analysis

The constitution's Supremacy Clause1 allows federal law to essentially "trump" state law in certain matters. For instance, lawyers generally can rely on the Supremacy

Clause to practice before certain federal agencies or specialized tribunals, even in states where the lawyers are not licensed to practice law.2

1 U.S. Const. art. VI, § 2. 2 Interestingly, courts have dealt with the effect of licences issued by other sovereigns – Native American Tribes. State of Arizona v. Lang, 323 P.3d 740, 742, 744, 745-46 (Ariz. Ct. App. 2014) (concluding that a lawyer was committing the unauthorized practice of law in Arizona even though he had been admitted to practice in a tribal court; affirming an injunction prohibiting the nonmember from practicing law in Arizona; "We hold that a person who presents himself as an attorney based in an Arizona office engages in the unauthorized practice of law unless he has been admitted to practice before the Arizona Supreme Court, even if he has been admitted to practice in a tribal court within the boundaries of Arizona."; "Lang holds a law degree and is admitted to practice law in the San Carlos Apache Tribal Court. He has never been admitted to the practice of law by the Arizona Supreme Court, and he is not a certified legal document preparer under section 7-208 of the Arizona Code of Judicial Administration."; "The Arizona Constitution gives our supreme court exclusive authority to regulate the practice of law in Arizona. . . . The supreme court has disciplinary jurisdiction over individuals who are not members of the Arizona Bar but are licensed to practice law in a different state or possession [sic] of the United States and have engaged in the unauthorized practice of law in Arizona. . . . The Bar is authorized to investigate, prosecute, and obtain enforceable judgments in the superior court against such individuals."; "That residents of other states were affected by Lang's representations and that other states also may have jurisdiction over his conduct does not deprive Arizona of jurisdiction."; "Likewise, even if Lang could have represented S.J. (former client) in the federal employment law matter, he committed the unauthorized practice of law by falsely representing that he was an attorney practicing in Arizona. He 1

35951124_10 UPL, MDP and MJP (Defining What Lawyers Do and McGuireWoods LLP Where They Can Do It): Part II T. Spahn (1/1/19) Hypotheticals and Analyses

However, most regulation of the legal profession occurs at the state level.

Lawyers receive their licenses from states, and can be professionally disciplined by

whatever state has given them a license.

This state-centric regulatory regime creates a number of complicating issues,

including the issue of "multijurisdictional" practice. That term involves lawyers practicing

law in states where they are not licensed to practice law.

This is not to say that the federal constitution does not affect states' power over

the legal profession.3 For instance, several states have dealt with constitutional

challenges to their requirement that lawyers licensed in those states maintain a

permanent office in the state.

In 2010, a New Jersey legal ethics opinion addressed New Jersey's requirement

that New Jersey lawyers can advertise in that state only if they have a "bona fide" office

there.

• New Jersey LEO 718 (3/25/10) (explaining that a lawyer can advertise only if he or she operates in a "bona fide" office, which can be a home office if the lawyer assures confidentiality when working at home; explaining that lawyers can meet clients in other places as long as they have a bona fide office; "Rule 1:21-1(a) also requires regular physical presence by the attorney at the office during business hours. An attorney who is out of the office during normal business hours does not violate the bona fide office rule provided the absence from the office is occasional and the attorney is otherwise reachable by telephone, email, or the like. If the attorney is regularly out of the office during normal business hours, then a responsible person must be present at the office."; explaining that a lawyer with a bona fide office can also have a

similarly engaged in the unauthorized practice of law by holding himself out to be an Arizona attorney when negotiating S.J.'s legal rights in the real estate dispute. Even if Lang had been lawfully working under the supervision of a licensed attorney, as he now claims, the document he authored made no mention of this alleged association and did not otherwise dispel his false representations about his own professional status."). 3 One recurring dispute involves Federal prosecutors' compliance with their host states' ethical rules. 2

35951124_10 UPL, MDP and MJP (Defining What Lawyers Do and McGuireWoods LLP Where They Can Do It): Part II T. Spahn (1/1/19) Hypotheticals and Analyses

virtual office, but would need to add a disclaimer in any marketing for that virtual office; also explaining that a lawyer cannot use another person's address for the lawyer' office, even if the lawyer has an agreement to be able to use conference rooms or other office space in the third party's office; "[A] law firm may not practice law from an office that is embedded within a nonlegal business office. That arrangement does not preserve the required 'separate practices and identities' of the two businesses and does not ensure client confidentiality.").

On February 1, 2013, New Jersey backed away from this strict requirement.

• New Jersey Supreme Court Rule 1:21-1(a)(1) ("An attorney need not maintain a fixed physical location, but must structure his or her practice in such a manner as to assure, as set forth in RPC 1.4, prompt and reliable communication with and accessibility by clients, other counsel, and judicial and administrative tribunals before which the attorney may practice, provided that an attorney must designate one or more fixed physical locations where client files and the attorney's business and financial records may be inspected on short notice by duly authorized regulatory authorities, where mail or hand-deliveries may be made and promptly received, and where process may be served on the attorney for all actions, including disciplinary actions, that may arise out of the practice of law and activities related thereto.").

Other states have been more stubborn. A New York statute seems to allow lawyers to freely practice in New York.

• N.Y. Jud. § 470 Law (LexisNexis 2019) ("A person, regularly admitted to practice as an attorney and counsellor, in the courts of record of this state, whose office for the transaction of law business is within the state, may practice as such attorney or counsellor, although he resides in an adjoining state.").

But the statute actually takes a very restrictive approach – requiring New Jersey and

Connecticut lawyers to maintain an office in New York if they want to practice law in

New York.

• Platinum Rapid Funding Grp. V. HDW of Raleigh, Inc., No. 605890-17, 2017 NYLJ LEXIS 3734, at *4, *5, *5-6, *7-8 (N.Y. Sup. Ct. Jan. 9, 2018) (disqualifying a law firm from acting as counsel of record in a New York case because a "virtual" firm did not satisfy New York’s requirement that lawyers practicing in New York have a physical office there; "Pursuant to

3

35951124_10 UPL, MDP and MJP (Defining What Lawyers Do and McGuireWoods LLP Where They Can Do It): Part II T. Spahn (1/1/19) Hypotheticals and Analyses

Judiciary Law §470 entitled 'Attorneys having offices in this state may reside in adjoining state.'"; "A person, regularly admitted to practice as an attorney and counsellor, in the courts of record of this state, whose office for the transaction of law business is within the state, may practice as such attorney or counsellor, although he resides in an adjoining state." (citation omitted); "That is, this statutory directive that nonresident attorneys maintain an office within the State 'for the transaction of law business' requires nonresident attorneys to maintain a physical office in New York . . . . Indeed, an attorney or firm that has appeared in an action while violating Judiciary Law §470 must be disqualified from continuing to appear or represent any of the parties in the action."; "Based upon the papers submitted herewith, this Court finds that, in this case, neither Higbee & Associates ('Higbee') nor Raymin L. Ngo ('Ngo') had a physical office in the State of New York at the time that they appeared in this action on behalf of the defendants (ld). Thus, said entities are all non-residents and have failed to comply with the Judiciary Law §470. Indeed, the papers herein establish that Ngo and Higbee’s pleading – the Verified Answer and Counterclaims – identified their principal office to be located in Santa Ana, California . . . ."; "In the end, this Court finds that there is no evidence on this record that Ngo and Higbee had physical addresses in New York. Moreover, this Court cannot overlook the fact that the defendants have failed to offer any competing evidence against the sworn affidavits of Steven Pena and Jakeen Penss, Sr., process servers who attest that they physically went to the 48 Wall Street and or 605 West Genesee addresses, respectively, and confirmed that neither Ngo nor Higbee had physical offices at these locations. Accordingly, this court herewith awards the plaintiff its instant motion to disqualify the defendants’ attorneys of record – Raymin L. Ngo and Higbee & Associates.").

• Schoenefeld v. Schneiderman, 821 F.3d 273, 276 (2d Cir. 2016) (upholding the constitutionality of a New York law which requires nonresident lawyers licensed in New York to maintain in New York an "office for the transaction of law business"; "On this appeal, we must decide whether New York violates the Constitution's Privileges and Immunities Clause, see U.S. Const. art. IV, § 2, by requiring nonresident members of its bar to maintain a physical 'office for the transaction of law business' within the state, when resident attorneys are not required to maintain offices distinct from their homes, N.Y. Judiciary Law § 470. Having now received the New York Court of Appeals' response to our certified question as to the 'minimum requirements necessary to satisfy' § 470's office mandate, see Schoenefeld v. New York, 748 F.3d 464 (2d Cir. 2014); Schoenefeld v. State, 25 N.Y.3d 22, 6 N.Y.S.3d 221, 29 N.E.3d 230 (2015) (holding § 470 to require physical office), we conclude that § 470 does not violate the Privileges and Immunities Clause because it was not enacted for the protectionist purpose of favoring New York residents in their ability to practice law. To the contrary, the statute was enacted to ensure that nonresident members of the New York bar could practice in the

4

35951124_10 UPL, MDP and MJP (Defining What Lawyers Do and McGuireWoods LLP Where They Can Do It): Part II T. Spahn (1/1/19) Hypotheticals and Analyses

state by providing a means, i.e., a New York office, for them to establish a physical presence in the state on a par with that of resident attorneys, thereby eliminating a service-of-process concern. We identify no protectionist intent in that action. Indeed, it is Schoenefeld who, in seeking to practice law in New York without a physical presence in the state, is looking to be treated differently from, not the same as, New York resident attorneys. Such differential treatment is not required by the Privileges and Immunities Clause. Accordingly, we reverse the judgment of the United States District Court for the Northern District of New York (Lawrence E. Kahn, Judge) declaring § 470's office requirement unconstitutional, see Schoenefeld v. New York, 907 F. Supp. 2d 252 (N.D.N.Y. 2011), and we remand the case with instructions to enter judgment in favor of defendants on Schoenefeld's Privileges and Immunities claim.").

This turf – protecting approach has faced severe criticism.

• Melissa Maleske, 'Antiquated' NY Law Harms Legal Profession, ACC Says, Law360, Jan. 19, 2017) ("An 'antiquated' and 'protectionist' New York law that requires attorneys practicing in-state to maintain offices there prevents clients from hiring the attorneys of their choice and conflicts with the global, interstate nature of law practice today, according to an Association of Corporate Counsel brief submitted Wednesday to the U.S. Supreme Court."; "The ACC brief urges the high court to consider a petition filed earlier this month by New Jersey attorney Ekaterina Schoenefeld, who has been challenging the law in court since 2009. After finding that despite being admitted to practice in New York that she could not practice there without establishing a physical base, Schoenefeld, who lives and practices about an hour from the state line, sued New York over Section 470 of its Judiciary Law, arguing that requiring nonresident attorneys to maintain a physical office space in the state clearly violates the Constitution.";"The law is of special concern to in-house attorneys and their corporate employers, the ACC said in its amicus brief."; "'The ability of in-house counsel to advise and represent their clients in connection with litigation, regulatory, and transactional matters anywhere in the United States -- including the freedom of in-house counsel to select, engage, and utilize the most suitable outside counsel for a particular matter -- is a subject of overarching importance to ACC and the in-house bar,' the ACC wrote."; "After Schoenefeld sued, a New York federal court in 2011 struck down the law, finding that it is unconstitutional and places an unfair burden on attorneys practicing in multiple states. New York appealed to the Second Circuit, which sought input from the New York Court of Appeals clarifying the minimum requirements to satisfy the law’s mandate before it would issue an opinion."; "In a ruling the ACC's latest brief said 'interpreted [the New York law] in the most discriminatory possible way,' the state court said in March 2015 that nonresidents must maintain a physical office in-state to be eligible to practice

5

35951124_10 UPL, MDP and MJP (Defining What Lawyers Do and McGuireWoods LLP Where They Can Do It): Part II T. Spahn (1/1/19) Hypotheticals and Analyses

there, and merely having an address at which the attorneys can be served will not suffice. The ACC found the New York Court of Appeals' reading particularly troublesome, saying its 'draconian impacts . . . are a compelling reason why' a Supreme Court rehearing is warranted." (alterations in original); "In reversing the district court's ruling, the Second Circuit found that the statute was enacted as a means of ensuring every licensed New York lawyer could practice in the state and that Schoenefeld didn't offer any evidence of a protectionist intent meant to favor resident New York lawyers."; "However, in his dissenting opinion, Second Circuit Judge Peter W. Hall also agreed that the statute discriminates against nonresident lawyers and said the majority's holding reverses 'the state's burden of demonstrating that it has a substantial interest justifying the discrimination.'").

Delaware has also maintained this exclusionary approach.

• In re Barakat, 99 A.3d 639, 640, 640-41, 641, 644 (Del. 2013) (suspending for two years a lawyer who primarily practiced law in Pennsylvania, because the lawyer had not maintained a "bona fide" office in Delaware although he was member of the Delaware Bar; "Barakat has been a member of the Delaware Bar since 1992. Since January 2005, Barakat's address of record with this Court has been 901 North Market Street, Suite 460, in Wilmington, Delaware. Barakat also works from his home in Chadds Ford, Pennsylvania." (footnote omitted); "Barakat's 901 North Market Street office is not an 'office' in the traditional sense. Barakat's lease does not include any designated office space that is exclusively his. Rather, the employees of the landlord collect Barakat's mail and greet any visitors Barakat may have. The building security guards direct visitors to the fourth floor, where a receptionist is stationed during normal business hours. Under this arrangement, Barakat is entitled, for additional fees, to rent a conference room or office space, and utilize secretarial, reproduction, facsimile, word processing, and shipping services." (footnotes omitted); "The landlord's billing records (the 'Occupant Ledger'), and the testimony of two employees who work on the fourth floor, evidence that Barakat's presence at 901 North Market Street is 'sporadic and unscheduled.' . . . In October 2011, Barakat informed the United States Internal Revenue Service ('IRS') that 'all of [his] work aside from meeting clients, court room appearances and depositions are conducted at [his] home [in Pennsylvania],' and that he has no employees at his Wilmington office." (footnotes omitted); "In 2005, the ODC inquired about Barakat's compliance with Supreme Court Rule 12, which requires Delaware attorneys to maintain a 'bona fide' office for the practice of law in Delaware."; "Barakat argues that he meets the requirements of Supreme Court Rule 12, because he is reachable by phone, and, therefore, has complied with the Rule. The Rule requires that the office 'be a place where the attorney or a responsible person acting on the attorney's behalf can be reached in person or by telephone,' and have 'the customary facilities

6

35951124_10 UPL, MDP and MJP (Defining What Lawyers Do and McGuireWoods LLP Where They Can Do It): Part II T. Spahn (1/1/19) Hypotheticals and Analyses

for engaging in the practice of law.'" (footnote omitted); "Barakat appears to suggest that Supreme Court Rule 12, as interpreted by the ODC, imposes an unconstitutional residency requirement, and violates the commerce clause of the United States Constitution. That claim is unsupported. Barakat cites Tolchin v. Supreme Court of the State of N.J. [111 F.3d 1099 (3d Cir. 1997)], a case that involved a challenge of New Jersey's bona fide office requirement. In Tolchin, the Third Circuit held that the requirement violated neither the commerce clause, nor the privileges and immunities clause, nor the equal protection clause." (footnote omitted)).

Some lawyers have challenged other residency requirements, citing constitutional principles. In 2012, the District of Columbia circuit court declined to dismiss a lawsuit challenging the federal agency's explicit exclusion of job applicants who do not live in the Washington, D.C. area.

• Pollack v. Hogan, No. 11-52-83, 2012 U.S. App. LEXIS 25549, at *1, *3 (D.C. Cir. Dec. 14, 2012) (reversing a district court's dismissal of a constitutional challenge to the United States Courts Administrative Office's rejection of a job applicant because she did not live in the Washington, D.C. area; "Malla Pollack, a lawyer and resident of Kentucky, would like to work for the Administrative Office of the United States Courts in Washington, D.C. The Administrative Office rejected her application because she did not live or work in the Washington metropolitan area. Thereafter, she brought suit against officials of the Administrative Office, solely in their official capacities, alleging that they rejected her job application in violation of her constitutional rights."; "she contended that the geographic limitation on applicants for employment violated her 'fundamental constitutional right to travel,' and she sent the Fair Employment Practices Staff a letter outlining that contention."; "Specifically, she sought an injunction prohibiting the defendants from discriminating against job applicants on the basis of their place of residence within the United States, and ordering the defendants to consider her past and future applications without regard to her place of residence.").

Other plaintiffs have challenged the constitutionality of states' common practice of allowing admission by motion only to lawyers from states which provide reciprocal rights. In 2012, an entity called Association for the Advancement of

Multijurisdiction Practice ("NAAMJP") challenged the Arizona Supreme Court rule

7

35951124_10 UPL, MDP and MJP (Defining What Lawyers Do and McGuireWoods LLP Where They Can Do It): Part II T. Spahn (1/1/19) Hypotheticals and Analyses

requiring lawyers to take the Arizona bar exam if they seek to move to Arizona from a state that does not allow admission by motion to Arizona lawyers.

• Maria Chutchian, Out-of-State Attorneys Challenge Arizona Bar Admissions Rules, Law360, August 21, 2012 ("The National Association for the Advancement of Multijurisdiction Practice (NAAMJP) last week challenged the Arizona Supreme Court's rules forcing attorneys already licensed in certain states to take the state's bar exam in order to practice law within its borders."; "In a complaint filed Aug. 13 in Arizona federal court, the association and two attorneys sued the Arizona Supreme Court and three of its justices, questioning the constitutionality of the state's bar admission rules. Though it is one of nine states to have adopted the uniform bar exam [UBE], Arizona only allows attorneys from other states admission on motion if Arizona lawyers are given reciprocity in their home states, known as 'tit-for-tat' rules."; "The suit seeks an order invalidating the Arizona Supreme Court's rule, finding tit-for-tat admission rules unconstitutional, and admitting plaintiffs Allison Girvin and Mark Anderson -- who have licenses to practice in California and Montana, respectively -- to the state bar."; "'The main thrust of our argument is that there's no reason you should be discriminated against based on your prior place of residence,' Grant Savoy of the NAAMJP said.").

Not surprisingly, many academics and practicing lawyers have urged a more centralized approach to regulating the legal profession. Among other things, they frequently point to the freedom of a motorist licensed in one state to drive anywhere in the United States without being separately licensed in those other states. The

Restatement suggests that the constitution may require that lawyers be permitted to freely practice in other states, as long as they do not appear in those other states' courts or establish a permanent presence in those states.

• Restatement (Third) of Law Governing Lawyers § 3 cmt. e (2000) ("Modern communications, including ready electronic connection to much of the law of every state, makes concern about a competent analysis of a distant state's law unfounded. Accordingly, there is much to be said for a rule permitting a lawyer to practice in any state, except for litigation matters of law or for the purpose of establishing a permanent in-state branch office. Results approaching that rule may arguably be required under the federal interstate commerce clause and the privileges and immunities clause" (emphasis added).

8

35951124_10 UPL, MDP and MJP (Defining What Lawyers Do and McGuireWoods LLP Where They Can Do It): Part II T. Spahn (1/1/19) Hypotheticals and Analyses

To be sure, some states have taken steps to nationalize legal practice. as of late

2018, 37 states have adopted the Uniform Bar Exam.

States have also had to address some recent trends in the legal profession,

including other countries' recognition of multidisciplinary practice and even nonlawyer

majority ownership of law firms. For instance, in 2012 the New York State Bar held that

even lawyers licensed in New York may not practice law in the state if they work for a

UK law firm with nonlawyer owners.

• New York LEO 911 (3/14/12) (holding that "[a] New York lawyer may not practice law principally in New York as an employee of an out-of-state entity that has non-lawyer owners or managers."; explaining the background: "Lawyers admitted to practice in New York ask whether they may enter into a business relationship with a United Kingdom ('UK') entity under the following circumstances. The UK entity would be formed as an Alternative Business Structure under the UK's Legal Services Act, which permits entities with non-lawyer supervisors and owners to render legal services. The entity would include UK non-lawyers in supervisory and ownership positions, raise capital in private equity financing, and have a professional management team. The New York lawyers would establish a New York office, where they would represent New York clients. They would be employees of the UK entity and would hold stock options and, in some cases, vested shares in the UK entity. Lawyers in the New York office would adhere to confidentiality rules and would not share confidences with UK non-lawyer managers. The entity would adhere to UK rules as well."; holding that New York Rule 5.4 applied to the lawyer's conduct because the lawyer would be practicing law from the New York office on behalf of New York clients, in contrast to New York LEO 889 (2011) in which the New York Bar held that a lawyer licensed both in New York and D.C. but who primarily practiced in D.C. could temporarily practice in New York even if the lawyer was a member of a firm that included a nonlawyer partner, as permitted by D.C. ethics rules).

At about the same time, the New York State Bar held that a lawyer admitted to practice in New York primarily practicing in another jurisdiction may occasionally practice in New York courts despite being employed by a Washington, D.C., law firm that is partially owned by nonlawyers (as permitted in D.C., but not New York).

9

35951124_10 UPL, MDP and MJP (Defining What Lawyers Do and McGuireWoods LLP Where They Can Do It): Part II T. Spahn (1/1/19) Hypotheticals and Analyses

• New York LEO 889 (11/15/11) (holding that "[a] lawyer who principally practices in another jurisdiction but is also admitted in New York may conduct occasional New York litigation, even if a non-lawyer would benefit from the resulting fees (either as a member of the lawyer's partnership in that other jurisdiction or as its employee compensated through a profit-sharing arrangement), if the arrangements comply with the ethics rules of that other jurisdiction."; explaining that the lawyer was admitted both in New York and D.C., which permits nonlawyer partners; holding that the lawyer's actions would be governed by D.C. ethics rules, which permit nonlawyer partners; "Our conclusion as to choice of law is premised on the particular facts of the inquiry. These include that the lawyer and the law firm, now and in the foreseeable future, have their principal place of business in the District of Columbia and that the bulk of their revenue is derived from matters unrelated to the State of New York. Different facts could lead to a different result. For example, if a major portion of the revenue of the lawyer or the law firm were derived from the practice of law in the State of New York, then, depending on the particular facts, Rule 8.5 could make applicable the prohibitions of New York Rule 5.4. Certainly if the partnership were created for the very purpose of litigation in New York, establishing it in the District of Columbia would be ineffective to circumvent the New York rules on fee sharing."; noting that states have taken differing positions on this issue; "Other jurisdictions have reached varying conclusions as to the choice of law that governs such situations. Compare Philadelphia Opinion 2010-7 (opining that a Pennsylvania lawyer could share fees with a non-Pennsylvania lawyer in the District of Columbia even though the DC firm had a non-lawyer partner) with ABA 91-360 (opining that lawyer practicing in a jurisdiction forbidding partnerships with non-lawyers would be subject to that prohibition even if a member of a DC firm), followed in Virginia Opinion 1584 (1994). It should be noted, however, that the ABA and Virginia opinions were based on codes without choice of law provisions similar to the current Rule 8.5 in New York. We believe that the Philadelphia opinion is the more instructive precedent, and for the reasons stated above, we believe the New York choice of law rules support a similar conclusion." (footnote omitted)).

These two opinions highlight the continuing power of states to regulate the practice of law within their borders.

So far, such moves away from state-centric regulation have not been very successful. Given states' frequently jealous guarding of their power over professionals, one would expect that the legal profession will in the future continue to be primarily regulated by states.

10

35951124_10 UPL, MDP and MJP (Defining What Lawyers Do and McGuireWoods LLP Where They Can Do It): Part II T. Spahn (1/1/19) Hypotheticals and Analyses

Best Answer

The best answer to this hypothetical is NO.

B 2/13

11

35951124_10 UPL, MDP and MJP (Defining What Lawyers Do and McGuireWoods LLP Where They Can Do It): Part II T. Spahn (1/1/19) Hypotheticals and Analyses

Jurisdiction Over Out-of-State Lawyers

Hypothetical 2

All of your law firm's lawyers are licensed in just one state, in which your one office is located. However, your lawyers occasionally travel to other states on a temporary basis to serve clients located in those other states (but with connections to your lawyers' home state). You wonder about the risk of being sued for malpractice in those other states.

May your firm be sued in states where your lawyers have temporarily provided legal services?

YES (PROBABLY)

Analysis

This issue arises when a client attempts to exercise jurisdiction over an out-of-

state lawyer in a malpractice or other case.

Courts sometimes find that they can exercise personal jurisdiction over an out-of-

state lawyer.

• Beverage v. Pullman & Comley, LLC, 316 P.3d 590, 590 (Ariz. 2014) ("This Court granted review to determine whether the court of appeals erred in concluding that Arizona courts have specific jurisdiction over a Connecticut lawyer and law firm retained by Arizona residents to provide an opinion letter regarding a tax-shelter transaction."; "After considering the briefs and oral arguments, we conclude that the court of appeals did not err and that Arizona courts have jurisdiction over the defendants in this case.")

• Wachovia Bank, N.A. v. Jones, Morrison & Womack, P.C., 42 So. 3d 667, 677 (Ala. 2009) (concluding that lawyers from other states who are not admitted to practice in Alabama may nevertheless be sued under the Alabama Legal Service Liability Act; "Jones Morrison does not dispute that it was providing legal services to the Bank in the present case. Indeed, Jones Morrison's central argument on appeal is that because it provided legal services to the Bank, the Bank was required to offer expert testimony establishing the applicable standard of care and establishing that Jones Morrison breached that standard.").

12

35951124_10 UPL, MDP and MJP (Defining What Lawyers Do and McGuireWoods LLP Where They Can Do It): Part II T. Spahn (1/1/19) Hypotheticals and Analyses

• Fogarty v. Parker, Poe, Adams, & Bernstein, L.L.P., 961 So. 2d 784 (Ala. 2007) (holding that an Alabama citizen could sue an out-of-state law firm for UPL in Alabama; allowing a lawsuit against the well-known North Carolina law firm of Parker, Poe; reversing the lower court's holding for the firm).

• Loeffelbein v. Milberg Weiss Bershad Hynes & Lerach LLP, 106 P.3d 74 (Kan. Ct. App.) (holding that Milberg Weiss could be sued in Kansas, based on the firm's activities in Kansas), review denied, 280 Kan. 983 (2005).

In 2007, a Delaware court issued a potentially frightening decision, pointing to a law firm's marketing slogans in upholding jurisdiction over that law firm in Delaware.

• Sample v. Morgan, 935 A.2d 1046, 1053-55 (Del. Ch. 2007) (holding that the plaintiff in a derivative case can sue the corporation's law firm (Baker & Hostetler), which had provided Delaware law advice to the corporation and which had advertised itself as "being able to handle the full range of any corporation's legal needs, regardless of its location in the United States" from a "'coast-to-coast' platform of offices").

On the other hand, some courts find that lawyers' activities in a state where they are not licensed are not sufficient to support personal jurisdiction over the lawyers in that state.

• Rockwood Select Asset Fund XI (6)-1, LLC v. Devine, Millimet & Branch, 750 F.3d 1178, 1179 (10th Cir. 2014) (finding that a New Hampshire law firm's preparation of an opinion letter and forwarding of the letter to Utah did not support personal jurisdiction at a law firm in Utah; "This appeal involves personal jurisdiction. A Utah company, Rockwood Select Asset Fund XI (6)- 1, LLC, was asked to loan money. In considering the request, Rockwood required the borrower to obtain an opinion letter from its New Hampshire law firm, Devine, Millimet & Branch. Devine provided the letter, which was picked up by someone (Todd Enright) and forwarded to Rockwood's owner in Utah. But Rockwood subsequently concluded that the opinion letter contained falsehoods and sued Devine in Utah federal court. The district court dismissed the suit based on lack of personal jurisdiction. Rockwood appeals, and we must decide whether Devine had sufficient contacts with Utah to permit the exercise of personal jurisdiction. We conclude that Devine's contacts with Utah were insufficient under two precedential decisions.")

• Thompson Hine, LLP v. Taieb, 734 F.3d 1187, 1188 (D.C. Cir. 2013) (holding that a District of Columbia court did not have jurisdiction to hear the -

13

35951124_10 UPL, MDP and MJP (Defining What Lawyers Do and McGuireWoods LLP Where They Can Do It): Part II T. Spahn (1/1/19) Hypotheticals and Analyses

based law firm Thompson Hine's lawsuit against a former client seeking unpaid fees; "A Florida resident retained lawyers in an Ohio law firm's [Thompson Hine] District of Columbia office to represent him in a matter pending in Oregon. When the client refused to pay for services rendered, the firm sued in the United States District Court for the District of Columbia, and the court dismissed the case for lack of personal jurisdiction. Because neither the retainer itself nor anything about the client's dealings with the law firm demonstrates that the client 'purposefully avail[ed] [him]self of the privilege of conducting activities within the [District],' . . . we affirm.")

• Gordon & Doner, P.A. v. Joros, 287 S.W.3d 325, 328, 332, 333, 335 (Tex. Ct. App. 2009) ("Is a Florida law firm that referred a Florida resident's case to a lawyer for filing in New York subject to personal jurisdiction in Texas? Because we hold that the Florida firm did not have sufficient minimum contacts with Texas to support the exercise of specific jurisdiction over it by a Texas court, we answer the question 'no.'"; "Merely contracting with a Texas resident is insufficient to establish the minimum contacts necessary to support the exercise of specific personal jurisdiction over the nonresident defendant."; "A letter from Bailey to Joros in the record indicates that the case was referred to Bailey by Gordon solely because of Bailey's experience in pharmaceutical litigation."; "But he overlooks Gordon's similar affidavit here, which was likewise uncontroverted, that Gordon performed all of its legal services on behalf of Joros in Florida. His argument also omits the additional undisputed language in the agreement that the legal work to be performed in Texas would be performed by Bailey, not by Gordon."; "[T]here is also no evidence that Gordon exercised any actual control or supervision over Bailey's performance. Consequently, there is no basis in the record for imputing Bailey's conduct to Gordon as a purposeful contact with Texas." (first emphasis added)).

• Proskauer Rose LLP v. Pelican Trading, Inc., No. 14-08-00283-CV, 2009 Tex. App. LEXIS 667, at *9, *9-10 (Tex. App. Feb. 3, 2009) (dismissing a malpractice claim based on lack of personal jurisdiction over the law firm; "[T]he engagement letter e-mailed from New York to Walsh was the result of Ernst & Young's introduction of Proskauer Rose to Pelican. Next, neither Pelican's transmittal of the signed engagement letter to Proskauer Rose nor the accompanying check drawn on Pelican's Texas bank qualifies as a 'purposeful' contact because they were Pelican's actions, not Proskauer Rose's."; "[T]he draft opinion letters and 'Certificate of Facts' transmitted both to Pelican and its Texas attorneys reflect legal services performed in New York, not Texas. . . . We observed in Markette, in agreement with other Texas courts, that neither the mere existence of an attorney-client relationship between a resident client and an out-of-state attorney nor the routine correspondence and interactions attendant to that relationship are enough to confer personal jurisdiction.").

14

35951124_10 UPL, MDP and MJP (Defining What Lawyers Do and McGuireWoods LLP Where They Can Do It): Part II T. Spahn (1/1/19) Hypotheticals and Analyses

• Stacy v. Johnson, Civ. A. No. 07-cv-51-SM, 2007 U.S. Dist. LEXIS 63983, at *6, *10-11 (D. N.H. Aug. 28, 2007) (finding the New Hampshire court could not exercise jurisdiction over Texas lawyer for alleged malpractice in the lawyer's participation in the creation of guardianship over the plaintiff; "Stacy has failed to establish that Johnson maintains a continuous and systematic presence in New Hampshire. Nothing presented shows that Johnson maintains a place of business in the state, or is licensed to practice law here, or owns property or transacts any business here, or that he has ever appeared as an attorney in a New Hampshire court. Indeed, Johnson has not visited the state since 1978. Accordingly, the court lacks general personal jurisdiction over Johnson."; "Stacy also argues that jurisdiction in New Hampshire is appropriate because all or substantially all of Wyatt's guardianship work took place while he was physically in New Hampshire, and because Johnson routinely sent his own bills to Wyatt's office here for payment by the conservatorship. But the allegedly improper conduct that forms the basis of Stacy's complaint - namely, the improper or unethical pursuit of a Texas guardianship over Stacy, and the formation of a relationship between Wyatt and Johnson, notwithstanding Wyatt's professional conflicts - all took place in Texas and all related to the Texas proceeding. While it is true that the Johnson's bills were paid by a New Hampshire conservatorship, from New Hampshire, the mere fact that some effects of the allegedly improper conduct were coincidentally felt in New Hampshire does not itself give rise to personal jurisdiction over Johnson in this state.").

Best Answer

The best answer to this hypothetical is YES (PROBABLY).

B 2/13, 3/17

15

35951124_10 UPL, MDP and MJP (Defining What Lawyers Do and McGuireWoods LLP Where They Can Do It): Part II T. Spahn (1/1/19) Hypotheticals and Analyses

Status of Pleadings Signed by Out-of-State Lawyers

Hypothetical 3

You just received an urgent call from one of your partners. She realized a few minutes ago that the statute of limitations is about to expire on a claim that her brother wants to bring against a trucking company in a neighboring state (just across the river from your office). Neither your partner nor any of your small boutique law firm's other lawyers is licensed in the other state. The courts close in about 45 minutes there, and your partner wants to know what she can do. She tells you that she can prepare a bare-bones complaint and file it in the courthouse across the river in about 40 minutes.

Will the neighboring state's court consider valid a complaint signed and filed by your partner, even though she is not licensed there?

NO (PROBABLY)

Analysis

Courts dealing with this situation have addressed both the unauthorized practice

of law aspects of such a lawyer's conduct, as well as the effect of the lawyer's conduct

on pleadings' validity.

Some courts have taken a liberal approach.

• DeCook v. Olmsted Med. Ctr., Inc., 875 N.W.2d 263, 264-65, 272 (Minn. 2016) (finding that a pleading filed by a non- lawyer could be amended at the trial court's discretion; "In this appeal from the denial of a motion to dismiss for insufficiency of process, we consider the question of whether a summons and complaint signed by an attorney licensed outside the state of Minnesota are legal nullities, or whether the defective signature may be cured. In the related conditional cross-appeal, we consider whether the district court erred in granting a motion to dismiss as to several defendants for insufficiency of service. Because we conclude that the Minnesota Rules of Civil Procedure afford a district court the discretion to allow an amendment to cure a defective signature on a summons and complaint, we affirm the court of appeals' judgment in part. However, because we conclude that the plaintiffs produced sufficient evidence of effective service to shift the burden to the defendants to prove that service was not effective and that burden was not carried, we reverse in part."; "In sum, we conclude that the summons and complaint bearing only the

16

35951124_10 UPL, MDP and MJP (Defining What Lawyers Do and McGuireWoods LLP Where They Can Do It): Part II T. Spahn (1/1/19) Hypotheticals and Analyses

signature of an attorney not licensed to practice in Minnesota were defective. That signature defect did not render the summons and complaint void, however, the district court did not abuse its discretion in allowing them to be amended. Thus, we affirm in part the judgment of the court of appeals."; "We further conclude that the district court clearly erred in finding that the record contained no evidence that the defendants consented to service. The DeCooks submitted evidence that the defendants had agreed to be served by email through the Medical Center's compliance officer. Because the DeCooks' evidence of service was wholly unrebutted, the defendants did not satisfy their burden to prove that service was insufficient. Accordingly, we reverse in part the judgment of the court of appeals, and remand to the district court for further proceedings consistent with this opinion.").

• Richardson v. First Tenn. Bank, N.A. (In re Estate of Cooper), 746 N.W.2d 653, 662 (Neb. 2008) (holding that a lawyer did not commit the unauthorized practice of law in Nebraska by filing a claim on behalf of a client; "The record reflects that First Tennessee is a Tennessee banking corporation, with its principal place of business in Memphis, Tennessee, the same city and state where Wright, the Tennessee lawyer who filed the demand for notice, maintains her law practice. First Tennessee is a client of Wright. The request for notice sought to have copies of all filings made in the underlying estate case mailed to Wright in the same state where she offices and First Tennessee has its principal place of business. The 'risk' involved to either the client, the public, or the courts was de minimis. The filing of the request for notice was effectively an administrative matter and did not in and of itself involve either rendering a legal opinion to First Tennessee or engaging in a legal contest on behalf of First Tennessee in Nebraska. Given these facts, we conclude that the county court erred as a matter of law when it determined that Wright's filing of the demand for notice constituted the unauthorized practice of law under either rule 5.5(c) or § 7-101 and ordered the demand struck. We reverse such order." (emphases added)).

Similarly, courts generally do not invalidate criminal convictions on the grounds that the prosecutor was not licensed in the state (or not even licensed at all).

• State v. Graham, 764 N.W.2d 340, 358, 358-59, 359 (Minn. 2009) ("[A criminal defendant] places considerable weight on the case of Dunson, 316 Ill. App. 3d 760, 737 N.E.2d 699, 250 Ill. Dec. 77, in urging us to adopt a rule which per se invalidates any conviction obtained by an unlicensed attorney. Under its common law, Illinois invalidates any prosecution by an attorney unauthorized to practice law. Id. at 702. But Illinois also appears to have a lengthy history of invalidating any action instituted by an unlicensed practitioner whether the case is civil or criminal in nature. See id. at 702, 704. Minnesota does not share a similar common

17

35951124_10 UPL, MDP and MJP (Defining What Lawyers Do and McGuireWoods LLP Where They Can Do It): Part II T. Spahn (1/1/19) Hypotheticals and Analyses

law approach."; "There does not appear to be any jurisdiction which has held that a defendant's due process rights are per se violated as a consequence of prosecution by an attorney unauthorized to practice law. We conclude there is no sound basis for applying a per se rule and decline to do so." (footnote omitted); "[W]e hold that convictions obtained by a prosecutor who was unlicensed to practice law at the time of the conviction should be set aside only when the defendant is able to show prejudice warranting reversal." (emphasis added)).

In contrast, other states take a very unforgiving view -- and find such a pleading to be a legal "nullity."

• Blume Constr., Inc. v. State, 872 N.W.2d 312, 314, 316, 319, 320 (N.D. 2015) (holding that an appeal of an administrative ruling was void because the Colorado lawyer who signed on behalf of appellant had not taken any steps to be admitted pro hac in North Dakota; "Prior to the hearing, Blume requested the hearing be postponed because it had retained the service of Fidler, who was identified as a licensed attorney from Colorado. Fidler was not licensed to practice law in North Dakota. In approximately May 2014, Fidler notified the referee he was unable to secure a sponsoring attorney licensed in North Dakota. During the same time period, the referee was informed a North Dakota attorney would be representing Blume."; "Blume is a corporation, and there is no evidence in the record that Fidler is employed by Blume or that he is an officer or on the corporation's board of directors. Evidence established that Fidler was a licensed attorney in Colorado, but he was not licensed to practice law in North Dakota."; "Evidence establishes Fidler signed the appeal request and Blume requested the hearing be postponed because it had retained Fidler's services. Fidler engaged in the practice of law in this state by signing the appeal request and being designated as counsel in the action before Job Service. There is no evidence in the record that he applied for pro hac vice admission."; "We have explained that a corporation may not be represented by a non-attorney agent in a legal proceeding, just as a unlicensed natural person may not act as an attorney for another natural person. . . . When a non-attorney agent represents a corporation in a legal proceeding, all legal documents signed and filed by the non-attorney are void. . . . Because Blume is a corporation and may not be represented by a non-attorney agent, Fidler's request for an appeal was not conduct that could be performed by a person without a license to practice law."; "We conclude Fidler's actions filling out and submitting the appear request form on behalf of a corporation were not services a person without a license to practice law could perform, and therefore the safe harbor of N.D.R. Prof. Conduct 5.5(A)(2) does not apply."; "We conclude Fidler's activities completing and submitting the appeal request on Blume's behalf were not protected by the safe harbor provisions of N.D.R.

18

35951124_10 UPL, MDP and MJP (Defining What Lawyers Do and McGuireWoods LLP Where They Can Do It): Part II T. Spahn (1/1/19) Hypotheticals and Analyses

Prof. Conduct 5.5(b). Fidler was required to move for pro hac vice admission under Admission to Practice R. 3(b)(2) within 45 days of filing the appeal request. There is no evidence in the record that Fidler moved for pro hac vice admission, and therefore we conclude Blume's appeal request by its non-attorney agent was void.").

• State ex rel. Hadley v. Pike, 2014-Ohio-3310, at ¶¶ 16, 17 (Ohio Ct. App. 2014) (holding that a complaint filed by a non-Ohio lawyer not admitted pro hac was void ab initio; "It is undisputed that Attorney Lucas did not register with the Ohio Supreme Court Office of Attorney Services until November 1, 2013, two weeks after the complaint was filed. We note that registering with the Ohio Supreme Court is only the preliminary step to being granted pro hace vice status. The attorney must subsequently file a motion with the trial court, and the trial court decides whether to grant pro hac vice admission. Lucas did not file this motion with the Columbiana County Court of Common pleas until November 25, 2013, two days after Relators filed their motion to dismiss the complaint for lack of jurisdiction. The court did not actually grant Lucas's motion for pro hac vice until December 26, 2013, the day it also overruled the motion to dismiss."; "Clearly, Attorney Lucas was not admitted to practice law in Ohio when he filed the complaint. Therefore, the complaint in Case No. 13 CV 631 was void ab initio. The trial court had no discretion in ruling on the motion to dismiss the complaint. The complaint should have been dismissed without prejudice for lack of subject matter jurisdiction. A writ of mandamus is appropriate because Relators have a clear legal right to dismissal of the complaint for lack of jurisdiction and Respondent has a clear legal duty to perform the act requested. In addition, a writ of prohibition is warranted because any further prosecution of the matter by Respondent is unauthorized and without jurisdiction under the law. Because the matters for review are jurisdictional in nature, it is unnecessary to determine whether Relators had other legal remedies available for relief.").

• Shipe v. Hunter, 699 S.E.2d 519, 520-21, 521, 522 (Va. 2010) (holding that a non-Virginia lawyer cannot be deputized by a Virginia lawyer to sign a pleading; "We have repeatedly held that a pleading, signed only by a person acting in a representative capacity who is not licensed to practice law in Virginia, is a nullity. . . . The plaintiff argues, however, that a person may make another his agent for the purpose of signing a pleading and that the signature of the agent, if properly authorized by the principal, would be effective as if the principal had personally signed the pleading." (emphasis added); "For the protection of the public from harassment by frivolous, oppressive, fraudulent or purely malicious litigation, the General Assembly has chosen to hold attorneys and pro se litigants to a high degree of accountability for the assertions they make in judicial proceedings."; upholding dismissal of the complaint "[b]ecause neither a pro se plaintiff nor

19

35951124_10 UPL, MDP and MJP (Defining What Lawyers Do and McGuireWoods LLP Where They Can Do It): Part II T. Spahn (1/1/19) Hypotheticals and Analyses

an attorney licensed to practice law in Virginia signed the complaint, and because that defect could not be cured by amendment.").

• Carlson v. Workforce Safety & Ins., 765 N.W.2d 691, 703, 704 (N.D. 2009) (holding that an insurance company's request for reconsideration in an administrative proceeding was void, because it had been filed by an Ohio lawyer who was not admitted in North Dakota; "Under the plain language of Admission to Practice R. 3(A), pro hac vice admission is required for all nonresident attorneys admitted to practice law in another state and not licensed in North Dakota who engage in the practice of law by appearing, either in person, by signing pleadings, or by being designated as counsel in actions filed in administrative agencies. Here, on October 13, 2006, GMR's president notified WSI that Plewacki and Stemen would serve as GMR's special counsel for Carlson's claim; on October 26, 2006, Stemen, on behalf of GMR, requested reconsideration of Carlson's claim; on November 11, 2006, Stemen advised WSI that, on behalf of Roetzel & Andress, she was GMR's authorized representative for Carlson's claim; and on November 16, 2006, Stemen and Plewacki submitted a legal brief and documents in support of GMR's request for reconsideration."; "GMR's nonresident attorneys were required to file a motion for pro hac vice admission under Admission to Practice R. 3(A) within 45 days of their appearance in this agency proceeding. Because GMR's nonresident attorneys failed to timely comply with the requirements for pro hac vice admission, we conclude GMR's request for reconsideration by its nonattorney agents was void." (emphasis added)).

• Wellmore Coal Corp. v. Harman Mining Corp., 568 S.E.2d 671, 672-73, 673 (Va. 2002) (holding that a notice of appeal amounted to a nullity because it was signed only by an out-of-state lawyer; explaining that "Wellmore filed a notice of appeal on June 5, 2001, within the 30-day period required by Rule 5:9(a). However, the notice of appeal was signed only by Woods, Wellmore's foreign counsel. Rule 1A:4 governs the practice of law by foreign attorneys in Virginia. In pertinent part, the Rule states: 'Except where a party conducts his own case, a pleading, or other paper required to be served (whether relating to discovery or otherwise) shall be invalid unless it is signed by a member of the Virginia State Bar.'"; rejecting Wellmore's argument that the out-of-state lawyer's signature was a "clerical error" that could be corrected pursuant to another Virginia statute; "We recognize that Code § 8.01-428(B) provides for the correction of clerical errors; however, the signature defect at issue in the present case is not a 'clerical error' as contemplated by the Code."; ultimately dismissing Wellmore's appeal; "The failure of Wellmore's Virginia counsel to append his signature to the notice of appeal, as required by Rule 1A:4, does not constitute a clerical error contemplated by Code § 8.01-418; accordingly, Code § 8.01-428(B) provides Wellmore no relief in the present case. Based upon the failure of the notice

20

35951124_10 UPL, MDP and MJP (Defining What Lawyers Do and McGuireWoods LLP Where They Can Do It): Part II T. Spahn (1/1/19) Hypotheticals and Analyses

of appeal to comply with Rule 1A:4 and the filing of the amended notice outside the 30-day requirement of Rule 5:9(a), we will grant Harman's and Sovereign's motion to dismiss Wellmore's appeal.").

• Preston v. Univ. of Ark., 128 S.W.3d 430, 437, 438 (Ark. 2003) (holding that pleadings filed in Arkansas by an Oklahoma lawyer who had not properly been admitted to practice in Arkansas amounted to a nullity; first rejecting the clients' claim that they could have represented themselves pro se; "The Prestons finally claim that, because their attorneys were not licensed to practice law in Arkansas, it is as if the Prestons appeared pro se. They, however, did not appear pro se, because they hired attorneys to handle their case, and those attorneys signed the complaint."; "In the case at bar, Richard Preston underwent surgery to his left femur on November 19, 1999. On February 17, 2000, x-rays were taken and revealed debris in the muscle tissue of the left thigh. Two years after his surgery on November 19, 2001, the unauthorized Oklahoma counsel filed the complaint on behalf of the Prestons. On July 25, 2002, these same attorneys filed a motion for admission to practice pro hac vice. On October 3, 2002, the circuit court found that, because the Prestons' attorneys were unauthorized to practice law in Arkansas, their complaint was a nullity, as if it had never been filed. Under § 16-114-203(a), the statute of limitations for medical malpractice expired on November 19, 2001, the day the complaint, which was a nullity, was filed. Accordingly, no valid complaint was filed within the required two- year period. Under § 16-114-203(b), even assuming arguendo that the debris found in Richard Preston's thigh falls within the 'foreign object' exception, the statute of limitations has run. However, at that time no valid and operable complaint had been filed. Thus, under either the general statute of limitations for medical malpractice or the statute of limitations for the 'foreign object' exception, the Prestons' attorneys were not authorized to practice law in Arkansas at the time the statute of limitations had run."; "We reiterate our holding that the complaint was a nullity due to Oklahoma counsel's unauthorized practice of law and, therefore, void and of no effect. We further hold that the circuit court correctly dismissed this matter because no valid complaint had been filed within the time period of the statute of limitations." (emphasis added)).

Among other things, this issue involves the fairness of punishing a client for the lawyer's mistake or misconduct -- by rendering the pleading a nullity. Depriving the client of some right or privilege based on a lawyer's mistake essentially condemns the client to seek compensation from the lawyer in a malpractice action.

21

35951124_10 UPL, MDP and MJP (Defining What Lawyers Do and McGuireWoods LLP Where They Can Do It): Part II T. Spahn (1/1/19) Hypotheticals and Analyses

Courts sometimes look for ways to sanction lawyers for their sloppiness or

intentional misconduct, without any "collateral damage" to the client. Here, the majority

of states seem to take the other approach -- considering pleadings filed by out-of-state lawyers to be a nullity, regardless of the effect on the client.

Another option in this situation would be for the lawyer to prepare a pro se pleading that the client could sign and file. However, there might be ethical limitations on such an action, and most courts condemn it.

Best Answer

The best answer to this hypothetical is NO (PROBABLY).

B 2/13

22

35951124_10 UPL, MDP and MJP (Defining What Lawyers Do and McGuireWoods LLP Where They Can Do It): Part II T. Spahn (1/1/19) Hypotheticals and Analyses

Out-of State Lawyers' Ability to Collect Fees

Hypothetical 4

You should have followed your initial instinct before beginning to represent an estate planning client -- she has been nothing but trouble since you began to work with her. The final straw came yesterday, when she told you that she would not pay your bill, because you are not licensed in the state where she lives.

Will you be able collect your bill for estate planning work you have performed for a client, who lives in a state where you are not licensed?

YES (PROBABLY)

Analysis

Before the ABA addressed the multijurisdictional practice issue, most states dealt

with that issue in connection with an out-of-state lawyer's attempt to collect fees for work performed in states where the lawyer was not licensed.

In some cases, the out-of-state lawyers lost their attempt to recover fees.

• Birbrower, Montalbano, Condon & Frank, P.C. v. Superior Court, 949 P.2d 1, 2, 5-6, 8, 9, 10, 11 (Cal. 1998) (concluding that New York lawyers who represented a California client in connection with a California arbitration could not recover fees for services performed physically or virtually in California, but could recover fees for work the New York lawyers performed in New York; noting that a California statute indicated that "[n]o person shall practice law in California unless the person is an active member of the State Bar"; noting that the California client hired New York lawyers to assist with a California arbitration, although the dispute was settled before the arbitration; explaining that the client sued the lawyer for malpractice, and that the lawyers counterclaimed for his fees; "Our definition does not necessarily depend on or require the unlicensed lawyer's physical presence in the state. Physical presence here is one factor we may consider in deciding whether the unlicensed lawyer has violated section 6125, but it is by no means exclusive. For example, one may practice law in the state in violation of section 6125 although not physically present here by advising a California client on California law in connection with a California legal dispute by telephone, fax, computer, or other modern technological means. Conversely, although we decline to provide a comprehensive list of what activities

23

35951124_10 UPL, MDP and MJP (Defining What Lawyers Do and McGuireWoods LLP Where They Can Do It): Part II T. Spahn (1/1/19) Hypotheticals and Analyses

constitute sufficient contact with the state, we do reject the notion that a person automatically practices law 'in California' whenever that person practices California law anywhere, or 'virtually' enters the state by telephone, fax, e-mail, or satellite."; "Birbrower next argues that we do not further the statute's intent and purpose -- to protect California citizens from incompetent attorneys -- by enforcing it against out-of-state attorneys. Birbrower argues that because out-of-state attorneys have been licensed to practice in other jurisdictions, they have already demonstrated sufficient competence to protect California clients. But Birbrower's argument overlooks the obvious fact that other states' laws may differ substantially from California law. Competence in one jurisdiction does not necessarily guarantee competence in another. By applying section 6125 to out-of-state attorneys who engage in the extensive practice of law in California without becoming licensed in our state, we serve the statute's goal of assuring the competence of all attorneys practicing in this state."; "Whether an attorney is duly admitted in another state and is, in fact, competent to practice in California is irrelevant in the face of section 6125's language and purpose."; "Birbrower alternatively asks us to create an exception to section 6125 for work incidental to private arbitration or other alternative dispute resolution proceedings."; "We decline Birbrower's invitation to craft an arbitration exception to section 6125's prohibition of the unlicensed practice of law in this state. Any exception for arbitration is best left to the Legislature, which has the authority to determine qualifications for admission to the State Bar and to decide what constitutes the practice of law."; "We agree with the Court of Appeal to the extent it barred Birbrower from recovering fees generated under the fee agreement for the unauthorized legal services it performed in California. We disagree with the same court to the extent it implicitly barred Birbrower from recovering fees generated under the fee agreement for the limited legal services the firm performed in New York." (emphasis added); "We agree with Birbrower that it may be able to recover fees under the fee agreement for the limited legal services it performed for ESQ in New York to the extent they did not constitute practicing law in California, even though those services were performed for a California client." (emphasis added)).

• Chandris, S.A. v. Yanakakis, 668 So.2d 180 (Fla. 1995) (lawyer living and practicing in Florida but not admitted to the Florida bar could not enter into a contingent fee arrangement with a Florida client, and therefore could not recover fees from that client; result would have been different if the lawyer had joined with a Florida lawyer in a joint representation).

• Perlah v. S.E.I. Corp., 612 A.2d 806, 808 (Conn. 1992) (New York lawyer advising a Connecticut client could not recover fees for the time before he was admitted in Connecticut; "'Generally, even though duly admitted in another state, an attorney may not recover compensation for legal services unless he has been admitted to practice in the jurisdiction where the services

24

35951124_10 UPL, MDP and MJP (Defining What Lawyers Do and McGuireWoods LLP Where They Can Do It): Part II T. Spahn (1/1/19) Hypotheticals and Analyses

were rendered.' 7 Am. Jur.2d 280, Attorneys at Law § 242" (emphasis added)).

• Martin & Martin v. Jones, 541 So.2d 1 (Ala. 1989) (Mississippi law firm could not recover attorney's fees from an Alabama resident under the terms of a retainer agreement with an Alabama client because it would have been improper for the Mississippi lawyers to practice law in Alabama).

• Ranta v. McCarney, 391 N.W.2d 161, 164, 165 (N.D. 1986) ("A problem develops, however, in relation to exceptions to the rule that many jurisdictions have developed. The exception of Federal court practice (as opposed to State court practice) does not apply because Ranta's conduct did not involve an appearance in a federal court."; holding that out-of-state lawyer could not recover fees for work in state in which the lawyer is not licensed, and remanding for a determination of which fees related to the practice of law conducted outside of North Dakota; "Nor do we perceive a justification for Ranta's conduct under the interstate practice exception. Although some States allow an out-of-State attorney to recover fees where the attorney made proper disclosure to the client and associated with local counsel . . . we need not reach the issue because such is not the situation here"); "Another State has adopted a 'sister-State' exception, allowing recovery when the attorney is licensed to practice law in a sister State and has not offended the spirit of intention of the statutes regulating the practice of law. . . . But even if we were to adopt an interstate or sister-State exception (a point which we do not reach), such an exception would not apply here in light of Ranta's long-term unauthorized practice in this State, his involvement with many other area clients, and his opening a branch office in Bismarck.").

• Lozoff v. Shore Heights, Ltd., 342 N.E.2d 475, 480 (Ill. App. Ct. 1976) ("We therefore find that plaintiff is precluded from recovering for legal services rendered in Illinois when he was not licensed to practice law in Illinois"), aff'd, 362 N.E.2d 1047 (Ill. 1977).

Other cases from that era went the other way -- permitting lawyers to recover their fees.

• Koscove v. Bolte, 30 P.3d 784, 786, 788 (Colo. Ct. App. 2001) (relying on the Colorado statute in allowing a client to recover fees paid to a lawyer that was not licensed to practice law in Colorado; determining first that the lawyer had agreed to provide legal services rather than non-legal services; "The agreement does not make specific reference to legal services, but it is replete with phrases that suggest that was its purpose. Bolte [lawyer] was required to 'investigate, examine, copy, analyze and interpret' documents

25

35951124_10 UPL, MDP and MJP (Defining What Lawyers Do and McGuireWoods LLP Where They Can Do It): Part II T. Spahn (1/1/19) Hypotheticals and Analyses

relating to the mineral lease. He was to 'pursue' the payments due Koscove [client]. Bolte's fees were based on a percentage share of royalties that were increased or enhanced, a provision almost indistinguishable from an attorney's contingent fee arrangement."; rejecting the lawyer's argument that the Colorado state court could not apply Colorado law to the lawyer's conduct because it related to federal litigation; "Bolte provided legal services to Koscove on Colorado state law claims before any decision was made to file a case in federal court and long before he was admitted pro hac vice in the federal court. He continued to operate under the same agreement after his pro hac vice admission. We do not suggest that the federal court does not have jurisdiction to decide whether, under federal law, Bolte is entitled to fees for his work before that court. But that fact, if true, does not deprive Colorado courts of jurisdiction over the agreement that the parties entered into long before the jurisdiction of the federal court was invoked."; quoting the Colorado statute, and permitting the client to recover the fees paid to the lawyer), cert. denied, 534 U.S. 1128 (2002).

• El Gemayel v. Seaman, 533 N.E.2d 245, 248-49 (N.Y. 1988) (Lebanese lawyer could recover fees from a New York client despite the client's allegation that the lawyer had improperly practiced law in New York; "[P]laintiff's contacts with New York were, as Supreme Court found, incidental and innocuous. Although plaintiff engaged in substantial litigation in Lebanon, where he was licensed, and even arguably provided legal services while in Washington, D.C., and , his contact with New York consisted entirely of phone calls to defendant and her daughter Mary in New York in which they discussed the progress of the legal proceedings in Lebanon. There was but a single visit to Phoenix, New York, after the successful completion of his legal services. We conclude that, in the circumstances of this case, phone calls to New York by plaintiff, an attorney licensed in a foreign jurisdiction, to advise his client of the progress of legal proceedings in that foreign jurisdiction, did not, without more, constitute the 'practice' of law in this State in violation of Judiciary Law § 478.").

• Dietrich Corp. v. King Resources Co., 596 F.2d 422, 426 (10th Cir. 1979) (holding that a law professor at the University of Colorado who acted as a consultant had not engaged in the unauthorized practice of law even though he was not licensed in Colorado, and therefore could recover fees; "Is what he did, providing services in the field of his legal expertise to or through established law firms, with no court appearances as an attorney, the practice of law in Colorado? The cases and ethics opinions we have seen have involved either court appearances as counsel for private clients or the rendering of legal services directly to a client who was not a lawyer or law firm authorized to practice in the jurisdiction. Law firms have always hired unlicensed student law clerks, paralegals and persons who have completed

26

35951124_10 UPL, MDP and MJP (Defining What Lawyers Do and McGuireWoods LLP Where They Can Do It): Part II T. Spahn (1/1/19) Hypotheticals and Analyses

their legal education but are awaiting admission to the bar, before or after taking a bar examination or fulfilling residency requirements. Virtually every lawyer has served in such a situation and performed services to or through other attorneys for some period prior to his or her own admission to practice in the state where such services were rendered. No one has treated this activity as the unauthorized practice of law, because the licensed attorneys alone remain responsible to the clients, there are no court appearances as attorney, and no holding out of the unlicensed person as an Independent giver of legal advice").

• Schaeffer v. Sykes (In re Estate of Waring), 221 A.2d 193, 198 (N.J. 1966) (New York law firm could recover fees for work on a New Jersey estate despite claims that the law firm was practicing in New Jersey without a license; noting that "its work was performed largely outside the borders of New Jersey and dealt for the most part with federal tax matters").

• Spivak v. Sachs, 211 N.E.2d 329, 331 (N.Y. 1965) (California lawyer was allowed to recover fees from a New York client despite the client's claim that the lawyer was practicing law in New York without a license for approximately 14 days; "Here we have a California lawyer brought to New York not for a conference or to look over a document but to advise directly with a New York resident as to most important marital rights and problems. Not only did he give her legal counsel as to those matters but essayed to give his opinion as to New York's being the proper jurisdiction for litigation concerning the marital res and as to related alimony and custody issues, and even went so far as to urge a change in New York counsel. . . . [W]e cannot penalize every instance in which an attorney from another State comes into our State for conferences or negotiations relating to a New York client and a transaction somehow tied to New York.").

• Appell v. Reiner, 204 A.2d 146, 148 (N.J. 1964) (finding that New York lawyer could foreclose on two notes given in payment for his services, despite former client's claim that the lawyer was not entitled to fees because he committed the unauthorized practice of law in New Jersey; "The Chancery Division correctly delineated the generally controlling principle that legal services to the furnished to New Jersey residents relating to New Jersey matters may be furnished only by New Jersey counsel. We nevertheless recognize that there are unusual situations in which a strict adherence to such a thesis is not in the public interest. In this connection recognition must be given to the numerous multi-state transactions arising in modern times. This is particularly true of our State, situated as it is in the midst of the financial and manufacturing center of the nation. An inflexible observance of the generally controlling doctrine may well occasion a result detrimental to the public interest, and it follows that there may be instances justifying such exceptional treatment warranting the ignoring of state lines. This is such a

27

35951124_10 UPL, MDP and MJP (Defining What Lawyers Do and McGuireWoods LLP Where They Can Do It): Part II T. Spahn (1/1/19) Hypotheticals and Analyses

situation. Under the peculiar facts here present, having in mind the nature of the services to be rendered, the inseparability of the New York and New Jersey transactions, and the substantial nature of the New York claim, we conclude that plaintiff's agreement to furnish services in New Jersey was not illegal and contrary to public policy.").

This trend has continued after most states began to follow the ABA's approach to permissible multijurisdictional practice -- which the ABA adopted in 2002.

• Beery v. Chandler, No. 4:14CV456 RLW, 2015 U.S. Dist. LEXIS 5990, at *10 (E.D. Mo. Jan. 20, 2015) (holding that a lawyer not licensed in Missouri may counterclaim for fee related to services the lawyer provided to a Missouri client; "At this stage of the proceedings, the Court finds that Roper [defendant, counter-claimant] has at least stated a claim for which relief may be granted to survive a motion to dismiss, and Plaintiff has not provided any specific case law indicating that Missouri law precludes Roper from asserting his claims. The Court also notes that discovery is still ongoing, and the disposition motion deadline has not yet passed. Thus, the Court will deny Plaintiff's motion to dismiss Roper's counterclaims on the basis of public policy." (footnote omitted))

• Forbes v. Hixson (In re Estate of St. Martin), 145 So. 3d 1124, 1130, 1137-38 (Miss. 2014) (holding that an out-of-state lawyer could recover a contingent fee despite having violated several ethics rules, and engaging in the unauthorized practice of law in Mississippi; "We agree that antisettlement and antitermination clauses in contingent-fee contracts have been held to be unenforceable. However, Mississippi caselaw is abundant that 'if a court strikes a portion of an agreement as being void, the remainder of the contract is binding.' Neither of these provisions was at issue in this case. No evidence was presented that these provisions caused any breach of duty or damages to Forbes. As such, we find that the contracts' two invalid provisions should be struck, but that the remainder of the contracts, including the contingency-fee provision, is still binding." (footnotes omitted); "[W]e find no support for the Court of Appeals' determination that the contract may not be enforceable because St. Martin entered into the contract knowing that he could not represent the Forbeses without making an appearance in Mississippi, which his lack of a Mississippi law license precluded him from doing. Attorneys routinely enter into contracts for representation with clients in matters that they know may be litigated in foreign jurisdictions in which they may not be licensed to practice. This is the purpose of our rules requiring attorneys to associate local counsel and to be admitted pro hac vice, and nothing in our rules or law indicate a contract for legal representation is illegal or void merely because an attorney entered into it with the knowledge that he likely will have to associate local counsel and/or

28

35951124_10 UPL, MDP and MJP (Defining What Lawyers Do and McGuireWoods LLP Where They Can Do It): Part II T. Spahn (1/1/19) Hypotheticals and Analyses

be admitted pro hac vice to fulfill the representation. There is no evidence that St. Martin could not have been successfully admitted pro hac vice in this matter if he had chosen to be. Furthermore, it is not unusual for some matters to be resolved through settlement before litigation commences, in which case an attorney would be able to fulfill his representation without every making an appearance in court. Therefore, we find that, absent some evidence of fraud or misrepresentation, a contract for legal representation is not void merely because the attorney knows or should know that he may be required to enter an appearance in a foreign jurisdiction.")

• Priestly v. Astrue, 651 F.3d 410, 417 (4th Cir. 2011) (holding that plaintiffs could recover fees under the Equal Access to Justice Act for the services of lawyers who are not licensed in the state where the lawsuit was litigated; "[T]he record strongly suggests that Martin and Naides [out-of-state lawyers] did not 'appear' before the district court. Indeed, their role was quite limited. Martin and Naides never physically appeared in court or before the judge; they never filed a brief or paper in the court; they never communicated with opposing counsel; they never even communicated with the plaintiffs; their work agreement was with McChesney, a member of the district court bar, and not with the plaintiffs; and they prepared only drafts of briefs and papers which they then submitted to McChesney for editing, signing, and filing with the court. Even though their names were sometimes included on briefs as attorneys for the plaintiffs and they stated in their EAJA affidavits that they were attorneys for plaintiffs, the nature of their brief-writing function in supporting McChesney never changed. . . . Indeed, McChesney could just as well have retained an English professor or law professor to assist him in writing briefs and submitted those charges as paralegal type fees, with a legitimate expectation that his clients would receive reimbursement.").

• Arizona LEO 10-04 (6/2010) ("An Arizona lawyer may divide a fee with a lawyer admitted in another United States jurisdiction if the client consents to the arrangement in writing, each lawyer receiving any portion of the fee assumes joint responsibility for the representation, and the total fee is reasonable. In addition to complying with these general rules regarding fee division, the out-of-state lawyer must be in good standing, admitted in a United States jurisdiction, and providing services to the Arizona client in association with a lawyer who is admitted to practice in Arizona and who actively participates in the matter. The client must consent in writing to the fee division, acknowledge the out-of-state lawyer is not admitted in Arizona, and consent to the out-of-state lawyer's representation. The out-of-state lawyer must either ensure that he or she is admitted pro hac vice in order to provide legal services that require pro hac vice admission or be eligible to provide temporary legal services in Arizona pursuant to ER 5.5."; explaining that in Arizona the term "joint responsibility" does not necessarily require "substantive involvement in the matter"; "Arizona fee division rules do not

29

35951124_10 UPL, MDP and MJP (Defining What Lawyers Do and McGuireWoods LLP Where They Can Do It): Part II T. Spahn (1/1/19) Hypotheticals and Analyses

require that a lawyer have substantive involvement in the matter on which the fee is divided. . . . The client must, however, consent to the fee division arrangement in writing and the lawyers must assume joint responsibility for the representation. . . . Therefore, in response to the first question, a fee may not be divided with the out-of-state lawyer due to non-compliance with ER 1.5(e).").

• Winterrowd v. Am. Gen. Annuity Ins. Co., 556 F.3d 815, 825 (9th Cir. 2009) ("We hold that because Wheatley, Sr., a non-member of the California Bar, did not physically appear before the Central District, did not sign pleadings in the case before the Central District, had minimal, nonexclusive contacts with the Winterrowd plaintiffs, that Wheatley, Jr., who was licensed to practice law in California, was the person who alone remained responsible to the plaintiffs, and that Wheatley, Sr. did not render legal services directly to the plaintiffs, the Winterrowd plaintiffs [represented by both lawyers] may recover attorney's fees for his work in the case prosecuted before the district court in the Central District.").

• Winston & Strawn, LLP v. Salt Lake Tribune Publ'g Co., Case No. 2:04-CV- 01182 PGC, 2006 U.S. Dist. LEXIS 24246, at *9-10, *12 (D. Utah Apr. 24, 2006) (denying a summary judgment motion by a former Winston & Strawn client, which had argued that Winston & Strawn could not recover fees for work in Utah because no Winston & Strawn lawyer was admitted to practice in Utah; "[I]t is undisputed that Winston & Strawn was at all times affiliated or working in conjunction with local Utah counsel that had been admitted to the Utah bar and the Federal District of Utah. This fact distinguished this case from all of the cases cited by Tribune Publishing; in these other cases, local counsel was not involved in planning or communication with the out-of-state attorney, nor did local counsel represent or file any of the documents written, researched, or discussed by the out-of-state attorney with the court."; noting that the client had paid Winston & Strawn over $200,000 without ever having complained about the issue; "Winston & Strawn has presented evidence supporting its theory that principles of equitable estoppel and waiver preclude Tribune Publishing from raising the pro hac vice question. Indeed, if anyone is entitled to summary judgment on this issue, it may well be Winston & Strawn.").

• Plummer v. Gittleman, Paskel, Tashman & Walker, P.C., 821 N.E.2d 825, 827-28, 828-29 (Ind. Ct. App. 2004) ("The issue of whether an out-of-state attorney, assisted by local counsel, may recover attorney's fees for work conducted in preparation of future litigation is a matter of first impression in Indiana courts."; "Through a common acquaintance, the Plummers contacted Gittleman to investigate any potential claims they might have had against Norfolk Southern. Gittleman associated himself with local counsel, Colvin, in anticipation of filing a lawsuit. Gittleman and Colvin executed a written

30

35951124_10 UPL, MDP and MJP (Defining What Lawyers Do and McGuireWoods LLP Where They Can Do It): Part II T. Spahn (1/1/19) Hypotheticals and Analyses

retainer agreement with the Plummers. Finally, if litigation had been commenced in this case, it appears Gittleman could have been admitted to practice law in Indiana pro hac vice. Therefore, the trial court did not err in awarding attorney's fees to Gittleman even though he was not licensed to practice law in the State of Indiana.").

The increasing acceptability of lawyers practicing in other states (under the liberal ABA Model Rule 5.5 approach) undoubtedly will result in this trend continuing.

Best Answer

The best answer to this hypothetical is YES (PROBABLY).

B 2/13, 3/17

31

35951124_10 UPL, MDP and MJP (Defining What Lawyers Do and McGuireWoods LLP Where They Can Do It): Part II T. Spahn (1/1/19) Hypotheticals and Analyses

Providing Advice about Another State's Law

Hypothetical 5

One of your clients conducts operations in ten Midwestern states. Your client's general counsel just asked you to analyze the franchise laws in all ten of the states in which it has operations. You quickly check your firm's roster, and realize that you have lawyers licensed in some of those states, but not all of them.

(a) May you provide advice about the law of a state in which one of your lawyers is licensed -- although that lawyer knows nothing about franchise law, and will not be involved in analyzing that state's law?

YES

(b) May you provide advice about the law of a state in which none of your lawyers is licensed?

YES (PROBABLY)

Analysis

Introduction

This hypothetical deals with a common situation facing transactional lawyers, who often find themselves asked to give opinions about other states' laws.

Of course, both transactional lawyers and litigators dealing with issues involving other states might be called upon to provide informal advice about other states' laws. In fact, a litigator who has never left her home state and is litigating a case in her home state might have to deal with another state's laws -- if her state's choice of laws principles require that the court handling the litigation apply another state's substantive law.

32

35951124_10 UPL, MDP and MJP (Defining What Lawyers Do and McGuireWoods LLP Where They Can Do It): Part II T. Spahn (1/1/19) Hypotheticals and Analyses

ABA Model Rules

The ABA Model Rules provide somewhat mixed signals about this general issue.

ABA Model Rule 5.5(b)(1) prohibits an out-of-state lawyer (other than an in- house lawyer) from establishing

an office or other systematic and continuous presence in this jurisdiction for the practice of law.

ABA Model Rule 5.5(b)(1). Comment [4] indicates that such a presence "may be

systemic and continuous even if the lawyer is not physically present here" (emphasis

added).

Although the ABA Model Rules do not explain this, presumably the comment

refers to communications in and out of the state -- an issue which has become much

more acute in recent years, as lawyers have been able to establish a "virtual" presence

in other states through electronic communications. Thus, the ABA Model Rules

recognize that a lawyer may impermissibly engage in the "systematic and continuous"

unauthorized practice of law in another state without ever traveling there.

On the other hand, another comment to the ABA Model Rules provides an

example of permissible activity that undoubtedly involves the lawyer providing advice

about various states in which the lawyer is not licensed. ABA Model Rule 5.5 cmt. [14]

indicates that a lawyer may assist a corporate client

when the client's activities or the legal issues involve multiple jurisdictions, such as when the officers of a multinational corporation survey potential business sites and seek the services of their lawyer in assessing the relative merits of each.

ABA Model Rule 5.5 cmt. [14] (emphasis added).

33

35951124_10 UPL, MDP and MJP (Defining What Lawyers Do and McGuireWoods LLP Where They Can Do It): Part II T. Spahn (1/1/19) Hypotheticals and Analyses

In its report to the ABA House of Delegates describing its proposed (and eventually adopted in February 2013) changes to Rule 5.5 (primarily dealing with foreign lawyers), the ABA Ethics 20/20 Commission provided its view on the permissibility of a lawyer providing legal advice about another jurisdiction's law.

Some commenters have suggested that the proposed constraints on foreign in-house counsel are too restrictive (e.g., it is not necessary to require such counsel to consult with U.S. counsel when advising on issues of U.S. law). They argue that these foreign lawyers could offer advice on U.S. law to their organizational clients from their home jurisdictions, so they should be able to offer the same advice to the same clients while on U.S. soil. The Commission rejected this argument because U.S. lawyers are subject to similar constraints on where they are permitted to offer their advice. For example, a New Hampshire lawyer can offer advice about Missouri law while in New Hampshire, but the New Hampshire lawyer is not permitted to relocate to Missouri and offer advice on Missouri law without becoming licensed to practice. Also, as referenced above, this limitation is consistent with the limitation already contained in the Model Foreign Legal Consultant Rule.

Am. Bar Ass'n Comm'n on Ethics 20/20 (June 19, 2012) (emphasis added). Although this comment appeared in a report dealing with foreign lawyers, it certainly is consistent with the broad ABA view of permissible multijurisdictional activities.

Restatement

The Restatement does not contain ABA Model Rule 5.5's explicit dichotomy between an impermissible "virtual systematic and continuous presence" and a permissible "temporary" presence.

In fact, the Restatement specifically indicates that lawyers may communicate electronically into other states where they are not licensed.

34

35951124_10 UPL, MDP and MJP (Defining What Lawyers Do and McGuireWoods LLP Where They Can Do It): Part II T. Spahn (1/1/19) Hypotheticals and Analyses

It is also clearly permissible for a lawyer from a home-state office to direct communications to persons and organizations in other states (in which the lawyer is not separately admitted), by letter, telephone, telecopier, or other forms of electronic communication.

Restatement (Third) of Law Governing Lawyers § 3 cmt. e (2000).

The Restatement also explicitly approves lawyers' providing opinions about the laws of states in which they are not licensed.

Some activities are clearly permissible. Thus, a lawyer conducting activities in the lawyer's home state may advise a client about the law of another state, a proceeding in another state, or a transaction there, including conducting research in the law of the other state, advising the client about the application of that law, and drafting legal documents intended to have legal effect there. There is no per se bar against such a lawyer giving a formal opinion based in whole or in part on the law of another jurisdiction, but a lawyer should do so only if the lawyer has adequate familiarity with the relevant law.

Id. (emphases added). The Restatement even explains why such activity should be permissible.

Modern communications, including ready electronic connection to much of the law of every state, makes concern about a competent analysis of a distant state's law unfounded. Accordingly, there is much to be said for a rule permitting a lawyer to practice in any state, except for litigation matters of law or for the purpose of establishing a permanent in-state branch office. Results approaching that rule may arguably be required under the federal interstate commerce clause and the privileges and immunities clause. The approach of the Section is more guarded. However, its primary focus is appropriately on the needs of clients.

Id. (emphasis added).

In what almost surely is not a coincidence, the Restatement also provides exactly the same example as the ABA Model Rules of a lawyer's permissible activity that

35

35951124_10 UPL, MDP and MJP (Defining What Lawyers Do and McGuireWoods LLP Where They Can Do It): Part II T. Spahn (1/1/19) Hypotheticals and Analyses

undoubtedly involves the lawyer providing advice about states other than those in which the lawyer is licensed. Restatement § 3 cmt. e mentions

a multinational corporation wishing to select a location in the United States to build a new facility [which] may engage a lawyer to accompany officers of the corporation to survey possible sites in several states, perhaps holding discussions with local governmental officers about such topics as zoning, taxation, environmental requirements, and the like.

Id. (emphases added).

ACTEC

The ACTEC Commentaries similarly explain that in some circumstances lawyers should be able to prepare documents governed by the law of a state where the lawyer is not licensed.

Comment 6 suggests a liberal interpretation of "temporary basis." This is particularly important for estate lawyers practicing in close proximity to another state. For example, a Chicago lawyer providing estate counseling for Illinois clients is likely to find multiple occasions to analyze and opine on the laws of , Iowa, Indiana, and Michigan regarding titling, tax, and similar issues. In addition, the Chicago lawyer may need to prepare deeds and other documents according to the laws of one or more of these jurisdictions. Provided the Chicago lawyer otherwise complies with paragraph (c), the lawyer's legal services regarding the surrounding non-admitted jurisdictions would constitute practicing law in those jurisdictions on a "temporary basis."

American College of Trust & Estate Counsel, Commentaries on the Model Rules of

Professional Conduct, Commentary on MRPC 5.5, at 203 (5th ed. 2016) (emphasis added).

36

35951124_10 UPL, MDP and MJP (Defining What Lawyers Do and McGuireWoods LLP Where They Can Do It): Part II T. Spahn (1/1/19) Hypotheticals and Analyses

States' Approach

Despite the ABA Model Rules' liberal approach (and the Restatement's even more liberal approach), states take differing positions on this issue.

The 1998 California Supreme Court case arguably responsible for triggering the national multijurisdictional practice debate recognized that a lawyer providing California law advice while physically present in another state might violate the California UPL statute.

Physical presence here is one factor we may consider in deciding whether the unlicensed lawyer has violated section 6125, but it is by no means exclusive. For example, one may practice law in the state in violation of section 6125 although not physically present here by advising a California client on California law in connection with a California legal dispute by telephone, fax, computer, or other modern technological means.

Birbrower, Montalbano, Condon & Frank, P.C. v. Superior Court, 949 P.2d 1, 5-6 (Cal.

1998) (emphasis added).

In contrast, some legal ethics opinions explicitly indicate that a lawyer providing advice about another state's law -- but not physically present in that other state -- has not engaged in improper multijurisdictional practice.

• Pennsylvania LEO 2001-62 (9/11/01) (indicating that a Pennsylvania lawyer could "undertake brief and limited engagements from a number of parties around the country," all of which have similar relationships with a single franchisor; explaining that "[t]hose parties would be advised by the franchisor that you are an available, knowledgeable attorney, who could promptly review and comment on proposed lease agreements which they are considering in connection with their franchise operation. You have explained by reason of your prior experience, you are familiar with and knowledgeable in the substantive area. Presumably, these clients have common or typical issues with respect to their space needs, in order to meet the needs of their franchise business, and your familiarity with those needs, in combination with your prior experience in representing business tenants in like circumstances,

37

35951124_10 UPL, MDP and MJP (Defining What Lawyers Do and McGuireWoods LLP Where They Can Do It): Part II T. Spahn (1/1/19) Hypotheticals and Analyses

makes you a particularly adept and suitable lawyer for such engagements."; noting that "[t]he provision of your legal services, i.e., your work, will occur within Pennsylvania, although the recipients of your services will, in many instances, be in other places."; noting that the lawyer "will not have an ongoing business relationship with these clients; this will be a limited, single and specialize [sic] engagement"; also noting that "[y]ou clearly will not maintain any office in another jurisdiction; you will not regularly advise clients in another jurisdiction; your work incident in any particular location will be likely a single engagement. Your communications with the clients will likely occur by letter, telecopy or other form of electronic communication. In this setting, and with these qualifications, it is my view that you can fairly assert that your work in review of a lease and commenting on it should not offend ULP regulations of another jurisdiction."; citing the Restatement of the Law Governing Lawyers, Section 3 cmt. [e]; "[T]here is no per se bar against such a lawyer giving a formal opinion based in whole or in part on the law of another jurisdiction. It is also clearly permissible for a lawyer from a home state office to direct communications to persons and organizations in other states (in which the lawyer is not separately admitted) by letter, telephone, telecopy, or other forms of electronic communication."; ultimately concluding that "I do believe you can proceed" with the representations).

This opinion took a far more liberal approach than an earlier Pennsylvania opinion.

• Pennsylvania LEO 90-02 (3/2/90) (explaining that a lawyer can provide an opinion about another state's law even if the lawyer is not admitted in that state, but should explain the lawyer's lack of expertise in that state's law; "There would appear to be no difficulty in the preparation of loan documentation for your lender clients at your offices to the extent such instruments are by their terms governed by the law of a jurisdiction in which you are admitted to practice. If that is not the case, the usual convention consistent with Rule 5.5 is to specify that any legal opinion rendered in respect of these instruments covers only the laws of the jurisdiction in which the lawyer is admitted to practice." (emphasis added); "This type of an opinion also would appear to be consistent with standard practice, and would not alone constitute an unauthorized practice of law. Care must be taken, however, to conform the opinion to the confines of the language appearing in the certificate issued." (emphasis added); "It is unlikely that the activity you describe in subpart (d) of your letter would alone constitute unauthorized practice, even if certain of the documents prepared and delivered to the borrower's counsel would by their own terms be governed by Pennsylvania law. However, if this activity were performed in combination with frequent visits to Pennsylvania in your capacity as counsel for the lender, the likelihood of a violation here (and in your own jurisdiction) would increase." (emphasis added); "Enforceability opinions, and exceptions thereto, have many permutations, and the form of such opinions as delivered by counsel

38

35951124_10 UPL, MDP and MJP (Defining What Lawyers Do and McGuireWoods LLP Where They Can Do It): Part II T. Spahn (1/1/19) Hypotheticals and Analyses

on the east coast is quite different from the usual practice on the west coast. It is fair to say, however, that the practice that has developed for competent counsel in many jurisdictions to opine regularly concerning the corporation law of Delaware has not yet grown to encompass Pennsylvania law."; "It also is fairly common practice for competent corporate counsel to opine on blue Sky law [sic] issues in foreign jurisdictions and on certain issues concerning perfection of security interests upon review of standard compilations, such as the CCH services, again informing the client that the scope of the inquiry has not been exhaustive. There is, however, no identifiable standard which in all instances will provide guidance as to whether it is necessary, or simply wise, to secure the opinion of local counsel in such matters. Moreover, the form of the opinion that the client may find satisfactory may determine the need for local counsel. By way of illustration, an opinion clearly denoting a lack of expertise may state the following: We are qualified to practice law in the State of New York and we do not purport to be experts on, or to express any opinion herein concerning, any law other than the law of the State of New York and the federal ." (emphases added)).

Thus, a lawyer's physical presence in a state dramatically increases the risk that

the lawyer will be engaging in the improper practice of law in that state. However,

avoiding such a physical presence does not automatically eliminate the risk. The trend

is in favor of allowing lawyers to provide advice about the law of states in which the

lawyers are not licensed.

A 2016 article raised the specter of trouble for lawyers providing advice about another country's laws.

• Randy Evans and Shari Klevens, Advice For Advising Clients Across Borders, Recorder (Online), Oct. 25, 2016 ("As modern legal practices expand nationally and globally, attorneys serve clients in regions all over the country and around the world. This expansion offers great opportunities for clients who have legal needs beyond simply their home state, as well as for law practices who can increase their base of work and enhance their expertise."; "No doubt this expansion is aided by the rapid development of technology, allowing attorneys to counsel clients in the most cost-effective and efficient manners without having to be in the same room. However, as with many developments of this age, these technological advances come with additional, albeit manageable, risk."; "Until recently, these risks seemed more hypothetical than real. Law practices are discovering that a national or international practice can create some hazards such as the potential

39

35951124_10 UPL, MDP and MJP (Defining What Lawyers Do and McGuireWoods LLP Where They Can Do It): Part II T. Spahn (1/1/19) Hypotheticals and Analyses

unauthorized practice of law, inconsistent ethics rules, and legal malpractice exposure under the laws of multiple states. For example, a law practice could be subject to the jurisdiction of a state in which it does not maintain an office, even where its contacts with the client have nothing to do with that state's laws or regulations. As a result, the law practice and individual attorneys may be forced to defend themselves in a foreign location, subject to the rules and laws regarding professionalism and malpractice that are completely different from those where the law practice operates."; "Importantly, the risk of uncertainty regarding jurisdiction extends internationally. The province of Ontario, Canada, for example, generally considers an attorney to be practicing law if the attorney merely gives legal advice on the laws of Ontario or the laws of Canada applicable to Ontario. Thus, an attorney might be found to have practiced law in Ontario even without physically entering the province. This could include giving legal advice regarding Ontario's laws by telephone, email, or other written correspondence that crosses provincial or international borders.").

(a) A lawyer whose colleague is licensed in the other state might arrange for

the colleague to participate in the transaction -- which presumably would satisfy any

state's requirement that only lawyers licensed in a state can provide opinions about that

state's law.

Of course the lawyer will also have to deal with the risk-management issues raised by lawyers providing advice on unfamiliar topics.

(b) Although risk management issues might deter lawyers from providing such

advice, the ABA Model Rules, the Restatement and the trend in most states'

approaches would probably permit such activity.

Best Answer

The best answer to (a) is YES; the best answer to (b) is YES (PROBABLY).

B 2/13

40

35951124_10 UPL, MDP and MJP (Defining What Lawyers Do and McGuireWoods LLP Where They Can Do It): Part II T. Spahn (1/1/19) Hypotheticals and Analyses

Establishing a "Virtual" Presence in Another State

Hypothetical 6

You have practiced for a number of years in Pennsylvania (where you are licensed), and primarily handle trust and estate matters. With the recent economic downturn, you have tried to expand your client base. Thanks to your good reputation and "word of mouth" from satisfied clients, you have begun to attract a number of clients who live in Delaware -- where you are not licensed. You communicate electronically and by telephone with these clients, and carefully avoid traveling into Delaware to meet with any of the clients.

May you continue to represent Delaware trust and estate clients as long as you avoid spending any time in Delaware providing those services?

MAYBE

Analysis

Multijurisdictional practice issues arise when lawyers practice law in a state

where they are not licensed. States' jealous hold over the practice of law within their

borders has led to a somewhat counter-intuitive result: lawyers fully licensed in another

jurisdiction are guilty of the unauthorized practice of law by practicing law in another

state just as if the lawyers had never spent a day in law school, passed a bar exam, or

met the rigorous standards for joining the professional. Although such lawyers might

face less severe sanctions than non-lawyers for practicing in a state where they are not

licensed, the conduct can still trigger even criminal penalties.

This harsh principle makes some sense when applied to a lawyer who moves to

another state and "hangs shingle" without taking some steps to join the new state's bar.

But the increasing ability of lawyers to practice "virtually" anywhere raises numerous

multijurisdictional practice issues -- with enormous stakes for the lawyers.

41

35951124_10 UPL, MDP and MJP (Defining What Lawyers Do and McGuireWoods LLP Where They Can Do It): Part II T. Spahn (1/1/19) Hypotheticals and Analyses

Lawyers' Temporary "Virtual" Practice Where They Are Not Licensed

Although ABA Model Rule 5.5 and state parallel ethics rules take a fairly generous approach to lawyers temporarily practicing in states where they are not licensed, there are lines -- which temporary "virtual" practice might cross.

States have severely punished (and even disbarred) lawyers who have virtually practiced improperly in other states. Some of those states have considered such virtual practice to have moved from "temporary" to "systematic and continuous."

• In re Velahos, Dkt. No. DRB 15-109, at 6 (N.J. Sup. Ct. Disciplinary Review Bd. May 23, 2016) (suspending for six months a lawyer for various ethics violations; noting the disciplinary review board's findings from 3/23/16; "In fact, respondent represented clients in multiple matters in jurisdictions in which he was not authorized to practice, without the assistance of local counsel. Respondent conducted no less than eighteen mortgage modifications in the States of Georgia, Washington, New York, Pennsylvania, Virginia, Maryland, Connecticut, Texas, or Florida. Respondent misrepresented to several of these out-of-state clients in the fee agreements that FLA 'has been retained as 'Of Counsel' to Loan Law Center.' Moreover, respondent engaged in credit and debt adjustment services in Maryland over a two-year period, even after the Commissioner of Financial Regulation for the State of Maryland issued a summary order, followed by a final order to Cease and Desist. When questioned by the OAE about the orders, respondent denied that he had 'taken any money' from Maryland. However, the OAE's review of respondent's records disclosed that, during that period, respondent actively represented several Maryland clients in that state and collected fees from them. Respondent's conduct in this respect violated RPC 1.16(a)(1), RPC 5.5(a), RPC 8.1(a), RPC 8.4(c), and RPC 8.4(d)." (emphasis added), aff'd, 137 A.3d 500 (N.J. May 25, 2016)).

• In re Lenard, Cal. Bar Court Review Dep't Case No. 09-O-11175 (Apr. 15, 2013) (disbarring a lawyer for engaging in the unauthorized practice of law by providing "credit repair" services to debtors in several states where the lawyer was not licensed; "Lenard contracted with three California consumer debt relief companies: Freedom Financial Management; Beacon Debt Service; and Pathway Financial Management (the Settlement Companies). These companies paid Lenard a flat fee to provide limited legal services for clients regarding their consumer debt. Lenard testified that he customarily charged the Settlement Companies between $75 to $100 per client and spent 15 to 20 minutes on each file. He also estimated that he had over

42

35951124_10 UPL, MDP and MJP (Defining What Lawyers Do and McGuireWoods LLP Where They Can Do It): Part II T. Spahn (1/1/19) Hypotheticals and Analyses

1,000 clients 'in credit repair' among all three companies. The Settlement Companies advertised through television and radio ads in a number of states. Clients who retained one of the Settlement Companies agreed to pay retainer fees of up to 12% of the balance of their debts, contingency fees of 8% of the amount by which their debts were reduced, and monthly maintenance fees of between $15 to $25. Clients also were required to make monthly payments into the Companies' 'client trust account,' and those funds were to be used to settle their debts. The Settlement Companies represented that the clients' accounts would be 'handled by our legal counsel.'"; "Lenard practiced law and held himself out as an attorney with the authority and knowledge to settle consumer debts to Wisconsin and New York clients Burgess and Manfredi, respectively. He also represented to their creditors that they should follow debt collection laws or his clients were prepared to take legal action. In addition, Lenard claims he their files to determine whether they should file bankruptcy, although he admitted he was 'not licensed to do a bankruptcy out of state.' Wisconsin and New York have both considered conduct similar to Lenard's to constitute UPL."; "The hearing judge found that Lenard established a systematic and continuous presence in each of the jurisdictions listed in the NDCs [Notice of Disciplinary Charge]. Based on the limited record, we do not find clear and convincing evidence of this proscription. However, we find that Lenard committed UPL by holding himself out as entitled to practice law in each of the severn states for a total of ten willful violations of rule 1-300(B)" (emphasis added) (footnote omitted); "By implying he was licensed in the relevant states, Lenard gave the false impression to his clients and their creditors that he held an advantage over a non-attorney debt negotiator. He explicitly represented to the clients that he would provide legal services, and informed creditors that he was representing each client utilizing his law office letterhead. The written communications Lenard provided to clients (and their creditors) in those states are evidence that he violated the applicable rules of professional conduct, as well as relevant case law and advisory authority."; "He [Lenard] contends that all work was done in California and any legal opinions rendered were based on California law. However, the factors defined in comment 14 of the ABA Model Rule compel our conclusion that Lenard was not entitled to practice law even on a temporary basis in these states. Analyzing those factors, we find that he had no prior contact with the clients and they never lived in California or had substantial contact with this state. There is no evidence that California law would be relevant to any of the consumer debts in these matters. Further, Lenard has no knowledge of the specific laws of the states in which the clients resided, where they faced state collection actions and may have had assets. As such, the contact with these out-of-state clients was not reasonably related to Lenard's practice in California, and he was not authorized to provide legal services on a temporary basis under the states' versions of ABA Model Rule 5.5(c)." (emphasis added) (footnote omitted); "[W]e reject any contention by Lenard

43

35951124_10 UPL, MDP and MJP (Defining What Lawyers Do and McGuireWoods LLP Where They Can Do It): Part II T. Spahn (1/1/19) Hypotheticals and Analyses

that ABA Model Rule 5.5(d)(2) enabled him to provide legal services related to bankruptcy law. Primarily, Lenard's proposed services were not limited to issues of bankruptcy.").

Such lawyers may face other threats.

• Angela Morris, Linebarger Goggan Law Firm Settles Class Action For $3.4 Million, Tex. Lawyer, Jan. 12, 2016 ("A federal judge has approved a settlement that requires Austin-based law firm Linebarger Goggan Blair & Sampson, LLC to pay $3.4 million -- including nearly $904,000 in attorney fees and expenses -- to settle a class action that alleged it engaged in the unauthorized practice of law in California."; "The settlement ends litigation spanning back to May 2013, when plaintiff 4EC Holdings sued Linebarger, a firm that contracts with governmental agencies to collect debts. 4EC alleged that Linebarger sent debt collection demand letters to California residents, even though the firm did not employ lawyers in California, as allegedly required under California law. Linebarger denied the allegations.")

Lawyers' Systematic And Continuous "Virtual" Practice Where They Are Not Licensed

The ABA Model Rules contain two flat prohibitions on lawyers' provisions of legal services in states where they are not licensed.

First, a lawyer not licensed in a jurisdiction

shall not . . . except as authorized by these Rules or other law, establish an office or other systematic and continuous presence in this jurisdiction for the practice of law.

ABA Model Rule 5.5(b)(1) (emphasis added).

Second, such a lawyer may not

hold out to the public or otherwise represent that the lawyer is admitted to practice law in this jurisdiction.

ABA Model Rule 5.5(b)(2).

A comment to the ABA Model Rules includes a twist -- which complicates the analysis.

Other than as authorized by law or this Rule, a lawyer who is not admitted to practice generally in this jurisdiction violates 44

35951124_10 UPL, MDP and MJP (Defining What Lawyers Do and McGuireWoods LLP Where They Can Do It): Part II T. Spahn (1/1/19) Hypotheticals and Analyses

paragraph (b)(1) if the lawyer establishes an office or other systematic and continuous presence in this jurisdiction for the practice of law. Presence may be systematic and continuous even if the lawyer is not physically present here.

ABA Model Rule 5.5 cmt. [4] (emphasis added).

The Restatement also deals with the rise in electronic communications, and the resulting ability of lawyers to engage in a "virtual" practice. In fact the Restatement points to this trend as a grounds for allowing lawyers licensed in one state to continuously practice in other states.

The Restatement essentially follows the ABA Model Rules standard.

The extent to which a lawyer may practice beyond the borders of the lawyer's home state depends on the circumstances in which the lawyer acts in both the lawyer's home state and the other state. At one extreme, it is clear that a lawyer's admission to practice in one jurisdiction does not authorize the lawyer to practice generally in another jurisdiction as if the lawyer were also fully admitted there. Thus, a lawyer admitted in State A may not open an office in State B for the general practice of law there or otherwise engage in the continuous, regular, or repeated representation of clients within the other state.

Restatement (Third) of Law Governing Lawyers § 3 cmt. e (2000) (emphasis added).

However, the Restatement clearly takes a more liberal view than the ABA Model

Rules of the type of "virtual" presence in a state that lawyers should be able to arrange.

It is also clearly permissible for a lawyer from a home-state office to direct communications to persons and organizations in other states (in which the lawyer is not separately admitted), by letter, telephone, telecopier, or other forms of electronic communication.

Restatement (Third) of Law Governing Lawyers § 3 cmt. e (2000) (emphasis added).

45

35951124_10 UPL, MDP and MJP (Defining What Lawyers Do and McGuireWoods LLP Where They Can Do It): Part II T. Spahn (1/1/19) Hypotheticals and Analyses

The ABA Ethics 20/20 Commission primarily focused on the rising use of electronic communications in the practice of law, and the increasing mobility of lawyers.

Thus, one would think that the issue of a "virtual" continuous presence in another state would have been an obvious choice for proposed rules changes.

The Commission tiptoed into the issue. In its June 19, 2012, Issue Paper, the

ABA Ethics 20/20 Commission described the earlier circulation of a draft proposed rules change.

The Commission previously circulated a draft proposal that would have addressed this ambiguity in a general way by adding new sentences to Comment [4] to Rule 5.5. The new sentences would have provided as follows:

For example, a lawyer may direct electronic or other forms of communications to potential clients in this jurisdiction and consequently establish a substantial practice representing clients in this jurisdiction, but without a physical presence here. At some point, such a virtual presence in this jurisdiction may be come [sic] systematic and continuous within the meaning of Rule 5.5(b)(1).

In response to this proposal, several commenters suggested that the sentences not only provide little additional guidance, but that they might have the unintended effect of deterring lawyers from engaging in forms of virtual practice that should be permissible.

Based on this response, the Commission asked its Uniformity, Choice of Law, and Conflicts of Interest Working Group to evaluate whether it is possible to provide enhanced guidance on this issue, and if so, how. The Working Group has identified several possible approaches.

Am. Bar Ass'n Comm'n on Ethics 20/20, Issue Paper (June 19, 2012).

The Commission tentatively floated the following "trial balloon" as a way to assess such a "virtual" presence. 46

35951124_10 UPL, MDP and MJP (Defining What Lawyers Do and McGuireWoods LLP Where They Can Do It): Part II T. Spahn (1/1/19) Hypotheticals and Analyses

One possible approach is to identify the factors that lawyers and disciplinary authorities should consider when deciding whether a lawyer's presence has become sufficiently systematic and continuous to trigger Rule 5.5(b)'s requirement that the lawyer become licensed. For example, those factors might include:

• the nature and volume of communications directed to potential clients in the jurisdiction;

• whether the purpose of the communications is to obtain new clients in the jurisdiction;

• the number of the lawyer's clients in the jurisdiction;

• the proportion of the lawyer's clients in the jurisdiction;

• the frequency of representing clients in the jurisdiction;

• the extent to which the legal services have their predominant effect in the jurisdiction; and

• the extent to which the representation of clients in the jurisdiction arises out of, or is reasonably related to, the lawyer's practice in a jurisdiction in which the lawyer is admitted to practice.

A second possibility is for the Commission to make no proposal in this area and to refer the issue to the Standing Committee on Ethics and Professional Responsibility for an opinion on the meaning of "systematic and continuous presence" in the context of virtual law practice.

A third possibility is for the Commission to make no proposal in this area, but identify the relevant issues in an informational report that the Commission could file with the ABA House of Delegates to help educate the profession about this issue.

Id. at 2-3.

The ABA 20/20 Commission eventually chose option No. 2 -- essentially punting the issue to the ABA Standing Committee on Ethics and Professional Responsibility.

The Commission described its decision in a February 2013 release. 47

35951124_10 UPL, MDP and MJP (Defining What Lawyers Do and McGuireWoods LLP Where They Can Do It): Part II T. Spahn (1/1/19) Hypotheticals and Analyses

Currently, Model Rule 5.5(b)(1) requires a lawyer to obtain a license in a jurisdiction if the lawyer has an office or a "systematic and continuous" presence there, unless the lawyer's work falls within one of the exceptions identified in Rule 5.5(d). The increased demand for cross-border practice and related changes in technology have raised new questions about the meaning of the phrase "systematic and continuous presence" in Rule 5.5(b). In particular, technology now enables lawyers to be physically present in one jurisdiction, yet have a substantial virtual practice in another. The problem is that it is not always clear when this virtual practice in a jurisdiction is sufficiently "systematic and continuous" to require a license in that jurisdiction.

Currently, Comment [4] to Model Rule 5.5 identifies these issues, but provides limited guidance as to how to resolve them. The Comment states that a lawyer's "[p]resence may be systematic and continuous even if the lawyer is not physically present" in the jurisdiction. Neither the Rule nor the Comment provides any clarity as to when a lawyer who is "not physically present" in a jurisdiction nevertheless has a systematic and continuous presence there.

The Commission released an issues paper, seeking feedback on a number of possible options for addressing these issues, including the identification of relevant factors when analyzing when a presence becomes "systematic and continuous" and referring the issue to the Standing Committee on Ethics and Professional Responsibility for a Formal Opinion on the meaning of "systematic and continuous presence" in the context of virtual law practice.

The Commission, after considerable deliberations, concluded that these issues may be best addressed in the future as the nature of virtual law practice becomes clearer and as relevant technology continues to evolve.

Am. Bar Ass'n Comm'n on Ethics 20/20, Introduction and Overview (Feb. 2013)

(emphasis added).

It might be fair to conclude that such "virtual" practice possibilities represent a huge threat to states' somewhat parochial and often "turf protecting" view of their power

48

35951124_10 UPL, MDP and MJP (Defining What Lawyers Do and McGuireWoods LLP Where They Can Do It): Part II T. Spahn (1/1/19) Hypotheticals and Analyses

to regulate the legal profession. This may be one reason that the ABA Ethics 20/20

Commission abandoned its efforts.

Just as the ABA has recognized but not dealt with this issue, courts and bars have wrestled with it too.

The 1988 California decision that arguably triggered the national multijurisdictional practice debate definitely moved away from an exclusive reliance on a lawyer's physical presence in a state, but without offering any concrete guidance about where to draw the line between permissible and impermissible conduct.

• Birbrower, Montalbano, Condon & Frank, P.C. v. Superior Court, 949 P.2d 1, 5-6 (Cal. 1998) ("Our definition does not necessarily depend on or require the unlicensed lawyer's physical presence in the state. Physical presence here is one factor we may consider in deciding whether the unlicensed lawyer has violated section 6125, but it is by no means exclusive. For example, one may practice law in the state in violation of section 6125 although not physically present here by advising a California client on California law in connection with a California legal dispute by telephone, fax, computer, or other modern technological means. Conversely, although we decline to provide a comprehensive list of what activities constitute sufficient contact with the state, we do reject the notion that a person automatically practices law 'in California' whenever that person practices California law anywhere, or 'virtually' enters the state by telephone, fax, e-mail, or satellite." (emphases added)).

The August 2002 ABA Commission on Multijurisdictional Practice's report recognized

the impact of the Birbrower decision.1

1 Am. Bar Ass’n Ctr. for Prof’l Responsibility, Report on Multijurisdictional Practice, at *3-4 (Aug. 2002) ("This concern was sharpened by the California Supreme Court decision, Birbrower, Montalbano, Condon & Frank, P.C. v. Superior Court of Santa Clara County, 949 P.2d 1 (Cal. 1998), which held that lawyers not licensed to practice law in California violated California's misdemeanor UPL provision when they assisted a California corporate client in connection with an impending California arbitration under California law, and were therefore barred from recovering fees under a written fee agreement for services the lawyers rendered while they were physically or 'virtually' in California. Although the state law was subsequently and temporarily amended to allow out-of-state lawyers to obtain permission to participate in certain California arbitrations, concerns have persisted."). 49

35951124_10 UPL, MDP and MJP (Defining What Lawyers Do and McGuireWoods LLP Where They Can Do It): Part II T. Spahn (1/1/19) Hypotheticals and Analyses

As "virtual" practice became easier and more tempting for lawyers, bars initially continued to focus on lawyers' continued presence in a jurisdiction when determining whether they met the multijurisdictional practice standards.

• Illinois LEO 02-04 (4/2003) (addressing the following situation: "An attorney licensed in State X who negotiates, from his office in State X, his clients' claim for medical matters in State Y, where no lawsuit has been filed and where the attorney is not licensed, does not engage in the unauthorized practice of law, and need not associate with an attorney in State Y to conduct this negotiation."; holding as follows: "[T]he Committee assumes that no lawsuit has been filed in State Y, and that the only services the attorney would provide would be to negotiate, from the attorney's office in State X, the couple's claim for medical matters in State Y. The Committee also assumes that the attorney is not habitually engaged in such negotiations, and that the attorney is merely doing so in this instance to assist a couple known to him, as set forth in the facts. Under this scenario, it is the Committee's opinion that while the attorney may be engaging in the practice of law, it is not the 'unauthorized' practice of law in State Y because the attorney is conducting the negotiation from State X, where he is licensed to practice law. Accordingly, under the facts presented, the Committee believes the attorney may settle the couple's claim for medical matters in State Y without associating with an attorney in State Y, and that doing so does not violate Rule 5.5(a) of the Illinois Rules of Professional Conduct. To the extent the attorney leaves State X, where he is licensed, and enters State Y to conduct the negotiation, the issue becomes less clear. See Lozoff v. Shore Heights, Ltd., 66 Ill. 2d 398, 362 N.E. 2d 1047 (1977) (holding that a lawyer licensed only in Wisconsin who had rendered legal services in connection with an Illinois real estate transaction had engaged in the unauthorized practice of law and could not recover fees)." (emphases added)).

• Philadelphia LEO 2007-4 (8/2007) (addressing a request by a Pennsylvania lawyer about how to respond to a request by someone outside Pennsylvania that the lawyer provide legal services; directing the lawyer to research the UPL laws of the other jurisdiction; explaining that the other jurisdiction's UPL rules would also govern the permissibility of the lawyer providing "advice and consultation on matters of federal law, general legal principles and common law"; explaining that the other pertinent state might find that actions taken in Pennsylvania violated the UPL laws of that other state; "The inquirer also specifically asks if it makes a difference whether he performs legal services in Pennsylvania or in the subject jurisdiction. The answer to this question again depends on the law of that jurisdiction. See, e.g., Comment 4 to ABA Model Rule 5.5, which states that a lawyer may violate Model Rule 5.5(b) (which proscribes a lawyer who is not admitted in a jurisdiction from

50

35951124_10 UPL, MDP and MJP (Defining What Lawyers Do and McGuireWoods LLP Where They Can Do It): Part II T. Spahn (1/1/19) Hypotheticals and Analyses

systematic and continuous presence in that jurisdiction) even if the lawyer is not physically present in the jurisdiction. Most recently the Delaware Supreme Court took action against an attorney not even admitted in Delaware see In re Tonwe, Del., No. 584, 2006, 5/25/07." (emphasis added)).

• Virginia UPL Op. 215 (3/18/08) (holding that in-house lawyers who work for a financial institution and are based outside of Virginia and not licensed in Virginia may provide advice to Virginia employees; "When these lawyers provide advice and counsel regarding Virginia law to employees of the financial institution employer located in branches in Virginia, they are not engaged in unauthorized practice. When they are providing the advice either from their offices outside of Virginia or when they visit the branches in-person in Virginia, this constitutes advising their regular employer which is permitted under Part 6, §I (B)(1) of the Rules of the Virginia Supreme Court. Should they have to prepare documents in either situation, again, these lawyers are providing this legal service only to their regular employer which is permitted under Part 6, §I (B)(2). These lawyers also fall within the scope of the temporary practice provisions of Part 6, §I (C). They represent the employer elsewhere and have occasion to have to come into Virginia in relation to that representation. This occurs only on a temporary or occasional basis. Nothing in this inquiry suggested that these lawyers were attempting to appear before any tribunal in Virginia on behalf of the employer, which would require association with Virginia-licensed counsel. Finally, two earlier opinions from the Committee, UPL Opinions 93 and 99[,] are also instructive on the issues presented in this inquiry. In these opinions the Committee found that it was not the unauthorized practice of law for a non-Virginia- licensed attorney to prepare legal documents for a Virginia client relating to a Virginia matter when the attorney did so from his/her office in the jurisdiction where he/she is licensed. Similarly, if any attorney is providing legal advice to or on behalf of a Virginia client while located in his/her licensing jurisdiction, this will not be the unauthorized practice of law in Virginia. The Committee cautions that an attorney licensed other than in Virginia must also be aware of any applicable rules and/or limitations of his/her licensing jurisdiction and/or the jurisdiction where he/she is practicing regarding the practice of another jurisdiction's law where the attorney is not licensed." (emphases added)).

Eventually, states began to de-emphasize lawyers' physical presence and acknowledge that lawyers can practice "virtually" and permanently in a state where they are not licensed -- a scenario the ABA acknowledged about the same time but never resolved.

51

35951124_10 UPL, MDP and MJP (Defining What Lawyers Do and McGuireWoods LLP Where They Can Do It): Part II T. Spahn (1/1/19) Hypotheticals and Analyses

The continuing uncertainty of these issues has generated warnings to lawyers to be careful when they "virtually" cross state lines.

• Samuel C. Stretton, Pennsylvania Lawyers Should Be Cautious About Offering Legal Advice in Delaware, Legal Intelligencer (Online), Sept. 9, 2014) ("Is it permissible for a Pennsylvania lawyer to represent people in Delaware if the lawyer has local counsel or an office with a Delaware lawyer in it?"; "Any Pennsylvania lawyer who wants to provide legal advice in Delaware, even with local counsel or an office that has a Delaware lawyer as a paid employee, should be very cautious. Unless a Pennsylvania lawyer is moved in pro hac vice or sits for the bar exam and passes it, there is an excellent chance a lawyer practicing down there with some regularity, even though there are no court appearances, will receive professional discipline."; "Delaware's disciplinary system is extremely vigorous in tracking down and prosecuting Pennsylvania lawyers who are not licensed in Delaware, even though their activities seem to be acceptable, at least from a Pennsylvania viewpoint. There are several lawyers in Delaware who seem to make it an avocation of reporting Pennsylvania lawyers in a vigorous fashion to the Delaware disciplinary authorities."; "The problem started about six or seven years ago when an attorney was licensed in Pennsylvania with an office in southern Chester County and was handling a number of first-party benefits insurance cases in Delaware. She had local counsel, but was settling these cases. Many of these cases had little third-party value and, therefore, were not of interest to members of the Delaware bar. The attorney was a black woman, and many people felt comfortable going to her since there were not a large number of minority lawyers actively practicing in Delaware. The cases came to her through her husband's medical practice and the church that she and her husband attended in Delaware."; "The problem in Delaware is that the Delaware Supreme Court will suspend or disbar a lawyer even if he or she is not admitted in Delaware. This is what happened in these cases. This was challenged in Pennsylvania, but the Pennsylvania Supreme Court will grant reciprocal discipline, as will the federal courts in Pennsylvania. A Pennsylvania lawyer disciplined in Delaware will lose his or her license in Pennsylvania under reciprocal discipline. The fact that the lawyer was never licensed in Delaware is not a defense to the reciprocal discipline, at least in Pennsylvania."; "Of great interest is the fact that many Delaware lawyers provide advice in Pennsylvania and other states on a regular basis, particularly through transactional, tax and other types of corporate law. The Delaware Supreme Court seems somewhat shortsighted. If Pennsylvania courts would be just as vigorous, there would be a large number of major law firms and lawyers in Delaware who would be under suspension if the same rationale applied."; "In conclusion, particularly in Delaware, lawyers who are licensed in Pennsylvania but providing advice across state lines should be very careful. It has always been risky to do that, 52

35951124_10 UPL, MDP and MJP (Defining What Lawyers Do and McGuireWoods LLP Where They Can Do It): Part II T. Spahn (1/1/19) Hypotheticals and Analyses

though with modern availability of legal research for various state laws, it is not as dangerous as it used to be, because one can check relevant state law on issues. But any lawyer providing advice or writing wills or other matters for people in another state should have local counsel. If a lawyer does it with regularity, he or she should be careful about being cited for unauthorized practice of law. In states like Delaware, the unauthorized practice of law results in major suspensions, which will then be ordered in Pennsylvania under a reciprocal system."; "Unless the bar leaders in Delaware and Pennsylvania can reach some understanding or accommodation, any lawyer who wants to provide advice across the Delaware state line should do so very cautiously. The best practice would be to move for pro hac vice admission." (emphases added)).

All in all, lawyers should be very wary of attracting most of their clients from

states where they are not licensed, and representing those clients "virtually." Although

the ABA has certainly abandoned (at least in the short run) its goal of determining when

such activities becomes the illegal unauthorized practice of law in those other states, it

has recognized that there is a line somewhere.

In 2012, an Illinois legal ethics opinion provided a more precise analysis --

holding that an Illinois lawyer could not partner with an out-of-state lawyer for the

practice of law in Illinois, even if the out-of-state lawyer did not hold himself out as an

Illinois lawyer and always acted under the direct supervision of the Illinois lawyer.

Among other things, the Illinois Bar recognized that the out-of-state lawyer might violate

Illinois Rule 5.5 through a "virtual" presence in Illinois.

In the context of a virtual law office involving lawyers from different states, each lawyer should take care that any out-of-state practice is not systematic and continuous. The proposed practice involves a lawyer from State X who wishes to practice regularly in Illinois, whether through a physical presence or a virtual presence. 'Presence may be systematic and continuous even if the lawyer is not physically present here.' RPC 5.5, Comment [4]. So even if the virtual office were not based in Illinois, the fact that the State X lawyer would do work for Illinois clients and would

53

35951124_10 UPL, MDP and MJP (Defining What Lawyers Do and McGuireWoods LLP Where They Can Do It): Part II T. Spahn (1/1/19) Hypotheticals and Analyses

seek legal work in Illinois establishes a systematic and continuous presence. . . . Because the State X lawyer wishes to practice regularly in Illinois, the Committee is of the opinion that Rule 5.5(b) bars the proposed practice, regardless of whether the lawyer's presence in Illinois is physical or virtual. Additionally, because the Illinois lawyer would be part and parcel of the project, he or she would be subject to discipline under Rule 5.5(a) for assisting the State X lawyer.

Illinois LEO 12-09 (3/2012) (emphases added).2 The Illinois legal ethics opinion

explained that both the out-of-state lawyer and the Illinois lawyer assisting in the

arrangement put themselves in harm's way.

Interestingly, the ramifications of a lawyer's "virtual" presence have arisen in

several disciplinary cases. In some situations, lawyers establishing a largely "virtual"

practice have also occasionally entered the state to meet with clients -- giving state

disciplinary authorities a "hook" to punish the lawyers under the traditional emphasis on

physical location. Not surprisingly, these generally involve lawyers practicing near a

2 Illinois LEO 12-09 (3/2012) (explaining that a non-Illinois lawyer may not practice physically or "virtually" in Illinois even if the law firm's co-owner is licensed in Illinois and directly supervises the non-Illinois lawyer on matters involving Illinois clients; "Two attorneys wish to establish a law practice owned 50/50 between them. One is licensed only in Illinois, one is licensed only in State X."; "Both live and primarily work in Illinois. However, the attorney licensed in State X makes frequent visits to State X for networking and to cultivate a client base there. The attorneys agree that the Illinois-licensed attorney will have direct supervision and ultimate authority over matters involving Illinois clients, although the State X-licensed attorney will interact with Illinois clients and dispense legal advice to them from time to time."; "The Illinois-licensed attorney will sign all pleadings in Illinois courts, make all Illinois court appearances, and conduct any Illinois real estate closings personally. The State X-licensed attorney will engage in networking and market himself in Illinois as an attorney, but will take precautions to ensure that potential clients do not get the impression that he is licensed in Illinois. All letterheads and business cards will clearly and correctly indicate the jurisdictions in which each attorney is licensed to practice. Both attorneys agree to make sure, at the time any client is acquired, that the client understands that the State X-licensed attorney is not licensed in Illinois. Retainer agreements will contain bold-type disclosures to this effect."; "[T]he State X lawyer would work primarily in Illinois, which means that he would have a systematic and continuous presence (presumably including an office) in Illinois for the practice of law, in violation of paragraph (b)(1). The fact that the state of admission is accurately displayed does not vitiate that violation, as Rule 5.5(b)(1) prohibits the systematic and continuous presence, independent of the lawyer's representation as to his bar admission."). 54

35951124_10 UPL, MDP and MJP (Defining What Lawyers Do and McGuireWoods LLP Where They Can Do It): Part II T. Spahn (1/1/19) Hypotheticals and Analyses

state border, and drawing clients from a neighboring state where the lawyer is not licensed to practice law.

In 2007, the Delaware Supreme Court punished such a lawyer.

Glover says that she did not provide legal services 'in Delaware' because she worked out of an office in Pennsylvania. Moreover, because she reasonably believed that the predominant effect of her legal work was in Pennsylvania, she should be protected by the 'safe harbor' provision in Rule 8.5(b). Glover's argument fails for several reasons. First, the record establishes that on three occasions she was physically present in Delaware, representing her Delaware clients. Second, physical presence is not required to establish that a person is providing, or offering to provide, legal services in this state. For several years, Glover accepted new clients who were: (1) Delaware residents, (2) involved in Delaware car accidents, and (3) seeking recovery under Delaware insurance policies. Glover did everything short of appearing in Delaware courts, and engaged Delaware attorneys as co-counsel only if she could not resolve the matter without litigation. We are satisfied that this regular pattern of representation of Delaware clients constituted the practice of law 'in Delaware' for purposes of Rule 8.5. (footnote omitted).; Glover may not have engaged in formal advertising to attract clients, but she certainly cultivated a network of Delaware contacts who accomplished the same result. After carefully reviewing the record, we are satisfied that there is substantial evidence to support the Board's finding that Glover established a systematic and continuous presence in Delaware for the practice of law in violation of Rule 5.5(b).

In re Tonwe, 929 A.2d 774, 778, 779-80 (Del. 2007) (emphases added). To be sure, the Delaware court might have been influenced by the lawyer's unsavory practice history and questionable representations during the disciplinary process.3

3 In re Tonwe, 929 A.2d 774, 776 (Del. 2007) (disbarring a lawyer for the unauthorized practice of law in Delaware; explaining the Delaware Office of Disciplinary Counsel had filed a petition alleging that the Pennsylvania-licensed lawyer had practiced law in Delaware; explaining that "Glover graduated from law school in 1985 and was admitted to the Ohio bar shortly thereafter. She moved to Delaware a few 55

35951124_10 UPL, MDP and MJP (Defining What Lawyers Do and McGuireWoods LLP Where They Can Do It): Part II T. Spahn (1/1/19) Hypotheticals and Analyses

A year later, the Delaware Supreme Court applied the same basic principle.

• In re Kingsley, No. 138, 2008 Del. LEXIS 255, at *13 (Del. June 4, 2008) (holding that a lawyer licensed in Pennsylvania and New Jersey committed the unauthorized practice of law in Delaware by accepting a monthly retainer to draft estate planning documents for clients of a Delaware accountant; concluding that the lawyer established a "systematic and continuous presence" in Delaware by engaging in these activities; prohibiting the lawyer from practicing law in Delaware).

Lawyers' Systematic and Continuous "Virtual" Practice Where They Are Licensed -- While Physically In A State Where They Are Not Licensed

Most lawyers analyzing "virtual" practice focus on the scenario discussed above -

- remaining physically in a state where they are licensed but "virtually" representing

clients located in states where the lawyers are not licensed.

But lawyers might instead choose to live in a state where they are not licensed,

while continuously practicing -- "virtually" -- in a state where they are licensed. There are several scenarios in which such a arrangement might be attractive. For instance, lawyers might want to continue practicing "big city" law while living in more attractive or less expensive rural settings. They might want to be near aging parents, or follow a spouse who will be attending graduate school for several years, etc.

One might wonder why the state where such lawyers will be physically present

would care about any multijurisdictional implications. Presumably, that state has an

interest in protecting its own citizens from lawyers representing them without local

years later. In 1989, Glover was admitted to practice in Pennsylvania and the District of Columbia. She took the Delaware bar examination, but did not pass. In 1990, Glover opened a law office in her home in Milford, Delaware. Glover's practice included federal immigration law and personal injury cases."; "The ODC first learned about Glover's Delaware legal practice as a result of an ongoing federal investigation. In 1991, Glover was convicted of bribing a federal immigration official, and served 37 months in prison. Following her conviction, Glover was disbarred in Pennsylvania, Ohio and the District of Columbia. She was reinstated in Pennsylvania in 2002."; rejecting the lawyer's argument that she had not practiced law in Delaware; noting that the lawyer's husband and children live in Delaware, but she claims to sleep in her Pennsylvania office – but denying that she and her husband are separated). 56

35951124_10 UPL, MDP and MJP (Defining What Lawyers Do and McGuireWoods LLP Where They Can Do It): Part II T. Spahn (1/1/19) Hypotheticals and Analyses

knowledge, without any supervision from that state's bar, etc. So why would that state be concerned, as long as those lawyers do not hold themselves out to practice in the state, do not represent any citizens of that state, etc.?

In this scenario, the lawyers rather than the bar would have an interest in rejecting the old "physical presence" standard, and instead focus on the "virtual" practice factors.

A February 2013 release of the ABA Ethics 20/20 Commission noted this issue, but without reaching any conclusions.

Conversely, a lawyer may be licensed in one jurisdiction, but live in a jurisdiction where the lawyer is not licensed. If the lawyer conducts a virtual practice from the latter jurisdiction and serves clients only in the jurisdiction where the lawyer is actually licensed, there is a question of whether the lawyer has a "systematic and continuous" presence in the jurisdiction where the lawyer is living and thus violates Rule 5.5(b) in that jurisdiction. The Rule is unclear in this regard as well.

Am. Bar Ass'n Comm'n on Ethics 20/20, Introduction and Overview, at 10 n.27 (Feb.

2013).

But other states continue to focus on lawyers' physical presence – generally barring lawyers domiciled in the state from practicing law in that state without a license -

- even if they do not provide advice about their host state's law.

Such states consider such an arrangement to be the illegal systematic and continuous practice of law. That could result in the lawyers' discipline in that state, which in turn can trigger discipline in the lawyers' home state.

57

35951124_10 UPL, MDP and MJP (Defining What Lawyers Do and McGuireWoods LLP Where They Can Do It): Part II T. Spahn (1/1/19) Hypotheticals and Analyses

Predictably, Florida takes such a protective approach. A Florida Rules comment

prohibits lawyers from establishing "an office or regular presence" in Florida – even if

they limit their practice to the law of a state where they are licensed.

• Florida Rule 4-5.5 cmt ("[A] lawyer licensed to practice law in New York could not establish an office or regular presence in Florida to practice New York law. Such activity would constitute the unlicensed practice of law. However, for purposes of this rule, a lawyer licensed in another jurisdiction who is in Florida for vacation or for a limited period of time, may provide services to their clients in the jurisdiction where admitted as this does not constitute a regular presence. The lawyer must not hold out to the public or otherwise represent that the lawyer is admitted to practice law in Florida. Presence may be regular even if the lawyer is not physically present here." (emphasis added)).

Florida is among the most jealous of all states about out-of-state lawyers practicing there (especially on a systematic basis). The reference to New York lawyers trying to horn in on Florida lawyers' practice is a blatant shot across the bow of New York lawyers seeking a warmer climate.

Some states have adopted the same approach, but in a more subtle fashion.

For instance, Colorado prevents lawyers domiciled in that state from taking

advantage of the ethics provisions permitting non-Colorado lawyers to temporarily

practice in the state.

• Rules Governing Admission To The Practice Of Law In Colorado; Rule 205.1 (Revised 2014) ("(1) Eligibility. An attorney who meets the following conditions is an out-of-state attorney for the purpose of this rule: (a) The attorney is licensed to practice law and is on active status in another jurisdiction in the United States; (b) The attorney is a member in good standing of the bar of all courts and jurisdictions in which he or she is admitted to practice; (c) The attorney has not established domicile in Colorado; and (d) The attorney has not established a place for the regular practice of law in Colorado from which the attorney holds himself or herself out to the public as practicing Colorado law or solicits or accepts Colorado clients." (emphasis added)).

58

35951124_10 UPL, MDP and MJP (Defining What Lawyers Do and McGuireWoods LLP Where They Can Do It): Part II T. Spahn (1/1/19) Hypotheticals and Analyses

In 2015, some New York lawyers unsuccessfully sought to move New York in the

direction of allowing such practice.

• ABA/BNA Lawyers' Manual on Professional Conduct, Vol. 31, No. 26 (Dec. 30, 2015) ("While characterizing the new temporary practice rule as a 'huge advance' for New York, Davis [Anthony Davis of Hinshaw & Culbertson LLP] pointed out that the court of appeals did not adopt a recommendation made by a group of large law firms that have offices in New York and other jurisdictions."; "That recommendation took aim at the situation where a lawyer lives in one state but practices law remotely from an office in another state where the lawyer is admitted to practice. For example, a lawyer may live in New York and commute to New Jersey or Connecticut. In this situation, Davis said, it's a stretch to say the lawyer is temporarily in New York, and the lawyer arguably may be engaging in unauthorized practice there."; "In comments on the proposed temporary practice rule, the large firms recommended additional language that would allow a lawyer authorized to practice law in a U.S. jurisdiction to provide legal services that exclusively involve federal law, the law of another jurisdiction or tribal law, provided the lawyer does not hold himself out in any way as having an office for the practice of law in New York."; "This language, the firms said, would prevent UPL accusations against lawyers who live in New York and use technology to practice remotely as if they were physically in their state of admission, provided they do not practice New York law or hold themselves out as doing so.).

One court has taken what seems to be a ridiculously overbroad approach to this issue -- holding that even a lawyer's triage of matters that the lawyer can and cannot handle amounts to the unauthorized practice of law, if the lawyer is physically in that state.

The issue came up in connection with a lawyer's practice of federal law in

Maryland. Of course, the Supremacy Clause allows lawyers to practice purely federal

law even if they are not licensed in the state where they are physically present.

In Kennedy v. Bar Ass'n, 561 A.2d 200 (Md. 1989), the court acknowledged the

possibility that a lawyer could properly draw the line between the permissible offering of

federal law advice and the impermissible offering of Maryland law advice. But the court

59

35951124_10 UPL, MDP and MJP (Defining What Lawyers Do and McGuireWoods LLP Where They Can Do It): Part II T. Spahn (1/1/19) Hypotheticals and Analyses

found as a practical matter that the lawyer could not adequately serve clients by trying to do so.

We will not go so far as to say that it is theoretically impossible for Kennedy to maintain a principal office in Maryland exclusively for engaging in a practice before the federal court in Maryland and the courts in the District of Columbia. It seems, however, that it would be practically impossible to do so. Nevertheless, we shall not foreclose the possibility of Kennedy's presenting to the Circuit Court of Montgomery County, in the exercise of its continuing jurisdiction over the injunction, any proposal whereby Kennedy, without holding himself out as practicing law in Maryland, could first pinpoint clients whose specific matters actually required counsel before those courts where Kennedy is currently admitted to practice, and thereby could limit his legal representation in Maryland to those specific matters.

Id. at 211 (emphases added).

Significantly, the court did not focus on what the lawyer would do for the clients he represented. Instead, the court noted that the lawyer would be engaging in the practice of law (in Maryland) when deciding whether he could represent them. The court explained that

advising clients by applying legal principles to the client's problem is practicing law. When Kennedy, who is unadmitted in Maryland, set up his principal office for the practice of law in Maryland and began advising clients and preparing legal documents for them from that office, he engaged in the unauthorized practice of law. This is so whether the legal principles he was applying were established by the law of Montgomery County, the State of Maryland, some other state of the United States, the United States of America, or a foreign nation. . . . He is not permitted to sort through clients who may present themselves at his Maryland office and represent only those whose legal matters would require suit or defense in a Washington, D.C. court or in the federal court in Maryland because the very acts of interview, analysis and explanation

60

35951124_10 UPL, MDP and MJP (Defining What Lawyers Do and McGuireWoods LLP Where They Can Do It): Part II T. Spahn (1/1/19) Hypotheticals and Analyses

of legal rights constitute practicing law in Maryland. For an unadmitted person to do so on a regular basis from a Maryland principal office is the unauthorized practice of law in Maryland.

Id. at 208-10 (emphases added). The District of Columbia Bar later suspended

Kennedy for nine months because of this infraction in Maryland. In re Kennedy, 605

A.2d 600 (D.C. 1992).

This is a frightening holding that could theoretically subject to criminal penalties even non-lawyers who handle some tasks for clients, but not other tasks (which would amount to the practice of law). For instance, a non-lawyer tax preparer's client might ask if the tax preparer can write the client's will. Under the Maryland court's approach, the tax preparer would presumably be committing the criminal unauthorized practice of law by explaining to the client that the tax preparer could not write the will because it would be the unauthorized practice of law. And as that unfortunate Maryland lawyer discovered after being punished by the D.C. Bar, the stakes are high.

A few states have allowed lawyers to systematically and continuously practice within a state without being licensed there -- as long as they do not give any clients advice about the law of that state. This approach does not focus specifically on "virtual" practice. For instance, a new associate might work in a large law firm's office without being licensed in that state, and exclusively conduct research into New York and

Delaware corporate law, etc. But focusing on the type of legal advice that such lawyers can give opens up the possibility of lawyers physically present in states where they are not licensed "virtually" representing clients in states where they are licensed.

61

35951124_10 UPL, MDP and MJP (Defining What Lawyers Do and McGuireWoods LLP Where They Can Do It): Part II T. Spahn (1/1/19) Hypotheticals and Analyses

In 2011, Virginia indicated that such practice does not run afoul of the MJP rules -- as long as the lawyer physically present in Virginia limits her legal advice to federal law or to the law of states where she is licensed.

• Virginia LEO 1856 (9/19/11) (explaining that under Virginia Rule 5.5, non- Virginia lawyers "may not practice Virginia law on a 'systematic and continuous' basis," unless they (1) limit their practice to the "law of the jurisdiction/s where they are licensed"; (2) practice "exclusively federal law" under the federal supremacy clause (such as "lawyers with practices limited to immigration or military law or who practice before the Internal Revenue Service, the United States Tax Court, or the United States Patent and Trademark Office," although lawyers such as bankruptcy, patent or federal procurement lawyers must abide by courts' possible limitation of practice before the courts to members of the Virginia Bar, and may provide advice "such as the debtor's homestead exemption and status or priority of claims or liens" or "the assignment of the patent to a third party or the organization of a corporate entity to market or franchise the invention" only under the conditions mentioned immediately below; (3) "provide advice about Virginia law or matters peripheral to federal law (described immediately above) only if they do so on a "temporary and occasional" basis and (as stated in UPL Opinion 195) "under the direct supervision of a Virginia licensed lawyer before any of the [non-Virginia] lawyer's work product is delivered to the client" or if they "associate with an active member of the Virginia State Bar."; noting that Rule 5.5 overrules an earlier UPL Opinion about which law applies to a non-Virginia lawyer's practice of another state's law while physically in Virginia; thus, "New York law should govern whether a foreign lawyer not authorized to practice in New York may advise New York clients on matters involving New York law. The [non-Virginia] lawyer's physical presence in Virginia may not be a sufficient basis to apply Virginia's rules over New York's rules governing foreign lawyer practice." Contract lawyers hired to "work on a matter involving Virginia law" must either "be licensed in Virginia or work in association with a Virginia licensed lawyer in the firm on a temporary basis" although such a lawyer's practice "could be regarded as 'continuous and systematic'" if the non-Virginia contract lawyer is hired "to work on several and various Virginia matters/cases over a period of time."; concluding that such contract lawyers need not be licensed in Virginia if the lawyer is "hired to work only on matters involving federal law or the law of the jurisdiction in which the [non-Virginia] contract lawyer is admitted.").

62

35951124_10 UPL, MDP and MJP (Defining What Lawyers Do and McGuireWoods LLP Where They Can Do It): Part II T. Spahn (1/1/19) Hypotheticals and Analyses

More recently, Arizona amended its ethics rules to permit lawyers to practice continuously in Arizona (without a license there) as long as they give advice only about the law of a state where they are licensed (or federal or tribal law).

A lawyer admitted in another United States jurisdiction, or a lawyer admitted in a jurisdiction outside the United States, not disbarred or suspended from practice in any jurisdiction may provide legal services in Arizona that exclusively involve federal law, the law of another jurisdiction, or tribal law.

Arizona Rule 5.5(d) (emphasis added). The rules change reversed a 2010 legal ethics opinion that had taken the restrictive approach.

• Arizona UPL Advisory Op. 10-02 (02/2010) (explaining out-of-state lawyers may not reside in Arizona and practice virtually in a state where they are licensed; "An out-of-state lawyer admitted to practice law in states other than Arizona wishes to relocate to Arizona and to practice law of the states in which he is admitted while physically present in Arizona. The lawyer seeks an advisory opinion regarding whether a lawyer may practice the law of states in which he is admitted while physically present in Arizona if the lawyer clearly discloses the jurisdictional limitations of his practice on his letterhead and business cards, does not solicit or advertise to Arizona residents, and does not practice Arizona law. The lawyer further inquires whether such work may be performed in an Arizona office of record, either that of the lawyer admitted to practice out of state or that of an attorney admitted to practice in Arizona."; "An out-of-state lawyer, not admitted to practice in Arizona but living in Arizona, may not practice law limited to the law of jurisdictions in which he is licensed. The out-of-state lawyer may not perform the practice of law in an Arizona office of record, either the office of the out- of-state lawyer or an admitted Arizona attorney."; "Consistent with the Supremacy Clause and preemption doctrine, Rule 31 exemptions and MJP rules at ER 5.5 permit out-of-state lawyers to practice federal law as authorized by federal law and rules. The Supremacy Clause does not extend to laws of other states. Neither the exceptions to Rule 31 nor the MJP rules at ER 5.5 permit the out-of-state lawyer to engage in the practice of law of the out-of-state lawyer's jurisidiction while s/he resides in Arizona."; "Rule 31(a)(2)(A) defines the 'Practice of law' as providing legal advice or services to or for another . . .' The rules do not limit the term 'practice of law' to Arizona law. ER 5.5(d)(2) assumes the practice of law is not limited to Arizona law. The multijurisdictional rule carves out a 'safe harbor' in 5.5(d) for practice of law by an out-of-state lawyer stating the 'lawyer admitted in another United States jurisdiction . . . may provide legal services in this

63

35951124_10 UPL, MDP and MJP (Defining What Lawyers Do and McGuireWoods LLP Where They Can Do It): Part II T. Spahn (1/1/19) Hypotheticals and Analyses

jurisdiction that . . . are services that the lawyer is authorized to provide by federal law or other law of this jurisdiction.' The rule permits that lawyer to provide legal services authorized by federal law and the law of Arizona, and does not permit the out-of-state lawyer to provide legal services authorized by another state's jurisdiction. One can not [sic] extend the authorization to engage in the practice of federal law to an authorization to engage in the practice of the out-of-state lawyer's state of admission.").

If non-Arizona lawyers can practice law under those limitations "in Arizona," they clearly can practice "virtually" in a state where they are licensed. That other state presumably would not object to such a practice, because the lawyer is licensed there

(although that other state might have some archaic requirements that the lawyer have a physical office in that state, etc.).

The Arizona rule change is more significant than Virginia's 2011 legal ethics opinion -- both because it is a formal rule, and because Arizona made the change in the midst of the nationwide debate about the multijurisdictional practice implications of a

"virtual" practice.

In one encouraging sign, even some turf-protecting states have respected other states' liberal approach.

• Smith v. Hewlett-Packard Co., Case No. 2:15-cv-484-GMN-VCF, 2016 U.S. Dist. LEXIS 75938, at *4, *4-5, *5, *5-6 (D. Nev. June 9, 2016) (rejecting plaintiff's motion to disqualify and sanction defendant's lawyer based on plaintiff's argument that defendant's lawyer lied about her practice of law in Pennsylvania; "Although the 'practice of law' may be difficult to define, it most assuredly encompasses: advising clients regarding the law; preparing documents for client which require a familiarity with legal principles beyond the ken of the ordinary layman such as wills and contracts; and appearing for client before public tribunals charged with the power of determining liberty or property rights.' Gmerek v. State Ethics Com'n, 751 A.2d 1241, 1251 (Pa. Commw. Ct. 2000). 'However, it is important to stress that the 'practice of law' is not limited to a lawyer's appearance in court.' Id."; "Plaintiffs argue that Elam's [lawyer] two appearances in Pennsylvania District Court do not constitute the 'regular practiced law.' Plaintiffs' argument adopts an extremely narrow definition of the term 'practice of law.' Pennsylvania courts

64

35951124_10 UPL, MDP and MJP (Defining What Lawyers Do and McGuireWoods LLP Where They Can Do It): Part II T. Spahn (1/1/19) Hypotheticals and Analyses

have addressed and rejected Plaintiffs' definition."; "Furthermore Plaintiffs' definition is out of place in a modern legal market. In the era of the legal specialist, Plaintiffs' definition leads to the absurd conclusion that every transactional lawyer or in-house counsel does not 'practice law' simply because they do not regularly appear in court. Since this court does not accept Plaintiffs' definition of 'practice of law,' it finds that Elam made no misrepresentation on her pro hac vice petition."; "Plaintiffs argue that Elam engaged in the unauthorized practice of law when she continued to work for HP in Virginia despite not being admitted to the Virginia state bar and without a corporate counsel certificate. Smith's argument ignores that fact that the Virginia Rules of Professional Conduct permit attorneys like Elam to provide legal services to out-of-state clients while residing in Virginia. Furthermore Elam promptly obtained a Virginia corporate counsel certificate once Plaintiff's counsel had brought this oversight to her attention. . . . Elam's conduct in Virginia does not constitute the unauthorized practice of law." (emphases added)).

• New York LEO 1054 (4/10/15) (Virginia ethics rules governing a New York- licensed lawyer's "virtual" practice in Virginia; "The inquirer is an attorney licensed to practice in both the State of New York and the Commonwealth of Pennsylvania. He now intends to open a solo law office in Virginia, for the sole purpose of representing veterans and their dependents in the United States Court of Appeals for the Fourth Circuit, the United States District Courts in Virginia, and the Administrative Board of Veterans Appeals."; "The inquirer seeks to practice in Virginia by using a physical office two days per month, using the street address of the office as his mailing address, having access to a private mailbox at that address five days a week; answering phone calls personally when in the office; forwarding calls to the inquirer's cell phone or to a personal voicemail account attached to the cell phone when he is not in the physical office; and using a recorded message when he is not available to answer a phone call."; "The inquirer formerly worked for the federal government, working on rulemakings pertaining to veterans' benefits and representing the government on appellate briefs."; "The inquirer states that he has obtained an advisory opinion from the Virginia State Bar Association’s ethics committee, advising that he is permitted to practice from an office address in Virginia, as long as the inquirer (a) limits his practice to federal court and (b) indicates on his letterhead, business cards and website that he is licensed to practice law only in New York and Pennsylvania. The inquirer also states that such opinion would permit the inquirer to operate using a 'virtual office.'"; "Here, the inquirer is not formally admitted to the bar in Virginia, the jurisdiction in which he intends to principally practice. However, in N.Y. State 815 (2007), we determined that, if a New York lawyer is permitted to engage in conduct in another jurisdiction without being formally admitted in that jurisdiction, then the lawyer should be deemed to be 'licensed to practice' in the other jurisdiction, even though such conduct

65

35951124_10 UPL, MDP and MJP (Defining What Lawyers Do and McGuireWoods LLP Where They Can Do It): Part II T. Spahn (1/1/19) Hypotheticals and Analyses

would constitute the practice of law if the lawyer were practicing in New York. According to the inquirer, the Virginia State Bar Association has opined that he may practice from an office address in Virginia, as long as he limits his practice to federal court, and indicates on his letterhead, business cards and website that he is licensed to practice law only in New York and Pennsylvania. Consequently, the inquirer is deemed 'licensed to practice' in Virginia, and the New York disciplinary authorities would ordinarily apply the Virginia Rules of Professional Conduct to his conduct. However, an exception will arise if the inquirer solicits business in New York or Pennsylvania. In that case, the lawyer's conduct regarding the solicitation would clearly have its 'predominant effect' in the admitting jurisdiction to which the solicitations are directed, and the disciplinary authorities would apply the rules of that jurisdiction to the solicitations. . . . Whether any ensuing business would also be subject to the rules of such admitting jurisdiction depends upon where such business has its 'predominant effect.' That is a factual question on which we do not opine." (footnote omitted); "If a New York lawyer has been admitted to practice (generally, or for purposes of a proceeding) before the Virginia courts, when the lawyer represents a client in a proceeding in a court in Virginia, the rules to be applied ordinarily will be the rules of Virginia, unless the court rules provide otherwise. If the lawyer does not represent a client in a proceeding in a court, the rules to be applied will be those of the 'admitting jurisdiction' in which the inquirer principally practices, unless the conduct clearly has its predominant effect in another jurisdiction in which the lawyer is licensed to practice. If the lawyer is permitted to practice in Virginia without being formally admitted there, the lawyer should be deemed to be 'licensed to practice' in Virginia for purposes of Rule 8.5(b)(2). However, if the lawyer solicits business in New York, the lawyer's conduct in connection with such solicitation would have its principal effect in New York and the disciplinary authorities would apply the rules of New York." (emphases added)).

As "virtual" practice became easier, some lawyers planned to establish law firms structured so lawyers would practice physically in one state but serve clients "virtually" in other states. It would be easy to see the attractiveness in such an arrangement in some circumstances. For instance, lawyers wishing to live in attractive locations where they attended law school (such as Palo Alto, Chapel Hill, Boston, etc.) could physically remain in those locations, but practice "virtually" in locations where their talents would be useful -- such as New York, Chicago, Los Angeles, Washington, etc. And those law

66

35951124_10 UPL, MDP and MJP (Defining What Lawyers Do and McGuireWoods LLP Where They Can Do It): Part II T. Spahn (1/1/19) Hypotheticals and Analyses

firms could take advantage of the frequently lower cost of living in those college towns,

compared to the high-price cities where sophisticated legal services would be more marketable.

It is difficult to gauge whether such law firm models have succeeded. A 2014

article described the demise of one such law firm.

• Andrew Strickler, 'Virtual' Firm Clearspire Dissolves, Sells Tech Platform, Law360, June 9, 2014 ("'Virtual' law firm Clearspire, which launched five years ago with a promise of a high-tech online platform and no hourly fees, has dissolved its legal branch, a firm founder confirmed Friday, with the firm's technology sold to a group of investors looking to bring it to market."; "'From the beginning, we knew we ultimately wanted this thing to be a 21st century platform that could scale to many lawyers and law firms,' Clearspire co- founder Bryce Arrowood said. 'We've had a very strong proof of concept.'"; "A Texas-based investment group with experience in legal technology purchased the firm's cloud-based platform, known as Coral, with the goal of marketing the technology to other firms, Arrowood said."; "As part of that sale completed in May, Clearspire's legal branch, known as Clearspire Law Company PLLC, was dissolved, with about two dozen attorneys moving on to other firms or businesses."; "The technology side of the company will continue under the Clearspire name and will be led by Eyal Iffergan, Clearspire's former chief investment officer and the chief architect of the platform."; "Clearspire launched in 2009 under a non-partnership business model in which employee-lawyers working remotely would collaborate and share work products with clients and others in a secure online environment."; "The Washington, D.C.-based firm at one time had a few dozen lawyers and a range of service offerings, including corporate, banking and finance, and employment.").

Lawyers Moving Permanently To Another State

Some but not all states allow lawyers to be admitted by motion to those states'

bar without taking a bar examination. Among other things, such states normally limit

such admission by motion to lawyers practicing in jurisdictions which offer reciprocal

rights to the state's lawyers.

67

35951124_10 UPL, MDP and MJP (Defining What Lawyers Do and McGuireWoods LLP Where They Can Do It): Part II T. Spahn (1/1/19) Hypotheticals and Analyses

In a somewhat analogous scenario, many federal courts limit their bar admission to lawyers practicing in the federal courts' host state.

Both of these admissions analyses normally require a state or the federal court to determine where the lawyers seeking admission are practicing law. This begs the questions of whether lawyers practicing "virtually" in a jurisdiction meet the admissions standards of state or federal courts considering the motions from such lawyers to be admitted to their bars.

This can therefore raise the familiar "virtual" practice issue.

• In re Carlton, 708 F. Supp. 2d 524, 526, 525, 526, 527 (D. Md. 2010) (holding that a lawyer who sometimes visits her firm's D.C. office and uses that office's computer for her work and conference rooms for client meetings can claim that a D.C. office is her "principal law office" when applying for admission to the District of Maryland bar, even though she does not physically work full-time in that office; explaining the factual background; "Ms. Carlton advised that from her home in Cambridge, she accesses a computer in the Washington, D.C. office of the firm that is designated for her use. She is thus able to use the firm's computer network and access all programs used by the firm's attorneys, including the internal firm email and firm time-keeping program. Thus, even though she is physically located in Cambridge, Massachusetts, she works off a computer and server located in Washington, D.C., and, just as when she physically worked in Washington, D.C., and, all of her correspondence is sent to the Washington, D.C. address and forwarded to her by the firm's office staff. Clients communicate with her by calling the firm's Washington, D.C. phone number which forwards those calls to her in Cambridge in the same manner as would be the case at an extension in the District of Columbia office. All of her outgoing client correspondence is sent from the D.C. office and all court pleadings are also prepared for filing and filed from the District of Columbia office, unless she is filing a pleading electronically which she can do from Cambridge. Finally, she stated that she only meets with clients when she is in Washington, D.C., and that she has traveled there several times over the past year to complete large projects and meet with clients."; noting that "Local Rule 701.1 (a) provides that 'in order for an attorney to be qualified for admission to the bar of this district, the attorney must be, and continuously remain, a member in good standing of the highest court of any state (or the District of Columbia) in which the attorney maintains his or her principal law office, or the Court of Appeals of Maryland.' (emphasis added)."; explaining that "[i]n recent years, 68

35951124_10 UPL, MDP and MJP (Defining What Lawyers Do and McGuireWoods LLP Where They Can Do It): Part II T. Spahn (1/1/19) Hypotheticals and Analyses

the concept of a 'principal law office' has evolved somewhat as a result of significant advances in technology which provide an attorney with the flexibility to carry out a variety of activities at different locations and under varying circumstances. The term does not necessarily mean continuous physical presence but, at a minimum it requires some physical presence sufficient to assure accountability of the attorney to clients and the Court."; noting that the Washington, D.C., office was more than just a "mail drop" in this situation, and that the lawyer was occasionally physically present in Washington, D.C. (emphasis added)).

"Virtual Offices"

Bars and courts have had to assess whether "virtual" offices constitute offices in several contexts where that concept is important.

First, can lawyers advertise such a "virtual" office as an office?

• New York City LEO 2014-2 (6/2014) (approving lawyer's principal law office a "virtual" office; "A New York lawyer may use the street address of a virtual law office ('VLO') located in New York state as the 'principal law office address' for the purposes of Rule 7.1(h) of the New York Rules of Professional Conduct (the 'New York Rules' or the 'Rules'), even if most of the lawyer's work is done at another location. In addition, a New York lawyer may use the address of a VLO as the office address on business cards, letterhead and law firm website. Given the lower overhead, improved encryption systems, expansion of mobile communication options, availability of electronic research, and the ease of storing and transmitting digital documents and information, VLOs are becoming an increasingly attractive option for attorneys throughout the country. A New York lawyer who uses a VLO must also comply with other New York Rules, including Rules 1.4, 1.6, 5.1, 5.3, 8.4(a) and 8.4(c)."; "A New York lawyer (the 'Lawyer') is considering becoming a solo practitioner and plans to do most of her work at her home. The Lawyer does not intend to maintain a separate physical office. Instead, she plans to use a VLO in New York State, as defined below, to meet with clients, hold 'office hours,' receive mail, or otherwise be present and available at various times. For privacy and security reasons, she does not wish to identify her home address as her business address. She would like to use the address of the VLO as her 'principal office address' for purposes of advertising her legal services under Rule 7.1(h). She would also like to use the VLO address on her letterhead, business cards and law firm website."; "The VLO, as used herein, has a physical street address where the Lawyer plans to make herself available for meetings with clients and where the Lawyer can receive service and delivery of legal papers. Accordingly, we conclude that the use of a VLO address in attorney advertising complies with

69

35951124_10 UPL, MDP and MJP (Defining What Lawyers Do and McGuireWoods LLP Where They Can Do It): Part II T. Spahn (1/1/19) Hypotheticals and Analyses

the requirement of 7.1(h) to disclose a physical street address."; "[L]awyers who use VLOs may need to take additional precautions to ensure that they are fulfilling their supervisory obligations. Notwithstanding the differences between VLOs and traditional law firms, the '[a]ttorney must take reasonable measures to ascertain that everyone under her supervision is complying with the Rules of Professional Conduct, including the duties of confidentiality and competence.'" (citation omitted); "A lawyer who uses the shared services and office space of a VLO to perform legal services and to meet with clients, witnesses, or other third parties must take reasonable steps to ensure that she does not expose or put the client's confidential information at risk. This should include, as appropriate, training and educating staff at the VLO on these obligations."; "Lawyers who use VLOs must be particularly mindful of these ethical obligations, given that the lawyers may frequently be away from the physical location that serves as their business address. Lawyers who use VLOs should also take steps to ensure that they are available to meet with and communicate with their clients and respond promptly to their requests for information." (emphasis added)).

• Michigan LEO RI-355 (10/26/12) ("Maintaining a part-time presence at an alternate law office, which is not staffed during normal business hours on a regular basis and occupies office space not reserved for use solely by the lawyer and shared with nonlawyers, raises a number of ethical issues— communication, confidentiality, safeguarding client property, competent and diligent representation, advertising, impermissible multidisciplinary practice, and facilitation of the unauthorized practice of law. The potential for the concern may be heightened by the fact that the lawyer's presence in the location is sporadic, rendering what transpires in the lawyer's absence largely unobserved."; "Lawyers seeking to add an alternate law office to their law practice must comply with the applicable Rules of Professional Conduct, which includes disseminating any information about their practice in a manner that is not false, fraudulent, misleading, or deceptive contrary to MRPC 7.1. A lawyer cannot communicate the existence of a physical office location unless the lawyer maintains dedicated office space, appropriately separate and distinct from other businesses."; "Lawyers are not precluded from meeting with clients or prospective clients at locations other than a permanent office maintained during normal business hours. However, in communications governed by MRPC 7.1, a lawyer cannot identify a physical location as a law office without having a dedicated office space that has the necessary separation from other businesses.").

Second, may lawyers admitted pro hac rely on local counsel with only a "virtual" office in the pertinent jurisidiction? Several courts have rejected that approach.

70

35951124_10 UPL, MDP and MJP (Defining What Lawyers Do and McGuireWoods LLP Where They Can Do It): Part II T. Spahn (1/1/19) Hypotheticals and Analyses

• Suevon Lee, Edelson DQ'd In False Ad Row For Using 'Virtual Office,' Law360, Apr. 20, 2016 ("A California federal judge on Wednesday disqualified Edelson PC as counsel in a putative class action accusing TP- Link USA Corporation of deceptive marketing of its network adapters, saying an out-of-state attorney's use of a virtual office address amounted to bad faith conduct."; "United States District Judge Percy Anderson also imposed sanctions against the leading Chicago-based technology class action litigation firm in the amount of $5,265 and denied a partner’s pro hac vice application to the Central District of California in the false advertising suit brought against the California-based United States subsidiary of China’s network adapter manufacturer, TP-Link Technologies Co."; "Judge Anderson's ruling cast a strict gaze on Edelson's long-held practice of applying for out-of-state court admission by using a so-called 'virtual office' address for designated local counsel, in light of the fact that the court considering such applications cannot discern from the address alone if it's for a bona fide office or not."; "'Edelson has abused this blind spot for more than six years, securing multiple pro hac vice admissions in flagrant violation of the court’s local rules,' Judge Anderson wrote in a nine-page order dated April 19."; "In this instance, Chicago-based Edelson partner Ari J. Scharg applied to appear pro hac vice in the Central District -- which includes Los Angeles -- designating an associate, Thomas Logan, as his local counsel. But attorneys for TP-Link USA filed an objection to that application last month, saying Logan lives in Chicago, practices out of the firm's San Francisco and Chicago offices, and that his listed office address in the Central District is actually for a virtual office in a downtown Los Angeles office building operated by Regus Business Center, which provides mail-and call-forwarding services and in-person meeting space."; "TP-Link's counsel at LTL Attorneys LLP also referenced the number of times Scharg had been admitted to practice in California on a pro hac vice basis -- 28 times over the last six years, with nine of those cases in the Central District."; "Over the last six years, Judge Anderson’s ruling said, Edelson has maintained a virtual office in the Central District -- in Santa Margarita, California, from January 2010 to January 2014 and in the downtown Los Angeles location since January 2014 to present."; "While the Chicago plaintiffs' firm, which frequently litigates cases in California due to its tech industry-geared focus, argued that the Central District's local rules permit use of a 'virtual office' by designated local counsel, the judge said the law firm failed to establish it could satisfy the rules' additional prong that this local counsel, through use of the virtual office, is someone with whom opposing counsel can 'readily communicate regarding the conduct of the case and upon whom documents may be served.'"; "The judge pointed out how Edelson 'readily admits that no Edelson attorney or staff operate from the Los Angeles virtual office.'"; "'Thus, it appears that the Los Angeles virtual office is not ‘maintain[ed] . . . for the practice of law’ so much as it is maintained for the purpose of

71

35951124_10 UPL, MDP and MJP (Defining What Lawyers Do and McGuireWoods LLP Where They Can Do It): Part II T. Spahn (1/1/19) Hypotheticals and Analyses

facilitating pro hac vice admissions for Edelson attorneys,' the judge held." (emphasis added)).

• Arroyo v. TP-Link USA Corp., No. CV 16-1044 PA (KKx), 2016 U.S. Dist. LEXIS 38952, at *2, *2-3, *3-4, *4-5 (C.D. Cal. Mar. 24, 2016) (denying lawyer's pro hac vice motion and issuing a show-cause order why the lawyer should not be sanctioned for improperly providing a required local counsel's address on what turned out to be a "virtual office"; "The Local Rules for the Central District of California govern who may appear before the Court pro hac vice."; "An attorney is disqualified under L.R. 83-2.1.3.2 if he or she: '(a) resides in California; (b) is regularly employed in California; or (c) is regularly engaged in business, professional, or other similar activities in California.' L.R. 83-2.1.3.2. Applicants seeking to appear pro hac vice must also designate local counsel pursuant to Local Rule 83-2.1.3.4, which provides: 'Every attorney seeking to appear pro hac vice must designate as Local Counsel an attorney with whom the Court and opposing counsel may readily communicate regarding the conduct of the case and upon whom documents may be served. An attorney may be designated as Local Counsel only if he or she: (1) is a member of the Bar of this court; and (2) maintains an office within the District.' L.R. 83-1.2.1.3.2."; "[O]n March 18, 2016, Ari J. Scharg filed an Application to Appear Pro Hac Vice (Docket No. 119), which also designated Mr. Logan as local counsel. In this application, Mr. Logan represented that his office was located at 555 West Fifth Street, 31st Floor, Los Angeles, California 90013. Additionally, Mr. Scharg declared, under penalty of perjury, that he had designated local counsel who 'maintains an office in the Central District of California for the practice of law.' Defendant's Opposition points out the inconsistency in the location of Mr. Logan's office, and asserts that the Los Angeles office is only a 'virtual office' which does not comply with the Local Rule's requirements."; "The Court therefore orders Plaintiff's counsel to show cause, in writing, no later than April 4, 2016, why they should not be sanctioned for abusing the pro hac vice application process. Plaintiff's response to this Order to Show Cause must include: (1) a declaration under penalty of perjury from Mr. Logan as to how many hours per week he physically spends in the Los Angeles office; how many Edelson employees, including other attorneys, are physically present at the Los Angeles office; and the city in which Mr. Logan maintains his permanent residence; [and] whether he maintains a residence in Los Angeles . . . . The Court warns that potential sanctions include the imposition of monetary sanctions, including Defendant's attorneys' fees incurred in bringing this matter before the Court, as well as the disqualification of any Edelson attorney from serving as either Plaintiff's individual counsel or as class counsel for the putative class.").

72

35951124_10 UPL, MDP and MJP (Defining What Lawyers Do and McGuireWoods LLP Where They Can Do It): Part II T. Spahn (1/1/19) Hypotheticals and Analyses

Third, do "virtual" offices satisfy specific state statutory rules requiring an "office"?

New York has taken a predictably narrow approach.

• Platinum Rapid Funding Grp. V. HDW of Raleigh, Inc., No. 605890-17, 2017 NYLJ LEXIS 3734, at *4, *5, *5-6, *7-8 (N.Y. Sup. Ct. Jan. 9, 2018) (disqualifying a California law firm from acting as counsel of record in a New York case because a "virtual" firm did not satisfy New York’s requirement that lawyers practicing in New York have a physical office there; "Pursuant to Judiciary Law §470 entitled 'Attorneys having offices in this state may reside in adjoining state.'"; "A person, regularly admitted to practice as an attorney and counsellor, in the courts of record of this state, whose office for the transaction of law business is within the state, may practice as such attorney or counsellor, although he resides in an adjoining state." (citation omitted); "That is, this statutory directive that nonresident attorneys maintain an office within the State 'for the transaction of law business' requires nonresident attorneys to maintain a physical office in New York . . . . Indeed, an attorney or firm that has appeared in an action while violating Judiciary Law §470 must be disqualified from continuing to appear or represent any of the parties in the action."; "Based upon the papers submitted herewith, this Court finds that, in this case, neither Higbee & Associates ('Higbee') nor Raymin L. Ngo ('Ngo') had a physical office in the State of New York at the time that they appeared in this action on behalf of the defendants (ld). Thus, said entities are all non-residents and have failed to comply with the Judiciary Law §470. Indeed, the papers herein establish that Ngo and Higbee’s pleading – the Verified Answer and Counterclaims – identified their principal office to be located in Santa Ana, California . . . ."; "In the end, this Court finds that there is no evidence on this record that Ngo and Higbee had physical addresses in New York. Moreover, this Court cannot overlook the fact that the defendants have failed to offer any competing evidence against the sworn affidavits of Steven Pena and Jakeen Penss, Sr., process servers who attest that they physically went to the 48 Wall Street and or 605 West Genesee addresses, respectively, and confirmed that neither Ngo nor Higbee had physical offices at these locations. Accordingly, this court herewith awards the plaintiff its instant motion to disqualify the defendants’ attorneys of record – Raymin L. Ngo and Higbee & Associates.").

• New York LEO 1054 (4/10/15) (Virginia ethics rules govern a New York- licensed lawyer's "virtual" practice in Virginia; "If a New York lawyer has been admitted to practice (generally, or for purposes of a proceeding) before the Virginia courts, when the lawyer represents a client in a proceeding in a court in Virginia, the rules to be applied ordinarily will be the rules of Virginia, unless the court rules provide otherwise. If the lawyer does not represent a client in a proceeding in a court, the rules to be applied will be those of the 'admitting jurisdiction' in which the inquirer principally practices, unless the

73

35951124_10 UPL, MDP and MJP (Defining What Lawyers Do and McGuireWoods LLP Where They Can Do It): Part II T. Spahn (1/1/19) Hypotheticals and Analyses

conduct clearly has its predominant effect in another jurisdiction in which the lawyer is licensed to practice. If the lawyer is permitted to practice in Virginia without being formally admitted there, the lawyer should be deemed to be 'licensed to practice' in Virginia for purposes of Rule 8.5(b)(2). However, if the lawyer solicits business in New York, the lawyer's conduct in connection with such solicitation would have its principal effect in New York and the disciplinary authorities would apply the rules of New York."; "Assuming the inquirer is soliciting business in New York, another question arises: must he have a local office in New York? This question is governed by law and not by the Rules. In N.Y. State 1025 (2014), we noted that Judiciary Law §470 has been interpreted by New York courts to require that attorneys have an office in New York if they practice, but do not live, in New York. See Lichtenstein, 251 A.D.2d 64; Haas, 237 A.D.2d 729; Matter of Larsen, 182 A.D.2d 149 (2d Dept 1992). We also determined that Rule 7.1(h), which requires that every lawyer advertisement include the 'principal law office address and telephone number of the lawyer or law firm whose services are being offered,' does not provide an independent basis for requiring a physical office in New York."; "In N.Y. State 1025, we noted the case of Schoenefeld v. New York, 748 F.3d 464 (2d Cir. 2014). There, the Northern District of New York found unconstitutional the interpretation of § 470 requiring a physical office. On appeal, the Second Circuit referred a certified question to the New York Court of Appeals, asking about the minimum requirements necessary to satisfy the requirement for a local office for the transaction of law business. Although the Court of Appeals had not responded when we published N.Y. State 1025, on March 31, 2015, it issued its response, confirming that the statute requires a physical office for the conduct of business. The Second Circuit must now decide whether enforcement of §470 as so interpreted would be constitutional."; "Assuming the inquirer is soliciting business from New York residents, the inquirer must comply with various duties imposed by the Rules. See N.Y. State 1025 (2014) (listing duties under various Rules, and noting that there is no 'virtual law office exception' to any of the Rules)." (emphasis added)).

Best Answer

The best answer to this hypothetical is MAYBE. B 2/13, 11/16, 3/17

74

35951124_10 UPL, MDP and MJP (Defining What Lawyers Do and McGuireWoods LLP Where They Can Do It): Part II T. Spahn (1/1/19) Hypotheticals and Analyses

Pro Hac Admission in Another State's Court

Hypothetical 7

Your largest corporate client wants you to try an employment discrimination case recently filed against it in a neighboring state's court. You are not admitted to practice in that state.

Will you be able to represent your corporate client in the employment discrimination case?

YES (PROBABLY)

Analysis

Every federal and state court can decide for itself which lawyers can practice

before it.

Courts presumably could prohibit lawyers not fully licensed in the court's

jurisdiction from practicing in that court. However, every court allows lawyers to appear

temporarily, under certain circumstances. This is admission "pro hac vice" -- meaning

"this time only" in Latin.

The ABA Model Rules mention this process in a comment.

Lawyers not admitted to practice generally in a jurisdiction may be authorized by law or order of a tribunal or an administrative agency to appear before the tribunal or agency. This authority may be granted pursuant to formal rules governing admission pro hac vice or pursuant to informal practice of the tribunal or agency. Under paragraph (c)(2), a lawyer does not violate this Rule when the lawyer appears before a tribunal or agency pursuant to such authority. To the extent that a court rule or other law of this jurisdiction requires a lawyer who is not admitted to practice in this jurisdiction to obtain admission pro hac vice before appearing before a tribunal or administrative agency, this Rule requires the lawyer to obtain that authority.

75

35951124_10 UPL, MDP and MJP (Defining What Lawyers Do and McGuireWoods LLP Where They Can Do It): Part II T. Spahn (1/1/19) Hypotheticals and Analyses

ABA Model Rule 5.5 cmt. [9].

The Restatement also discusses such pro hac admissions.

Certainty is provided in litigated matters by procedures for securing the right to practice elsewhere, although the arrangement is limited to appearances as counsel in individual litigated matters. Apparently all states provide such a procedure for temporary admission of an unadmitted lawyer, usually termed admission pro hac vice. . . . Although the decision is sometimes described as discretionary, a court will grant admission pro hac vice if the lawyer applying for admission is in good standing in the bar of another jurisdiction and has complied with applicable requirements (sometimes requiring the association of local counsel), and if no reason is shown why the lawyer cannot be relied upon to provide competent representation to the lawyer's client in conformance with the local lawyer code. Such temporary admission is recognized in Subsection (2). Courts are particularly apt to grant such applications in criminal-defense representations. Some jurisdictions impose limitations, such as a maximum number of such admissions in a specified period. Admission pro hac vice normally permits the lawyer to engage within the jurisdiction in all customary and appropriate activities in conducting the litigation, including appropriate office practice. Activities in contemplation of such admission are also authorized, such as investigating facts or consulting with the client within the jurisdiction prior to drafting a complaint and filing the action.

Restatement (Third) of Law Governing Lawyers § 3 cmt. e (2000) (emphasis added).

Courts seem to generally agree on some basic principles.

First, a lawyer seeking to appear pro hac must be admitted in some state.

• In re Hall, 377 P.3d 1149, 1156-57, 1157 (Kan. 2016) (suspending for sixty days an inactive Kansas lawyer for having improperly been admitted pro hac vice to appear in Kansas court decades after abandoning his Kansas license; "In 1990, respondent went on inactive status in Kansas and then, in 1996, failed to pay the inactive fee. Due to his failure to pay the fee, this court ordered an administrative suspension of his license. In 2003 and 2009, respondent called the office of the Clerk of the Appellate Courts to learn what steps he could take to reinstate his license. But he failed to complete the necessary steps. Respondent's administrative suspension qualifies as a

76

35951124_10 UPL, MDP and MJP (Defining What Lawyers Do and McGuireWoods LLP Where They Can Do It): Part II T. Spahn (1/1/19) Hypotheticals and Analyses

suspension for Rule 218(c)(1) purposes. See In re Thompson, 301 Kan. 428, 433, 343 P. 3d 108 (2015) (an administrative suspension was sufficient to constitute a violation of Kansas Supreme Court Rule 218 for failure to notify clients, opposing counsel, and the courts of a suspension)."; "Respondent claims that despite his administrative suspension, he did not engage in the unauthorized practice of law because he was authorized through his pro hac vice admissions. In 2012, respondent submitted two applications to appear pro hac vice on behalf of two separate clients. On both applications, he failed to list his Kansas bar admittance, inform the court his license to practice law in Kansas was not in good standing, or inform the court his license in Kansas was on administrative suspension. Subsequently, respondent was admitted pro hac vice in both cases."; "The panel merely found respondent's pro hac vice admission invalid: 'Respondent obtained admission pro hac vice improperly as the respondent was not eligible for admission pro hac vice.' The Disciplinary Administrator correctly points out that, under Kansas Supreme Court Rule 116 (2015 Kan. Ct. R. Annot. 222), only out-of-state attorneys who are not admitted to practice in Kansas are eligible for pro hac vice admission. That rule states: 'An attorney not admitted to practice law in Kansas may be admitted on motion to practice law in a Kansas court or administrative tribunal – for a particular case only[.]' In the instant case, respondent was admitted to practice law in Kansas and therefore could not be admitted pro hac vice.").

• Arizona UPL Op. 10-01 (1/2010) (holding that a lawyer "authorized to practice in a legal services organization, pursuant to Arizona Supreme Court Rule 38(f) must be an active attorney in at least one jurisdiction where he or she is licensed to practice"; explaining that "[j]ust as a lawyer admitted pro hac vice is admitted on a temporary basis and must remain active in their home state, so too must an out-of-state lawyer who is authorized to practice law under Rule 38(f). Neither Rule 38(a), the pro hac vice Rule, nor Rule 38(f) specifically note that home admission must remain active. This is because it is presumed in both instances. Both authorizations are temporary and conditioned on an active license in another state.").

Second, lawyers hoping to be admitted pro hac in another state's court must follow the procedural guidelines and meet the deadlines.1

• Christopher Massaro & Neoma Ayala, What Out-Of-Staters Must Know When Hiring Local NJ Counsel, Law360, Aug. 28, 2015 ("Most state and federal courts have very specific and individualized rules governing the

1 Courts also seem to view pro hac admissions as an increasing source of revenue. Every court requires a fee, which frequently seems out of line with the cost of administering such admission applications. 77

35951124_10 UPL, MDP and MJP (Defining What Lawyers Do and McGuireWoods LLP Where They Can Do It): Part II T. Spahn (1/1/19) Hypotheticals and Analyses

admission of attorneys pro hac vice. In New Jersey, local counsel must file a motion supported by an affidavit or certification confirming that: (i) the attorney seeking admission is a member in good standing of the state bar where the attorney is domiciled or principally practices law; (ii) the attorney has associated with New Jersey counsel; (iii) the client has asked the out-of- state attorney for representation; and (iv) the attorney seeking admission has no past or pending disciplinary record (or the details of any such record or pending proceedings are disclosed). Such motions are granted only upon a showing of 'good cause,' which includes at least one of the following situations: (i) the case involves a complex area of law in which the attorney is a specialist; (ii) the attorney and client have a long-standing professional relationship; (iii) a lack of local counsel with the necessary expertise to handle the case; (iv) the case involves the law of the jurisdiction where the attorney is regularly admitted; (v) extensive discovery or proceedings will occur in a foreign jurisdiction; or (vi) other similar reasons for the pro hac vice admission exists." (footnote omitted); "Similar requirements apply for pro hac vice admission in the United States District Court for the District of New Jersey."; "Whether the litigation is pending in state or federal court, local attorneys are expected to: (i) supervise admitted attorneys and educate them regarding practice rules and customs; (ii) take responsibility for court filings and make appearances; and (iii) serve as the liaison between the court and any attorney admitted pro hac vice.").

• Ky. Bar Ass'n v. Yocum, 294 S.W.3d 437. 440 (Ky. 2009) (prohibiting an Indiana lawyer from applying to appear in Kentucky pro hac vice, because the lawyer had not filed the application in time; "[B]ecause Yocum's pro hac vice status has lapsed and he is no longer licensed to practice law in this Commonwealth, the effect of his suspension will be that he is prohibited from requesting permission to practice law in this Commonwealth, pro hac vice or otherwise, for 120 days.").Perhaps the least surprising provisions in every state's pro hac rules try to assure the involvement of a local lawyer.

In a frightening 2018 case, the Supreme Court of the Virgin Islands found that three Akin Gump lawyers had committed the unauthorized practice of law in the Virgin

Islands, and referred them, two Virginia-based colleagues and local counsel to bar disciplinary authorities and the Virgin Islands Attorney General (mentioning the possibility of prosecution).

78

35951124_10 UPL, MDP and MJP (Defining What Lawyers Do and McGuireWoods LLP Where They Can Do It): Part II T. Spahn (1/1/19) Hypotheticals and Analyses

In In re Jindal, S. Ct. BA Nos. 2018-0018, 2018-0019, 2018-0020, 2018 V.I.

Supreme LEXIS 38 (V.I. Nov. 29, 2018)2, three Akin Gump lawyers sought admission

pro hac vice in the Virgin Island courts to represent a client in an antitrust case. Those

lawyers had unwisely neglected to fulfill all the pro hac vice admissions requirements.

Although this Court granted those motions, it advised Attorney Eckard and the applicants that their authorization to practice on a pro hac vice basis would not commence until and unless they were administered the pro hac vice attorney oath. However, neither Jindal, Roush, nor Schmitten ever took the pro hac vice attorney oath.

Id. at *2.

Just a few months later, the lawyer's adversaries voluntarily dismissed all claims

against Akin Gump's client. Akin Gump's client soon sought approximately $75,000 in

attorneys fees. In addition to the time spent by the three lawyers, two other Virginia-

based Akin Gump lawyers' "legal services" appeared on the fee petition.

[T]he motion included affidavits from Amanda B. Lowe and Patrick O'Brien – two Virginia attorneys employed by Akin Gump, who are not admitted to practice in the Virgin Islands

2 In re Jindal, S. Ct. BA. Nos. 2018-0018 to 2018-0020, 2018 V.I. Supreme LEXIS 38, at *10 n.3, *13, *4-5, *2-3 (V.I. Nov. 29, 2018) (per curiam) (denying pro hac admission petitions of three Akin Gump lawyers after they sought fees for successfully representing their client in the Virgin Islands courts; acknowledging that their pro hac vice motions had been granted, but noting that they had never taken the necessary attorney oath and therefore were not authorized to practice in the Virgin Islands; explaining that their litigation adversary had asked for a stay of their request for attorneys fees because they had violated the Virgin Islands unauthorized practice of law regulations – but that the three lawyers had never responded to the unauthorized practice of law complaint; rejecting the lawyers' argument that they were practicing federal law (noting that the antitrust case involved Virgin Islands law, and that they could not ethically have practiced in the Virgin Island courts even if the case had involved federal law); also rejecting their argument that requiring pro hac admission in the Virgin Islands by "every attorney who works on a matter that 'touches' the Virgin Islands" would "'cripple cross-border transactions and litigation'"; warning that "[s]ince the underlying conduct may potentially warrant action beyond the denial of pro hac vice admission" (having earlier issued a show cause order demanding that the lawyers show cause why they should not be referred "for further investigation and prosecution"), referring to two Virgin Islands bar disciplinary entities and the Virgin Islands Attorney General): (1) the three Akin Gump lawyers; (2) two other Virginia-based Akin Gump lawyers who were listed as having provided "legal services," but who were never admitted pro hac; (3) the Akin Gump law firm; and (4) Akin Gump's local counsel). 79

35951124_10 UPL, MDP and MJP (Defining What Lawyers Do and McGuireWoods LLP Where They Can Do It): Part II T. Spahn (1/1/19) Hypotheticals and Analyses

and who never applied to this Court for pro hac vice admission – in which they also stated that they represented U.S. Concrete and included itemized lists of "legal services."

Id. at *2-3.

Plaintiffs' lawyer sought a stay of the fee issue, along with filing a "Notice to Court of Possible Violation of Pro Hac Vice Order." That Notice mentioned the Akin Gump lawyers' violation of "the prohibition on the unauthorized practice of law" codified in

Virgin Islands law and the Virgin Islands Rules of Professional Conduct. Id. at *4.

The three Akin Gump lawyers' second unwise move was to ignore what the court called plaintiffs' "unauthorized practice of law complaint."

Neither Attorney Eckard [Akin Gump's local counsel, who had filed the pro hac admission motions] nor Applicants Jindal, Roush or Schmitten responded to the May 23, 2018 unauthorized practice of law complaint. This Court, in a June 15, 2018 order, noted the seriousness of the allegations and directed Attorney Eckard and Applicants Jindal, Roush, and Schmitten to show cause, in writing, as to why their conduct (1) should not constitute the unauthorized practice of law or the aiding and abetting of the unauthorized practice of law; (2) should not result in denial of the applications byJindal, Roush, and Schmitten for pro hac vice admission; and (3) should not be referred to the Board on Unauthorized Practice of Law, the Board on Professional Responsibility, the Office of Disciplinary Counsel, the Virgin Islands Attorney General, or other authorities for further investigation and prosecution.

Id. at *4-5 (emphasis added). The three lawyers responded to the show cause order.

The court rejected several arguments advanced by the Akin Gump lawyers.

First, the court dismissed the Akin Gump lawyers' complaint that plaintiffs were using the UPL complaint "for leverage." Id. at *5 n.1. The court found plaintiffs' motives

80

35951124_10 UPL, MDP and MJP (Defining What Lawyers Do and McGuireWoods LLP Where They Can Do It): Part II T. Spahn (1/1/19) Hypotheticals and Analyses

irrelevant, and also pointed to the Virgin Islands ethics rule obligating lawyers to report

other lawyers' misconduct.

Second, the court found as "wholly without merit" the Akin Gump lawyers'

argument that their work was "consistent with ordinary cross-jurisdictional litigation practice," and that their acts were performed while they were physically present in

Washington, D.C. Id. at *6.

Although Jindal, Roush, and Schmitten were physically based in Washington, D.C., their conduct clearly relates to the Gilbert case, which was pending before the Superior Court of the Virgin Islands. Moreover, when they applied for pro hace vice admission, Jindal, Roush, and Schmitten agreed to submit to the jurisdiction of this Court and to be bound by the disciplinary rules applicable to Virgin Islands attorneys. See V.I.S.Ct.R. 201. Therefore, the Virgin Islands definition of the practice of law – including Virgin Islands rules and statutes relating to the unauthorized practice of law – apply to their conduct.

Id. at *7. The court noted that the Virgin Islands had not adopted ABA Model Rule 5.5,

which authorizes temporary practice in some jurisdictions under certain conditions.

Third, the court rejected the Akin Gump lawyers' argument that (as the court put

it) "an individual can 'engage in any conduct he wishe[s] without obtaining Virgin Islands

Bar membership, with the sole exception of signing motions in his own name and

arguing in court without the presence of a licensed attorney.'" Id. at *9 (alteration in

original). The court concluded that the three Akin Gump lawyers (and their Virginia-

based colleagues) had impermissibly practiced law in the Virgin Islands.

Fourth, the court vehemently rejected the Akin Gump lawyers' seemingly tone-

deaf position that the courts' approach would "cripple cross-border transactions and

litigation."

81

35951124_10 UPL, MDP and MJP (Defining What Lawyers Do and McGuireWoods LLP Where They Can Do It): Part II T. Spahn (1/1/19) Hypotheticals and Analyses

In their response, Eckard, Jindal, Roush, and Schmitten argue that "a rule requiring every attorney who works on a matter that 'touches' the Virgin Islands to be admitted to the Virgin Islands Bar is unworkable" and "cripple cross-border transactions and litigation.". . . However, the rule announced in Motylinski [In re Motylinski, 60 V.I. 621 (V.I. 2014)] has now been in place for more than four years, and no evidence has been presented to support a claim that these matters have been "crippled" as a result.

Id. at *10 n.3.

Fifth, the court rejected the Akin Gump lawyers' argument that their client understood their license restrictions and were satisfied with their work.

[T]hey cite to absolutely no authority – and this Court can find none – to support the proposition that individuals may engage in the unauthorized practice of law so long as the client is aware and satisfied.

Id. at *10 n.4.

Sixth, the court rejected the Akin Gump lawyers' reliance on caselaw allowing nonlawyers to assist Virgin Island lawyers in performing tasks.

[I]t is clear that Jindal, Roush, and Schmitten had held themselves out as attorneys in connection with this matter – even if only in conjunction with the attorneys' fees motion – and that any work they claimed to have performed necessary exceeded the acts – if any – that could permissibly be performed by a paralegal or secretary.

Id. at *11-12.

Seventh, the court rejected the Akin Gump lawyers' argument that they never held themselves out as Virgin Island lawyers.

Although the Applicants vehemently deny holding themselves out as Virgin Island attorneys, and in fact note that the attorneys' fee motion disclosed that their pro hac vice applications were pending, this Court has previously held that it "emphatically reject[s] any implication that one

82

35951124_10 UPL, MDP and MJP (Defining What Lawyers Do and McGuireWoods LLP Where They Can Do It): Part II T. Spahn (1/1/19) Hypotheticals and Analyses

can simply state 'I'm not currently admitted into the Virgin Islands Bar,' proceed to engage in precisely the activities forbidden by section 443, and then rely on those magic words as a talisman to escape liability under the statute."

Id. at *12 n.5 (alteration in original) (emphasis added).

Eighth, the court rejected the Akin Gump lawyers' argument that they were antitrust counsel for their client and therefore somehow authorized to practice in the

Virgin Islands. The court noted that the case "did not raise a claim under federal antitrust law, or assert any federal claim for that matter." Id. at *12. The court then indicated that even if the case had involved federal antitrust law, the Akin Gump lawyers would still have had to follow the Virgin Islands pro hac process.

But even if the Gilbert lawsuit involved a federal claim, the Supreme Court of the United States has held that "[t]here is no right of federal origin that permits . . . lawyers to appear in state courts without meeting that State's bar admission requirements." Leis v. Flynt, 439 U.S. 438, 443, 99 S. Ct. 698, 58 L. Ed. 2d 717 (1979). Neither Supreme Court Rule 201, Virgin Islands Rule of Professional Conduct 211.5.5, section 443 of title 4 of the Virgin Islands Code, nor any other applicable Virgin Islands rule or statute codifies a federal practice exception to either the prohibition on the unauthorized practice of law or the requirement that one cannot practice law in the Virgin Islands without being a member of the Virgin Islands Bar, and we agree with the courts that have declined to recognize such an exception. See Chandris, S.A. v. Yanakakis, 668 So.2d 180, 184 (Fla. 1995) ("We find no merit to [the] argument that there is a general federal law exception to Florida's bar admission requirement."); Kennedy v. Bar Ass'n of Montgomery County, Inc., 561 A.2d 200, 208 (Md. 1989) ("[A]dvising clients by applying legal principles to the client's problem is practicing law . . . . This is so whether the legal principles [the attorney] was applying were established by the law of Montgomery County, the State of Maryland, some other state of the United States, the United States of America, or a foreign nation.").

83

35951124_10 UPL, MDP and MJP (Defining What Lawyers Do and McGuireWoods LLP Where They Can Do It): Part II T. Spahn (1/1/19) Hypotheticals and Analyses

Id. at *12-13 (alterations in original).

The court ultimately denied the three Akin Gump lawyers' pro hac vice petitions.

The court then ended with an ominious paragraph.

Since the underlying conduct may potentially warrant action beyond the denial of pro hac vice admission, we also refer this matter to the Office of Disciplinary Counsel, the Board on Professional Responsibility, the Board on Unauthorized Practice of Law, and the Virgin Islands Attorney General for the purpose of taking any additional action which they may find appropriate with respect to the conduct of Eckard [Virgin Islands local counsel], Jindal, Roush, Schmitten [the three Akin Gump lawyers who had been admitted pro hac, but who had never taken the oath], Lowe, O'Brien [the two Virginia- based Akin Gump lawyers who had never applied for pro hac admission, but who had sought reimbursement for the "legal services" they had performed on the case], and the Akin Gump law firm.

Id. at *13-14 (emphases added). The court added a footnote describing those entities' possible steps, and the limited nature of their future role.

As this Court has previously explained, multiple entities may exercise concurrent jurisdiction to investigate and adjudicate unauthorized practice of law complaints, with different remedies beng available to those entities. For instance, the Board on Professional Responsibility may impose attorney discipline on Eckard and the three applicants for pro hac vice admission, whereas the Board on Unauthorized Practice of Law may obtain an order, enforceable by contempt, to direct that the unauthorized practice of law be ceased immediately. However, ultimately it is this Court that possesses the final word on what constitutes thte unauthorized practice of law in the Virgin Islands, and whether the prohibition on unauthorized practice has been violated. Because we find that the prohibition on unauthorized practice has occurred in this case, any proceeding before the Board on Professional Responsibility or the Board on Unauthorized Practice of Law will be limited to the issue of remedy, and not the question of whether the prohibition on the unauthorized practice of law has been violated.

84

35951124_10 UPL, MDP and MJP (Defining What Lawyers Do and McGuireWoods LLP Where They Can Do It): Part II T. Spahn (1/1/19) Hypotheticals and Analyses

Id. at *14 n.6 (emphases added).

Third, pro hac rules typically require the local lawyer in sign, and ultimately be responsible for, all pleadings filed in the case.

• Utah LEO 17-04 (9/26/17) ("Acting as local counsel for a pro hac vice attorney is not a minor or perfunctory undertaking. Local counsel violates the Utah Rules of Professional Conduct when local counsel acts as nothing more than a mail drop or messenger for the pro hac vice attorney. All attorneys admitted to the Utah State Bar are required to comply with all of the Utah Rules of Professional Conduct, including when they are acting as local counsel. Under Rule 5.1 of the Utah Rules of Professional Conduct, local counsel has a general duty to adequately supervise pro hac vice counsel and to provide expertise regarding Utah law, statutes, cases, rules, procedures, and customs in Utah. Local counsel is responsible to the client and responsible for the conduct of the Utah court proceedings. Under Rule 1.2 of the Utah Rules of Professional Conduct, local counsel may be able to limit the scope of the representation if the limitation is reasonable under the circumstances and the client gives informed consent. Regardless of any limited scope representation agreement, if local counsel determines that the pro hac vice attorney is engaging in conduct that is likely to seriously prejudice the client's interests, or the administration of justice, local counsel must communicate local counsel's independent judgment to the client, and, if necessary, to the court or tribunal.").

• New York City LEO 2015-4 (2015) ("Attorneys who act as “local counsel” are subject to the same ethical rules as all lawyers. An attorney who is retained as local counsel may circumscribe her role by entering into an agreement to limit the scope of representation, provided the agreement complies with Rule 1.2(c). It is the attorney’s obligation to communicate to the client any limits on the scope of the representation, rather than to rely on undefined terms, such as “local counsel.” Any limitations to the scope of representation must be reasonable under the circumstances and the client must give informed consent. Local counsel must also comply with any relevant court rules governing the responsibilities of counsel. Such rules are beyond the scope of this Committee’s jurisdiction, which is limited to interpreting the New York Rules of Professional Conduct.").

• Orange County LEO 2014-1 (2014) (holding that a lawyer had not acted unethically in filing a pleading prepared by an out-of-state lawyer; addressing the following situation; "California Counsel of Record is a member of Law Firm. California Counsel of Record hires Out-of-State Lawyer to draft pleadings and other documents in connection with a case pending in California state court in which California Counsel of Record and Law Firm

85

35951124_10 UPL, MDP and MJP (Defining What Lawyers Do and McGuireWoods LLP Where They Can Do It): Part II T. Spahn (1/1/19) Hypotheticals and Analyses

represent Client. Out-of-State Lawyer is licensed in and resides in a state other than California, and is not admitted to practice law in California or otherwise authorized to practice in California temporarily. Nor is Out-of- State Lawyer associated with Law Firm. The documents, which are submitted to the California court, are reviewed and signed by California Counsel of Record, but do not in any way indicate Out-of-State Lawyer participated in drafting the documents."; "In our opinion, Out-of-State Lawyer is not violating the duty of candor to the court or the duty of honesty under California law because Out-of-State Lawyer has made no affirmative statement to the court or in any other way misled the court or opposing counsel. As for California Counsel of Record and Law Firm, we conclude that they need not disclose the involvement of Out-of-State Lawyer to the court unless a fee request is made because the involvement of Out-of-State Lawyer is not material to the court."; "Whether Out-of-State Lawyer is engaging in the unauthorized practice of law in California, and whether California Counsel of Record and Law Firm are aiding in his unauthorized practice of law, will depend on the specific factual situation and the scope of Out-of-State Lawyer's involvement. The analysis will consider the significance of the contact with the client and the activity in the state, which may occur in person or remotely through the use of technology, as discussed in Birbrower [Birbrower, Montalbano, Condon & Frank, P.C. v. Superior Court, 949 P.2d 1 (Cal. 1998)]. Assuming there is no significant involvement with Client, however, the mere act of Out-of-State Lawyer's ghostwriting a document for California Counsel of Record is not likely to constitute the unauthorized practice of law in California."; also addressing a second scenario; "An independent Contract Lawyer not associated with Law Firm, who is licensed to practice law in California, also drafts documents for California Counsel of Record. Contract Lawyer's role in preparing the documents is not revealed in the documents, which are submitted to a California state court under the name of California Counsel of Record and Law Firm. The Client is not aware of the involvement of contract lawyer."; "In our opinion, Contract Lawyer is not violating his duty of candor to the court or the duty of honesty under California law and ethics rules because the Contract Lawyer has made no affirmative statement to the court. California Counsel of Record has not violated the duty of candor as long as he has not misled the court regarding the lawyer or lawyers working on the case. Depending on the scope of Contract Lawyer's services, however, his involvement may be a material development that should be disclosed to Client.").

• Iowa LEO 13-02 (8/6/13) ("Unfortunately, some have come to view the role of the 'local' counsel as subservant to the non-Iowa lawyer who will, in effect, be the 'lead' counsel. While some states may view the role of the 'local' lawyer to be nothing more than a mail drop, Iowa does not. In Iowa there is no such thing as 'local counsel' within the meaning ascribed above. At best, the

86

35951124_10 UPL, MDP and MJP (Defining What Lawyers Do and McGuireWoods LLP Where They Can Do It): Part II T. Spahn (1/1/19) Hypotheticals and Analyses

relationship is one of 'co-counsel' with the Iowa lawyer having certain responsibilities to both the client and the court."; "In Iowa the term 'local' counsel must be understood in the context of Iowa Supreme Court Rule 31 pertaining to professional regulation and the Iowa Rules of Professional Conduct. Lawyers practicing in a limited capacity as pro hac vice do so only by operation of the fact of their sponsorship by a fully admitted Iowa lawyer who must, at all times, actively participate and take responsibility for the conduct of the matter before the Iowa court of agency. The Iowa lawyer may, after exercising due diligence, exercise discretion in determing the level of participation required by Iowa Sup. Ct. R. 31.14(3). However in doing so the Iowa lawyer should be guided by the overriding ethical obligations to the client.").

• In re Anonymous, 932 N.E.2d 1247, 1249, 1249-50 (Ind. 2010) (issuing a private reprimand against an Indiana lawyer who assisted a Kentucky lawyer in Indiana litigation, without complying with all of Indiana's pro hac rules; "The participation of Indiana co-counsel in the temporary admission process is of vital importance to this Court's ability to supervise out-of-state attorneys practicing in this state. This is no minor or perfunctory duty. Not all attorneys seeking temporary admission will be granted the privilege of practicing in Indiana. See Matter of Fieger, 887 N.E.2d 87 (Ind. 2008) (two-year bar on applying for temporary admission imposed for misrepresentations in petition for temporary admission). Thus, an out-of-state attorney may seek temporary admission in an Indiana court only if a member of the bar of this state has appeared and agreed to act as co-counsel. See Admis. Disc. R. 3(2)(a)(1). Indiana co-counsel must co-sign the out-of-state attorney's petition for temporary admission, which must include the attorney's temporary admission number and a receipt showing that the attorney has paid the temporary admission fee. See Admis. Disc. R. 3(2)(a)(4). Indiana co-counsel must also sign all briefs, papers and pleadings in the case and is jointly responsible for them. See Admis. Disc. R. 3(2)(d). This signature constitutes a certificate that, to the best of co-counsel's knowledge, information and belief, there is good ground to support the document. See id. Indiana co-counsel is subject to discipline if the out-of-state attorney fails to satisfy the requirements of the rule governing temporary admission." (emphasis added); "In the current case, Kentucky lawyer Redelberger ignored the rules setting forth the procedures for obtaining temporary admission in Indiana. Respondent failed to discharge his responsibility to ensure that Redelberger was properly admitted to practice in the state before signing an appearance with him and allowing him to appear in court and take depositions in Indiana. Respondent abdicated his joint responsibility for documents in the case by allowing Redelberger alone to sign answers to interrogatories. When notified by the trial court that Redelberger was not admitted to practice in Indiana, Respondent's inadequate response was to give Redelberger a copy of the applicable rule, which apparently neither

87

35951124_10 UPL, MDP and MJP (Defining What Lawyers Do and McGuireWoods LLP Where They Can Do It): Part II T. Spahn (1/1/19) Hypotheticals and Analyses

Redelberger nor Respondent had consulted up to that point. As a result, this Court had no notice that Redelberger was practicing law in Indiana.").

Although such rules are "justified" by the courts' desire to have a knowledgeable local

lawyer protecting the client, one cannot help but suspect that the courts are seeking to

protect job opportunities for local lawyers.

Fourth, several courts have held that such local counsel must have a real physical office – not a virtual office – in the pertinent jurisdiction.

• Suevon Lee, Edelson DQ'd In False Ad Row For Using 'Virtual Office,' Law360, Apr. 20, 2016 ("A California federal judge on Wednesday disqualified Edelson PC as counsel in a putative class action accusing TP- Link USA Corporation of deceptive marketing of its network adapters, saying an out-of-state attorney's use of a virtual office address amounted to bad faith conduct."; "United States District Judge Percy Anderson also imposed sanctions against the leading Chicago-based technology class action litigation firm in the amount of $5,265 and denied a partner’s pro hac vice application to the Central District of California in the false advertising suit brought against the California-based United States subsidiary of China’s network adapter manufacturer, TP-Link Technologies Co."; "Judge Anderson's ruling cast a strict gaze on Edelson's long-held practice of applying for out-of-state court admission by using a so-called 'virtual office' address for designated local counsel, in light of the fact that the court considering such applications cannot discern from the address alone if it's for a bona fide office or not."; "'Edelson has abused this blind spot for more than six years, securing multiple pro hac vice admissions in flagrant violation of the court’s local rules,' Judge Anderson wrote in a nine-page order dated April 19."; "In this instance, Chicago-based Edelson partner Ari J. Scharg applied to appear pro hac vice in the Central District -- which includes Los Angeles -- designating an associate, Thomas Logan, as his local counsel. But attorneys for TP-Link USA filed an objection to that application last month, saying Logan lives in Chicago, practices out of the firm's San Francisco and Chicago offices, and that his listed office address in the Central District is actually for a virtual office in a downtown Los Angeles office building operated by Regus Business Center, which provides mail-and call-forwarding services and in-person meeting space."; "TP-Link's counsel at LTL Attorneys LLP also referenced the number of times Scharg had been admitted to practice in California on a pro hac vice basis -- 28 times over the last six years, with nine of those cases in the Central District."; "Over the last six years, Judge Anderson’s ruling said, Edelson has maintained a virtual office in the Central District -- in Santa Margarita, California, from January

88

35951124_10 UPL, MDP and MJP (Defining What Lawyers Do and McGuireWoods LLP Where They Can Do It): Part II T. Spahn (1/1/19) Hypotheticals and Analyses

2010 to January 2014 and in the downtown Los Angeles location since January 2014 to present."; "While the Chicago plaintiffs' firm, which frequently litigates cases in California due to its tech industry-geared focus, argued that the Central District's local rules permit use of a 'virtual office' by designated local counsel, the judge said the law firm failed to establish it could satisfy the rules' additional prong that this local counsel, through use of the virtual office, is someone with whom opposing counsel can 'readily communicate regarding the conduct of the case and upon whom documents may be served.'"; "The judge pointed out how Edelson 'readily admits that no Edelson attorney or staff operate from the Los Angeles virtual office.'"; "'Thus, it appears that the Los Angeles virtual office is not ‘maintain[ed] . . . for the practice of law’ so much as it is maintained for the purpose of facilitating pro hac vice admissions for Edelson attorneys,' the judge held." (emphasis added)).

• Arroyo v. TP-Link USA Corp., No. CV 16-1044 PA (KKx), 2016 U.S. Dist. LEXIS 38952, at *2, *2-3, *3-4, *4-5 (C.D. Cal. Mar. 24, 2016) (denying lawyer's pro hac vice motion and issuing a show-cause order why the lawyer should not be sanctioned for improperly providing a required local counsel's address on what turned out to be a "virtual office"; "The Local Rules for the Central District of California govern who may appear before the Court pro hac vice."; "An attorney is disqualified under L.R. 83-2.1.3.2 if he or she: '(a) resides in California; (b) is regularly employed in California; or (c) is regularly engaged in business, professional, or other similar activities in California.' L.R. 83-2.1.3.2. Applicants seeking to appear pro hac vice must also designate local counsel pursuant to Local Rule 83-2.1.3.4, which provides: 'Every attorney seeking to appear pro hac vice must designate as Local Counsel an attorney with whom the Court and opposing counsel may readily communicate regarding the conduct of the case and upon whom documents may be served. An attorney may be designated as Local Counsel only if he or she: (1) is a member of the Bar of this court; and (2) maintains an office within the District.' L.R. 83-1.2.1.3.2."; "[O]n March 18, 2016, Ari J. Scharg filed an Application to Appear Pro Hac Vice (Docket No. 119), which also designated Mr. Logan as local counsel. In this application, Mr. Logan represented that his office was located at 555 West Fifth Street, 31st Floor, Los Angeles, California 90013. Additionally, Mr. Scharg declared, under penalty of perjury, that he had designated local counsel who 'maintains an office in the Central District of California for the practice of law.' Defendant's Opposition points out the inconsistency in the location of Mr. Logan's office, and asserts that the Los Angeles office is only a 'virtual office' which does not comply with the Local Rule's requirements."; "The Court therefore orders Plaintiff's counsel to show cause, in writing, no later than April 4, 2016, why they should not be sanctioned for abusing the pro hac vice application process. Plaintiff's response to this Order to Show Cause must include: (1) a declaration under penalty of perjury from Mr. Logan as to how many

89

35951124_10 UPL, MDP and MJP (Defining What Lawyers Do and McGuireWoods LLP Where They Can Do It): Part II T. Spahn (1/1/19) Hypotheticals and Analyses

hours per week he physically spends in the Los Angeles office; how many Edelson employees, including other attorneys, are physically present at the Los Angeles office; and the city in which Mr. Logan maintains his permanent residence; [and] whether he maintains a residence in Los Angeles . . . . The Court warns that potential sanctions include the imposition of monetary sanctions, including Defendant's attorneys' fees incurred in bringing this matter before the Court, as well as the disqualification of any Edelson attorney from serving as either Plaintiff's individual counsel or as class counsel for the putative class.").

Fifth, such lawyers generally must also disclose any past or pending disciplinary actions against them.

• Martin Bricketto, NJ High Court Reprimands Atty In Legal Malpratice Row, Law360, July 1, 2013 ("The New Jersey Supreme Court has reprimanded a Roper & Twardowsky LLC lawyer based on a grievance pursued by Rivkin Radler LLP, the attorney's foe in bitter malpractice litigation accusing a Rivkin client, Leeds Morelli & Brown PC, of forsaking employment claims for commercial bribes. Rivkin had tapped Pashman Stein co-founder Michael Stein to file the grievance against Kenneth Thyne, and the state Disciplinary Review Board (DRB) in March found that misrepresentations and omissions by Thyne in an application for admission to the United States Court of Appeals for the Second Circuit [pro hac vice] had violated state conduct rules. Another Pasham Stein co-founder, Louis Pashman, chairs the DRB, though he recused himself from the case. In an order posted Monday, the high court backed the DRB's call for a reprimand. The board decided on that discipline after finding that Thyne had improperly held himself out to be in good legal standing in Minnesota and withheld information about a vacated pro hac vice admission in Colorado, criminal contempt proceedings and grievances against him. 'Respondent's arguments that he forgot that he was not in good standing or that he misinterpreted the questions on the application for admission are specious and strain credulity. Respondent was clearly an able attorney with 20 years of experience,' the DRB decision said.").

• State ex rel. Ind. Sup. Ct. Disciplinary Comm'n v. Farmer, 978 N.E.2d 409, 414 (Ind. 2012) (holding that the Indiana Supreme Court Disciplinary Commission could not obtain an injunction prohibiting an Ohio lawyer from occasionally representing Indiana clients; explaining that the Indiana lawyer (Farmer) represented an Indiana criminal client for three years; "The Commission's other argument with regard to this phase is that Farmer could not have reasonably expected to be authorized to appear in Ivy's post-conviction case because, at least beginning in December 2004, 'Farmer had no reasonable belief that he could gain temporary admission to practice

90

35951124_10 UPL, MDP and MJP (Defining What Lawyers Do and McGuireWoods LLP Where They Can Do It): Part II T. Spahn (1/1/19) Hypotheticals and Analyses

in the State of Indiana due to a pending disciplinary proceeding in Ohio.'" (internal citation omitted); "This argument, too, fails. A temporary admission to an out-of-state lawyer pursuant to Admission and Discipline Rule 3(2) is within the discretion of the trial court. . . . Nothing in this temporary admission rule per se disqualified Farmer from seeking admission in Ivy's post-conviction case simply because a disciplinary proceeding was pending against Farmer in Ohio." (emphasis added)).

Sixth, most bars limit the number of times a lawyer can be admitted pro hac in a defined period.

• In re Amendments to Rules Regulating Fla. Bar, 101 So. 3d 807, 812 (Fla. 2012) (explaining that new Florida rule 1.310(a)(2) involves pro hac admissions; "This rule does not prohibit a non-Florida lawyer from participating in more than 3 cases during any 365-day period; instead, it prohibits a non-Florida lawyer from making an initial or first appearance in more than 3 cases during any 365-day period.").

• D.C. Ct. App. R. 49 ("(a) No person shall engage in the practice of law in the District of Columbia or in any manner hold out as authorized or competent to practice law in the District of Columbia unless enrolled as an active member of the District of Columbia Bar, except as otherwise permitted by these Rules."; recognizing exception (c)(7) "[p]roviding legal services in or reasonably related to a pending or potential proceeding in a court of the District of Columbia, if the person has been or reasonably expects to be admitted pro hac vice, provided: (i) . . . No person may apply for admission pro hac vice in more than five (5) cases pending in the courts of the District of Columbia per calendar year, except for exceptional cause shown to the court." (emphasis added); also recognizing exception (c)(8): "Practicing law from a principal office located in the District of Columbia, while an active member in good standing of the highest court of a state or territory, and while not disbarred or suspended for disciplinary reasons or after resignation with charges pending in any jurisdiction or court, under the direct supervision of an enrolled, active member of the District of Columbia Bar, for one period not to exceed 360 days from the commencement of such practice, during pendency of a person's first application for admission to the District of Columbia Bar; provided that the practitioner has submitted the application for admission within ninety (90) days of commencing practice in the District of Columbia, that the District of Columbia Bar member takes responsibility for the quality of the work and complaints concerning the services, that the practitioner or the District of Columbia Bar member gives notice to the public of the member's supervision and the practitioner's bar status, and that the practitioner is admitted pro hac vice to the extent he or she provides legal services in the courts of the District of Columbia.").

91

35951124_10 UPL, MDP and MJP (Defining What Lawyers Do and McGuireWoods LLP Where They Can Do It): Part II T. Spahn (1/1/19) Hypotheticals and Analyses

• Golba v. Dick's Sporting Goods, Inc., 190 Cal. Rptr. 3d 337, 340 (Cal. Ct. App. 2015) (confirming the denial of pro hac application by a lawyer who had been admitted pro hac twelve times in eleven months to a California court; "A fundamental principle of California law, enshrined in the State Bar Act (Bus. & Prof. Code, § 6000 et seq.), is that no person may 'practice law in California' unless that person is an active member of the State Bar. (Bus. & Prof. Code § 6125 (section 6125).) As a corollary principle, no person may recover compensation for practicing law 'in California' unless that person was a member of the State Bar or admitted pro hac vice at the time the services were performed, or the legal services fall within an exception."; "In this case, those principles lead us to conclude the trial court did not err by denying recovery of attorney fees for work performed by out-of-state counsel who represented the named plaintiff in a class action in California but who had not been admitted pro hac vice. We affirm the trial court's order awarding $11,000 in attorney fees and costs out of the $210,000 sought as part of a class action settlement. We also affirm the trial court's decision to reduce the amount of the plaintiff incentive award.").

• Whiteside v. Empire Plaza, LLC, Civ. A. No. 14-3062 (SRC) (CLW), 2014 U.S. Dist. LEXIS 151860, at *3, *4, *6-7, *7, *8-9 (D.N.J. Oct. 24, 2014) (reversing a magistrate judge's rejection of a Florida lawyer's pro-hac admission application because the administrative judge concluded that the lawyer had appeared too many times pro hac in New Jersey; "The District of New Jersey allows for pro hac vice admission."; "An earlier version of the rule had provided that an attorney could only be admitted pro hac vice three times annually. In 1984, however, the District revised the rule to eliminate that limitation, and it now contains no numerical cap on the number of times an attorney may be admitted pro hac vice."; "The only potential limitation, then, is the recognition that Mr. Fuller has appeared many times in our courts. Under the old version of the pro hac vice rule, Mr. Fuller would have far exceeded the three appearances permitted. Yet the District of New Jersey revised the rule in 1984 to remove that numerical cap."; "Defendant argues that Mr. Fuller should not be allowed to use the pro hac vice procedures to substitute full admission to the New Jersey Bar. While that expresses an intuitive and understandable concern, the Court cannot endorse it here. There are three reasons for that."; "First, Mr. Fuller does not appear to actually be engaged in the general practice of law in New Jersey. He has no office in this State; he has not solicited clients here; he has not appeared in State courts, and has pursued only one kind of filing -- those pertaining to compliance with civil rights provisions like those in the ADA. Such a particularized practice is different from setting up shop in New Jersey and handing out a shingle."; "Second, more fundamentally, District Courts are national institutions intended to obviate concerns about provincial and local biases."; "Third, even if the Court were to endorse the principle that an outside lawyer cannot appear too often in this District, that limitation could

92

35951124_10 UPL, MDP and MJP (Defining What Lawyers Do and McGuireWoods LLP Where They Can Do It): Part II T. Spahn (1/1/19) Hypotheticals and Analyses

not be enforced on an ad hoc basis. Exactly how many pro hac vice appearances would be permitted? That which strikes one judge as an acceptable pro hac vice admission could strike another as one-time-too many. Such an approach would invite arbitrary results. Without a consistent, uniformly applied limit, attorneys would have no way to predict whether they would be admitted in a case, and those who were denied admission would not know how long their ineligibility would last. This District is authorized to revise its rules to implement a numerical cap if it chooses to, but that has not taken place.").

Presumably, the purpose of these limitations is to avoid a situation in which a lawyer essentially establishes an improper "continuous presence in another state."

Some states also limit the number of pro hac appearances by a law firm's

lawyers during a defined period of time. This type of collective limitation occasionally

causes problems for large law firms. Unless they monitor each of their lawyer's pro hac

admissions, the law firm might find itself precluded from taking a major case in another

state -- because the firm's lawyers have used up the law firm's "quota" of pro hac

admissions on earlier minor matters in that state.

Seventh, courts possess the power to deny or to revoke an out-of-state lawyer's

pro hac admission for some misconduct.

• Tri-Cities Holdings, LLC v. Tenn. Health Servs. & Dev. Agency, No. M2015- 00058-COA-R3-CV, 2016 Tenn. App. LEXIS 135, at *8, *8-9, *28 (Tenn. Ct. App. Feb. 22, 2016) (upholding an administrative law judge's revocation of a Georgia lawyer's pro hac admission, based on the lawyer's threat to sue the administrative law judge; quoting the lawyer's pleading: "'. . . [I]f this tribunal does 'take HSDA's bait' and takes any action to decide this appeal before a federal court or DOJ has spoken on this case, including scheduling a hearing, Petitioner respectfully indicates that it will have no choice but to join Your Honor, in an official capacity, and this tribunal, as defendants in the pending federal court action.'" (internal citation omitted); explaining the aftermath of the pleading; "The next day, the AJ [Administrative Judge] issued an order revoking Mr. Dunlap's [plaintiff’s lawyer] permission to appear pro hac vice."; "On the basis of the factual findings discussed above, the AJ concluded that Mr. Dunlap breached the conditions on which he was granted pro hac vice admission and did not conduct himself in conformity

93

35951124_10 UPL, MDP and MJP (Defining What Lawyers Do and McGuireWoods LLP Where They Can Do It): Part II T. Spahn (1/1/19) Hypotheticals and Analyses

with professional standards because: he made an 'unveiled threat that the Administrative Judge and the tribunal will face an ADA enforcement action by the Department of Justice should the Administrative Judge fail to provide the requested relief; he misrepresented to the tribunal the status of the federal litigation and used the misrepresentation to attempt to coerce a decision in favor of his client without the benefit of a hearing; his coercion and misrepresentation [we]re 'a flagrant attempt to improperly influence a judge in violation of Rules 3.3, 3.5, and 8.4 of the Tennessee Rules of Professional Conduct, as well as Tenn. Code Ann. § 39-14-112'; and he expressed contempt for the tribunal and impeded a resolution of the CON appeal.").

• Cruz-Aponte v. Caribbean Petroleum Corp., 123 F. Supp. 3d 276, 279, 279- 80, 280, 280-81, 271 (D.P.R. 2015) (holding that a lawyer's reference to another lawyer's possible menopause symptoms justified a monetary sanction but not revocation of pro hac vice admission; "Ms. Monserrate contends that Mr. Salas's [Lawyer accused of the misconduct] comment [during a deposition], 'You're not getting menopause, I hope,' was disparaging and discriminatory, and that it 'humiliated, embarrassed, and demeaned' her."; "Mr. Salas insists that he made the comment 'out of concern about Ms. Monserrate's medical condition.'. . . He explains that future depositions were scheduled to take place in the same room and that he knows 'that a hot room is a trigger for hot flashes in women who are going through menopause.'" (internal citation omitted); "The Court unequivocally rejects Mr. Sala's post hoc explanation. If Mr. Salas was genuinely concerned that Ms. Monserrate had a 'medical condition' triggered by the room's temperature, then he would have asked Ms. Monserrate in a more private setting and in a more respectful way whether there was anything he could do to alleviate her symptoms. Mr. Salas instead chose to tell Ms. Monserrate in the presence of fourteen other attorneys, eleven of whom were male, that he hopes that she is not menopausal."; "The public nature of Mr. Salas's comment combined with the personal and private nature of menopause leads the Court to conclude that the comment was made to embarrass Ms. Monserrate and was not intended to serve any other purpose. This is a clear violation of Model Rule 4.4."; "Discriminatory comments like this undoubtedly occur on a daily basis in the legal profession and are routinely swept under the rug. But the concealment does not diminish the effect. An ABA report published this year, for example, identified 'inappropriate or stereotypical comments' directed at female attorneys by opposing counsel as one of the causes of the marked underrepresentation of women in lead trial attorney roles. Discriminatory conduct on the part of an attorney is 'palpably adverse to the goals of justice and the legal profession.'. . . When an attorney engages in discriminatory behavior, it reflects not only on the attorney's lack of professionalism, but also tarnishes the image of the entire profession and disgraces our system of justice." (footnote omitted); "Having determined that Mr. Salas committed professional

94

35951124_10 UPL, MDP and MJP (Defining What Lawyers Do and McGuireWoods LLP Where They Can Do It): Part II T. Spahn (1/1/19) Hypotheticals and Analyses

misconduct in a proceeding pending before the Court, the Court now discusses whether sanctions are warranted, and if so, whether revocation of Mr. Salas's pro hac vice admission, as requested by Ms. Monserrate, is appropriate."; "The fact that Mr. Salas's improper comment was an isolated incident mitigates his misconduct. In other cases, including those cited in Ms. Monserrate's motion, where male attorneys were sanctioned for discriminatory comments made to female attorneys, the courts found repeated misconduct that cumulatively warranted sanctions. See Mullaney v. Aude, 126 Md. App. 639, 644-45, 659, 730 A.2d 759 (1999) (affirming protective order and attorney's fees sanction where male attorney made sexist remark to female deponent, addressed female attorney as 'babe,' and said calling her 'babe' was better than calling her a 'bimbo' during deposition); In re Valcarel Mulero, 142 D.P.R. 41, 1996 Juris P.R. 154 (1996) (suspending male attorney from practice for period of three months for referring to female attorney as a 'crazy chicken' and 'girl,' repeatedly raising his voice, and constantly interrupting the judge during a court hearing); Principe, 586 N.Y.S. 2d at 184-88, 191 (sanctioning male attorney in the form of attorney's fees for calling female attorney 'little lady,' 'little mouse,' and 'little girl' repeatedly during deposition); cf. Laddcap Value Partners, LP v. Lowenstein Sandler P.C., 18 Misc. 3d 1130[A], 859 N.Y.S. 2d 895, 2007 NY Slip Op 52538[U], 2007 WL 4901555, at *2-7 [N.Y. Sup. Ct. 2007] (ordering referee supervision of future depositions after male attorney addressed female attorney as 'dear,' 'hon,' and 'sorry girl,' said she had a 'cute little thing going on,' and asked why she was not wearing her wedding ring during deposition)."; "Further mitigating the misconduct are Mr. Salas's immediate and subsequent apologies. Mr. Salas attempted to apologize to Ms. Monserrate during a break in the deposition. . . . At the conclusion of the deposition, Mr. Salas apologized to Ms. Monserrate on the record and acknowledged that his comment was improper. . . . He also apologized to the Court when he responded to Ms. Monserrate's motion."; "Given these mitigating circumstances, the Court finds that the harsh sanction of revocation of Mr. Salas's pro hac vice admission is not warranted. Nonetheless, Mr. Salas's comment intended to humiliate Ms. Monserrate on the basis of her age and gender. This conduct is adverse to the goals of justice and cannot be permitted to find a safe haven in the practice of law. The Court therefore finds that the following sanctions are warranted. First, to ensure that he bears some of the burden of the costs of bringing his discriminatory conduct to light, Mr. Salas should pay Ms. Monserrate reasonable attorney's fees for bringing the motion. Second, Mr. Salas should complete a continuing legal education course on attorney professionalism and professional conduct.").

• Plante v. Stack, 109 A.3d 846 (R.I. 2015) (finding that a lawyer's deposition tactics were improper, but did not justify the trial court's revocation of the lawyer's pro hac admission).

95

35951124_10 UPL, MDP and MJP (Defining What Lawyers Do and McGuireWoods LLP Where They Can Do It): Part II T. Spahn (1/1/19) Hypotheticals and Analyses

• Ryan v. Astra Tech, Inc., 772 F.3d 50, 52, 54, 55, 59, 63 (1st Cir. 2014) (affirming the trial court's revocation of a lawyer's pro hac admission; "Attorney Jeffrey Ryan ('Ryan') appeals from the district court's revocation of his permission to practice pro hac vice for the plaintiff in the underlying lawsuit that gave rise to these proceedings. The district court revoked Ryan's pro hac vice admission after finding that he lied to the court about attempting to interfere with the deposition of his client." (footnote ommittd); finding that the lawyer lied; "In short, the district court found as a matter of fact that Ryan attempted to communicate surreptitiously with his client while a question was pending at a deposition, that Ryan manufactured false evidence, and that Ryan lied to the court. The court put these findings in the context of Ryan's prior conduct during the case. This conduct included adding a jury demand to a filed amended complaint after obtaining opposing counsel's assent and the court's permission to file an amended complaint without a jury demand, disobeying a court order about discovery, attaching personal and irrelevant information as exhibits to a hearing request, and threatening to embarrass a defendant's wife. The court ordered Ryan to pay monetary sanctions consisting of fees and costs for litigating the motion for sanctions, the court reporter's time, and videotaping the plaintiff's and all subsequent depositions. The court also directed Ryan to show cause why it should not revoke his pro hac vice admission for flagrant violations of the Massachusetts Rules of Professional Conduct."; "At a subsequent January 16, 2013, status conference, Ryan, now represented by his own attorney, voiced to the district court due process concerns about the revocation of his pro hac vice admission. In particular, Ryan asked the court either to grant another evidentiary hearing or to strike the sanctions because of the potential adverse consequences the revocation could have for Ryan's legal career."; "That brings us to Ryan's more general challenge that the district court violated his right to procedural due process under the Fifth Amendment. In particular, Ryan claims that he was entitled to prior notice, an opportunity to be heard, and an evidentiary hearing. We have not yet decided whether an attorney's pro hac vice admission is a property interest protected by due process, and, if so, what procedural safeguards due process requires before the district court may revoke that admission. We need not answer these questions today, however, because the district court afforded Ryan each of the procedural protections to which he claims he was entitled."; "Reduced to its nub, this is a simple, but important case. With notice, an opportunity to be heard, and ample, very convincing evidentiary support, the district court found that Ryan falsified evidence and lied point blank with premeditation to the court. Rejecting no procedure timely requested by Ryan or required by due process, and after further notice and opportunity to be heard, the court decided to withdraw its permission that Ryan could appear as counsel in this case. Anyone who thinks it important that lawyers not lie to judges would be surprised if the court had done otherwise. We therefore affirm the orders revoking Ryan's pro se vice admission and imposing monetary sanctions."). 96

35951124_10 UPL, MDP and MJP (Defining What Lawyers Do and McGuireWoods LLP Where They Can Do It): Part II T. Spahn (1/1/19) Hypotheticals and Analyses

• Mary Pat Gallagher, N.J. Judge Removes Pro Hac Vice Lawyer for Signing Court Papers, N.J. Law J., July 24, 2014 ("In a case that illustrates the potential pitfalls out-of-state attorneys face when practicing pro hac vice, a New York lawyer defending a New Jersey lawsuit was removed from the case after he signed papers that were to be signed only by a lawyer licensed to practice in New Jersey."; "Noah Katz of Lester Schwab Katz & Dwyer was removed from Pender v. Beiter, a personal injury case, by Hudson County Presiding Civil Judge Mary Costello on July 11."; "Lester Schwab has an office in Millburn, New Jersey, but, according to its website, all of its lawyers work out of its Manhattan location."; "Katz was admitted pro hac vice on the motion of attorney Barry Semel-Weinstein, who is admitted in New Jersey and subsequently left Lester Schwab. He is now at D'Amato & Lynch in New York."; "An April 19, 2013, order by Judge Martha Royster allowed Katz to appear in the Pender suit on behalf of defendants United Parcel Service and driver Pablo Francisco Berlanga Jr., citing a preexisting attorney-client relationship with those clients and Katz's good standing in New York."; "The order recited a variety of conditions, including compliance with New Jersey court and ethics rules and payment of the requisite fees."; "Royster's order required Katz to 'have all pleadings, briefs and other papers filed with the court signed by an attorney of record authorized to practice in New Jersey, who shall be responsible for them, the conduct of the litigation and the attorney admitted therein.'"; "In a July 7, 2014, letter to Costello, attorney Barry Epstein, who represents plaintiffs Rory and Michelle Pender, said Katz began signing papers, starting with the answer to the cross-claim, on May 8, 2013."; "Katz continued to do so, signing his name to discovery notices, a subpoena and notices of motion, Epstein, of The Epstein Law Firm in Rochelle Park, wrote in the letter. Katz responded in a July 8 letter to Costello, saying, 'I apologize for the inadvertence in signing pleadings and certifications in my own hand which were filed with the court.'"; "He expressed his intention to ask the court at an upcoming July 11 court date if he could refile the documents with signatures by a New Jersey lawyer.").

• Fein v. Chenault, 767 S.E.2d 766, 766 (Ga. Ct. App. 2014) (holding that a lawyer admitted pro hac could not directly appeal a lower court order restricting what actions the lawyer could undertake; "Florida attorney Frederick J. Fein and his client Continental Tire The Americas, LLC (Continental), filed a direct appeal of the trial court's order in this personal injury case finding that Fein violated the Georgia Rules of Professional Conduct but declining to revoke his pro hac vice status. Instead, the court restricted Fein's advocacy by designating local counsel as lead counsel for defendant Continental and directing Fein not to prepare or file pleadings, contact the court or its staff, or present argument or evidence at any future court proceedings. The court noted that Fein would be permitted to attend all future proceedings, confer with his client and local counsel, and sit at counsel table during court. Because this court lacks jurisdiction to consider this issue

97

35951124_10 UPL, MDP and MJP (Defining What Lawyers Do and McGuireWoods LLP Where They Can Do It): Part II T. Spahn (1/1/19) Hypotheticals and Analyses

on direct appeal while the case remains pending in the trial court, we must dismiss the appeal.").

• Mack Sperling, Business Court Refuses To Admit University Of Maryland's Lawyers On A Pro Hac Basis, North Carolina Business Litigation Report, July 3, 2014 ("The lawsuit filed by the Atlantic Coast Conference [ACC] against the University of Maryland continues to percolate in the North Carolina Business Court. But the University will have to proceed without its chosen attorneys, as the Court last week refused to admit them on a pro hac vice basis. The decision came in an Order in Atlantic Coast Conference v. University of Maryland."; "If you have forgotten about the ACC's lawsuit against the University of Maryland, it was filed by the ACC to recover the $50 million exit fee it says is due from the University upon its departure from the ACC to join the Big Ten Conference. The University disputes the validity of the exit fee, and has counterclaimed in very detailed claims for violation of antitrust laws and unfair competition."; "The Maryland University was represented by a lawyer from the Maryland Attorney General's office, and two lawyers from the Milwaukee firm Foley & Lardner. When these lawyers moved to be admitted pro hac, the ACC objected. It argued that the counsel from Foley & Lardner were in violation of Rule 1.7 of the North Carolina Rules of Professional Conduct due to their representation in other matters of Florida State, Virginia Tech, and the University of Virginia, members of the ACC, and the University of Louisville, which officially joined the ACC on July 1st."; "The ACC lawyers argued that the University of Maryland's lawyers were acting adversely to the interests of their other University clients. One of the comments to Rule 1.7 is favorable to the University's position. Comment 34 says that '[a] lawyer who represents a corporation or other organization does not, by virtue of that representation, necessarily represent any constituent or affiliated organization.'"; "But Comment 34 ends on a bad note for the University's argument. It says that this qualification does not apply if 'the lawyer's obligations to either the organizational client or the new client are likely to limit materially the lawyer's representation of the other client.'"; "The University's (former) counsel argued that they could ameliorate any conflict by hiring independent counsel to take the depositions of any institution which was a member of the ACC and represented by them. There is an American Bar Association (ABA) Opinion that supports this position, ABA Standing Committee on Ethics and Professional Responsibility, Formal Opinion 92-367, but it also says that if the 'conflict is clearly forseeable, then the solution, absent client consent, is clear enough: the prospective engagement must be declined.'"; "The Court refused to admit the University's attorneys on a pro hac basis, noting the lack of any conflict waiver, and stating that: '[w]hile it is true that a party's right to choose its own counsel is generally considered fundamental, 'an out-of-state attorney has no absolute right to practice law in another forum.'").

98

35951124_10 UPL, MDP and MJP (Defining What Lawyers Do and McGuireWoods LLP Where They Can Do It): Part II T. Spahn (1/1/19) Hypotheticals and Analyses

• YTC Dream Homes, Inc. v. DirectBuy, Inc., 30 N.E.3d 701, 702, 703 (Ind. 2014) (remanding to the trial court applications for pro hac vice admission by out-of-state lawyers, directing the trial court to use its discretion in analyzing "good cause for temporary admission"; agreeing with the Court of Appeals that a local court rule "does not create a presumption against pro hac vice admissions"; noting that the trial court had denied the pro hac vice admissions because they were qualified local lawyers; quoting the trial court's analysis, which the Indiana Supreme Court rejected: "'There is no doubt that the five [Attorneys] are eminently qualified, knowledgeable[,] and have a high level of competence in the area of franchise law.'"; "'[But there are] no less than seventeen licensed Indiana attorneys . . . that are members of the American Bar Association Forum on Franchising.'"; "'Even assuming that the plaintiffs have shown good cause to admit these attorneys because of their specialized skills, the pro hac vice petitioner must overcome the presumption under Lake County Local Rule 5(C) that an attorney not licensed in Indiana is not permitted to practice before it. . . . [T]he Court is not persuaded that the plaintiffs cannot locate attorneys licensed in the State of Indiana that have expertise in the field of franchise law.'"; "We agree with the Court of Appeals' conclusion that Local Rule 5(C) does not create a presumption against pro hac vice admissions.").

• Southerland v. Woo, 998 F. Supp. 2d 89, 91-92, 92 (E.D.N.Y. 2014) (denying pro hac vice admission for a lawyer seeking to handle the second trial before the court, following disputes between the lawyer and the judge in the first trial; "It follows from this that an attorney who is admitted pro hac vice must comport himself in the manner of attorneys who have standing admission to this Court. Having had Mr. King try this case before me previously, it is clear that Mr. King does not meet this standard. This Court has had no attorney before it in any case who has demonstrated the defiance, lack of respect, and unawareness of local practice and the Federal Rules of Civil Procedure as Mr. King demonstrated during the trial of this case."; "The record contains numerous examples but a few will make the point. First, Mr. King had difficulty with arriving on time for conferences or trial. That of course happens from time to time; when it does, attorneys apologize and the case goes on. Mr. King, however, refused to acknowledge his obligation to be on time."; noting that the transcript included the following comment by the lawyer: "'Your Honor, I can never assure anyone I won't be late, okay.'").

• Ford Motor Co. v. Young, No. A12A2335, 2013 Ga. App. LEXIS 522, at *1 (Ga. App. Ct. June 20, 2013) ("Attorneys Alan Thomas and Paul F. Malek appeal an order revoking their admission to practice as trial counsel pro hac vice in this product liability wrongful death case. They argue that their appeal is not moot, even though the underlying case has settled, and that they have standing to appeal, even though they were not parties below. They also argue that the trial court erred by considering the Georgia Rules of

99

35951124_10 UPL, MDP and MJP (Defining What Lawyers Do and McGuireWoods LLP Where They Can Do It): Part II T. Spahn (1/1/19) Hypotheticals and Analyses

Professional Conduct and by revoking their admissions. Finally, they argue that they were entitled to notice and a hearing before the court revoked their admissions. We agree with Thomas and Malek that the issue of the revocation of their admissions pro hac vice is not moot and that they have standing to appeal. We hold that the trial court properly considered the Georgia Rules of Professional Conduct. But we agree with Thomas and Malek that they were entitled to notice and an opportunity to be heard before the trial court revoked their admissions. We therefore vacate the trial court's order and remand the case for the court to allow the attorneys such due process.").

• Liebnow v. Boston Enters. Inc., 296 P.3d 108 (Colo. 2013) (denying the pro hac vice admission in Colorado of a plaintiff's law firm in a food poisoning case because defense counsel had spoken briefly to a lawyer in the plaintiff's law firm about the case).

• Siupa v. Astra Tech, Inc., Civ. A. No. 10-10525-LTS, 2012 U.S. Dist. LEXIS 178789, at *8-9 (D. Mass. Dec. 18, 2012) ("Consistent with such power, and with the discretionary nature of admission pursuant to Local Rule 83.5.3(b), courts have disqualified attorneys and revoked pro hac vice admission as a sanction for improper or unethical conduct. . . . There are, however, 'no national standards' on disqualification of pro hac vice counsel, whether and to what limited extent an attorney has a property right in retaining pro hac vice status after it has been granted, or what level of procedural due process is required before revocation of pro hac vice admission." (citation omitted)).

• Sisk v. Transylvania Cmty. Hosp., Inc., 695 S.E.2d 429, 434, 436, 431 (N.C. 2010) (approving a trial court's revocation of a pro hac vice admission of two Kentucky lawyers, although a Kentucky court found that the lawyers' interaction with another side's consulting expert did not amount to a knowing violation of the Kentucky ethics rules; explaining what the North Carolina Court of Appeals had concluded; "Apparently assuming that the predominant effect of the conduct would occur in Kentucky, the Court of Appeals held that because counsel's behavior did not violate the rules of that state, Rule 8.5 did not allow the conduct to be subject to discipline under the rules of North Carolina."; holding that the Court of Appeals had improperly analyzed the situation; "However, in focusing on the Rules of Professional Conduct, the Court of Appeals did not consider the trial court's independent inherent authority to discipline attorneys."; explaining that North Carolina courts can rely on their "inherent power" to revoke a pro hac admission; acknowledging that "[n]evertheless, in exercising its discretion, a trial court may consider the Rules of Professional Conduct when deciding whether to revoke pro hac vice status. Rule 4.3 of the North Carolina Revised Rules of Professional Conduct provides in pertinent part: 'In dealing on behalf of a client with a person who is not represented by counsel, a lawyer shall not: . . . (b) state or

100

35951124_10 UPL, MDP and MJP (Defining What Lawyers Do and McGuireWoods LLP Where They Can Do It): Part II T. Spahn (1/1/19) Hypotheticals and Analyses

imply that the lawyer is disinterested. When the lawyer knows or reasonably should know that the unrepresented person misunderstands the lawyer's role in the matter, the lawyer shall make reasonable efforts to correct the misunderstanding.'"; ultimately concluding that "the North Carolina Rules of Professional Conduct do not limit the trial court's discretion to revoke pro hac vice status. Because we find that the trial court did not abuse its discretion, we reverse the Court of Appeals.").

Although the exact contours are unclear, some courts recognize that such lawyers

admitted pro hac must be given at least "some modicum" of due process before losing

the privilege.

• Belue v. Leventhal, 640 F.3d 567, 577 (4th Cir. 2011) (reversing District Court Judge Ross Anderson's revocation of a pro hac admission after denying the out-of-state's lawyers' motion to disqualify Judge Anderson; finding that a district court judge cannot revoke a pro hac admission without at least "some modicum" of due process).

Eighth, a home-state lawyer might also face punishment for misconduct by an

out-of-state lawyer, based on the home state lawyer's failure to properly assure the out- of-state lawyer's compliance with ethics or court rules.

• In re Jindal, S. Ct. BA. Nos. 2018-0018 to 2018-0020, 2018 V.I. Supreme LEXIS 38, at *10 n.3, *13, *4-5, *2-3 (V.I. Nov. 29, 2018) (per curiam) (denying pro hac admission petitions of three Akin Gump lawyers after they sought fees for successfully representing their client in the Virgin Islands courts; acknowledging that their pro hac vice motions had been granted, but noting that they had never taken the necessary attorney oath and therefore were not authorized to practice in the Virgin Islands; explaining that their litigation adversary had asked for a stay of their request for attorneys fees because they had violated the Virgin Islands unauthorized practice of law regulations – but that the three lawyers had never responded to the unauthorized practice of law complaint; rejecting the lawyers' argument that they were practicing federal law (noting that the antitrust case involved Virgin Islands law, and that they could not ethically have practiced in the Virgin Island courts even if the case had involved federal law); also rejecting their argument that requiring pro hac admission in the Virgin Islands by "every attorney who works on a matter that 'touches' the Virgin Islands" would "'cripple cross-border transactions and litigation'"; warning that "[s]ince the underlying conduct may potentially warrant action beyond the denial of pro hac vice admission" (having earlier issued a show cause order demanding

101

35951124_10 UPL, MDP and MJP (Defining What Lawyers Do and McGuireWoods LLP Where They Can Do It): Part II T. Spahn (1/1/19) Hypotheticals and Analyses

that the lawyers show cause why they should not be referred "for further investigation and prosecution"), referring to two Virgin Islands bar disciplinary entities and the Virgin Islands Attorney General): (1) the three Akin Gump lawyers; (2) two other Virginia-based Akin Gump lawyers who were listed as having provided "legal services," but who were never admitted pro hac; (3) the Akin Gump law firm; and (4) Akin Gump's local counsel).

• Disciplinary Counsel v. Lorenzon, 978 N.E.2d 183, 184-85, 187(Ohio Oct. 16, 2012) (suspending for six month an Ohio lawyer who agreed to act as Ohio counsel for a Florida law firm, but essentially allowing a Florida law firm to use his electronic signature without his significant involvement in the debt collection efforts undertaken by the Florida firm in Ohio; "On September 15, 2008, Lorenzon entered into an 'of counsel' agreement with Consumer Law Group, P.A. ('CLG'), a Florida law firm that negotiates debt on behalf of consumers. The agreement provided that Lorenzo would be paid $1,000 annually to serve as local counsel for CLG and that he would execute a contract with each Ohio client. To facilitate the execution of the contracts, the agreement required Lorenzon to provide CLG with his electronic signature and Ohio attorney-registration number. The agreement specified that Lorenzon's only duty to Ohio clients was to engage in 'episodic phone calls where [he] may be needed to answer client questions from time to time' -- a duty that CLG estimated would require no more than three hours per year -- and expressly provided that Lorenzon would not be obligated to represent the clients in court or before any agency or panel."; "Lorenzon admitted that he had preapproved the blank form that was to be used for every Ohio client and acknowledged that the contract with CLG did not require his approval for every individual debt-negotiation contract. Thus, Lorenzon provided his electronic signature without retaining any appreciable means to control or supervise its use. His only justification for the arrangement was to save the time that it would otherwise have taken him to sign each individual contract, scan it, and send it back to CLG -- a process that he estimated would delay contract execution by a couple of hours. One panel member likened the process to giving CLG 'what amounts to your ATM identification and your pin number.'"; "Lorenzon's failure to properly supervise the use of his signature exposed him to unanticipated professional liability at the hands of an allegedly unscrupulous person. But more importantly, by loaning his good name without retaining adequate control over its use, he exposed vulnerable and unsuspecting Ohio customers to the questionable practices of an out-of-state debt-negotiation firm.").

• In re Anonymous, 932 N.E.2d 1247, 1249, 1249-50 (Ind. 2010) (issuing a private reprimand against an Indiana lawyer who assisted a Kentucky lawyer in Indiana litigation, without complying with all of Indiana's pro hac rules; "The participation of Indiana co-counsel in the temporary admission process is of vital importance to this Court's ability to supervise out-of-state attorneys

102

35951124_10 UPL, MDP and MJP (Defining What Lawyers Do and McGuireWoods LLP Where They Can Do It): Part II T. Spahn (1/1/19) Hypotheticals and Analyses

practicing in this state. This is no minor or perfunctory duty. Not all attorneys seeking temporary admission will be granted the privilege of practicing in Indiana. See Matter of Fieger, 887 N.E.2d 87 (Ind. 2008) (two-year bar on applying for temporary admission imposed for misrepresentations in petition for temporary admission). Thus, an out-of-state attorney may seek temporary admission in an Indiana court only if a member of the bar of this state has appeared and agreed to act as co-counsel. See Admis. Disc. R. 3(2)(a)(1). Indiana co-counsel must co-sign the out-of-state attorney's petition for temporary admission, which must include the attorney's temporary admission number and a receipt showing that the attorney has paid the temporary admission fee. See Admis. Disc. R. 3(2)(a)(4). Indiana co-counsel must also sign all briefs, papers and pleadings in the case and is jointly responsible for them. See Admis. Disc. R. 3(2)(d). This signature constitutes a certificate that, to the best of co-counsel's knowledge, information and belief, there is good ground to support the document. See id. Indiana co-counsel is subject to discipline if the out-of-state attorney fails to satisfy the requirements of the rule governing temporary admission."; "In the current case, Kentucky lawyer Redelberger ignored the rules setting forth the procedures for obtaining temporary admission in Indiana. Respondent failed to discharge his responsibility to ensure that Redelberger was properly admitted to practice in the state before signing an appearance with him and allowing him to appear in court and take depositions in Indiana. Respondent abdicated his joint responsibility for documents in the case by allowing Redelberger alone to sign answers to interrogatories. When notified by the trial court that Redelberger was not admitted to practice in Indiana, Respondent's inadequate response was to give Redelberger a copy of the applicable rule, which apparently neither Redelberger nor Respondent had consulted up to that point. As a result, this Court had no notice that Redelberger was practicing law in Indiana.").

Best Answer

The best answer to this hypothetical is YES (PROBABLY).

B 2/13, 3/17

103

35951124_10 UPL, MDP and MJP (Defining What Lawyers Do and McGuireWoods LLP Where They Can Do It): Part II T. Spahn (1/1/19) Hypotheticals and Analyses

Involvement in Pre-Litigation Activities

Hypothetical 8

You are exploring the possibility of representing your client in a lawsuit against one of its business competitors -- which you would plan to file in a neighboring state where you are not licensed. To prepare for the lawsuit, you need to interview several witnesses who live in that neighboring state. Based on the results of your interviews, your client might ask you to represent several of those witnesses, many of whom formerly worked for your client.

(a) In connection with pending or potential litigation, may you interview witnesses in a state in which you are not licensed to practice law?

YES (PROBABLY)

(b) In connection with pending or potential litigation, may you represent witnesses who live in states where you are not licensed to practice law?

YES (PROBABLY)

(c) May you represent a client based in that other state in trying to resolve the dispute – so long as you retain local counsel if litigation ensues?

MAYBE

Analysis

ABA Model Rule 5.5 allows lawyers admitted in a United States jurisdiction to practice law in another United States jurisdiction under certain conditions.

A lawyer admitted in another United States jurisdiction, and not disbarred or suspended from practice in any jurisdiction, may provide legal services on a temporary basis in this jurisdiction that:

(1) are undertaken in association with a lawyer who is admitted to practice in this jurisdiction and who actively participates in the matter;

104

35951124_10 UPL, MDP and MJP (Defining What Lawyers Do and McGuireWoods LLP Where They Can Do It): Part II T. Spahn (1/1/19) Hypotheticals and Analyses

(2) are in or reasonably related to a pending or potential proceeding before a tribunal in this or another jurisdiction, if the lawyer, or a person the lawyer is assisting, is authorized by law or order to appear in such proceeding or reasonably expects to be so authorized;

(3) are in or reasonably related to a pending or potential arbitration, mediation, or other alternative dispute resolution proceeding in this or another jurisdiction, if the services arise out of or are reasonably related to the lawyer's practice in a jurisdiction in which the lawyer is admitted to practice and are not services for which the forum requires pro hac vice admission; or

(4) are not within paragraphs (c)(2) or (c)(3) and arise out of or are reasonably related to the lawyer's practice in a jurisdiction in which the lawyer is admitted to practice.

ABA Model Rule 5.5(c) (emphasis added).

This hypothetical deals with the stage of the litigation process that comes before the actual court proceeding or formal discovery such as depositions.

(a) Both the ABA Model Rules and the Restatement take a very broad view of such pre-filing activities.

As long as a litigator undertakes such activities in another state on a "temporary basis," the ABA Model Rules permit such activities.

A lawyer admitted in another United States jurisdiction, and not disbarred or suspended from practice in any jurisdiction, may provide legal services on a temporary basis in this jurisdiction that . . . (1) are undertaken in association with a lawyer who is admitted to practice in this jurisdiction and who actively participates in the matter; (2) are in or reasonably related to a pending or potential proceeding before a tribunal in this or another jurisdiction, if the lawyer, or a person the lawyer is assisting, is authorized by the law or order to appear in such proceeding or reasonably expects to be so authorized.

ABA Model Rule 5.5 (c)(1)- (2) (emphases added).

105

35951124_10 UPL, MDP and MJP (Defining What Lawyers Do and McGuireWoods LLP Where They Can Do It): Part II T. Spahn (1/1/19) Hypotheticals and Analyses

This approach allows a remarkably broad range of activities that lawyers may ethically undertake in another state where they are not licensed. For instance, lawyers may undertake activities in another state if they are "reasonably related" to "potential" proceedings before a tribunal in that other state -- as long as the lawyer or another lawyer that she is "assisting" (presumably in the same firm or some other firm)

"reasonably expects" to be eventually authorized by a pro hac order to appear in the proceeding.

Comment [10] deals with this situation, and specifically approves lawyers' involvement in pre-litigation activities.

Paragraph (c)(2) also provides that a lawyer rendering services in this jurisdiction on a temporary basis does not violate the Rule when the lawyer engages in conduct in anticipation of a proceeding or hearing in a jurisdiction in which the lawyer is authorized to practice law or in which the lawyer reasonably expects to be admitted pro hac vice. Examples of such conduct include meetings with the client, interviews of potential witnesses, and the review of documents.

ABA Model Rule 5.5 cmt. [10] (emphasis added).

Comment [11] also approves this type of pre-litigation activities by a lawyer assisting the lawyer who expects to be admitted.

When a lawyer has been or reasonably expects to be admitted to appear before a court or administrative agency, paragraph (c)(2) also permits conduct by lawyers who are associated with that lawyer in the matter, but who do not expect to appear before the court or administrative agency. For example, subordinate lawyers may conduct research, review documents, and attend meetings with witnesses in support of the lawyer responsible for the litigation.

ABA Model Rule 5.5 cmt. [11] (emphases added).

106

35951124_10 UPL, MDP and MJP (Defining What Lawyers Do and McGuireWoods LLP Where They Can Do It): Part II T. Spahn (1/1/19) Hypotheticals and Analyses

The Restatement also takes a very broad approach.

A lawyer who is properly admitted to practice in a state with respect to litigation pending there, either generally or pro hac vice, may need to conduct proceedings and activities ancillary to the litigation in other states, such as counseling clients, dealing with co-counsel or opposing counsel, conducting depositions, examining documents, interviewing witnesses, negotiating settlements, and the like. Such activities incidental to permissible practice are appropriate and permissible.

Restatement (Third) of Law Governing Lawyers § 3 cmt. e (2000) (emphasis added).

Some states take a similarly broad view of this principle, and allow lawyers to take a fairly active role in pre-trial activity in a state where they are not licensed.

• State ex rel. Ind. Sup. Ct. Disciplinary Comm'n v. Farmer, 978 N.E.2d 409, 414 (Ind. 2012) (holding that the Indiana Supreme Court Disciplinary Commission could not obtain an injunction prohibiting an Ohio lawyer from occasionally representing Indiana clients; explaining that the Indiana lawyer (Farmer) represented an Indiana criminal client for three years; "[T]he charged conduct of Farmer involves occasional visits to Indiana for a single client in a single legal matter, not multiple matters or clients or any systematic or continuous presence in Indiana. Under these circumstances, the Court concludes that the Commission has failed to prove by clear and convincing evidence that Farmer's provision of legal service to Ivy was more than 'temporary.'").

(b) The very liberal ABA Model Rule approach does not explain whether

lawyers preparing for litigation may represent witnesses in other states. For instance,

ABA Model Rule 5.5 cmt. [11] mentions "attend[ing] meetings with witnesses" -- but not

representing witnesses. The Restatement takes the same approach.1

As long as the representation is temporary, such activity would seem acceptable.

Subject to the conflicts rules that might otherwise prohibit or discourage such

1 Restatement (Third) of Law Governing Lawyers § 3 cmt. e (2000). 107

35951124_10 UPL, MDP and MJP (Defining What Lawyers Do and McGuireWoods LLP Where They Can Do It): Part II T. Spahn (1/1/19) Hypotheticals and Analyses

representation, the ABA Model Rule and Restatement approach would presumably

permit such representations.

Of course, states taking a narrower view of the permissible lawyer activity

generally might prohibit such activity in states where the lawyer is not licensed.

(c) Although ABA Model Rule 5.5 takes a very broad approach to lawyers

preparing for litigation in which they expect themselves or others to eventually practice

before a tribunal, most state courts do not follow such a liberal approach.

States taking this narrow approach have sometimes severely punished out-of- state lawyers who have represented clients in claims that do not end up in active litigation.

Of course, if such activity becomes "systematic and continuous," a different and more demanding ethics rule applies. Not surprisingly, courts sometimes take a turf- protective view of that line. And most states take a restrictive view of out-of-state lawyer's even temporary practice in such situations – requiring some connection with

the lawyers' home state, and compliance with all the other requirements.

Delaware takes this turf-protection approach.

• Matt Fair, Pa. Justices Reinstate Atty After Unlicensed Del. Practice, Law360, June 29, 2015 ("The Pennsylvania Supreme Court on Friday reinstated the license of a New Jersey attorney who faced disciplinary action in 2013 after he was found to have practiced law in Delaware for four years without being admitted to the bar there."; "The justices in Pennsylvania had slapped Raymond Nadel with a one-year suspension last May based on an order by the Delaware Supreme Court finding that the New Jersey-based attorney had improperly practiced law in the state from April 2009 to September 2012."; "The justices did not comment on Nadel’s disciplinary proceedings in inking the reinstatement order."; "The Delaware Supreme Court suspended Nadel in December 2013 after rejecting a series of objections the attorney raised to findings made by the state’s Board on Professional Responsibility."; "Nadel was admitted to the bars in both 108

35951124_10 UPL, MDP and MJP (Defining What Lawyers Do and McGuireWoods LLP Where They Can Do It): Part II T. Spahn (1/1/19) Hypotheticals and Analyses

Pennsylvania and New Jersey in 1982."; "According to court records, Nadel’s unauthorized work in the state began after he was approached by a Delaware-based doctor who asked the attorney to assist one of his patients."; "The attorney went on to meet with more than 75 Delaware residents about potential auto accident claims. According to the Delaware Supreme Court's opinion, the accidents all occurred in Delaware and involved insurance policies governed by Delaware law. About half of Nadel’s meetings with the clients, the Delaware Supreme Court said, were held in Wilmington."; "'Nadel never filed a lawsuit in Delaware or made any representations to a Delaware court,' the state's Supreme Court said in their opinion suspending the attorney. 'Further, Nadel never advertised or actively solicited clients. Nor did he ever represent to a Delaware citizen that he was a member of the Delaware bar. But Nadel does admit that by meeting with his Delaware clients in Delaware, he could have unintentionally created the impression that he was licensed to practice law in Delaware.'"; "The state's Supreme Court said that the Delaware-based clients represented upwards of 15 percent of his practice."; "Nadel said that he was not aware at the time that Delaware rules of professional conduct barred attorneys from representing clients in pre-litigation matters without being licensed in the state.").

• Samuel C. Stretton, Pennsylvania Lawyers Should Be Cautious About Offering Legal Advice in Delaware, Legal Intelligencer (Online), Sept. 9, 2014) ("Is it permissible for a Pennsylvania lawyer to represent people in Delaware if the lawyer has local counsel or an office with a Delaware lawyer in it?"; "Any Pennsylvania lawyer who wants to provide legal advice in Delaware, even with local counsel or an office that has a Delaware lawyer as a paid employee, should be very cautious. Unless a Pennsylvania lawyer is moved in pro hac vice or sits for the bar exam and passes it, there is an excellent chance a lawyer practicing down there with some regularity, even though there are no court appearances, will receive professional discipline."; "Delaware's disciplinary system is extremely vigorous in tracking down and prosecuting Pennsylvania lawyers who are not licensed in Delaware, even though their activities seem to be acceptable, at least from a Pennsylvania viewpoint. There are several lawyers in Delaware who seem to make it an avocation of reporting Pennsylvania lawyers in a vigorous fashion to the Delaware disciplinary authorities."; "The problem started about six or seven years ago when an attorney was licensed in Pennsylvania with an office in southern Chester County and was handling a number of first-party benefits insurance cases in Delaware. She had local counsel, but was settling these cases. Many of these cases had little third-party value and, therefore, were not of interest to members of the Delaware bar. The attorney was a black woman, and many people felt comfortable going to her since there were not a large number of minority lawyers actively practicing in Delaware. The cases came to her through her husband's medical practice and the church

109

35951124_10 UPL, MDP and MJP (Defining What Lawyers Do and McGuireWoods LLP Where They Can Do It): Part II T. Spahn (1/1/19) Hypotheticals and Analyses

that she and her husband attended in Delaware."; "The problem in Delaware is that the Delaware Supreme Court will suspend or disbar a lawyer even if he or she is not admitted in Delaware. This is what happened in these cases. This was challenged in Pennsylvania, but the Pennsylvania Supreme Court will grant reciprocal discipline, as will the federal courts in Pennsylvania. A Pennsylvania lawyer disciplined in Delaware will lose his or her license in Pennsylvania under reciprocal discipline. The fact that the lawyer was never licensed in Delaware is not a defense to the reciprocal discipline, at least in Pennsylvania."; "Of great interest is the fact that many Delaware lawyers provide advice in Pennsylvania and other states on a regular basis, particularly through transactional, tax and other types of corporate law. The Delaware Supreme Court seems somewhat shortsighted. If Pennsylvania courts would be just as vigorous, there would be a large number of major law firms and lawyers in Delaware who would be under suspension if the same rationale applied."; "In conclusion, particularly in Delaware, lawyers who are licensed in Pennsylvania but providing advice across state lines should be very careful. It has always been risky to do that, though with modern availability of legal research for various state laws, it is not as dangerous as it used to be, because one can check relevant state law on issues. But any lawyer providing advice or writing wills or other matters for people in another state should have local counsel. If a lawyer does it with regularity, he or she should be careful about being cited for unauthorized practice of law. In states like Delaware, the unauthorized practice of law results in major suspensions, which will then be ordered in Pennsylvania under a reciprocal system."; "Unless the bar leaders in Delaware and Pennsylvania can reach some understanding or accommodation, any lawyer who wants to provide advice across the Delaware state line should do so very cautiously. The best practice would be to move for pro hac vice admission.").

• In re Tonwe, 929 A.2d 774, 775-76, 776, 776-77, 778, 779 n.7, 781 (Del. 2007) (enjoining a Pennsylvania lawyer from engaging in the practice of law in Delaware without being admitted pro hac in Delaware; also suggesting that Pennsylvania disbar the lawyer; "This is a lawyer disciplinary proceeding. The respondent, Valerie Glover Tonwe ('Glover'), is a Pennsylvania attorney who has never been admitted to practice law in Delaware. In 1991, when the Office of the Disciplinary Counsel ('ODC') learned that she was maintaining an office and practicing law in Delaware, Glover consented to the entry of a cease and desist order. In 2006, the ODC filed a petition alleging that Glover has been violating the cease and desist order, and that her activities in Delaware constitute the unauthorized practice of law. A panel of the Board of Professional Responsibility ('Board') issued its report recommending, among other things, that Glover be declared permanently unfit for admission to the Delaware bar. Glover objected to the Board's findings and recommendations. After careful consideration, we conclude that Glover must

110

35951124_10 UPL, MDP and MJP (Defining What Lawyers Do and McGuireWoods LLP Where They Can Do It): Part II T. Spahn (1/1/19) Hypotheticals and Analyses

be disbarred." (footnote omitted); "The ODC first learned about Glover's Delaware legal practice as a result of an ongoing federal investigation. In 1991, Glover was convicted of bribing a federal immigration official, and served 37 months in prison. Following her conviction, Glover was disbarred in Pennsylvania, Ohio and the District of Columbia. She was reinstated in Pennsylvania in 2002." (emphasis added); "Shortly after she was reinstated, Glover claims that she contacted the Pennsylvania Ethics Hotline to find out whether she could conduct 'pre-litigation' meetings with Delaware clients who had personal injury claims arising from Delaware accidents. Louise M. Lamoreaux, the person who would have responded to such a call, did not have any records showing whether Glover had called. Lamoreaux explained that she would have advised Glover to contact the appropriate Delaware authorities. Lamoreaux also would have advised Glover that pre-litigation meetings with Delaware clients would not violate the Pennsylvania Rules of Professional Conduct as long as the Pennsylvania lawyer was not appearing in Delaware tribunals."; "Glover never contacted anyone in Delaware to determine whether she could provide any legal services to Delaware clients. Instead, over the next few years, Glover developed a sizeable Delaware clientele. Glover claims that she never solicited business from Delaware residents, and that her Delaware practice grew because she was involved in church groups and other civic activities. In June 2005, after following up on information provided by a Delaware lawyer, the ODC notified Glover of its concerns about her Delaware activities. Shortly thereafter, she transferred more than 60 Delaware files to Delaware lawyers. In all, the ODC established that, from 2003-2006 Glover represented more than 100 Delaware residents in Delaware matters." (emphases added); "Glover says that she did not provide legal services 'in Delaware' because she worked out of an office in Pennsylvania. Moreover, because she reasonably believed that the predominant effect of her legal work was in Pennsylvania, she should be protected by the 'safe harbor' provision in Rule 8.5(b)."; "Glover's argument fails for several reasons. First, the record establishes that on three occasions she was physically present in Delaware, representing her Delaware clients. Second, physical presence is not required to establish that a person is providing, or offering to provide, legal services in this state. For several years, Glover accepted new clients who were: (1) Delaware residents, (2) involved in Delware car accidents, and (3) seeking recovery under Delaware insurance policies. Glover did everything short of appearing in Delaware courts, and engaged Delaware attorneys as co-counsel only if she could not resolve the matter without litigation. We are satisfied that this regular pattern of representation of Delaware clients constituted the practice of law 'in Delaware' for purposes of Rule 8.5." (emphases added)(footnote omitted); "'Although it has limited relevance, the evidence suggests that Glover still lives in Delaware. She testified that her husband and children live in their home in Dover, Delaware, but that she is 'legally' a resident of Pennsylvania and that she sleeps in her office. When asked whether she

111

35951124_10 UPL, MDP and MJP (Defining What Lawyers Do and McGuireWoods LLP Where They Can Do It): Part II T. Spahn (1/1/19) Hypotheticals and Analyses

and her husband are separated, however, Glover said, 'No.'"; "The Board recommended a series of sanctions that effectively result in disbarment: (1) Respondent be declared permanently unfit for admission to the Delaware bar; (2) Immediate prohibition on Respondent's ability to appear in Delaware pro hac vice; (3) An Order directing Respondent to cease and desist from all practice of law in Delaware; and (4) Request and recommend disbarment by the Supreme Court of Pennsylvania." (emphases added)).

In 2016, the Minnesota Supreme Court reprimanded a Colorado lawyer for exchanging "approximately two dozen emails" with a Minnesota lawyer to assist his

Minnesota in-laws in a $2,368.13 Minnesota dispute.

Among other things, the Minnesota Supreme Court rejected the Colorado lawyer's argument that his temporary virtual practice in Minnesota satisfied Minnesota

Rule 5.5's exception because he anticipated associating with local counsel and being admitted pro hac vice if his efforts to resolve his in-laws' Minnesota dispute was unsuccessful.

• In re Charges of Unprofessional Conduct in Panel File No. 39302, 884 N.W.2d 661, 663, 663-64, 664, 665, 666, 666-67, 667, 667-68, 668, 668-69, 672, 672-73, 673 (Minn. 2016) (privately reprimanding a Colorado lawyer for assisting his in-laws in a Minnesota dispute; "We hold that engaging in e-mail communications with people in Minnesota may constitute the unauthorized practice of law in Minnesota, in violation of Minn. R. Prof. Conduct 5.5(a), even if the lawyer is not physically present in Minnesota. . . . Appellant represented a Minnesota couple with respect to a Minnesota judgment and attempted to negotiate, via e-mail, the satisfaction of that judgment with a Minnesota lawyer, and was not authorized to practice law in Minnesota temporarily. We further conclude that the appropriate disposition for this misconduct is an admonition."; "Appellant is an attorney licensed to practice law in the state of Colorado, where he maintains an office and has been practicing environmental law since 1986."; "Appellant's mother-and father-in- law live in Minnesota. They contacted appellant in May 2014 to obtain assistance regarding a judgment entered against them in conciliation court in Minnesota for $2,368.13 in favor of their condominium association, Voyager Condominium Homeowners' Association, Inc. (VCHA). The couple told appellant that VCHA's attorney, D.R., a Minnesota-based lawyer and the complainant in this case, was harassing them with telephone calls attempting to collect on the judgment. The couple asked appellant for his assistance in

112

35951124_10 UPL, MDP and MJP (Defining What Lawyers Do and McGuireWoods LLP Where They Can Do It): Part II T. Spahn (1/1/19) Hypotheticals and Analyses

negotiating with D.R. regarding payment of the outstanding judgment."; "Appellant sent an e-mail to D.R. in late May 2014, informing D.R. that he was representing his in-laws and instructing D.R. to direct all future communications to him instead. Appellant and D.R. exchanged approximately two dozen e-mails between May 2014 and September 2014. In his first responsive e-mail to appellant, D.R. asked whether appellant was licensed to practice law in Minnesota. Appellant replied that he was not licensed in Minnesota and that if he needed to file suit in Minnesota he would hire local counsel."; "Nothing in the record shows that appellant researched whether his activities constituted the unauthorized practice of law under the Minnesota Rules of Professional Conduct. When asked by the Panel at the evidentiary hearing whether he researched the rules in Minnesota, appellant said that he did not recall. Appellant admitted that he had not researched Minnesota law on foreclosure and how it would apply to his in-laws' case. Appellant also admitted that when he considered the relevant law and the rules of professional conduct, he was more familiar with the laws and rules in Colorado."; concluding that the lawyer practiced law in Minnesota despite not having been physically present there; "Appellant contends that he did not violate Rule 5.5(a) because he did not practice law in Minnesota."; "Other courts have addressed the issue of whether an attorney practices law in a jurisdiction even though the attorney was not physically present in that jurisdiction."; "[W]e conclude that the Panel did not clearly err by finding that appellant practiced law in Minnesota, in violation of Minn. R. Conduct 5.5(a). Appellant contacted D.R., a Minnesota lawyer, and stated that he represented Minnesota clients in a Minnesota legal dispute. This legal dispute was not interjurisdictional; instead, it involved only Minnesota residents and a debt arising from a judgment entered by a Minnesota court. Appellant instructed D.R. to refer all future correspondence to him, and he continued to engage in correspondence and negotiations with D.R. over the course of several months. Appellant requested and received financial documents from his Minnesota clients and advised them on their legal options. By multiple e-mails sent over several months, appellant advised Minnesota clients on Minnesota law in connection with a Minnesota legal dispute and attempted to negotiate a resolution of that dispute with a Minnesota attorney. Appellant had a clear, ongoing attorney-client relationship with his Minnesota clients, and his contacts with Minnesota were not fortuitous or attenuated. Thus, there is ample support for the Panel's finding that appellant practiced law in Minnesota." (footnote omitted); "Next, we turn to appellant's claim that even if the Panel did not err in determining that he was practicing law in Minnesota in violation of Minn. R. Prof. Conduct 5.5(a), his conduct was permitted under one of the exceptions in Minn R. Prof. Conduct 5.5(c). Appellant argues that Rule 5.5(c)(2) authorized his conduct because he reasonably believed that he would be able to associate with local counsel and be admitted pro hac vice if necessary. Appellant further claims that Rule 5.5(c)(4) authorized his conduct because his in-laws

113

35951124_10 UPL, MDP and MJP (Defining What Lawyers Do and McGuireWoods LLP Where They Can Do It): Part II T. Spahn (1/1/19) Hypotheticals and Analyses

reached out to him for assistance on a matter within his expertise; thus the matter 'arose out of [Appellant's] law practice.'"; also finding that he could not rely on the exception finding temporary practice in Minnesota to a lawyer who expects to be admitted in litigation there; "Under Minnesota Rules of Professional Conduct 5.5(c)(2), a lawyer admitted in another jurisdiction may provide legal services in Minnesota on a temporary basis if the lawyer's services are reasonably related to a pending or potential proceeding before a tribunal and the lawyer reasonably expects to be authorized by law to appear in the proceeding. Comment 10 explains that a lawyer rendering services in Minnesota on a temporary basis is permitted to engage in conduct in anticipation of a proceeding or hearing in which the lawyer reasonably expects to be admitted pro hac vice."; "Rule 5.5(c)(2), by its plain language, requires more than an attorney's speculation that the attorney can find local counsel and be admitted pro hac vice. Appellant's e-mail correspondence does not indicate that he took steps to secure local counsel or investigate the possibility of pro hac vice admission. Thus, we conclude there is no support for appellant's claim that his conduct was authorized by Rule 5.5(c)(2)."; also finding that there was no connection between the Minnesota matter and his home state of Colorado; "Under Minnesota Rules of Professional Conduct 5.5(c)(4), a lawyer admitted in another jurisdiction may provide legal services in Minnesota on a temporary basis if the lawyer's services are not covered by paragraphs (c)(2) and (c)(3) and 'arise out of or are reasonably related to the lawyer's practice in a jurisdiction in which the lawyer is admitted to practice.' Appellant contends that his services arose out of or were reasonably related to his practice in Colorado because the clients are his relatives who 'reached out to him for assistance' and appellant's environmental and personal-injury practice involves debt collection."; "The legal services appellant provided to his in-laws were unrelated to his environmental and personal-injury practice in Colorado."; "Moreover, appellant's representation of his in-laws did not 'arise out of' or 'reasonably relate' to his practice in Colorado simply because his in-laws contacted him in Colorado or appellant has done collections work in Colorado. As the Director notes, appellant's in-laws were not long- standing clients; nor was there any connection between the in-laws' case and the state or laws of Colorado. And while appellant's Colorado practice may involve judgment collections work, nothing in the record establishes that this work was based on a body of federal or nationally uniform law. To the contrary, appellant's clients were Minnesota residents with a debt that arose in Minnesota that they owed to a Minnesota resident and that was governed by Minnesota law. Accordingly, Rule 5.5(c)(4) does not apply to appellant's conduct."; in dissent, three judges found a sufficient connection between the Minnesota temporary practice and Colorado; "[T]he clients' relationship to appellant, including their familial connection and the clients' contacts with appellant in his home state, should be considered in the 'reasonable relationship' analysis. The comments to the Restatement advise that, in determining whether an out-of-state lawyer's activities 'reasonably relate' to

114

35951124_10 UPL, MDP and MJP (Defining What Lawyers Do and McGuireWoods LLP Where They Can Do It): Part II T. Spahn (1/1/19) Hypotheticals and Analyses

the lawyer's practice in a state of admission, 'several factors are relevant, including the following: . . . [Whether the client] is from the lawyer's home state, has extensive contacts with that state, or contacted the lawyer there,' Restatement (Third) of the Law Governing Lawyers § 3 cmt. e (emphasis added). Here, the clients contacted their son-in-law, appellant, in his home state of Colorado."; "[A]nalogous support is provided in comment 14 to Rule 5.5(c)(4), which states that one factor to consider is whether the 'lawyer's client may have been previously represented by the lawyer.' Minn. R. Prof. Conduct 5.5(c)(4) cmt. 14. Although the record does not indicate whether appellate ever previously represented his parents-in-law, the principle underlying this comment – a relationship of trust and familiarity with the lawyer's capabilities – is applicable here. The recognition that a sustained lawyer-client relationship would allow an attorney to perform legal work for the client in other jurisdictions, based on confidence and trust, is reflected in the ABA's recommendation for the proposed Model Rule 5.5."; also finding that as a policy matter the private admonition was improper; "[A]s a policy matter, the implications of the court's decision are troubling and counterproductive. . . . Today's decision represents a step backwards. By the court's reasoning, when family members or friends – an abundant source of clients – email or call a practitioner admitted in another state, seeking assistance in areas in which the practitioner is experienced and competent, relying on a relationship of trust and confidence, they must be turned away. Those potential clients must then expend unnecessary time and resources to research and hire local counsel – even for minor, temporary services in which the out-of-state lawyer could have provided efficient, inexpensive, and competent service. Simply put, the court's decision is contrary to the principles and policy goals intended by Rule 5.5(c)."; "In sum, this case involves clients contacting an attorney, their son-in-law, in his home state of Colorado, to request his assistance regarding a small collection matter – an area that reasonably relates to appellant's expertise and experience in his Colorado litigation practice. Based on the relationship and contacts between the clients, appellant, and appellant's practice of law in Colorado, there is a sufficient 'reasonable relationship' here to satisfy the broad, catch-all exception under Rule 5.5(c)(4). For the above reasons, I conclude that appellant did not engage in professional misconduct because the exception in Rule 5.5(c)(4) applies. Therefore, I would reverse the Panel's decision to admonish appellant. I respectfully dissent.").

Other states also follow this narrower approach.

• Angela Morris, Linebarger Goggan Law Firm Settles Class Action For $3.4 Million, Tex. Lawyer, Jan. 12, 2016 ("A federal judge has approved a settlement that requires Austin-based law firm Linebarger Goggan Blair & Sampson, LLC to pay $3.4 million -- including nearly $904,000 in attorney fees and expenses -- to settle a class action that alleged it engaged in the

115

35951124_10 UPL, MDP and MJP (Defining What Lawyers Do and McGuireWoods LLP Where They Can Do It): Part II T. Spahn (1/1/19) Hypotheticals and Analyses

unauthorized practice of law in California."; "The settlement ends litigation spanning back to May 2013, when plaintiff 4EC Holdings sued Linebarger, a firm that contracts with governmental agencies to collect debts. 4EC alleged that Linebarger sent debt collection demand letters to California residents, even though the firm did not employ lawyers in California, as allegedly required under California law. Linebarger denied the allegations.").

• In re Velahos, Dkt. No. DRB 15-409, at 6 (N.J. Sup. Ct. Disciplinary Review Bd., May 23, 2016) (suspending for six months a lawyer for various ethics violations; "In fact, respondent represented clients in multiple matters in jurisdictions in which he was not authorized to practice, without the assistance of local counsel. Respondent conducted no less than eighteen mortgage modifications in the States of Georgia, Washington, New York, Pennsylvania, Virginia, Maryland, Connecticut, Texas, or Florida. Respondent misrepresented to several of these out-of-state clients in the fee agreements that FLA 'has been retained as 'Of Counsel' to Loan Law Center.' Moreover, respondent engaged in credit and debt adjustment services in Maryland over a two-year period, even after the Commissioner of Financial Regulation for the State of Maryland issued a summary order, followed by a final order to Cease and Desist. When questioned by the OAE about the orders, respondent denied that he had 'taken any money' from Maryland. However, the OAE's review of respondent's records disclosed that, during that period, respondent actively represented several Maryland clients in that state and collected fees from them. Respondent's conduct in this respect violated RPC 1.16(a)(1), RPC 5.5(a), RPC 8.1(a), RPC 8.4(c), and RPC 8.4(d)."; the suspension was ordered by 137 A.3d 500 (N.J. 2016), aff'd, 137 A.3d 500 (N.J. May 25, 2016)).

• Blume Constr., Inc. v. State, 872 N.W.2d 312, 314, 316, 319, 320 (N.D. 2015) (holding that an appeal of an administrative ruling was void because the Colorado lawyer who signed on behalf of appellant had not taken any steps to be admitted pro hac in North Dakota; "Prior to the hearing, Blume requested the hearing be postponed because it had retained the service of Fidler, who was identified as a licensed attorney from Colorado. Fidler was not licensed to practice law in North Dakota. In approximately May 2014, Fidler notified the referee he was unable to secure a sponsoring attorney licensed in North Dakota. During the same time period, the referee was informed a North Dakota attorney would be representing Blume."; "Blume is a corporation, and there is no evidence in the record that Fidler is employed by Blume or that he is an officer or on the corporation's board of directors. Evidence established that Fidler was a licensed attorney in Colorado, but he was not licensed to practice law in North Dakota."; "Evidence establishes Fidler signed the appeal request and Blume requested the hearing be postponed because it had retained Fidler's services. Fidler engaged in the practice of law in this state by signing the appeal request and being

116

35951124_10 UPL, MDP and MJP (Defining What Lawyers Do and McGuireWoods LLP Where They Can Do It): Part II T. Spahn (1/1/19) Hypotheticals and Analyses

designated as counsel in the action before Job Service. There is no evidence in the record that he applied for pro hac vice admission."; "We have explained that a corporation may not be represented by a non-attorney agent in a legal proceeding, just as a unlicensed natural person may not act as an attorney for another natural person. . . . When a non-attorney agent represents a corporation in a legal proceeding, all legal documents signed and filed by the non-attorney are void. . . . Because Blume is a corporation and may not be represented by a non-attorney agent, Fidler's request for an appeal was not conduct that could be performed by a person without a license to practice law."; "We conclude Fidler's actions filling out and submitting the appear request form on behalf of a corporation were not services a person without a license to practice law could perform, and therefore the safe harbor of N.D.R. Prof. Conduct 5.5(A)(2) does not apply."; "We conclude Fidler's activities completing and submitting the appeal request on Blume's behalf were not protected by the safe harbor provisions of N.D.R. Prof. Conduct 5.5(b). Fidler was required to move for pro hac vice admission under Admission to Practice R. 3(b)(2) within 45 days of filing the appeal request. There is no evidence in the record that Fidler moved for pro hac vice admission, and therefore we conclude Blume's appeal request by its non-attorney agent was void.").

• Peter Vieth, California Lawyer Sanctioned For Virginia Court Appearance, Va. Laws. Weekly, Dec. 22, 2014 ("A California lawyer -- lacking a Virginia law license -- was both fined by a judge and disciplined by the Virginia State Bar for allegedly representing a criminal defendant in Arlington. The lawyer says he got a raw deal."; "When he went to general district court in April, George F. Braun was just helping a buddy who faced a misdemeanor charge, Braun said. He never professed to be a lawyer and he merely helped his friend consider a plea deal from the prosecutor, he said."; "The friend got a favorable result, but a judge thought Braun crossed the line, and he was hauled into court on a contempt charge."; "The judge found Braun guilty of contempt and imposed a $250 fine."; "When the Virginia State Bar also brought a charge of practicing without a Virginia license, 'I threw myself on my sword,' Braun said. He accepted a public reprimand and agreed not to provide legal services of any kind in Virginia through January 1, 2016."; "The bar discipline was imposed December 10."; "'No good deed goes unpunished,' Braun said.").

• Gsell v. Rubin & Yates, LLC, 417 F. Supp. 3d 442, 444, 447, 450, 452 (E.D. Pa. 2014) (refusing to award attorney's fees to a successful lawyer who had played more than a supporting role in the case but had not been admitted pro hac; "This case raises the question of whether a lawyer, who is not admitted to practice in the Eastern District of Pennsylvania ('E.D. Pa.'), generally or pro hac vice, may recover attorney's fees as a 'consulting' attorney, under a fee-shifting statute, for work performed in a case in the E.D. Pa. in which his

117

35951124_10 UPL, MDP and MJP (Defining What Lawyers Do and McGuireWoods LLP Where They Can Do It): Part II T. Spahn (1/1/19) Hypotheticals and Analyses

client was the prevailing party. The answer is that, under the circumstances of this case, he may not."; "Plaintiff concedes that Mr. Lee has failed to obtain admission within the E.D. Pa., in either a general or pro hac vice capacity. However, Plaintiff asserts that compensation for Mr. Lee's work in the case is still appropriate, as his role was limited to that of a 'consulting' attorney. Thus, the Court must decide whether Mr. Lee actively participated in the case, thus requiring pro hac vice admission, or whether he acted merely as a 'consulting' attorney."; "From the various decisions considering whether an out-of-state attorney occupied the sort of 'consulting' role that would allow for an award of attorney's fees despite the attorney's failure to seek pro hac vice admission in the local court, a consistent list of relevant factors emerges. The weight that each factor commands necessarily depends on the facts of the underlying case."; "Where an out-of-state attorney's participation was limited to that of a consultant whose services are compensable despite lack of pro hac vice admission, that attorney likely . . . 'refrained from direct client contact; . . . refrained from contact with opposing counsel; . . . did not sign or draft substantial portions of pleadings, especially the complaint; . . . restricted his participation in the case to reviewing motions, drafting internal memos, and advising lead counsel, such that his work was supervised by, and ultimately 'filtered' through the lead attorney; [and] recorded only a modest number of hours on a case, relative to lead counsel and other admitted attorneys working on a case"; "On balance, the Court thus finds that Mr. Lee 'actively participated' in the case, rising well above what could be attributed to a 'consulting' attorney, and therefore he may not excuse his failure to obtain pro hac vice admission as required by Local Rule 83.5.2(b) on this ground."; "Having determined that Mr. Lee was in violation of E.D. Pa. Local Rule 83.5.2(b) by failing to obtain pro hac vice admission in this case, the Court must next determine what attorney's fees, if any, may be awarded for Mr. Lee's participation in this case."; "In light of Mr. Lee's willful violation of Local Rule 83.5.2(b), the Court concludes that Mr. Lee is not eligible to receive attorney's fees in this case."; "The Court finds that because Mr. Lee was not admitted to practice in Pennsylvania or the E.D. Pa. and yet actively participated in the case without obtaining pro hac vice admission, Mr. Lee violated Local Rule 83.5.2(b0 and therefore may not receive attorney's fees."; "The Court will approve an award of attorney's fees for services rendered by Mr. Rettig, who, in essence, acted as local counsel in this case. Additionally, the costs associated with filing the action will be granted pursuant to 15 U.S.C. § 1692k(a)(3).").

• In re Roswold, 249 P.3d 1199, 1207, 1207-08, 1208, 1208-09 (Kan. 2011) (suspending for one year a Kansas lawyer who had not been sufficiently active in cases in which one of his partners (not licensed in Kansas, but practicing with the lawyer in a Missouri office) pretended to settle a case that had been dismissed against all defendants; "We find the record supports a different conclusion that respondent knowingly disregarded his professional

118

35951124_10 UPL, MDP and MJP (Defining What Lawyers Do and McGuireWoods LLP Where They Can Do It): Part II T. Spahn (1/1/19) Hypotheticals and Analyses

responsibilities, especially as it concerns Schmid's unauthorized practice of law in Kansas. We base this conclusion on the panel's findings that respondent: (1) knew Schmid was meeting with, advising, and representing J.C. in her Kansas lawsuit when Schmid was not authorized to practice law in Kansas and not admitted pro hac vice by the district court; (2) knowingly assisted Schmid in the unauthorized practice of law by signing notices to take depositions that only Schmid planned to attend; and (3) knew about a prior problem relating to Schmid's representation of clients in a Kansas- based case."; "From these circumstances, we find respondent was more than negligent. He had actual knowledge of at least some of the ethical violations to which he has admitted. In some other incidents, respondent remained purposefully oblivious to the pitfalls attendant to a law practice straddling different states when the firm's attorneys were not authorized to practice law in one of those states."; "Supreme Court Rule 116(a) provides for admission pro hac vice in a specific case filed in Kansas only if: (1) out- of-state attorneys are in good standing with the respective jurisdictions in which they practice; and (2) the out-of-state attorney associates with an attorney of record who regularly practices law in Kansas. The rule requires the Kansas attorney be 'actively engaged' in the conduct of the case; sign all pleadings, documents, and briefs; and be present throughout all court or administrative appearances. The admission of an out-of-state attorney to appear pro hac vice is a privilege authorized and regulated by this rule and always subject to judicial discretion."; "Without admission pro hac vice, out- of-state attorneys appearing in Kansas courts, or actively participating in pretrial proceedings such as depositions or mediations, would be engaged in the unauthorized practice of law in this state."; "Applications to appear pro hac vice must be filed as soon as reasonably possible with the court or administrative tribunal where the case is pending. Supreme Court Rule 116(b). And to ensure compliance with the rule, admission pro hac vice is necessary before out-of-state counsel's active involvement with any pretrial proceedings associated with the Kansas litigation. Here, respondent testified it was his practice not to seek Schmid's admission pro hac vice until there was an actual court appearance in a case. This, respondent conceded, resulted in Schmid participating in depositions, other pretrial matters with opposing counsel, and even mediation, without ever complying with Supreme Court Rule 116." (emphasis added); "That said, we emphasize our rule does not require local counsel to actively present the case in person each time there is a proceeding. . . . But it does require local counsel's substantive attention by being actively engaged in the conduct of the case, signing all court filings, and being present at all court or administrative appearances. What constitutes 'court or administrative appearances' is universally understood to include appearances before the court or tribunal, but it does not necessarily extend to attendance at discovery depositions or mediations unless so ordered by the court or required by local rule." (emphasis added)).

119

35951124_10 UPL, MDP and MJP (Defining What Lawyers Do and McGuireWoods LLP Where They Can Do It): Part II T. Spahn (1/1/19) Hypotheticals and Analyses

Best Answer

The best answer to (a) is YES (PROBABLY); the best answer to (b) is YES

(PROBABLY); the best answer to (c) is MAYBE.

B 2/13, 3/17

120

35951124_10 UPL, MDP and MJP (Defining What Lawyers Do and McGuireWoods LLP Where They Can Do It): Part II T. Spahn (1/1/19) Hypotheticals and Analyses

Involvement in Discovery

Hypothetical 9

You represent your corporate client in several cases pending in your home state. Among other things, you need to schedule several depositions of witnesses who are living or working in several other states around the country. You are not admitted in any of those states.

May you take the deposition of witnesses in those other states -- where you are not admitted to practice law?

YES (PROBABLY)

Analysis

This hypothetical deals with lawyers engaging in discovery in states where they

are not licensed -- in connect with litigation pending in a state where they are licensed.

This is different from lawyers being admitted pro hac to litigate in litigation pending in a

state where the lawyer is not licensed.

ABA Model Rule 5.5 allows lawyers admitted in a United States jurisdiction to practice law in another United States jurisdiction under certain conditions.

A lawyer admitted in another United States jurisdiction, and not disbarred or suspended from practice in any jurisdiction, may provide legal services on a temporary basis in this jurisdiction that:

(1) are undertaken in association with a lawyer who is admitted to practice in this jurisdiction and who actively participates in the matter;

(2) are in or reasonably related to a pending or potential proceeding before a tribunal in this or another jurisdiction, if the lawyer, or a person the lawyer is assisting, is authorized by law or order to appear in such proceeding or reasonably expects to be so authorized;

121

35951124_10 UPL, MDP and MJP (Defining What Lawyers Do and McGuireWoods LLP Where They Can Do It): Part II T. Spahn (1/1/19) Hypotheticals and Analyses

(3) are in or reasonably related to a pending or potential arbitration, mediation, or other alternative dispute resolution proceeding in this or another jurisdiction, if the services arise out of or are reasonably related to the lawyer's practice in a jurisdiction in which the lawyer is admitted to practice and are not services for which the forum requires pro hac vice admission; or

(4) are not within paragraphs (c)(2) or (c)(3) and arise out of or are reasonably related to the lawyer's practice in a jurisdiction in which the lawyer is admitted to practice.

ABA Model Rule 5.5(c).

As long as a litigator undertakes such activities in another state on a "temporary

basis," the ABA Model Rules permit such activities.

A lawyer admitted in another United States jurisdiction, and not disbarred or suspended from practice in any jurisdiction, may provide legal services on a temporary basis in this jurisdiction that: (1) are undertaken in association with a lawyer who is admitted to practice in this jurisdiction and who actively participates in the matter; (2) are in or reasonably related to a pending or potential proceeding before a tribunal in this or another jurisdiction, if the lawyer, or a person the lawyer is assisting, is authorized by law or order to appear in such proceeding or reasonably expects to be so authorized.

ABA Model Rule 5.5 (c)(1), (2) (emphases added).

The first exception (requiring the association of a local lawyer) clearly satisfies every state's UPL regulation. Cynics would say that it also assures that local lawyers do not lose any business to out-of-state "interlopers."

However, arranging for a local lawyer's help also increases the client's cost -- so

many lawyers look for ways to avoid the extra expense if they do not need any local

expertise.

122

35951124_10 UPL, MDP and MJP (Defining What Lawyers Do and McGuireWoods LLP Where They Can Do It): Part II T. Spahn (1/1/19) Hypotheticals and Analyses

This second exception allows a remarkably broad range of activities that lawyers may ethically undertake in a state where the lawyers are not licensed. For instance, lawyers may undertake activities in another state:

• if the services are "reasonably related" to "potential" proceedings before a tribunal in that or even some other state and

• as long as the lawyer or another lawyer that she is "assisting" (presumably either in the same firm or some other firm)

• "reasonably expects" to be eventually authorized by a pro hac vice order to appear in the proceeding.

Comment [10] deals with this situation, and specifically approves lawyers' involvement in depositions.

Paragraph (c)(2) also provides that a lawyer rendering services in this jurisdiction on a temporary basis does not violate the Rule when the lawyer engages in conduct in anticipation of a proceeding or hearing in a jurisdiction in which the lawyer is authorized to practice law or in which the lawyer reasonably expects to be admitted pro hac vice. Examples of such conduct include meetings with the client, interviews of potential witnesses, and the review of documents. Similarly, a lawyer admitted only in another jurisdiction may engage in conduct temporarily in this jurisdiction in connection with pending litigation in another jurisdiction in which the lawyer is or reasonably expects to be authorized to appear, including taking depositions in the jurisdictions.

ABA Model Rule 5.5 cmt. [10] (emphases added).

The Restatement takes essentially the same liberal approach.

A lawyer who is properly admitted to practice in a state with respect to litigation pending there, either generally or pro hac vice, may need to conduct proceedings and activities ancillary to the litigation in other states, such as counseling clients, dealing with co-counsel or opposing counsel, conducting depositions, examining documents, interviewing witnesses, negotiating settlements, and the like.

123

35951124_10 UPL, MDP and MJP (Defining What Lawyers Do and McGuireWoods LLP Where They Can Do It): Part II T. Spahn (1/1/19) Hypotheticals and Analyses

Such activities incidental to permissible practice are appropriate and permissible.

Restatement (Third) of Law Governing Lawyers § 3 cmt. e (2000) (emphasis added).

Most states seem to permit lawyers to take depositions in those states even if they are not licensed there -- as long as the depositions involve pre-trial discovery in a case pending in another state where the lawyer is licensed.

• North Carolina State Bar Authorized Practice Comm., Guidelines for Attorneys Licensed in other Jurisdictions (July 2003) (explaining that a lawyer licensed in some other state but not licensed in North Carolina "can perform only the same legal activities that can be done by a non-lawyer under appropriate supervision unless the legal activities fall under a recognized exception to the unauthorized practice rules such as a limited practice before the federal courts or agencies or employment as in-house counsel corporate counsel."; also explaining that such a lawyer "may not perform the same or similar legal activities in North Carolina as he or she did in his or her licensed jurisdiction. Any legal activities by an unlicensed attorney in North Carolina must be done under the appropriate supervision of a responsible North Carolina attorney as with other non-lawyers. An unlicensed attorney may not establish a North Carolina law office or provide independent legal advice or counsel to members of the public."; explaining that such a lawyer may not provide independent legal advice to a member of the public, but may relay advice from a licensed North Carolina lawyer; prohibiting such a lawyer from providing a "written legal opinion to a member of the public"; also warning that such a lawyer may not "negotiate a legal claim directly with an opposing person"; "An unlicensed attorney, however, may convey to an opposing person a settlement offer or position provided or specifically approved by a responsible North Carolina attorney. The unlicensed attorney may not alter or negotiate such settlement offers or positions not specifically provided or approved by a responsible North Carolina attorney, or otherwise exercise independent legal judgment in negotiating or settling a legal claim directly with an opposing person."; allowing such a lawyer to draft legal documents, research memoranda and briefs, but only under the supervision of a licensed North Carolina lawyer, who "must review" such documents before they are submitted to a court or given to a member of the public; warning that such a lawyer residing in the North Carolina is not eligible to be admitted pro hac vice in a North Carolina court, but "may take or defend a deposition of a person physically located in North Carolina if the deposition relates to an action pending or to be filed in the attorney's licensed jurisdiction." (emphasis added); and "may oversee the execution of closing documents without a responsible North Carolina attorney represent, provided clients are informed

124

35951124_10 UPL, MDP and MJP (Defining What Lawyers Do and McGuireWoods LLP Where They Can Do It): Part II T. Spahn (1/1/19) Hypotheticals and Analyses

that the unlicensed attorney is not admitted to practice in North Carolina as a nonlawyer may do. However, an unlicensed attorney may not provide legal advice or opinion on any legal questions related to the closing or prepare the legal documents without the review and approval of a North Carolina attorney in the exercise of appropriate supervision.").

In contrast, some states take a very narrow view of what out-of-state lawyers may do in the state, absent a pro hac admission or the very active involvement of a lawyer licensed in that state.

Best Answer

The best answer to this hypothetical is YES.

B 2/13

125

35951124_10 UPL, MDP and MJP (Defining What Lawyers Do and McGuireWoods LLP Where They Can Do It): Part II T. Spahn (1/1/19) Hypotheticals and Analyses

Participation in Non-Court Proceedings

Hypothetical 10

You frequently represent doctors whose privileges have been terminated by hospitals. You just received a call from a doctor in Wisconsin who wants you to represent her in a termination process instituted and supervised by the hospital which just terminated her privileges. The hospital's lawyer called you a few hours later to say that he will object to your appearing on behalf of the doctor in the hospital process -- because you are not admitted to practice in Wisconsin.

May you represent the doctor in the Wisconsin hospital hearing?

MAYBE

Analysis

This hypothetical deals with out-of-state lawyers' ability to appear in proceedings that do not have the trappings of a court -- but which have more formality than private arbitrations.

A Restatement comment deals with this issue.

Transactional and similar out-of-court representation of clients may raise similar issues, yet there is no equivalent of temporary admission pro hac vice for such representation, as there is in litigation. Even activities that bear close resemblance to in-court litigation, such as representation of clients in arbitration or in administrative hearings, may not include measures for pro hac vice appearance.

Restatement (Third) of Law Governing Lawyers § 3 cmt. e (2000) (emphases added).

In some situations, courts or bars must analyze whether the body handling the

proceedings should be considered to be a "tribunal" for purposes of that state's

unauthorized practice of law rules.

126

35951124_10 UPL, MDP and MJP (Defining What Lawyers Do and McGuireWoods LLP Where They Can Do It): Part II T. Spahn (1/1/19) Hypotheticals and Analyses

Courts and bars focus on such a characterization because ABA Model Rule

5.5(c) (and its state counterparts) permit lawyers to engage in activities in other states

that are "reasonably related" to pending or potential proceedings "before a tribunal" -- if

the lawyer or someone else the lawyer is assisting is authorized by law or order to

appear in such proceeding or reasonably expects to be so authorized. ABA Model Rule

5.5(c)(2) (emphasis added). In other words, a lawyer cannot engage in such activity in

connection with some proceeding before a "tribunal" if the lawyer has no hope of being

admitted to practice before that tribunal.

This provision could theoretically even stymie a lawyer who might be ineligible to

appear pro hac before a tribunal -- because the lawyer or the law firm has reached the maximum number of such admissions permitted by the court, or for some other reason.

However, the provision normally becomes important if the lawyer is involved in a

non-court proceeding before somebody other than a "tribunal." It might seem odd that a

tribunal-like body would not have such a pro hac admissions process. In many

situations, the absence of such a process presumably is some accident of history or

procedural oversight. However, lawyers sometimes find themselves in a "Catch 22"

situation.

For instance, in Virginia UPL Opinion 209 (10/21/05), the Virginia Bar held that

an out-of-state lawyer could not appear by himself before the Virginia Gas and Oil

Board. The Virginia Bar correctly noted that whether the lawyer could appear before

that Board "will depend upon the determination of whether the Virginia Gas and Oil

Board is a 'tribunal.'"

127

35951124_10 UPL, MDP and MJP (Defining What Lawyers Do and McGuireWoods LLP Where They Can Do It): Part II T. Spahn (1/1/19) Hypotheticals and Analyses

The Virginia Bar extensively analyzed the Virginia Gas and Oil Board.

Interestingly, the Board's procedural rules did not allow nonlawyers to represent others

before the Board (if they did, the out-of-state lawyer could have represented a client before the board even without a law degree). The Virginia Bar also found that proceedings before the Board could not be properly analogized to an arbitration, which an out-of-state lawyer could handle without any problem.

The Virginia Bar ultimately determined that the Board should be considered to be

a "tribunal" -- because of the way it conducts adversarial proceedings. Significantly, the

Board did not have a process analogous to a pro hac admission process that every

court has adopted.

Under Rule 1A:4 of the Rules of the Supreme Court of Virginia, a foreign attorney may be permitted to appear and conduct a particular case before a Virginia court pro hac vice in association with a member of the Virginia State Bar. If an agency, in this case the Board, had a procedure for pro hac vice appearance the foreign attorney could avail himself of that process and appropriately appear before the Board. The Committee is presently not aware of any such process being available with the Board.

Id. Thus, the lawyer had to associate another lawyer for the Gas and Oil Board

proceedings.

Similarly, in 2001 the Rhode Island Supreme Court found that a non-court administrative tribunal was not authorized to grant pro hac admissions to out-of-state

lawyers to appear before it -- finding that only Rhode Island courts could do so.

• In re Ferrey, 774 A.2d 62, 64-65, 65 (R.I. 2001) (analyzing the ability of non- court administrative tribunals to permit out-of-state lawyers to appear before them; "Thus, it is abundantly clear that since 1917, chapter 27 of title 11 has made unlawful the practice of law in this state by any lawyer who is not a member of the Rhode Island bar and who has not been given prior pro hac

128

35951124_10 UPL, MDP and MJP (Defining What Lawyers Do and McGuireWoods LLP Where They Can Do It): Part II T. Spahn (1/1/19) Hypotheticals and Analyses

vice permission to practice here, regardless of whether that attorney appears before any court or before any municipal or state agency, board or commission. This Supreme Court alone possesses sole authority to determine who may, and who may not, engage in the practice of law in this state. No municipal or state board, agency or commission shares in that authority, and none has ever been delegated by this Court to any municipal or state board, agency or commission."; "[A]s a nonresident who had been given permission, albeit unauthorized, to practice in this state, it is somewhat understandable that Attorney Ferrey might not have known that chapter 27 of title 11 of our General Laws prohibited his practice before the particular state agency when he initially appeared. That assumed absence of knowledge, coupled with the Energy Facility Siting Board's previous permission for him to appear before it based upon its mistaken belief that it had been delegated authority to do so, prompted our decision to grant his pro hac vice request for permission to continue to represent his client in the continuing legal proceedings before that board." (footnote omitted); ultimately granting the lawyer's pro hac vice motion to appear before the board, but denying retroactive effect).

This hypothetical comes from a Wisconsin case. Although Wisconsin changed its applicable rules in recent years, it is worth explaining its earlier restrictive approach -- which traditionally was fairly typical, but which has been receding in recent years.

In that 2004 case, the Wisconsin Supreme Court held that a New Jersey lawyer could not assist a Wisconsin lawyer in representing a doctor during a hospital peer review hearing -- because the hospital's hearing process did not contain an analogy to a pro hac vice admission process.

• Seitzinger v. Community Health Network, 676 N.W.2d 426, 436, 452 n.79 (Wis. 2004) (assessing the ability of a lawyer admitted only in New Jersey to help a doctor in connection with a peer review hearing in Wisconsin; explaining that the New Jersey lawyer (who was also a board certified obstetrician-gynecologist and a board certified subspecialist in gynecologic oncology) filed a petition to be admitted pro hac vice for the hospital hearing process, and also promised to associate with a licensed Wisconsin lawyer; holding the hospital bylaws allowed doctors appearing in the peer review process to be represented by "legal counsel," which meant lawyers licensed in Wisconsin; rejecting the New Jersey lawyer's argument that he would be assisting the Wisconsin lawyer "only by marshalling the scientific and medical evidence presented at the hearing" and therefore could appear at

129

35951124_10 UPL, MDP and MJP (Defining What Lawyers Do and McGuireWoods LLP Where They Can Do It): Part II T. Spahn (1/1/19) Hypotheticals and Analyses

the hearing -- because only members of the hospital's staff could act as doctors in that role, meaning that the only role the New Jersey lawyer could theoretically play at the hearing was the role of the Wisconsin doctor's lawyer; concluding that the hospital process did not allow for the admission pro hac vice of any out-of-state lawyer, and that Wisconsin's general pro hac regulations deal only with lawyers appearing in court, rather than hospital peer review hearings).

Even worse (for the New Jersey lawyer and the Wisconsin doctor), the Wisconsin

Supreme Court held that the New Jersey lawyer could not even appear before the

hospital's peer review process in association with the Wisconsin lawyer -- because that

exception was "also linked to the requirement that the proceeding be before the judge permitting the non-resident attorney to represent the client in his or her court." Id. at

437.

Effective January 1, 2009, the Wisconsin Supreme Court adopted the ABA Model

Rule multijurisdictional approach of Rule 5.5. At the same time, the Wisconsin Supreme

Court amended its Rule 10.03(4), which allows a judge to authorize out-of-state lawyers to "appear and participate in a particular action or proceeding" even if that proceeding was not in the judge's own court -- which created the extreme result in the 2004

Seitzinger case.

Best Answer

The best answer to this hypothetical is MAYBE.

B 2/13

130

35951124_10 UPL, MDP and MJP (Defining What Lawyers Do and McGuireWoods LLP Where They Can Do It): Part II T. Spahn (1/1/19) Hypotheticals and Analyses

Lawyers Representing Their Clients in Arbitrations Where the Lawyers Are Not Licensed

Hypothetical 11

Over the years, you have shifted from being a trial lawyer to primarily representing your clients in employment law arbitrations. One of your clients just asked if you could represent it in an arbitration scheduled to take place in a state where you are not licensed.

May you represent a client in an arbitration taking place in a state where you are not licensed?

YES (PROBABLY)

Analysis

ABA Model Rule 5.5 allows lawyers admitted in a United States jurisdiction to practice law in another United States jurisdiction under certain conditions.

A lawyer admitted in another United States jurisdiction, and not disbarred or suspended from practice in any jurisdiction, may provide legal services on a temporary basis in this jurisdiction that:

(1) are undertaken in association with a lawyer who is admitted to practice in this jurisdiction and who actively participates in the matter;

(2) are in or reasonably related to a pending or potential proceeding before a tribunal in this or another jurisdiction, if the lawyer, or a person the lawyer is assisting, is authorized by law or order to appear in such proceeding or reasonably expects to be so authorized;

(3) are in or reasonably related to a pending or potential arbitration, mediation, or other alternative dispute resolution proceeding in this or another jurisdiction, if the services arise out of or are reasonably related to the lawyer's practice in a jurisdiction in which the lawyer is admitted to practice and are not services for which the forum requires pro hac vice admission; or

131

35951124_10 UPL, MDP and MJP (Defining What Lawyers Do and McGuireWoods LLP Where They Can Do It): Part II T. Spahn (1/1/19) Hypotheticals and Analyses

(4) are not within paragraphs (c)(2) or (c)(3) and arise out of or are reasonably related to the lawyer's practice in a jurisdiction in which the lawyer is admitted to practice.

ABA Model Rule 5.5(c) (emphasis added).

This hypothetical deals with the requirement that the temporary legal services

"are in or reasonably related to a pending or potential arbitration, mediation, or other

alternative dispute resolution proceeding."

A comment to ABA Model Rule 5.5 makes it clear that lawyers may temporarily

participate in arbitrations pending in states where they are not licensed.

Paragraph (c)(3) permits a lawyer admitted to practice law in another jurisdiction to perform services on a temporary basis in this jurisdiction if those services are in or reasonably related to a pending or potential arbitration, mediation, or other alternative dispute resolution proceeding in this or another jurisdiction, if the services arise out of or are reasonably related to the lawyer's practice in a jurisdiction in which the lawyer is admitted to practice. The lawyer, however, must obtain admission pro hac vice in the case of a court-annexed arbitration or otherwise if court rules or law so require.

ABA Model Rule 5.5 cmt. [12] (emphasis added).

The Restatement takes a more wary approach. It recognizes that some states

might require pro hac admission by out-of-state lawyers in arbitration or administrative hearings -- but do not include any pro hac process for such arbitrations.

Transactional and similar out-of-court representation of clients may raise similar issues, yet there is no equivalent of temporary admission pro hac vice for such representation, as there is in litigation. Even activities that bear close resemblance to in-court litigation, such as representation of clients in arbitration or in administrative hearings, may not include measures for pro hac vice appearance.

Restatement (Third) of Law Governing Lawyers § 3 cmt. e (2000) (emphasis added).

132

35951124_10 UPL, MDP and MJP (Defining What Lawyers Do and McGuireWoods LLP Where They Can Do It): Part II T. Spahn (1/1/19) Hypotheticals and Analyses

The ACTEC Commentaries also discuss this principle.

Subject to the "temporary basis" threshold requirement, paragraphs (c)(2) and (3) expand the situations in which lawyers may render services in a non-admitted jurisdiction regarding litigation and alternative dispute resolution ("ADR"). Regarding trials, preliminary work in preparation for the trial is acceptable, provided the lawyer is either authorized to appear or reasonably expects to be so authorized. Thus, a lawyer asked to assist or handle estate litigation could investigate the underlying facts, meet with and counsel clients, and provide related services, provided the lawyer reasonably expected to be admitted pro hac vice. While this exception is available to allow the lawyer to investigate the matter before seeking admission, the lawyer should not rely on the exception except where necessary. Instead, the lawyer should seek and obtain admission pro hac vice at the earliest opportunity.

On the other hand, the exception for ADR applies only when the non-admitted jurisdiction does not require admission pro hac vice to participate in the ADR and the lawyer's services "arise out of or are reasonably related to the lawyer's practice" in an admitted jurisdiction [MPRC 5.5(c)(3)]. If admission pro hac vice is required to participate in the ADR, then the ADR, then the lawyer must comply with MRPC 5.5(c)(2). Where admission pro hac vice is not required, the lawyer may provide legal services in the non-admitted jurisdiction regarding the client's ADR, provided those legal services are "reasonably related" to the lawyer's practice in an admitted jurisdiction. Like litigation, a lawyer engaged to assist a client's efforts to resolve estate litigation in a non-admitted jurisdiction through ADR may provide legal services both in preparation for ADR and during ADR.

While paragraph (c)(3) is silent regarding whether this exception would apply to settlement negotiations alone, logically a lawyer should be able to assist a client with settlement negotiations in a non-admitted jurisdiction, if the lawyer could assist the client with ADR. Although silent regarding this matter, paragraph (c)(3) does not apply to both "pending" and "potential" ADR. Since every settlement negotiation could "potentially" lead to ADR, a lawyer may rely on (c)(3) to authorize participation in settlement

133

35951124_10 UPL, MDP and MJP (Defining What Lawyers Do and McGuireWoods LLP Where They Can Do It): Part II T. Spahn (1/1/19) Hypotheticals and Analyses

discussions alone. If a lawyer is asked to represent a client in settlement negotiations regarding estate litigation in a non-admitted jurisdiction, the lawyer should consider specifically raising the possibility of "potential ADR" in written communications with the client.

American College of Trust & Estate Counsel, Commentaries on the Model Rules of

Professional Conduct, Commentary on MRPC 5.5, at 204-05 (5th ed. 2016).

Most bars take the broad ABA Model Rule approach.

• Illinois LEO 13-03 (1/2013) (holding that a nonlawyer could not represent a party to an arbitration under the Financial Industry Regulatory Authority ("FINRA") Code of Arbitration Procedure for Customer Disputes; "It is thus clear that an out-of-state attorney complying with the provisions of RPC 5.5(c)(3) may represent parties to an Illinois arbitration. The more difficult question, however, is whether the representation by a nonlawyer, who is not licensed to practice in any jurisdiction, of parties to an Illinois FINRA arbitration constitutes the unauthorized practice of law in Illinois.") (emphasis added)

• New Jersey UPL Op. 43 (1/8/07) (affirming an earlier New Jersey legal ethics opinion that permitted out-of-state lawyers to engage in New Jersey arbitrations; noting that New Jersey adopted its version of Rule 5.5 after that earlier opinion; "While RPC 5.5 does not change the ultimate opinion of the Committee in Opinion 28, i.e., that an out-of-state attorney may appear in an AAA arbitration, RPC 5.5 does change the prerequisites for this appearance. In Opinion 28 the Committee required that no related action was pending in the attorney's state of admission. This is not a requirement of RPC 5.5 and so is no longer required by this Committee. Further, RPC 5.5(c)(1) through (6) provides additional requirements, the most important of which is that the out-of-state attorney must register with the Clerk of the Supreme Court, authorize the Clerk to accept service of process on the attorney's behalf, and comply with New Jersey Rules regarding registration and fees. These requirements are therefore added to Opinion 28 in this Supplemental Opinion. Additionally, the question has been posed whether a multi- jurisdictional practitioner may represent an existing out-of-state client in mediation in New Jersey. The Committee finds that this is akin to arbitration and that an out-of-state attorney may participate in mediation and may prepare an order for the court reflecting a memorandum of understanding/agreement reached in mediation, provided that the out-of- state attorney has satisfied the requirements of RPC 5.5."; noting an additional requirement for such lawyers -- that they must "maintain a bona fide office in conformance with R. 1:21-1(a), except that, when admitted pro

134

35951124_10 UPL, MDP and MJP (Defining What Lawyers Do and McGuireWoods LLP Where They Can Do It): Part II T. Spahn (1/1/19) Hypotheticals and Analyses

hac vice, the lawyer may maintain the bona fide office within the bona fide law office of the associated New Jersey attorney pursuant to R. 1:21- 2(a)(1)(B)").

• Virginia UPL Op. 200 (1/22/01) (permitting a lawyer licensed only in Maryland to represent a client in an arbitration in Virginia; "The committee is of the opinion that the foreign attorney is authorized to represent his client in an arbitration proceeding in Virginia if it would be incidental to the foreign attorney's representation of the client whom the attorney represents elsewhere." (emphasis added)).

• Virginia UPL Op. 92 (5/2/86) ("It is not the unauthorized practice of law for a non-Virginia licensed attorney to present evidence and argue matters of law before an arbitration panel of the American Association in Virginia in order to represent a client from the attorney's jurisdiction in a franchise contract dispute.").

The trend is clearly in favor of permitting such activity by out-of-state lawyers.

For instance, in 2003 the Florida Supreme Court prohibited an out-of-state lawyer from engaging in securities arbitration proceedings in Florida.

• Fla. Bar v. Rapoport, 845 So. 2d 874, 875, 876 (Fla. 2003) ("The Bar filed its petition for an injunction in January 2001, claiming that Rapoport was engaged in the unlicensed practice of law (UPL) because he (1) represents parties in Florida in securities arbitration proceedings by entities such as the American Arbitration Association, the National Association of Securities Dealers, and the New York Stock Exchange; and (2) advertises his securities arbitration services in the Fort Lauderdale Sun-Sentinel."; "In Sperry v. Florida ex rel. Florida Bar, 373 U.S. 379, 10 L. Ed. 2d 428, 83 S. Ct. 1322, 1963 Dec. Comm'r Pat. 211 (1963), the United States Supreme Court, although acknowledging Florida's substantial interest in regulating the practice of law within the state, held that Florida could not enjoin a nonlawyer registered to practice before the U.S. Patent Office from preparing and prosecuting patent applications in Florida because a federal statute and Patent Office regulations authorized the practice. Rapoport provides a long list of federal cases concerning securities arbitration that involve preemption of state law by the FAA. None of the cases, however, concerns the authorization of the practice of law in securities arbitration proceedings." (footnote omitted); enjoining a Washington, D.C., lawyer not admitted in Florida from participating in Florida arbitrations).

135

35951124_10 UPL, MDP and MJP (Defining What Lawyers Do and McGuireWoods LLP Where They Can Do It): Part II T. Spahn (1/1/19) Hypotheticals and Analyses

However, current Florida Rule 4-5.5(c)(3) permits out-of-state lawyers to provide legal services in connection with Florida ADR in two scenarios:

(A) if the services are performed for a client who resides in or has an office in the jurisdiction in which the lawyer is admitted to practice, or

(B) where the services arise out of or are reasonably related to the lawyer's practice in a jurisdiction in which the lawyer is admitted to practice.

Florida Rule 4-5.5(c)(3).

This expands the situations in which a non-Florida lawyer can participate in

Florida ADR. Under ABA Model Rule 5.5(c)(3), the services must be related to the lawyer's practice in his or her home jurisdiction. Under the Florida alternative, lawyers may also participate in a Florida ADR if the client for whom the lawyer provides the services has an office in the lawyer's home state -- presumably even if the ADR is not related to the lawyer's practice in her home state.

However, Florida still keeps a tight leash on out-of-state lawyers hoping to represent clients in Florida arbitrations. Under every state's multijurisdictional practice rule, the exceptions apply to lawyers' temporary practice in the state. In other words, out-of-state lawyers may not establish a "systematic and continuous" presence in the state unless they satisfy some other exception. Florida takes the position that lawyers filing three arbitration demands in a one year period is no longer practicing temporarily in Florida, but instead is practicing regularly.

• Florida Rule 4-5.5 cmt. ("For the purposes of this rule, a lawyer who is not admitted to practice law in Florida who files more than 3 demands for arbitration or responses to arbitration in separate arbitration proceedings in a 365-day period is presumed to be providing legal services on a regular, not temporary, basis; however, this presumption does not apply to a lawyer

136

35951124_10 UPL, MDP and MJP (Defining What Lawyers Do and McGuireWoods LLP Where They Can Do It): Part II T. Spahn (1/1/19) Hypotheticals and Analyses

appearing in international arbitrations as defined in the comment to rule 1- 3.11.").

Other states have taken the same basic approach, but without Florida's specificity.

• Illinois LEO 94-5 (7/1994) ("Regular representation of Illinois parties to arbitration proceedings by lawyer not licensed in Illinois constitutes unauthorized practice of law. If a lawyer not licensed in Illinois seeks to advertise for or solicit Illinois clients, the lawyer should disclose the lack of an Illinois license in any advertising and solicitation materials."; explaining that "[t]he threshold issue presented is whether the representation of a party to an arbitration proceeding is the practice of law. In general, the courts have held that a person practices law when the person applies the law to the facts of a particular case. Rotunda, Professional Responsibility 123 (3d ed. 1992). The Illinois position is consistent with the general rule. The Supreme Court has held that the practice of law involves more than the representation of parties in litigation and includes the giving of advice or the rendering of any services requiring the use of legal skill or knowledge. People v. Schafer, 404 Ill. 45, 87 N.E.2d 773, 776 (1949). In a case directly relevant to the present inquiry, the Supreme Court held that the representation of parties in contested workers' compensation matters before an arbitrator of the Illinois Industrial Commission constituted the practice of law. People v. Goodman, 366 Ill. 346, 8 N.E.2d 941, (1937). The respondent in Goodman had argued that he was not practicing law because he was representing parties before an administrative agency rather than a court. The Supreme Court responded that the 'character of the act done, and not the place where it is committed' is the decisive factor. 8 N.E.2d at 947. In view of these authorities, the Committee concludes that the representation of a party in a contested arbitration proceeding would be considered the practice of law."; "the present inquiry involves matters by an unlicensed person. For these reasons, the Committee believes that the Illinois courts would find that a lawyer licensed only in another state who regularly represents Illinois parties in arbitration proceedings in Illinois engages in the unauthorized practice of law." (emphases added)).

The issue sometimes comes up in court, when the losing party in an arbitration

challenges the award -- by arguing that an out-of-state lawyer impermissibly represented one of the arbitration parties. These arguments normally fail.

• Nolan v. Kenner, 250 P.3d 236, 238 (Ariz. Ct. App. 2011) (holding that an arbitration award cannot be overturned because an out-of-state lawyer improperly represented a party in the arbitration; "Defendants-appellants Philip A. Kenner, Standard Advisors L.L.C., and Standard Advisors Inc.

137

35951124_10 UPL, MDP and MJP (Defining What Lawyers Do and McGuireWoods LLP Where They Can Do It): Part II T. Spahn (1/1/19) Hypotheticals and Analyses

(collectively 'Kenner') appeal the superior court's order confirming an arbitration award granting Plaintiffs-Appellees Owen Nolan and Diana Nolan (collectively, 'Nolan') approximately $2,700,000 in damages and attorneys' fees for Kenner's breach of fiduciary duty. Kenner contends that the arbitration award should be vacated because Nolan's counsel during arbitration was neither a member of the State Bar of Arizona nor admitted to appear pro hac vice. We hold that open representation by a foreign attorney is not the type of undue means permitting a court to vacate an arbitration award pursuant to Arizona Revised Statutes ('A.R.S.') section 12-1512(A)(1) (2003). Accordingly, we affirm the judgment of the superior court.").

• Superadio L.P. v. Winstar Radio Prods., LLC, 844 N.E.2d 246 (Mass. 2006) (upholding an arbitration award despite the fact that a non-Massachusetts lawyer had represented a party in the arbitration proceeding; declining to decide whether the lawyer's participation in the arbitration constituted unauthorized practice of law; concluding that the arbitration should be upheld even if the lawyer had engaged in the unauthorized practice of law in Massachusetts).

• Colmar, Ltd. v. Fremantlemedia N. Am., Inc., 801 N.E.2d 1017, 1022, 1022- 23, 1026 (Ill. App. Ct. 2003) (affirming an arbitration award; "We are called upon to determine for the first time what effect, if any, an out-of-state attorney's representation of an out-of-state client during arbitration in Illinois has on an arbitration award. We find that, for the reasons that follow, Anderson's representation has no effect on the arbitration award in this case."; "No Illinois decision has considered whether the general voidance rule applies to cases where the representation occurred strictly during arbitration proceedings. After considering the applicable Illinois cases, the modern trend in the jurisprudence of multijurisdictional practice, and the public policy reasons promoting both the rule prohibiting unauthorized practice and the general voidance rule, we find that the harsh general rule should not be applied in the instant case."; "Though the ABA Model Rule 5.5 has not been adopted by the Illinois Supreme Court at the time that we decide this case, we find it persuasive in that it reflects the modern trend in the law of multijurisdictional practice and is also in keeping with well- reasoned decisions from other jurisdictions that have found that an out-of- state attorney's representation of a client during arbitration does not violate the rules prohibiting the unauthorized practice of the law.").

Although these decisions arise in a different context (attacks on an existing arbitration award rather than a pre-arbitration analysis of the lawyer's role), the

138

35951124_10 UPL, MDP and MJP (Defining What Lawyers Do and McGuireWoods LLP Where They Can Do It): Part II T. Spahn (1/1/19) Hypotheticals and Analyses

decisions reflect most states' liberal approach to out-of-state lawyers engaging in arbitrations.

Some courts have taken a remarkably narrow view of what out-of-state lawyers may safely do within a state absent a pro hac vice admission. Until the West Virginia

Supreme Court took a different approach effective January 1, 2015, West Virginia followed an incredibly draconian approach -- described in a 2010 unauthorized practice of law opinion.

• West Virginia UPL Op. 10-001 (2010) (interpreting the then-current Rule 8.0 of the Rules for Admission to the Practice of Law, which explains the circumstances under which an out-of-state lawyer must apply for admission in West Virginia pro hac vice; "'Except in conformity with this Rule, members of the bar of any jurisdiction other than the State of West Virginia may not in this state do any act, or hold themselves out as entitled to do any act, within the definition of the practice of law, as prescribed by the Supreme Court of Appeals of West Virginia.'" (citation omitted); explaining the broad reach of that Rule; "As noted, Rule 8.0 (a) states that the requirement of admission pro hac vice is necessary before an applicant attorney can perform any act that falls within the definition of the practice of law. Based upon the clear language of the Rule, the Committee finds that attorneys licensed in states other than West Virginia must apply for admission pro hac vice in conformity with Rule 8 of the West Virginia Rules of Admission to the Practice of Law prior to engaging in any act that would fall within the definition of the practice of law. This requirement exists notwithstanding the absence of a pending action, suit or proceeding within which the applicant can seek to obtain an order granting admission pro hac vice. Under those circumstances, and along with the other requirements contained in Rule 8, the applicant must file a miscellaneous action in a West Virginia court of general jurisdiction to seek an order granting the applicant's admission."; holding that local West Virginia counsel must attend court proceedings, depositions and other events; "It is the concerted opinion of the Committee that these 'other actions' include any events that are brought about because of the existence of the in-court or out- of-court proceedings; that is, if the event is a necessary part of the proceedings -- such as depositions, arbitration, mediation, scheduling conference before a court employee other than the presiding judge, etc. -- then the responsible local counsel is required to attend." (emphasis added); explaining that the local lawyer can attend by video or telephone if the out-of- state lawyer can also attend in the same fashion; "Assuming that personal attendance is not required by the presiding judge, tribunal or other body of 139

35951124_10 UPL, MDP and MJP (Defining What Lawyers Do and McGuireWoods LLP Where They Can Do It): Part II T. Spahn (1/1/19) Hypotheticals and Analyses

the State of West Virginia, the responsible local counsel may attend the proceeding, deposition or other action by telephone or video-conferencing if the attorney admitted pro hac vice appears in a similar manner."; also concluding that a court cannot relieve a local lawyer of these obligations; "Any order purporting to release a responsible local counsel that was entered after the entry of the 2000 order of the West Virginia Supreme Court amending Rule 8 is, therefore, void and is of no effect. Responsible local counsel shall appear at all proceedings, depositions and other actions consistent with this Advisory Opinion."; also concluding that courts can decide whether to deny a motion for admission pro hac vice; "'if the applicant's appearances within the State of West Virginia within the past 24 months are numerous or frequent or involve improper conduct, the court or tribunal shall deny such person the continuing privilege of appearance.'" (citation omitted)).

As of January 1, 2015, West Virginia adopted a more measured approach. The new rule allows out-of-state lawyers to conduct certain activities in West Virginia without being admitted pro hac, including: providing legal services "pertaining to or an aid of" proceedings pending in other states; consulting with a West Virginia lawyer about a

"pending or potential proceeding in West Virginia" in which the out-of-state lawyer

"reasonably believes he is eligible for admission pro hac"; providing legal services in

West Virginia in preparation for a "potential case" to be filed in West Virginia on behalf of a client residing in West Virginia or elsewhere; preparing for and participating in an

ADR proceeding (including arbitrations and mediations) regardless of where it is

"expected to take place or actually takes place." West Virginia Rule of Prof'l Conduct

8(k).

Revised West Virginia Rule 8 requires out-of-state lawyers admitted pro hac to associate with a West Virginia lawyer who maintains "an actual physical office equipped to conduct a practice of law in the State of West Virginia, which office is the primary location from which local counsel practices law on a daily basis"; adds his or her name

140

35951124_10 UPL, MDP and MJP (Defining What Lawyers Do and McGuireWoods LLP Where They Can Do It): Part II T. Spahn (1/1/19) Hypotheticals and Analyses

and West Virginia Bar ID number to all pleadings; "physically or electronically" signs all pleadings; personally appears and participates in all proceedings before a tribunal; attends depositions and other events, unless the court permits him or her to appear by telephone. West Virginia Rule of Prof'l Conduct 8(b).

As long as lawyers temporarily participate in arbitrations or mediations in states where they are not licensed, they should not run afoul of the other state's MJP rules. Of course, the analysis changes if the lawyer engages in such activities with sufficient frequency that a court or bar might find that the lawyer has established a "systematic and continuous" presence in the other state -- in which case the lawyer probably will cross the line into improper multijurisdictional practice in that other state.

Best Answer

The best answer to this hypothetical is YES (PROBABLY).

b 2/13, 3/17

141

35951124_10 UPL, MDP and MJP (Defining What Lawyers Do and McGuireWoods LLP Where They Can Do It): Part II T. Spahn (1/1/19) Hypotheticals and Analyses

Non-Litigation Work in Another State -- Definition of "Temporary" Presence

Hypothetical 12

Your client has asked you to handle a complex transaction with a company in a neighboring state -- where you are not licensed. You expect to spend as much as 18 months at the other company's headquarters negotiating and consummating the transaction.

May you spend 18 months working on a transaction in another state where you are not licensed?

YES

Analysis

ABA Model Rule 5.5 allows lawyers admitted in a United States jurisdiction to

practice law in another United States jurisdiction under certain conditions.

A lawyer admitted in another United States jurisdiction, and not disbarred or suspended from practice in any jurisdiction, may provide legal services on a temporary basis in this jurisdiction that:

(1) are undertaken in association with a lawyer who is admitted to practice in this jurisdiction and who actively participates in the matter;

(2) are in or reasonably related to a pending or potential proceeding before a tribunal in this or another jurisdiction, if the lawyer, or a person the lawyer is assisting, is authorized by law or order to appear in such proceeding or reasonably expects to be so authorized;

(3) are in or reasonably related to a pending or potential arbitration, mediation, or other alternative dispute resolution proceeding in this or another jurisdiction, if the services arise out of or are reasonably related to the lawyer's practice in a jurisdiction in which the lawyer is admitted to practice and are not services for which the forum requires pro hac vice admission; or

142

35951124_10 UPL, MDP and MJP (Defining What Lawyers Do and McGuireWoods LLP Where They Can Do It): Part II T. Spahn (1/1/19) Hypotheticals and Analyses

(4) are not within paragraphs (c)(2) or (c)(3) and arise out of or are reasonably related to the lawyer's practice in a jurisdiction in which the lawyer is admitted to practice.

ABA Model Rule 5.5(c).

This hypothetical addresses the requirement that the lawyer's provision of legal services be "on a temporary basis."

As in every other aspect of the multijurisdictional practice issue, the ABA takes a surprisingly broad approach to the meaning of the term "temporary basis."

There is no single test to determine whether a lawyer's services are provided on a "temporary basis" in this jurisdiction, and may therefore be permissible under paragraph (c). Services may be "temporary" even though the lawyer provides services in this jurisdiction on a recurring basis, or for an extended period of time, as when the lawyer is representing a client in a single lengthy negotiation or litigation.

ABA Model Rule 5.5 cmt. [6] (emphasis added).

The ACTEC Commentaries take a similarly broad approach.

Comment 6 suggests a liberal interpretation of "temporary basis." This is particularly important for estate lawyers practicing in close proximity to another state. For example, a Chicago lawyer providing estate counseling for Illinois clients is likely to find multiple occasions to analyze and opine on the laws of Wisconsin, Iowa, Indiana, and Michigan regarding titling, tax, and similar issues. In addition, the Chicago lawyer may need to prepare deeds and other documents according to the laws of one or more of these jurisdictions. Provided the Chicago lawyer otherwise complies with paragraph (c), the lawyer's legal services regarding the surrounding non-admitted jurisdictions would constitute practicing law in those jurisdictions on a "temporary basis."

On the other hand, a lawyer who is engaged to provide estate planning services by clients in a non-admitted jurisdiction and makes personal visits to those clients on a

143

35951124_10 UPL, MDP and MJP (Defining What Lawyers Do and McGuireWoods LLP Where They Can Do It): Part II T. Spahn (1/1/19) Hypotheticals and Analyses

recurring basis should be cautious in relying upon MPRC 5.5(c). While Comment 6 might lead the courts in the non-admitted jurisdiction to interpret "temporary basis" broadly, the comments are not binding. Thus, a lawyer in such circumstances should consider the desirability of joining the non-admitted jurisdiction's bar.

American College of Trust & Estate Counsel, Commentaries on the Model Rules of

Professional Conduct, Commentary on MRPC 5.5, at 203 (5th ed. 2016).

Very few cases or legal ethics opinions provide guidance on the meaning of the

term "temporary" in states' Rule 5.5. For instance, a 2011 Virginia legal ethics opinion1

interpreted the word "temporary" to mean the lawyers' involvement with Virginia law, not

his or her physical presence in Virginia. In Virginia LEO 1856 (9/19/11), the Virginia Bar

held that non-Virginia lawyers could continuously practice in Virginia, as long as the

lawyers practiced "exclusively federal law" or limited his or her practice to the "law of the

1 Virginia LEO 1856 (9/19/11) (explaining that Under Virginia Rule 5.5, non-Virginia lawyers "may not practice Virginia law on a 'systematic and continuous' basis," unless they (1) limit their practice to the "law of the jurisdiction/s where they are licensed"; (2) practice "exclusively federal law" under the federal supremacy clause (such as "lawyers with practices limited to immigration or military law or who practice before the Internal Revenue Service, the United States Tax Court, or the United States Patent and Trademark Office," although lawyers such as bankruptcy, patent or federal procurement lawyers must abide by courts' possible limitation of practice before the courts to members of the Virginia Bar, and may provide advice "such as the debtor's homestead exemption and status or priority of claims or liens" or "the assignment of the patent to a third party or the organization of a corporate entity to market or franchise the invention" only under the conditions mentioned immediately below; (3) "provide advice about Virginia law or matters peripheral to federal law (described immediately above) only if they do so on a "temporary and occasional" basis and (as stated in UPL Opinion 195) "under the direct supervision of a Virginia licensed lawyer before any of the [non-Virginia] lawyer's work product is delivered to the client" or if they "associate with an active member of the Virginia State Bar."; noting that Rule 5.5 overrules an earlier UPL Opinion about which law applies to a non-Virginia lawyer's practice of another state's law while physically in Virginia; thus, "New York law should govern whether a foreign lawyer not authorized to practice in New York may advise New York clients on matters involving New York law. The [non-Virginia] lawyer's physical presence in Virginia may not be a sufficient basis to apply Virginia's rules over New York's rules governing foreign lawyer practice." Contract lawyers hired to "work on a matter involving Virginia law" must either "be licensed in Virginia or work in association with a Virginia licensed lawyer in the firm on a temporary basis" -- although such a lawyer's practice "could be regarded as 'continuous and systematic'" if the non-Virginia contract lawyer is hired "to work on several and various Virginia matters/cases over a period of time."; concluding that such contract lawyers need not be licensed in Virginia if the lawyer is "hired to work only on matters involving federal law or the law of the jurisdiction in which the [non-Virginia] contract lawyer is admitted."). 144

35951124_10 UPL, MDP and MJP (Defining What Lawyers Do and McGuireWoods LLP Where They Can Do It): Part II T. Spahn (1/1/19) Hypotheticals and Analyses

jurisdiction/s where they are licensed." The Virginia Bar also indicated that such lawyers could "provide advice about Virginia law or matters peripheral to federal law . . .

only if they do so on a 'temporary and occasional' basis" (and working under the

supervision of a Virginia lawyer). Thus, Virginia apparently takes the position that a

non-Virginia lawyer continuously present in Virginia may take advantage of the

"temporary" exception in Virginia Rule 5.5 by providing "temporary and occasional"

advice about Virginia law. This contrasts sharply with the majority view, which interprets

the term "temporary" as referring to lawyer's physical presence in a state.

Other states take a more restrictive view. In 2008, the Delaware Supreme Court

indicated that a non-Delaware lawyer had violated Delaware's UPL provision by

accepting a monthly retainer to draft estate planning documents for clients of a

Delaware accountant.

• In re Kingsley, No. 138, 2008 Del. LEXIS 255, at *5 (Del. June 4, 2008) (holding that a lawyer licensed in Pennsylvania and New Jersey committed the unauthorized practice of law in Delaware by accepting a monthly retainer to draft estate planning documents for clients of a Delaware accountant; concluding that the lawyer established a "systematic and continuous legal presence" in Delaware by engaging in these activities; excluding the lawyer from practicing law in Delaware).

• Philadelphia LEO 2007-4 (8/2007) (addressing a request by a Pennsylvania lawyer about how to respond to a request by someone outside Pennsylvania that the lawyer provide legal services; directing the lawyer to research the UPL laws of the other jurisdiction; explaining that the other jurisdiction's UPL rules would also govern the permissibility of the lawyer providing "advice and consultation on matters of federal law, general legal principles and common law"; explaining that the other pertinent state might find that actions taken in Pennsylvania violated the UPL laws of that other state; "The inquirer also specifically asks if it makes a difference whether he performs legal services in Pennsylvania or in the subject jurisdiction. The answer to this question again depends on the law of that jurisdiction. See, e.g., Comment 4 to ABA Model Rule 5.5, which states that a lawyer may violate Model Rule 5.5(b) (which proscribes a lawyer who is not admitted in a jurisdiction from

145

35951124_10 UPL, MDP and MJP (Defining What Lawyers Do and McGuireWoods LLP Where They Can Do It): Part II T. Spahn (1/1/19) Hypotheticals and Analyses

systematic and continuous presence in that jurisdiction) even if the lawyer is not physically present in the jurisdiction. Most recently the Delaware Supreme Court took action against an attorney not even admitted in Delaware see In Re Tonwe, Del., No. 584, 2006, 5/25/07.").

Taking a similarly restrictive view in 2005, the District of Columbia Bar warned

lawyers from outside D.C. that they may violate that jurisdiction's UPL rule by handling

multiple short-term assignments in D.C. (which presumably would fall within the ABA

Model Rules' permissible "recurring" services in a state where the lawyer is not licensed).

• District of Columbia UPL Op. 16-05 (6/17/05) ("Rule 49 also applies if a contract lawyer regularly takes short-term assignments in the District of Columbia. A contract lawyer, as defined for purposes of this Opinion, is employed for a limited period or project, and a contract lawyer may accept assignments outside the District of Columbia or have periods of non- employment between assignments. However, a contract lawyer whose presence in the District of Columbia as a contract lawyer is not occasional or incidental within the meaning of Rule 49(b)(3) is subject to Rule 49, even if each assignment, considered in isolation, might constitute only incidental or occasional presence here. For example, a contract lawyer who regularly practices in the District despite occasional projects in other jurisdictions is engaged in the practice of law, and holding out as authorized to practice law, 'in' the District of Columbia within the meaning of Rule 49(b)(3). Conversely, a contract lawyer who practices primarily in another jurisdiction generally is not engaging in the practice of law 'in' the District of Columbia simply by taking on one discrete project here on an incidental basis." (emphases added)).2

2 District of Columbia UPL Op. 16-05 (6/17/05) (holding that "practicing law in the District of Columbia as a contract lawyer is no different than practicing law as a non-contract partner, associate or other employee. Unless the contract lawyer independently qualifies for one of the exceptions to Rule 49, the lawyer must be a member of the D.C. Bar."; explaining that a lawyer from other jurisdiction seeking to waive into the D.C. Bar must act within 90 days "after commencing practice in the District"; explaining that the lawyer rules do not apply to "paralegal work" such as document or privilege review; explaining that determining whether a contract lawyer performing such paralegal work must comply with the D.C. UPL Rules "depends on whether the person is being held out, and billed out, as a lawyer or as a paralegal. Rule 49 does not regulate the hiring of a person as a paralegal or a law clerk, even though the person may be admitted to the practice of law in another jurisdiction. When a person is hired and billed as a lawyer, however, the person is generally engaged in the practice of law, and is certainly being held out as authorized or competent to practice law. Clients would reasonably assume that the person held out as a contract lawyer performs functions that are different in degree, if not in kind, from those performed by paralegals or law clerks, and that the cost of services performed by contract lawyers reflects the legal 146

35951124_10 UPL, MDP and MJP (Defining What Lawyers Do and McGuireWoods LLP Where They Can Do It): Part II T. Spahn (1/1/19) Hypotheticals and Analyses

Thus, the District of Columbia Bar did not consider each of the assignments discretely,

but instead lumped them together to find that lawyers engaging in such activities in D.C.

would be considered "continuously" practicing law there.

Best Answer

The best answer to this hypothetical is YES.

B 2/13

training and judgment that they bring to the work they perform. When a client is paying for the services of a lawyer, and not a paralegal or a law clerk, the person providing the services and the person's employer must comply with Rule 49."; also explaining that "if a contract lawyer is supervised not as a paralegal or law clerk but as a subordinate attorney would be supervised, the contract lawyer is engaged in the practice of law"; also explaining that "Rule 49 also applies if a contract lawyer regularly takes short-term assignments in the District of Columbia. A contract lawyer, as defined for purposes of this Opinion, is employed for a limited period or project, and a contract lawyer may accept assignments outside the District of Columbia or have periods of non-employment between assignments. However, a contract lawyer whose presence in the District of Columbia as a contract lawyer is not occasional or incidental within the meaning of Rule 49(b)(3) is subject to Rule 49, even if each assignment, considered in isolation, might constitute only incidental or occasional presence here. For example, a contract lawyer who regularly practices in the District despite occasional projects in other jurisdictions is engaged in the practice of law, and holding out as authorized to practice law, 'in' the District of Columbia within the meaning of Rule 49(b)(3). Conversely, a contract lawyer who practices primarily in another jurisdiction generally is not engaging in the practice of law 'in' the District of Columbia simply by taking on one discrete project here on an incidental basis."). 147

35951124_10 UPL, MDP and MJP (Defining What Lawyers Do and McGuireWoods LLP Where They Can Do It): Part II T. Spahn (1/1/19) Hypotheticals and Analyses

Requirement of a Relationship with the Lawyer's Home State: Narrow View

Hypothetical 13

You graduated from Vanderbilt Law School, and formerly served as Dean of Student Life at Georgia State University. Along with your partner (a University of Georgia Law School graduate and former regional counsel of the U.S. Department of Energy) you were retained by a North Carolina college to conduct an internal investigation into the college's president. The president was accused of manipulating a basketball player's grade point average so that the student could continue to play basketball. You are a member of the Tennessee and Georgia Bars, and your partner is a member of the Georgia Bar. Neither of you are members of the North Carolina Bar. As part of your investigation, you conducted interviews in North Carolina and provided your report to the North Carolina college's Board of Trustees.

Have you engaged in the improper multijurisdictional practice of law in North Carolina?

NO (PROBABLY)

Analysis

ABA Model Rule 5.5 allows lawyers admitted in a United States jurisdiction to practice law in another United States jurisdiction under certain conditions.

A lawyer admitted in another United States jurisdiction, and not disbarred or suspended from practice in any jurisdiction, may provide legal services on a temporary basis in this jurisdiction that:

(1) are undertaken in association with a lawyer who is admitted to practice in this jurisdiction and who actively participates in the matter;

(2) are in or reasonably related to a pending or potential proceeding before a tribunal in this or another jurisdiction, if the lawyer, or a person the lawyer is assisting, is authorized by law or order to appear in such proceeding or reasonably expects to be so authorized;

(3) are in or reasonably related to a pending or potential arbitration, mediation, or other alternative dispute resolution

148

35951124_10 UPL, MDP and MJP (Defining What Lawyers Do and McGuireWoods LLP Where They Can Do It): Part II T. Spahn (1/1/19) Hypotheticals and Analyses

proceeding in this or another jurisdiction, if the services arise out of or are reasonably related to the lawyer's practice in a jurisdiction in which the lawyer is admitted to practice and are not services for which the forum requires pro hac vice admission; or

(4) are not within paragraphs (c)(2) or (c)(3) and arise out of or are reasonably related to the lawyer's practice in a jurisdiction in which the lawyer is admitted to practice.

ABA Model Rule 5.5(c). Most states have adopted ABA Model Rule 5.5 or a variation of that rule.

The occasional aberrational decision creates some uncertainty about what is becoming a national consensus on the basic contours of acceptable multijurisdictional practice.

Perhaps most notably, in 2004 a North Carolina grand jury indicted two Georgia lawyers and their law firm for engaging in the unauthorized practice of law in North

Carolina.1

1 Jonathan Ringel, Georgia Lawyers Indicted for Advising North Carolina College, Fulton County Daily Report, Apr. 8, 2004 ("A North Carolina grand jury has indicted Atlanta lawyers F. Edwin Hallman Jr. and H. King Buttermore III -- along with their firm Decker, Hallman, Barber & Briggs -- for the unauthorized practice of law."; "Cleveland County District Attorney William C. Young said the misdemeanor charges could result in probated sentences for Hallman and Buttermore and an order that the firm return fees paid by Gardner-Webb University, a 3,500-student Baptist school in Boiling Springs, North Carolina."; "While a bitter internal dispute at the university seems to have prompted the charges, the action raises the question of how far law firms can go in serving clients in states where they are not members of the bar."; "The firm in 2002 conducted an investigation that largely cleared M. Christopher White, Gardner-Webb's president, of wrongdoing for ordering that a basketball player's grade point average be recalculated in a way that made the student eligible for the season. The firm's report, however, led the school's board of trustees to reassign two faculty members who had been critical of White's actions -- moves that led several faculty members to resign in protest. White later resigned."; "The National Collegiate Athletic Association last month cited White's grade change order, among other things, in its decision to put Gardner-Webb on a three-year probation."; "In response to calls to Hallman and Buttermore, the firm issued a statement to the Daily Report declaring that its lawyers conducted interviews and provided a report to the board of trustees at the school -- but did not practice law."; "'It is difficult for us to understand what is motivating these actions against us,' the statement read. 'If former university employees, or those unhappy with the steps taken by the Board of Trustees, are disgruntled and seeking some form of retribution against this firm, their ire is misguided.'"; noting that "Hallman, 58, is a graduate of the University of Georgia School of Law and is a former regional counsel of the U.S. Department of Energy. He is a member of the State Bar of Georgia, but not of North Carolina. 149

35951124_10 UPL, MDP and MJP (Defining What Lawyers Do and McGuireWoods LLP Where They Can Do It): Part II T. Spahn (1/1/19) Hypotheticals and Analyses

The two lawyers had conducted an internal investigation into alleged wrongdoing

by Gardner-Webb University's then-president, who had been accused of manipulating a

student's grade to make him eligible for basketball season. A newspaper article

reporting the indictment noted the lawyers' impressive pedigrees.

Hallman, 58, is a graduate of the University of Georgia School of Law and is a former regional counsel of the U.S. Department of Energy. He is a member of the State Bar of Georgia, but not of North Carolina. Buttermore, 59, is a graduate of Vanderbilt University School of Law, a former dean of student life at Georgia State University, and a member of the Tennessee and Georgia bars, but not North Carolina.

Jonathan Ringel, Georgia Lawyers Indicted for Advising North Carolina College, Fulton

County Daily Report, Apr. 8, 2004.

The article also noted that the North Carolina State Bar's unauthorized practice

committee issued a "Letter of Caution" to the lawyers' North Carolina lawyer. According

to the newspaper article,

'[t]he letter said the firm's report to the university went beyond an investigation into the alleged NCAA and school

Buttermore, 59, is a graduate of Vanderbilt University School of Law, a former dean of student life at Georgia State University, and a member of the Tennessee and Georgia bars, but not North Carolina."; noting that "[t]he lawyers' letters defending their report and their representation ultimately became evidence that Gardner used against the firm. One letter by Hallman tells a critic that, 'As attorneys for the University, we have a duty to respond to the University's concerns. . . . We represent the University. . . . We stand by our legal advice given to the Trustees.'"; "The North Carolina State Bar's Authorized Practice Committee appeared to agree. On July 31, it issued a 'Letter of Caution' to Thomas J. Dimmock, a Raleigh, N.C., lawyer who represented Hallman, Buttermore and the firm before the committee."; "The letter said the firm's report to the university went beyond an investigation into the alleged NCAA and school violations and 'also outlined and advised the Board concerning the University's potential legal liabilities.'"; "The caution letter cited other letters and communications in which Hallman or Buttermore provided legal assessments for the trustees."; "'It is the unauthorized practice of law for any persons other than licensed members of the North Carolina State Bar to practice law in the State of North Carolina,' the letter said. 'The practice of law includes preparation of legal documents, engaging in legal advice or counsel, acting as attorney, or furnishing the services as an attorney. North Carolina General Statutes § 84-2.1 and 4."; "'Your clients held themselves out as attorneys for the University, both to the Board of Trustees and to outside parties,' the letter continued. 'The Committee concluded that the conduct of your clients constituted the unauthorized practice of law in violation of these statutes.'"). 150

35951124_10 UPL, MDP and MJP (Defining What Lawyers Do and McGuireWoods LLP Where They Can Do It): Part II T. Spahn (1/1/19) Hypotheticals and Analyses

violations and 'also outlined and advised the Board concerning the University's potential legal liabilities.'

Id.

These indictments created quite a shock among lawyers expecting that most states would abandon their traditional parochial approach to multijurisdictional practice, and move in the direction of the ABA Model Rules. These indictments may stand as a tribute to some university trustee's influence with the local sheriff, rather than represent some jurisprudential trend.

North Carolina has since then adopted Rule 5.5 provisions that presumably would be more forgiving.

But other states have also taken a harsh approach to out-of-state lawyers.

• Ohio State Bar Ass'n v. Klosk, --- N.E.3d ---, 2018-Ohio-4864, at ¶¶ 1, 2, 9, 11, 12 (Ohio Dec. 11, 2018) (holding that a lawyer licensed in California, New Jersey and New York committed the illegal unauthorized practice of law in Ohio by representing an Ohio resident in negotiating resolution of his debt with an Ohio-based creditor's lawyer; enjoining the further illegal practice of law in Ohio and assessing a $2,000 civil penalty; inexplicably failing to mention Ohio Rule 5.5, the multijurisdictional practice rule that allows (among other things) out-of-state lawyers to temporarily practice in Ohio if there is a reasonable connection with the lawyer's home state; "Relator, Ohio State Bar Association, charged respondents, Michael J. Klosk, an attorney licensed to practice law in California, and Klosk Law Firm, Inc., a California law firm, with engaging in the unauthorized practice of law in Ohio. Relator alleged that Klosk and his form provided legal advice to an Ohio resident and/or negotiated the resolution of an Ohio resident's debt with a creditor's counsel who was also located in the state of Ohio." (footnote omitted); "Finding that Klosk and his firm practiced law in violation of Ohio licensure requirements, the Board on the Unauthorized Practice of Law recommends that we enjoin them from committing further illegal acts and assess a $2,000 civil penalty. We agree that Klosk and Klosk Law Firm, Inc., engaged in the unauthorized practice of law in Ohio and that an injunction and $2,000 civil penalty are warranted."; "[I]t is undisputed that Klosk and Klosk Law Firm, Inc., contacted Ohio counsel for CitiFinancial and indicated that they represented an Ohio resident in debt negotiations without the benefit of any legal counsel licensed to practice in Ohio."; "[A]n individual who is not licensed to practice law in

151

35951124_10 UPL, MDP and MJP (Defining What Lawyers Do and McGuireWoods LLP Where They Can Do It): Part II T. Spahn (1/1/19) Hypotheticals and Analyses

Ohio who negotiates legal claims on behalf of Ohio residents or advises Ohio residents of their legal rights or the terms and conditions of settlement is engaged in the unauthorized practice of law."; "Based on the facts in the record and the applicable law, there is no doubt that Klosk and the Klosk Law Firm, Inc., engaged in the unauthorized practice of law. They have conceded that neither Klosk nor the firm has ever been admitted to the practice of law in Ohio and that no other member of the firm is so admitted. Despite those facts, they represented an Ohio resident in debtor-creditor negotiations with creditor's Ohio counsel. We therefore find that Klosk and the Klosk Law Firm, Inc., engaged in the unauthorized practice of law as defined in Gov. Bar R. VII(2)(A) and hold that summary judgment in realtor's favor is appropriate.").

• In re Nadel, 82 A.3d 716, 718, 718-19, 719, 723 (Del. 2013) ("The Panel recommended that Nadel be suspended from practicing law for one year; prohibited from providing advice to any Delaware clients for a period of one year; prohibited from any admission pro hac vice for a period of three years; be publicly sanctioned; and pay the costs of the disciplinary proceedings."; "Nadel is not a member of the Bar of the Supreme Court of Delaware. He was admitted to the Bars of the State of New Jersey and the Commonwealth of Pennsylvania in 1982. Nadel currently practices in a private firm located in Cherry Hill and Pennsauken, New Jersey."; "From April 2009 through September 21, 2012, Nadel engaged in the unauthorized practice of law in Delaware. After he was initially asked to help a patient by a Delaware doctor, Dr. Morris Peterzell, Nadel met with more than seventy-five Delaware residents who were involved in auto accidents. These accidents occurred in Delaware and involved Delaware insurance policies governed by Delaware law. Nadel met with roughly half of his Delaware clients at Dr. Peterzell's medical office in Wilmington. But in each instance, Nadel would attempt to settle the insurance claims on behalf of his Delaware clients. If settlement proved unsuccessful, Nadel would turn the case over to local Delaware counsel to pursue the litigation."; "Nadel never filed a lawsuit in Delaware or made any representations to a Delaware court. Further, Nadel never advertised or actively solicited clients. Nor did he ever represent to a Delaware citizen that he was a member of the Delaware bar. But Nadel does admit that by meeting with his Delaware clients in Delaware, he could have unintentionally created the impression that he was licensed to practice law in Delaware. No actual harm resulted from Nadel's representation. But these Delaware clients accounted for ten to fifteen percent of Nadel's legal practice."; "The Panel recommended a series of sanctions in addition to a suspension. The recommended sanctions included: 1) [Nadel] be suspended from the practice of law in the State of Delaware for a period of one year; 2) [Nadel] be prohibited from providing advice to any Delaware clients on matters of Delaware law for period of one year; 3) [Nadel] be prohibited from acting pro hac vice on any matter in Delaware for a period of

152

35951124_10 UPL, MDP and MJP (Defining What Lawyers Do and McGuireWoods LLP Where They Can Do It): Part II T. Spahn (1/1/19) Hypotheticals and Analyses

three years; 4) The contents of [its] report be made public; and 5) [Nadel] pay the costs of these proceedings." (emphases added)).

• In re Lenard, Cal. Bar Court Review Dep't Case No. 09-O-11175 (Apr. 15, 2013) (disbarring a lawyer for engaging in the unauthorized practice of law by providing "credit repair" services to debtors in several states where the lawyer was not licensed; "Lenard contracted with three California consumer debt relief companies: Freedom Financial Management; Beacon Debt Service; and Pathway Financial Management (the Settlement Companies). These companies paid Lenard a flat fee to provide limited legal services for clients regarding their consumer debt. Lenard testified that he customarily charged the Settlement Companies between $75 to $100 per client and spent 15 to 20 minutes on each file. He also estimated that he had over 1,000 clients 'in credit repair' among all three companies. The Settlement Companies advertised through television and radio ads in a number of states. Clients who retained one of the Settlement Companies agreed to pay retainer fees of up to 12% of the balance of their debts, contingency fees of 8% of the amount by which their debts were reduced, and monthly maintenance fees of between $15 to $25. Clients also were required to make monthly payments into the Companies' 'client trust account,' and those funds were to be used to settle their debts. The Settlement Companies represented that the clients' accounts would be 'handled by our legal counsel.'"; "Lenard practiced law and held himself out as an attorney with the authority and knowledge to settle consumer debts to Wisconsin and New York clients Burgess and Manfredi, respectively. He also represented to their creditors that they should follow debt collection laws or his clients were prepared to take legal action. In addition, Lenard claims he reviewed their files to determine whether they should file bankruptcy, although he admitted he was 'not licensed to do a bankruptcy out of state.' Wisconsin and New York have both considered conduct similar to Lenard's to constitute UPL."; "The hearing judge found that Lenard established a systematic and continuous presence in each of the jurisdictions listed in the NDCs. Based on the limited record, we do not find clear and convincing evidence of this proscription. However, we find that Lenard committed UPL by holding himself out as entitled to practice law in each of the seven states for a total of ten willful violations of rule 1-300(B)" (footnote omitted); "By implying he was licensed in the relevant states, Lenard gave the false impression to his clients and their creditors that he held an advantage over a non-attorney debt negotiator. He explicitly represented to the clients that he would provide legal services, and informed creditors that he was representing each client utilizing his law office letterhead. The written communications Lenard provided to clients (and their creditors) in those states are evidence that he violated the applicable rules of professional conduct, as well as relevant case law and advisory authority."; "He [Lenard] contends that all work was done in California and any legal opinions rendered were based on California law.

153

35951124_10 UPL, MDP and MJP (Defining What Lawyers Do and McGuireWoods LLP Where They Can Do It): Part II T. Spahn (1/1/19) Hypotheticals and Analyses

However, the factors defined in comment 14 of the ABA Model Rule compel our conclusion that Lenard was not entitled to practice law even on a temporary basis in these states. Analyzing those factors, we find that he had no prior contact with the clients and they never lived in California or had substantial contact with this state. There is no evidence that California law would be relevant to any of the consumer debts in these matters. Further, Lenard has no knowledge of the specific laws of the states in which the clients resided, where they faced state collection actions and may have had assets. As such, the contact with these out-of-state clients was not reasonably related to Lenard's practice in California, and he was not authorized to provide legal services on a temporary basis under the states' versions of ABA Model Rule 5.5(c)." (footnote omitted); "[W]e reject any contention by Lenard that ABA Model Rule 5.5(d)(2) enabled him to provide legal services related to bankruptcy law. Primarily, Lenard's proposed services were not limited to issues of bankruptcy." (emphases added)).

• Ohio LEO 2011-2 (10/7/11) ("An out-of-state lawyer is not authorized to provide debt settlement legal services, including investigation, negotiation, and other nonlitigation activities, on a temporary basis in Ohio under Prof. Cond. Rule 5.5(c)(4) when the matter is not connected to the lawyer's jurisdiction of admission to the practice of law, there is not a pre-existing relationship between the lawyer and the client, and the lawyer does not have a recognized expertise in a particular body of federal, nationally-uniform, foreign, or international law that is applicable to the matter. Under these circumstances, the nonlitigation activities do not arise out of, and are not reasonably related to, the lawyer's practice in his or her jurisdiction of admission for purposes of Prof. Cond. Rule 5.5(c)(4)."; "Applying the seven factors outlined in Prof. Cond. Rule 5.5, Comment [14], and the concepts detailed in the legislative history of Model Rule 5.5, the Board concludes that the hypothetical out-of-state debt settlement lawyers are not authorized to practice law temporarily in Ohio. The Ohio clients became aware of the out-of-state lawyers for the first time through an internet search, and had no prior contact or relationship with the lawyers. The clients are not residents of the lawyers' home jurisdictions of licensure, and the work done for the client crosses into a number of jurisdictions where the creditors are located. The lawyers may be physically located in their state of licensure, but a significant portion of the work done for the clients is not centrally located in that home state. The debt settlement work performed by the out-of-state lawyers is likely not governed by the law of the lawyers' home state, as the Ohio clients may be facing state law collection actions and presumably have Ohio assets as well as income and debt incurrent in Ohio."; "In regard to any 'recognized expertise' in a 'particular body of federal, nationally-uniform, foreign, or international law,' it is the Board's opinion that general debtor/credit law is not the type of nationally-uniform law intended by the drafters that, without more of a nexus between lawyer and client, would permit multijurisdictional

154

35951124_10 UPL, MDP and MJP (Defining What Lawyers Do and McGuireWoods LLP Where They Can Do It): Part II T. Spahn (1/1/19) Hypotheticals and Analyses

practice under Prof. Cond. Rule 5.5(c)(4). The primary areas of concern for clients seeking debt settlement legal services are consumer protection, debt collection, garnishment, repossession, foreclosure, and general contract law. Even though some federal statutes may come into play, such as the Fair Debt Collection Practices Act, all of these areas implicate state law in Ohio. In addition, an internet search for 'debt settlement law firms,' similar to the search probably conducted by the Ohio clients, produces approximately two million results. Facing this many website options, a typical consumer could not realistically identify lawyers with a 'recognized expertise' in an area of nationally-uniform law that may justify temporary practice in Ohio." (footnotes omitted); "[T]emporary practice by out-of-state lawyers serves the interests of the public and clients when there is a connection to a home state matter, an existing client-lawyer relationship, or a recognized expertise in an area of federal, nationally-uniform, foreign, or international law. The debt settlement hypothetical presented to the Board does not fulfill these criteria, especially given that lawyers are exempt from the Ohio Debt Adjusters Act, which protects Ohio consumers receiving debt settlement services. The Board cannot conclude that it is in the best interests of clients and the public to allow out-of-state lawyers to provide debt settlement legal services to Ohio clients whose sole connection to the lawyers is an internet search." (footnote omitted)).

Best Answer

The best answer to this hypothetical is NO (PROBABLY).

B 2/13, 3/17

155

35951124_10 UPL, MDP and MJP (Defining What Lawyers Do and McGuireWoods LLP Where They Can Do It): Part II T. Spahn (1/1/19) Hypotheticals and Analyses

Requirement of a Relationship with the Lawyer's Home State: State Variations

Hypothetical 14

You have practiced environmental law in Colorado for approximately twenty years. About five months ago, you received a call from your in-laws, who live in Minnesota. Their condo association had a judgment entered against them for a little over $2,000, which they think was unfair. They asked you whether you could try to resolve their dispute with their condo association. You have exchanged approximately two dozen emails with the condo association's lawyer. You were shocked to learn that the Minnesota lawyer just filed a bar complaint in Minnesota, alleging that your email exchanges amounted to the "practice of law" in Minnesota, and that you have engaged in the illegal unauthorized practice of law in Minnesota.

Have you engaged in the illegal unauthorized practice of law in Minnesota?

MAYBE

Analysis

Some states have not adopted any variation of the ABA Model Rule in their ethics rules. Notably, California has not done so, despite having moved on November

1, 2018 to the ABA Model Rules format.

A lawyer admitted to practice law in California shall not: (1) practice law in a jurisdiction where to do so would be in violation of reguations of the profession in that jurisdiction; or (2) knowingly assist a person in the unauthorized practice of law in that jurisdiction.

California Rule 5.5(a).

A comment provides some meager guidance

Paragraph (b)(1) prohibits lawyers from practicing law in California unless otherwise entitled to practice law in this state by court rule or other law. (See, e.g., Bus. & Prof. Code, § 6125 et seq.; see also Cal. Rules of Court, rules 9.40 [counsel pro hac vice], 9.41 [appearances by military counsel], 9.42 [certified law students], 9.43 [out-of-state attorney arbitration counsel program], 9.44 [registered

156

35951124_10 UPL, MDP and MJP (Defining What Lawyers Do and McGuireWoods LLP Where They Can Do It): Part II T. Spahn (1/1/19) Hypotheticals and Analyses

foreign legal consultant], 9.45 [registered legal services attorneys], 9.46 [registered in-house counsel], 9.47 [attorneys practicing temporarily in California as part of litigation], 9.48 [non-litigating attorneys temporarily in California to provide legal services].

California Rule 5.5(a) cmt. California deals with litigators' temporary practice in

California in Cal. Rule of Court 9.47, and transactional lawyers' temporary practice in

California in Cal. Rule of Court 9.48.

Other states have made the same decision to leave the issue open.

• Hawaii Rule 5.5 ("A lawyer shall not: (a) practice law in a jurisdiction where doing so violates the regulation of the legal profession in that jurisdiction; or (b) assist a person who is not a member of the bar in the performance of activity that constitutes the unauthorized practice of law; or (c) allow any person who has been suspended or disbarred and who maintains a presence in an office where the practice of law is conducted by the lawyer to have any contact with the clients of the lawyer either in person, by telephone, or in writing or to have contact with persons who have legal dealings with the office either in person, by telephone, or in writing.").

Out-of-state lawyers wondering what they may legally do in these and other similar states have no guidance. And they must worry both about what those states might do to them and what their home states might do to them.

• In re Velahos, Dkt. No. DRB 15-109, at 6 (N.J. Sup. Ct. Disciplinary Review Bd. May 23, 2016) (suspending for six months a lawyer for various ethics violations; noting the disciplinary review board's findings from 3/23/16; "In fact, respondent represented clients in multiple matters in jurisdictions in which he was not authorized to practice, without the assistance of local counsel. Respondent conducted no less than eighteen mortgage modifications in the States of Georgia, Washington, New York, Pennsylvania, Virginia, Maryland, Connecticut, Texas, or Florida. Respondent misrepresented to several of these out-of-state clients in the fee agreements that FLA 'has been retained as 'Of Counsel' to Loan Law Center.' Moreover, respondent engaged in credit and debt adjustment services in Maryland over a two-year period, even after the Commissioner of Financial Regulation for the State of Maryland issued a summary order, followed by a final order to Cease and Desist. When questioned by the OAE about the orders, respondent denied that he had 'taken any money' from Maryland. However,

157

35951124_10 UPL, MDP and MJP (Defining What Lawyers Do and McGuireWoods LLP Where They Can Do It): Part II T. Spahn (1/1/19) Hypotheticals and Analyses

the OAE's review of respondent's records disclosed that, during that period, respondent actively represented several Maryland clients in that state and collected fees from them. Respondent's conduct in this respect violated RPC 1.16(a)(1), RPC 5.5(a), RPC 8.1(a), RPC 8.4(c), and RPC 8.4(d)." (emphasis added), aff'd, 137 A.3d 500 (N.J. May 25, 2016)).

ABA Model Rules

ABA Model Rule 5.5 allows lawyers admitted in a United States jurisdiction to

practice law in another United States jurisdiction under certain conditions.

A lawyer admitted in another United States jurisdiction, and not disbarred or suspended from practice in any jurisdiction, may provide legal services on a temporary basis in this jurisdiction that:

(1) are undertaken in association with a lawyer who is admitted to practice in this jurisdiction and who actively participates in the matter;

(2) are in or reasonably related to a pending or potential proceeding before a tribunal in this or another jurisdiction, if the lawyer, or a person the lawyer is assisting, is authorized by law or order to appear in such proceeding or reasonably expects to be so authorized;

(3) are in or reasonably related to a pending or potential arbitration, mediation, or other alternative dispute resolution proceeding in this or another jurisdiction, if the services arise out of or are reasonably related to the lawyer's practice in a jurisdiction in which the lawyer is admitted to practice and are not services for which the forum requires pro hac vice admission; or

(4) are not within paragraphs (c)(2) or (c)(3) and arise out of or are reasonably related to the lawyer's practice in a jurisdiction in which the lawyer is admitted to practice.

ABA Model Rule 5.5(c).

This hypothetical deals with the requirement that the temporary legal services

"arise out of or are reasonably related to the lawyer's practice" in the lawyer's home

state. 158

35951124_10 UPL, MDP and MJP (Defining What Lawyers Do and McGuireWoods LLP Where They Can Do It): Part II T. Spahn (1/1/19) Hypotheticals and Analyses

A comment shows just how broadly the ABA interprets this provision.

Paragraph (c)(4) permits a lawyer admitted in another jurisdiction to provide certain legal services on a temporary basis in this jurisdiction that arise out of or are reasonably related to the lawyer's practice in a jurisdiction in which the lawyer is admitted but are not within paragraphs (c)(2) or (c)(3). These services include both legal services and services that non-lawyers may perform but that are considered the practice of law when performed by lawyers.

ABA Model Rule 5.5 cmt. [13] (emphasis added).

The next comment takes a remarkably broad view of what type of "relationship" to the lawyer's home state suffices under this catch-all provision.

Paragraph (c)(3) (c)(4) require that the services arise out of or be reasonably related to the lawyer's practice in a jurisdiction in which the lawyer is admitted. A variety of factors evidence such a relationship. The lawyer's client may have been previously represented by the lawyer, or may be resident in or have substantial contacts with the jurisdiction in which the lawyer is admitted. The matter, although involving other jurisdictions, may have a significant connection with that jurisdiction. In other cases, significant aspects of the lawyer's work might be conducted in that jurisdiction or a significant aspect of the matter may involve the law of that jurisdiction. The necessary relationship might arise when the client's activities or the legal issues involve multiple jurisdictions, such as when the officers of a multinational corporation survey potential business sites and seek the services of their lawyer in assessing the relative merits of each. In addition, the services may draw on the lawyer's recognized expertise developed through the regular practice of law on behalf of clients in matters involving a particular body of federal, nationally-uniform, foreign, or international law. Lawyers desiring to provide pro bono legal services on a temporary basis in a jurisdiction that has been affected by a major disaster, but in which they are not otherwise authorized to practice law, as well as lawyers from the affected jurisdiction who seek to practice law temporarily in another jurisdiction, but in which they are not otherwise authorized to practice law, should consult the [Model Court

159

35951124_10 UPL, MDP and MJP (Defining What Lawyers Do and McGuireWoods LLP Where They Can Do It): Part II T. Spahn (1/1/19) Hypotheticals and Analyses

Rule on Provision of Legal Services Following Determination of Major Disaster.]

ABA Model Rule 5.5 cmt. [14] (emphases added).

Thus, the client does not have to be based in the lawyer's home jurisdiction -- as long as the client has "substantial contacts" with that jurisdiction.

In fact, the client does not have to have any connection to the lawyer's home jurisdiction -- it is enough that the "matter" has a "significant connection" to the home jurisdiction, even if it involves other jurisdictions. All in all, the ABA's catch-all provision permits nearly any conceivable type of "temporary" provision of legal services in other states.

Restatement

The Restatement takes a similarly broad approach.

A lawyer currently admitted to practice in a jurisdiction may provide legal services to a client . . . at a place within a jurisdiction in which the lawyer is not admitted to the extent that the lawyer's activities arise out of or are otherwise reasonably related to the lawyer's practice [where the lawyer is admitted].

Restatement (Third) of Law Governing Lawyers § 3(3) (2000). Thus, the Restatement uses the same basic "reasonably related to" standard as the ABA Model Rules.

The Restatement's comments match the ABA's expansive comments.

When other activities of a lawyer in a non-home state are challenged as impermissible for lack of admission to the state's bar, the context in which and purposes for which the lawyer acts should be carefully assessed. Beyond home- state activities, proper representation of clients often requires a lawyer to conduct activities while physically present in one or more other states. Such practice is customary in many areas of legal representation. As stated in Subsection (3), such activities should be recognized as 160

35951124_10 UPL, MDP and MJP (Defining What Lawyers Do and McGuireWoods LLP Where They Can Do It): Part II T. Spahn (1/1/19) Hypotheticals and Analyses

permissible so long as they arise out of or otherwise reasonably relate to the lawyer's practice in a state of admission. In determining that issue, several factors are relevant, including the following: whether the lawyer's client is a regular client of the lawyer or, if a new client, is from the lawyer's home state, has extensive contacts with that state, or contacted the lawyer there; whether a multistate transaction has other significant connections with the lawyer's home state; whether significant aspects of the lawyer's activities are conducted in the lawyer's home state; whether a significant aspect of the matter involves the law of the lawyer's home state; and whether either the activities of the client involve multiple jurisdictions or the legal issues involved are primarily either multistate or federal in nature.

Restatement (Third) of Law Governing Lawyers § 3 cmt. e (2000) (emphasis added).

The Restatement's approach essentially mirrors the ABA Model Rules approach, but contains a number of factors that reflect an even more liberal approach than the

ABA Model Rules.

• First, one Restatement factor in determining the "relationship" to a lawyer's home state is whether the prospective client "contacted the lawyer there." That provision (which does not appear in ABA Model Rule 5.5 cmt. [14]), would dramatically expand a lawyer's freedom to temporarily practice in other states.

• Second, the Restatement's focus on legal issues includes a reference to legal issues that "are primarily either multistate or federal in nature." ABA Model Rule 5.5 cmt. [14] uses a slightly different term: "a particular body of federal, nationally-uniform, foreign or international law." At least on its face, the Restatement seems to take a broader approach -- legal issues can be "multistate" even if they do not involve "nationally-uniform" law. However, ABA Model Rule 5.5 cmt. [14] also explains that lawyers may assist corporate clients "when the client's activities or the legal issues involve multiple jurisdictions" -- which presumably could include "multistate" legal issues that are not nationally uniform.

The Restatement also deals with lawyers engaged in multistate type practices -- paralleling the ABA Model Rules approach.

161

35951124_10 UPL, MDP and MJP (Defining What Lawyers Do and McGuireWoods LLP Where They Can Do It): Part II T. Spahn (1/1/19) Hypotheticals and Analyses

Particularly in the situation of a lawyer representing a multistate or multinational organization, the question of geographical connection may be difficult to assess or establish. Thus, a multinational corporation wishing to select a location in the United States to build a new facility may engage a lawyer to accompany officers of the corporation to survey possible sites in several states, perhaps holding discussions with local governmental officers about such topics as zoning, taxation, environmental requirements, and the like. Such occasional, temporary in-state services, when reasonable and appropriate in performing the lawyer's functions for the client, are a proper aspect of practice and do not constitute impermissible practice in the other state.

Id.

ACTEC

The ACTEC Commentaries also take a very broad approach.

Subject to the "temporary basis" threshold requirement, under paragraph (c)(4), a lawyer may provide legal services in a non-admitted jurisdiction that arise out of or are reasonably related to the lawyer's practice in an admitted jurisdiction. Comment 14 states that a variety of factors may establish that the services performed are reasonably related to the lawyer's practice in the admitted jurisdiction. For example, a lawyer provides estate planning services for a client in the lawyer's admitted jurisdiction. The client then moves to a non-admitted jurisdiction. The lawyer may continue to provide estate planning services for the client. Similarly, where a client retains the lawyer to represent the client in a fiduciary administration and the admitted jurisdiction is the natural situs for administration, the lawyer could provide legal services for ancillary administrations in non-admitted jurisdictions.

American College of Trust & Estate Counsel, Commentaries on the Model Rules of

Professional Conduct, Commentary on MRPC 5.5, at 203 (5th ed. 2016).

The ACTEC Commentaries also point to the ABA's "catch-all" provision.

While the language of paragraph (c) appears to state all of the exceptions available to a lawyer seeking to practice law 162

35951124_10 UPL, MDP and MJP (Defining What Lawyers Do and McGuireWoods LLP Where They Can Do It): Part II T. Spahn (1/1/19) Hypotheticals and Analyses

in a non-admitted jurisdiction on a "temporary basis," Comment 5 specifically provides: "The fact that conduct is not [stated in (c)(1) through (4)] does not imply that the conduct is or is not authorized" (Comment 5 to MRPC 5.5, emphasis added). Given the diversity of legal services that can be offered in estate planning and administration matters, there may be other situations in which a lawyer may provide legal services in a non-admitted jurisdiction or concerning the laws of a non-admitted jurisdiction not expressly covered in paragraphs (c)(1) through (4). In analyzing whether the lawyer may act on a "temporary basis" with regard to the requested services, the lawyer should consider whether or not the "circumstances . . . create an unreasonable risk to the interests of their clients, the public or the courts" (Comment 5 to MRPC 5.5). If the lawyer can demonstrate that there is no unreasonable risk, the lawyer may proceed with the requested representation on a "temporary basis." In the event, the lawyer should consider seeking an opinion of the non-admitted jurisdiction's bar counsel.

American College of Trust & Estate Counsel, Commentaries on the Model Rules of

Professional Conduct, Commentary on MRPC 5.5, at 204 (5th ed. 2016).

Most jurisdictions have adopted ABA Model Rule 5.5 or a variation of that rule.

An ABA Ethics 20/20 Commission Report described the current status of states' adoption of ABA Model Rule 5.5.

Forty-four United States jurisdictions now have some form of Model Rule 5.5. . . . Fourteen jurisdictions have adopted a rule identical to the ABA Model Rule, and thirty jurisdictions have adopted a rule similar to ABA Model Rule 5.5. Of the thirty jurisdictions that have adopted a rule similar to ABA Model Rule 5.5, some of the differences between what has been adopted and the ABA Model Rule are as follows:

• Eight jurisdictions (CT, ID, KY, ME, NJ, NC, SC, and TN) require that the temporary legal services provided in the host jurisdiction be reasonably related to the representation of an existing client in the jurisdiction where the lawyer is licensed. The Model Rule provides that the services need only be reasonably related to the

163

35951124_10 UPL, MDP and MJP (Defining What Lawyers Do and McGuireWoods LLP Where They Can Do It): Part II T. Spahn (1/1/19) Hypotheticals and Analyses

lawyer's practice in the jurisdiction where the lawyer is licensed.

• Six jurisdictions (DE, DC, FL, GA, PA, and VA) expressly allow temporary practice by foreign (non-U.S.) lawyers. In addition, North Carolina's rule appears to permit such temporary practice by omitting reference to the words "U.S. jurisdiction."

• Four jurisdictions (CT, NV, NJ, and SC) require out-of state lawyers to register in the host jurisdiction and pay a fee.

• Two jurisdictions (NM and ND) require the out-of-state lawyer to associate with an in-state lawyer in transactions involving issues specific to the host jurisdiction's law.

• Two jurisdictions (MN and WI) provide that the lawyer would not be disciplined in the state in which the lawyer is licensed for engaging in conduct in the host jurisdiction that is permitted in the home jurisdiction.

• One jurisdiction (SD) requires the out-of-state lawyer to obtain a sales tax license and to pay applicable taxes.

Of the remaining jurisdictions, two states have declined to adopt Model Rule 5.5; one state has a recommendation pending to adopt a version of Rule 5.5 that does not authorize multijurisdictional practice; and four states are still studying whether to adopt a version of Rule 5.5 that authorizes multijurisdictional practice.

Am. Bar Ass'n Comm'n on Ethics 20/20, Issues Paper Concerning Multijurisdictional

Practice, at 3 n.7 (Mar. 29, 2011).

Ironically, one of the last states to adopt explicit multijurisdictional practice provisions was New York State. As of December 30, 2015 New York essentially used the ABA Model Rule 5.5 approach, but placed the provisions in its judiciary law rather than its ethics rules. 22 NYCRR §522 mirrors ABA Model Rule 5.5. Reports at the time

164

35951124_10 UPL, MDP and MJP (Defining What Lawyers Do and McGuireWoods LLP Where They Can Do It): Part II T. Spahn (1/1/19) Hypotheticals and Analyses

indicated that New York was the forty-seventh state to adopt a variation of ABA Model

Rule 5.5. ABA/BNA Manual on Professional Conduct, Vol. 31, No. 26 (12/30/15).

Some states' Rule 5.5 and related rules or regulations take a remarkably narrow

view.

In 2012 a New Jersey unauthorized practice of law opinion dealt with this issue.1

After analyzing the unique New Jersey version of Rule 5.5(b)(3)(i), the New Jersey Bar

took the majority view of what out-of-state transactional lawyers could do in New Jersey.

1 New Jersey UPL Op. 49 (10/3/12) ("In sum, Rule of Professional Conduct 5.5(b)(3) permits out-of-state lawyers to engage in limited practice of New Jersey law provided all criteria in the pertinent 'safe harbor' subparagraph are met. Lawyers who engage in the practice of New Jersey law as multijurisdictional or crossborder practitioners under Rule of Professional Conduct 5.5(b)(3)(i), (iv), or (v) must first 'register' with the Clerk of the Supreme Court and pay the annual assessment. RPC 5.5(c)(3) and (6)."; explaining that New Jersey did not consider an MJP Rule 5.5 until 2001; noting that the New Jersey Supreme Court adopted its Rule 5.5 in 2003; explaining that "[i]n July 2004, the Court added language to Rule of Professional Conduct 5.5(c) requiring multijurisdictional practitioners to maintain a bona fide office and comply with annual assessment and registration rules 'during the period of practice.' In July 2010, the Court further amended Rule of Professional Conduct 5.5(b)(3) to include a new subparagraph (iv). This new subparagraph permitted out-of-state lawyers to engage in practice of New Jersey law if: the lawyer associates in a matter with a lawyer admitted to the Bar of this State who shall be held responsible for the conduct of the out-of-state lawyer in the matter . . . . Thereafter, in July 2012, the Court clarified this new subparagraph by stating that the out-of-state lawyer's practice in New Jersey must be 'occasional' and the out-of-state lawyer must designate and disclose to all interested parties the New Jersey lawyer with whom the out-of-state lawyer associates in the matter. This amendment became effective September 4, 2012."; explaining New Jersey Rule 5.5; "To ensure that out-of-state lawyers who are not admitted to practice in New Jersey do not establish a continuous or systematic presence in New Jersey, each of the 'safe harbor' provisions of Rule of Professional Conduct 5.5(b)(3) describe [sic] restricted practice activities. Subparagraph (b)(3)(i) is limited by the narrow circumstances in which the practice is permitted: the out-of-state lawyer engages in negotiation of the terms of a transaction in furtherance of representation of an existing client in a jurisdiction in which the lawyer is admitted to practice and the transaction originates in or is otherwise related to a jurisdiction in which the lawyer is admitted to practice. Subparagraph (b)(3)(ii) is similarly limited, in that the out-of-state lawyer may engage in representation of a party in complementary dispute resolution only when the services arise out of or are reasonably related to the lawyer's practice in a jurisdiction in which the lawyer is admitted to practice. Subparagraph (b)(3)(iii) is limited to certain litigation-related activities in New Jersey for a proceeding pending or anticipated to be instituted in a jurisdiction in which the lawyer is admitted to practice. Subparagraph (iv) is limited by the word 'occasional' -- the out-of-state lawyer's practice in New Jersey must be only 'occasional' and the lawyer must associate with a New Jersey lawyer in the matter. Lastly, subparagraph (b)(3)(v) is limited both by the condition that the practice is 'occasional' and that the practice activity arises directly out of the lawyer's representation on behalf of an existing client in a jurisdiction in which the lawyer is admitted to practice."; "Rule of Professional Conduct 5.5(b)(3) permits an out-of-state lawyer to engage in certain transactions or other nonlitigation matters in New Jersey. Pro hac vice admission under Rule 1:21-2 permits an out-of-state lawyer to appear, with local counsel, in a New Jersey court. These two paths to permitted practice in New Jersey are mutually exclusive."; 165

35951124_10 UPL, MDP and MJP (Defining What Lawyers Do and McGuireWoods LLP Where They Can Do It): Part II T. Spahn (1/1/19) Hypotheticals and Analyses

Rule of Professional Conduct 5.5(b)(3)(i) permits an out-of-state lawyer to 'negotiat[e] the terms of a transaction' when the lawyer represents 'an existing client in a jurisdiction where the lawyer is admitted to practice and the transaction originates in or is otherwise related to a jurisdiction in which the lawyer is admitted to practice.' Inquirer stated that the out-of-state purchaser of the New Jersey real estate is a developer. Presumably, the transaction relates to the developer's out-of-state business; the out-of-state lawyer is licensed in the jurisdiction where the developer and its business are located; and the developer is an 'existing client' of the out-of-state lawyer. Therefore, the out-of-state lawyer may represent this developer in a limited role, to negotiate the terms of the New Jersey real estate transaction.

Id. (emphases added).

But then the New Jersey Bar surprisingly indicated that out-of-state lawyers could not write up what they had negotiated.

"Lawyers with offices and practices in a neighboring state have asked whether they may engage in an ongoing practice of New Jersey law by bringing a New Jersey lawyer into the firm, thereby 'associating' with New Jersey counsel. Again, the answer is no. Rule of Professional Conduct 5.5(b)(3)(iv) does not permit recurring practice in New Jersey by an out-of-state lawyer who has associated with a New Jersey lawyer. The practice must be 'occasional.'"; "The Committee now turns to the specific inquiry about an out-of-state lawyer representing an out-of-state client in a commercial real estate transaction, negotiating the terms of the transaction, and preparing the contract and other related documents. Preparing real estate sale and lease contracts for a third person is the practice of law."; "Further, negotiating the terms of a legal document such as a contract for the purchase or sale of real estate as an advocate for another person is the practice of law."; "Rule of Professional Conduct 5.5(b)(3)(i) permits an out-of-state lawyer to 'negotiat[e] the terms of a transaction' when the lawyer represents 'an existing client in a jurisdiction where the lawyer is admitted to practice and the transaction originates in or is otherwise related to a jurisdiction in which the lawyer is admitted to practice.' Inquirer stated that the out-of-state purchaser of the New Jersey real estate is a developer. Presumably, the transaction relates to the developer's out-of-state business; the out-of-state lawyer is licensed in the jurisdiction where the developer and its business are located; and the developer is an 'existing client' of the out-of-state lawyer. Therefore, the out-of-state lawyer may represent this developer in a limited role, to negotiate the terms of the New Jersey real estate transaction. Notably, however, Rule of Professional Conduct 5.5(b)(3)(i) does not further authorize the out-of-state lawyer to prepare the contract of sale or other pertinent legal documents."; "Here, the out-of-state client is a developer seeking to purchase New Jersey commercial real estate in the course of its business. The transaction relates to the developer's out-of-state business; the out-of-state lawyer is licensed in the jurisdiction where the developer and its business are located; and the developer has a preexisting relationship with its lawyer. The lawyer's disengagement may result in 'substantial inefficiency, impracticality or detriment' to a sophisticated client in these circumstances. If the practice in New Jersey is also 'occasional,' these circumstances could satisfy the criteria of Rule of Professional Conduct 5.5(b)(3)(v). The out-of-state lawyer, however, must 'register' with the Clerk of the Court and pay the annual assessment pursuant to Rule of Professional Conduct 5.5(c)."). 166

35951124_10 UPL, MDP and MJP (Defining What Lawyers Do and McGuireWoods LLP Where They Can Do It): Part II T. Spahn (1/1/19) Hypotheticals and Analyses

Notably, however, Rule of Professional Conduct 5.5(b)(3)(i) does not further authorize the out-of-state lawyer to prepare the contract of sale or other pertinent legal documents.

Id. The New Jersey Bar also reminded the out-of-state lawyer of another surprising requirement.

The out-of-state lawyer . . . must 'register' with the Clerk of the Court and pay the annual assessment pursuant to Rule of Professional Conduct 5.5(c).

New Jersey UPL Op. 49 (10/3/12).

New Jersey is not alone in taking such a narrow approach to out-of-state lawyers temporarily practicing within its boarders.

More recently, the Minnesota Supreme Court reprimanded a Colorado lawyer for exchanging "approximately two dozen emails" with a Minnesota lawyer to assist his

Minnesota in-laws in a $2,368.13 Minnesota dispute.

Among other things, the Minnesota Supreme Court rejected the Colorado lawyer's argument that his temporary virtual practice in Minnesota had a sufficient connection with his home state of Colorado to satisfy Minnesota Rule 5.5's exception for temporary practice if the lawyer's services "arise out of or are reasonably related to the lawyer's practice in a jurisdiction in which the lawyer is admitted to practice."

• In re Charges of Unprofessional Conduct in Panel File No. 39302, 884 N.W.2d 661, 663, 663-64, 664, 665, 666, 666-67, 667, 667-68, 668, 668-69, 672, 672-73, 673 (Minn. 2016) (privately reprimanding a Colorado lawyer for assisting his in-laws in a Minnesota dispute; "We hold that engaging in e-mail communications with people in Minnesota may constitute the unauthorized practice of law in Minnesota, in violation of Minn. R. Prof. Conduct 5.5(a), even if the lawyer is not physically present in Minnesota. . . . Appellant represented a Minnesota couple with respect to a Minnesota judgment and attempted to negotiate, via e-mail, the satisfaction of that judgment with a Minnesota lawyer, and was not authorized to practice law in Minnesota temporarily. We further conclude that the appropriate disposition for this

167

35951124_10 UPL, MDP and MJP (Defining What Lawyers Do and McGuireWoods LLP Where They Can Do It): Part II T. Spahn (1/1/19) Hypotheticals and Analyses

misconduct is an admonition."; "Appellant is an attorney licensed to practice law in the state of Colorado, where he maintains an office and has been practicing environmental law since 1986."; "Appellant's mother-and father-in- law live in Minnesota. They contacted appellant in May 2014 to obtain assistance regarding a judgment entered against them in conciliation court in Minnesota for $2,368.13 in favor of their condominium association, Voyager Condominium Homeowners' Association, Inc. (VCHA). The couple told appellant that VCHA's attorney, D.R., a Minnesota-based lawyer and the complainant in this case, was harassing them with telephone calls attempting to collect on the judgment. The couple asked appellant for his assistance in negotiating with D.R. regarding payment of the outstanding judgment."; "Appellant sent an e-mail to D.R. in late May 2014, informing D.R. that he was representing his in-laws and instructing D.R. to direct all future communications to him instead. Appellant and D.R. exchanged approximately two dozen e-mails between May 2014 and September 2014. In his first responsive e-mail to appellant, D.R. asked whether appellant was licensed to practice law in Minnesota. Appellant replied that he was not licensed in Minnesota and that if he needed to file suit in Minnesota he would hire local counsel."; "Nothing in the record shows that appellant researched whether his activities constituted the unauthorized practice of law under the Minnesota Rules of Professional Conduct. When asked by the Panel at the evidentiary hearing whether he researched the rules in Minnesota, appellant said that he did not recall. Appellant admitted that he had not researched Minnesota law on foreclosure and how it would apply to his in-laws' case. Appellant also admitted that when he considered the relevant law and the rules of professional conduct, he was more familiar with the laws and rules in Colorado."; concluding that the lawyer practiced law in Minnesota despite not having been physically present there; "Appellant contends that he did not violate Rule 5.5(a) because he did not practice law in Minnesota."; "Other courts have addressed the issue of whether an attorney practices law in a jurisdiction even though the attorney was not physically present in that jurisdiction."; "[W]e conclude that the Panel did not clearly err by finding that appellant practiced law in Minnesota, in violation of Minn. R. Conduct 5.5(a). Appellant contacted D.R., a Minnesota lawyer, and stated that he represented Minnesota clients in a Minnesota legal dispute. This legal dispute was not interjurisdictional; instead, it involved only Minnesota residents and a debt arising from a judgment entered by a Minnesota court. Appellant instructed D.R. to refer all future correspondence to him, and he continued to engage in correspondence and negotiations with D.R. over the course of several months. Appellant requested and received financial documents from his Minnesota clients and advised them on their legal options. By multiple e-mails sent over several months, appellant advised Minnesota clients on Minnesota law in connection with a Minnesota legal dispute and attempted to negotiate a resolution of that dispute with a Minnesota attorney. Appellant had a clear, ongoing attorney-client

168

35951124_10 UPL, MDP and MJP (Defining What Lawyers Do and McGuireWoods LLP Where They Can Do It): Part II T. Spahn (1/1/19) Hypotheticals and Analyses

relationship with his Minnesota clients, and his contacts with Minnesota were not fortuitous or attenuated. Thus, there is ample support for the Panel's finding that appellant practiced law in Minnesota." (footnote omitted); "Next, we turn to appellant's claim that even if the Panel did not err in determining that he was practicing law in Minnesota in violation of Minn. R. Prof. Conduct 5.5(a), his conduct was permitted under one of the exceptions in Minn R. Prof. Conduct 5.5(c). Appellant argues that Rule 5.5(c)(2) authorized his conduct because he reasonably believed that he would be able to associate with local counsel and be admitted pro hac vice if necessary. Appellant further claims that Rule 5.5(c)(4) authorized his conduct because his in-laws reached out to him for assistance on a matter within his expertise; thus the matter 'arose out of [Appellant's] law practice.'"; also finding that he could not rely on the exception finding temporary practice in Minnesota to a lawyer who expects to be admitted in litigation there; "Under Minnesota Rules of Professional Conduct 5.5(c)(2), a lawyer admitted in another jurisdiction may provide legal services in Minnesota on a temporary basis if the lawyer's services are reasonably related to a pending or potential proceeding before a tribunal and the lawyer reasonably expects to be authorized by law to appear in the proceeding. Comment 10 explains that a lawyer rendering services in Minnesota on a temporary basis is permitted to engage in conduct in anticipation of a proceeding or hearing in which the lawyer reasonably expects to be admitted pro hac vice."; "Rule 5.5(c)(2), by its plain language, requires more than an attorney's speculation that the attorney can find local counsel and be admitted pro hac vice. Appellant's e-mail correspondence does not indicate that he took steps to secure local counsel or investigate the possibility of pro hac vice admission. Thus, we conclude there is no support for appellant's claim that his conduct was authorized by Rule 5.5(c)(2)."; also finding that there was no connection between the Minnesota matter and his home state of Colorado; "Under Minnesota Rules of Professional Conduct 5.5(c)(4), a lawyer admitted in another jurisdiction may provide legal services in Minnesota on a temporary basis if the lawyer's services are not covered by paragraphs (c)(2) and (c)(3) and 'arise out of or are reasonably related to the lawyer's practice in a jurisdiction in which the lawyer is admitted to practice.' Appellant contends that his services arose out of or were reasonably related to his practice in Colorado because the clients are his relatives who 'reached out to him for assistance' and appellant's environmental and personal-injury practice involves debt collection."; "The legal services appellant provided to his in-laws were unrelated to his environmental and personal-injury practice in Colorado."; "Moreover, appellant's representation of his in-laws did not 'arise out of' or 'reasonably relate' to his practice in Colorado simply because his in-laws contacted him in Colorado or appellant has done collections work in Colorado. As the Director notes, appellant's in-laws were not long- standing clients; nor was there any connection between the in-laws' case and the state or laws of Colorado. And while appellant's Colorado practice may involve judgment collections work, nothing in the record establishes that this

169

35951124_10 UPL, MDP and MJP (Defining What Lawyers Do and McGuireWoods LLP Where They Can Do It): Part II T. Spahn (1/1/19) Hypotheticals and Analyses

work was based on a body of federal or nationally uniform law. To the contrary, appellant's clients were Minnesota residents with a debt that arose in Minnesota that they owed to a Minnesota resident and that was governed by Minnesota law. Accordingly, Rule 5.5(c)(4) does not apply to appellant's conduct."; in dissent, three judges found a sufficient connection between the Minnesota temporary practice and Colorado; "[T]he clients' relationship to appellant, including their familial connection and the clients' contacts with appellant in his home state, should be considered in the 'reasonable relationship' analysis. The comments to the Restatement advise that, in determining whether an out-of-state lawyer's activities 'reasonably relate' to the lawyer's practice in a state of admission, 'several factors are relevant, including the following: . . . [Whether the client] is from the lawyer's home state, has extensive contacts with that state, or contacted the lawyer there,' Restatement (Third) of the Law Governing Lawyers § 3 cmt. e (emphasis added). Here, the clients contacted their son-in-law, appellant, in his home state of Colorado."; "[A]nalogous support is provided in comment 14 to Rule 5.5(c)(4), which states that one factor to consider is whether the 'lawyer's client may have been previously represented by the lawyer.' Minn. R. Prof. Conduct 5.5(c)(4) cmt. 14. Although the record does not indicate whether appellate ever previously represented his parents-in-law, the principle underlying this comment – a relationship of trust and familiarity with the lawyer's capabilities – is applicable here. The recognition that a sustained lawyer-client relationship would allow an attorney to perform legal work for the client in other jurisdictions, based on confidence and trust, is reflected in the ABA's recommendation for the proposed Model Rule 5.5."; also finding that as a policy matter the private admonition was improper; "[A]s a policy matter, the implications of the court's decision are troubling and counterproductive. . . . Today's decision represents a step backwards. By the court's reasoning, when family members or friends – an abundant source of clients – email or call a practitioner admitted in another state, seeking assistance in areas in which the practitioner is experienced and competent, relying on a relationship of trust and confidence, they must be turned away. Those potential clients must then expend unnecessary time and resources to research and hire local counsel – even for minor, temporary services in which the out-of-state lawyer could have provided efficient, inexpensive, and competent service. Simply put, the court's decision is contrary to the principles and policy goals intended by Rule 5.5(c)."; "In sum, this case involves clients contacting an attorney, their son-in-law, in his home state of Colorado, to request his assistance regarding a small collection matter – an area that reasonably relates to appellant's expertise and experience in his Colorado litigation practice. Based on the relationship and contacts between the clients, appellant, and appellant's practice of law in Colorado, there is a sufficient 'reasonable relationship' here to satisfy the broad, catch-all exception under Rule 5.5(c)(4). For the above reasons, I conclude that appellant did not engage in professional misconduct because the exception

170

35951124_10 UPL, MDP and MJP (Defining What Lawyers Do and McGuireWoods LLP Where They Can Do It): Part II T. Spahn (1/1/19) Hypotheticals and Analyses

in Rule 5.5(c)(4) applies. Therefore, I would reverse the Panel's decision to admonish appellant. I respectfully dissent.").

Best Answer

The best answer to this hypothetical is MAYBE.

B 2/13, 3/17, n

171

35951124_10 UPL, MDP and MJP (Defining What Lawyers Do and McGuireWoods LLP Where They Can Do It): Part II T. Spahn (1/1/19) Hypotheticals and Analyses

Requirement of a Relationship with the Lawyer's Home State: Broad View

Hypothetical 15

You handle the trust and estate work for several wealthy individuals who spend summers in your state but winters in Florida (where you are not licensed). Several multijurisdictional questions have arisen in connection with your practice.

(a) May you travel to Florida in February and meet with one of your clients to go over her estate plan?

YES

(b) May you represent your client's neighbor in preparing her estate plan (she lives permanently in Florida, but heard favorable reports about you from your client who lives next door)?

MAYBE

Analysis

This hypothetical comes directly from a Restatement (Third) of Law Governing

Lawyers illustration -- and highlights the surprisingly broad approach that the

Restatement takes.

(a) The ABA Model Rules would clearly allow a lawyer to temporarily follow a client from her home state to another state to provide legal services.

For instance, ABA Model Rule 5.5 cmt. [14] recognizes that the "reasonable relationship" that permits an out-of-state lawyer to perform legal services in a state can include such factors as:

The lawyer's client may have been previously represented by the lawyer, or may be resident in or have substantial contacts with this jurisdiction in which the lawyer is admitted.

172

35951124_10 UPL, MDP and MJP (Defining What Lawyers Do and McGuireWoods LLP Where They Can Do It): Part II T. Spahn (1/1/19) Hypotheticals and Analyses

ABA Model Rule 5.5 cmt. [14] (emphasis added).

As explained immediately below, the Restatement takes even a broader approach, and thus would clearly permit this conduct.

(b) The ABA Model Rules might permit a lawyer to assist the neighbor in this setting, but probably not because of the neighbor's contact with the lawyer's home state.

Instead, the lawyer presumably would have to rely on such factors as the lawyer's expertise in "a particular body of federal, [or] nationally-uniform" law. ABA

Model Rule 5.5 cmt. [14]. That factor could presumably justify the lawyer's assistance in federal tax law questions, but not state tax law questions.

The ACTEC Commentaries would probably take the same approach.

Where the lawyer has developed a recognized expertise in federal, nationally-uniform, foreign or international law, Comment 14 suggests that the lawyer's practice in non-admitted jurisdictions will be considered reasonably related to the lawyer's practice in the lawyer's admitted jurisdiction. For example, a lawyer with recognized expertise in retirement planning, charitable planning, estate and gift tax planning, or international estate planning may be able to practice in non-admitted jurisdictions, again on a temporary basis. Because the comments are not binding, a lawyer who intends to rely on this analysis should consider seeking an opinion of the non-admitted jurisdiction's bar association. In addition, since this exception is based on "recognized expertise," a lawyer who chooses to rely on this exception should take steps to insure that the lawyer is recognized as an expert. These steps could include: obtaining certification as a specialist in those jurisdictions offering such programs; participating actively in bar sections related to the lawyer's expertise; participating in national associations of lawyers related to the lawyer's expertise; writing scholarly articles; teaching; participating in seminars and panel discussions; or any other activity that demonstrates the lawyer's expertise.

173

35951124_10 UPL, MDP and MJP (Defining What Lawyers Do and McGuireWoods LLP Where They Can Do It): Part II T. Spahn (1/1/19) Hypotheticals and Analyses

American College of Trust & Estate Counsel, Commentaries on the Model Rules of

Professional Conduct, Commentary on MRPC 5.5, at 203-04 (5th ed. 2016) (emphases added).

This hypothetical comes from a Restatement illustration -- which represents perhaps the most liberal view of multijurisdictional practice.

Lawyer is admitted to practice and has an office in Illinois, where Lawyer practices in the area of trusts and estates, an area involving, among other things, both the law of wills, property, taxation, and trusts of a particular state and federal income, estate, and gift tax law. Client A, whom Lawyer has represented in estate-planning matters, has recently moved to Florida and calls Lawyer from there with a request that leads to Lawyer's preparation of a codicil to A's will, which Lawyer takes to Florida to obtain the necessary signatures. While there, A introduces Lawyer to B, a friend of A, who, after learning of A's estate-planning arrangements from A, wishes Lawyer to prepare a similar estate arrangement for B. Lawyer prepares the necessary documents and conducts legal research in Lawyer's office in Illinois, frequently conferring by telephone and letter with B in Florida. Lawyer then takes the documents to Florida for execution by B and necessary witnesses. Lawyer's activities in Florida on behalf of both A and B were permissible.

Restatement (Third) of Law Governing Lawyers § 3 illus. 5 (2000). Thus, the

Restatement would even allow this type of out-of-state activity.

Presumably, the Restatement deliberately picked Florida as the jurisdiction in which this question arose. Undoubtedly driven by Florida lawyers' worry that northern lawyers will follow their wealthy "snowbirds" south (or work part-time themselves in

Florida during the winter months), Florida seems more concerned than any other state with out-of-state lawyers impermissibly providing legal services within its borders. The

174

35951124_10 UPL, MDP and MJP (Defining What Lawyers Do and McGuireWoods LLP Where They Can Do It): Part II T. Spahn (1/1/19) Hypotheticals and Analyses

Restatement illustration represents an explicit challenge to this type of parochial turf- protection.

Not every state takes the narrow view that Florida has adopted (or polices multijurisdictional practice issues as aggressively as Florida), but all states protect their own lawyers to at least some extent.

Best Answer

The best answer to (a) is YES; the best answer to (b) is MAYBE.

B 12/10

175

35951124_10 UPL, MDP and MJP (Defining What Lawyers Do and McGuireWoods LLP Where They Can Do It): Part II T. Spahn (1/1/19) Hypotheticals and Analyses

Non-Litigation Work Without a Relationship with the Lawyer's Home State, but Based on Nationally-Uniform Law

Hypothetical 16

You have gained a national reputation for representing aircraft part vendors in complying with very specific (and complex) UCC regulations. You just received a call from a small vendor which would like to hire you to help with such UCC issues. The vendor is located in a state where you are not licensed to practice law.

May you represent the vendor, although you are not licensed to practice law in that state?

MAYBE

Analysis

ABA Model Rule 5.5 allows lawyers admitted in a United States jurisdiction to practice law in another United States jurisdiction under certain conditions.

A lawyer admitted in another United States jurisdiction, and not disbarred or suspended from practice in any jurisdiction, may provide legal services on a temporary basis in this jurisdiction that:

(1) are undertaken in association with a lawyer who is admitted to practice in this jurisdiction and who actively participates in the matter;

(2) are in or reasonably related to a pending or potential proceeding before a tribunal in this or another jurisdiction, if the lawyer, or a person the lawyer is assisting, is authorized by law or order to appear in such proceeding or reasonably expects to be so authorized;

(3) are in or reasonably related to a pending or potential arbitration, mediation, or other alternative dispute resolution proceeding in this or another jurisdiction, if the services arise out of or are reasonably related to the lawyer's practice in a jurisdiction in which the lawyer is admitted to practice and are not services for which the forum requires pro hac vice admission; or

176

35951124_10 UPL, MDP and MJP (Defining What Lawyers Do and McGuireWoods LLP Where They Can Do It): Part II T. Spahn (1/1/19) Hypotheticals and Analyses

(4) are not within paragraphs (c)(2) or (c)(3) and arise out of or are reasonably related to the lawyer's practice in a jurisdiction in which the lawyer is admitted to practice.

ABA Model Rule 5.5(c).

This hypothetical deals with the requirement that the temporary legal services

"arise out of or are reasonably related to the lawyer's practice" in the lawyer's home state.

A comment to ABA Model Rule 5.5 takes a remarkably broad view of what type of "relationship" to the lawyer's home state suffices under this catch-all provision.

Paragraph (c)(3) (c)(4) require that the services arise out of or be reasonably related to the lawyer's practice in a jurisdiction in which the lawyer is admitted. A variety of factors evidence such a relationship. The lawyer's client may have been previously represented by the lawyer, or may be resident in or have substantial contacts with the jurisdiction in which the lawyer is admitted. The matter, although involving other jurisdictions, may have a significant connection with that jurisdiction. In other cases, significant aspects of the lawyer's work might be conducted in that jurisdiction or a significant aspect of the matter may involve the law of that jurisdiction. The necessary relationship might arise when the client's activities or the legal issues involve multiple jurisdictions, such as when the officers of a multinational corporation survey potential business sites and seek the services of their lawyer in assessing the relative merits of each. In addition, the services may draw on the lawyer's recognized expertise developed through the regular practice of law on behalf of clients in matters involving a particular body of federal, nationally-uniform, foreign, or international law. Lawyers desiring to provide pro bono legal services on a temporary basis in a jurisdiction that has been affected by a major disaster, but in which they are not otherwise authorized to practice law, as well as lawyers from the affected jurisdiction who seek to practice law temporarily in another jurisdiction, but in which they are not otherwise authorized to practice law, should consult the [Model Court Rule on Provision of Legal Services Following Determination of Major Disaster.]

177

35951124_10 UPL, MDP and MJP (Defining What Lawyers Do and McGuireWoods LLP Where They Can Do It): Part II T. Spahn (1/1/19) Hypotheticals and Analyses

ABA Model Rule 5.5 cmt. [14] (emphases added).

The Restatement similarly notes that the pertinent factors include whether "the legal issues involved are primarily either multistate or federal in nature." Restatement

(Third) of Law Governing Lawyers § 3 cmt. e (2000).

Theoretically, the Restatement's reference to "multistate" legal issues is somewhat broader than the ABA Model Rule's reference to "nationally-uniform" law -- but ABA Model Rule 5.5 cmt. [14] explains that lawyers may assist corporate clients

"when the client's activities or the legal issues involve multiple jurisdictions" -- which presumably could include "multistate" legal issues that are not nationally uniform.

The Restatement also indicates that the permissibility of a lawyer's action in another state depends in part on whether a new client "contacted the lawyer" in the lawyer's home state. Restatement (Third) of Law Governing Lawyers § 3 cmt. e (2000).

Not surprisingly, the American College of Trust and Estate Counsel

Commentaries emphasize this MJP rule.

Where the lawyer has developed a recognized expertise in federal, nationally-uniform, foreign or international law, Comment 14 suggests that the lawyer's practice in non-admitted jurisdictions will be considered reasonably related to the lawyer's practice in the lawyer's admitted jurisdiction. For example, a lawyer with recognized expertise in retirement planning, charitable planning, estate and gift tax planning, or international estate planning may be able to practice in non-admitted jurisdictions, again on a temporary basis. Because the comments are not binding, a lawyer who intends to rely on this analysis should consider seeking an opinion of the non-admitted jurisdiction's bar association. In addition, since this exception is based on "recognized expertise," a lawyer who chooses to rely on this exception should take steps to insure that the lawyer is recognized as an expert. These steps could include: obtaining certification as a specialist in those

178

35951124_10 UPL, MDP and MJP (Defining What Lawyers Do and McGuireWoods LLP Where They Can Do It): Part II T. Spahn (1/1/19) Hypotheticals and Analyses

jurisdictions offering such programs; participating actively in bar sections related to the lawyer's expertise; participating in national associations of lawyers related to the lawyer's expertise; writing scholarly articles; teaching; participating in seminars and panel discussions; or any other activity that demonstrates the lawyer's expertise.

American College of Trust & Estate Counsel, Commentaries on the Model Rules of

Professional Conduct, Commentary on MRPC 5.5, at 203-04 (5th ed. 2016) (emphases

added).

Many states do not seem to have adopted as broad a view as that reflected in

these ABA Model Rules, Restatement, or ACTEC provisions. However, the apparent

lack of any disciplinary actions involving lawyers engaging in this type of

multijurisdictional practice may reflect states' acquiescence in such behavior.

In 2011, an Ohio legal ethics opinion took a parochial view.1

1 Ohio LEO 2011-2 (10/7/11) ("An out-of-state lawyer is not authorized to provide debt settlement legal services, including investigation, negotiation, and other nonlitigation activities, on a temporary basis in Ohio under Prof. Cond. Rule 5.5(c)(4) when the matter is not connected to the lawyer's jurisdiction of admission to the practice of law, there is not a pre-existing relationship between the lawyer and the client, and the lawyer does not have a recognized expertise in a particular body of federal, nationally-uniform, foreign, or international law that is applicable to the matter. Under these circumstances, the nonlitigation activities do not arise out of, and are not reasonably related to, the lawyer's practice in his or her jurisdiction of admission for purposes of Prof. Cond. Rule 5.5(c)(4)."; "Applying the seven factors outlined in Prof. Cond. Rule 5.5, Comment [14], and the concepts detailed in the legislative history of Model Rule 5.5, the Board concludes that the hypothetical out-of-state debt settlement lawyers are not authorized to practice law temporarily in Ohio. The Ohio clients became aware of the out-of-state lawyers for the first time through an internet search, and had no prior contact or relationship with the lawyers. The clients are not residents of the lawyers' home jurisdictions of licensure, and the work done for the client crosses into a number of jurisdictions where the creditors are located. The lawyers may be physically located in their state of licensure, but a significant portion of the work done for the clients is not centrally located in that home state. The debt settlement work performed by the out-of-state lawyers is likely not governed by the law of the lawyers' home state, as the Ohio clients may be facing state law collection actions and presumably have Ohio assets as well as income and debt incurrent in Ohio."; "In regard to any 'recognized expertise' in a 'particular body of federal, nationally-uniform, foreign, or international law,' it is the Board's opinion that general debtor/credit law is not the type of nationally-uniform law intended by the drafters that, without more of a nexus between lawyer and client, would permit multijurisdictional practice under Prof. Cond. Rule 5.5(c)(4). The primary areas of concern for clients seeking debt settlement legal services are consumer protection, debt collection, garnishment, repossession, foreclosure, and general contract law. Even though some federal statutes may come into play, such as the Fair Debt Collection Practices Act, all of these areas implicate state law in Ohio. In addition, an internet search for 'debt 179

35951124_10 UPL, MDP and MJP (Defining What Lawyers Do and McGuireWoods LLP Where They Can Do It): Part II T. Spahn (1/1/19) Hypotheticals and Analyses

An out-of-state lawyer is not authorized to provide debt settlement legal services, including investigation, negotiation, and other nonlitigation activities, on a temporary basis in Ohio under Prof. Cond. Rule 5.5(c)(4) when the matter is not connected to the lawyer's jurisdiction of admission to the practice of law, there is not a pre-existing relationship between the lawyer and the client, and the lawyer does not have a recognized expertise in a particular body of federal, nationally-uniform, foreign, or international law that is applicable to the matter.

Ohio LEO 2011-2 (10/7/11).

The Ohio Bar noted the lack of any substantial relationship between the out-of-

state lawyer and his or her home state.

The Ohio clients became aware of the out-of-state lawyers for the first time through an internet search, and had no prior contact or relationship with the lawyers. The clients are not residents of the lawyers' home jurisdictions of licensure, and the work done for the client crosses into a number of jurisdictions where the creditors are located. The lawyers may be physically located in their state of licensure, but a significant portion of the work done for the clients is not centrally located in that home state. The debt settlement work performed by the out-of-state lawyers is likely not governed by the law of the lawyers' home state, as the Ohio clients may be facing state law collection actions and presumably have Ohio assets as well as income and debt incurrent in Ohio.

settlement law firms,' similar to the search probably conducted by the Ohio clients, produces approximately two million results. Facing this many website options, a typical consumer could not realistically identify lawyers with a 'recognized expertise' in an area of nationally-uniform law that may justify temporary practice in Ohio." (footnotes omitted); "[T]emporary practice by out-of-state lawyers serves the interests of the public and clients when there is a connection to a home state matter, an existing client-lawyer relationship, or a recognized expertise in an area of federal, nationally-uniform, foreign, or international law. The debt settlement hypothetical presented to the Board does not fulfill these criteria, especially given that lawyers are exempt from the Ohio Debt Adjusters Act, which protects Ohio consumers receiving debt settlement services. The Board cannot conclude that it is in the best interests of clients and the public to allow out-of-state lawyers to provide debt settlement legal services to Ohio clients whose sole connection to the lawyers is an internet search." (footnote omitted)). 180

35951124_10 UPL, MDP and MJP (Defining What Lawyers Do and McGuireWoods LLP Where They Can Do It): Part II T. Spahn (1/1/19) Hypotheticals and Analyses

Id. The Ohio opinion also found that the out-of-state lawyer's collection practice did not amount to a sufficiently particular "body of federal, national uniform, foreign, or international law."

In regard to any 'recognized expertise' in a 'particular body of federal, nationally-uniform, foreign, or international law,' it is the Board's opinion that general debtor/credit law is not the type of nationally-uniform law intended by the drafters that, without more of a nexus between lawyer and client, would permit multijurisdictional practice under Prof. Cond. Rule 5.5(c)(4). The primary areas of concern for clients seeking debt settlement legal services are consumer protection, debt collection, garnishment, repossession, foreclosure, and general contract law. Even though some federal statutes may come into play, such as the Fair Debt Collection Practices Act, all of these areas implicate state law in Ohio. In addition, an internet search for 'debt settlement law firms,' similar to the search probably conducted by the Ohio clients, produces approximately two million results. Facing this many website options, a typical consumer could not realistically identify lawyers with a 'recognized expertise' in an area of nationally-uniform law that may justify temporary practice in Ohio.

Id. (footnotes omitted).

Best Answer

The best answer to this hypothetical is MAYBE.

B 2/13, 3/17

181

35951124_10 UPL, MDP and MJP (Defining What Lawyers Do and McGuireWoods LLP Where They Can Do It): Part II T. Spahn (1/1/19) Hypotheticals and Analyses

Lawyers Systematically and Continuously Practicing Law in a State Where They Are Not Licensed

Hypothetical 17

Your landlord just terminated your lease, so you are looking for new office space. You have always lived near the border of two states, and are used to crossing the border on a nearly daily basis as you shop, try new restaurants, etc. You are licensed only in the state where you live -- focusing your practice on elder-law issues. You just read about a small office that would be perfect for your practice. It is actually closer to your home than your current office, and just a few minutes away from a large retirement community. However, the office is in the neighboring state, where you are not licensed to practice law.

(a) May you continuously practice in the neighboring state, as long as you very carefully explain in all of your marketing materials and to your clients that you are not licensed there?

NO (PROBABLY)

(b) May you continuously practice in the neighboring state, as long as you follow the step discussed above, and also work under the direct supervision of a partner who is licensed in that neighboring state?

NO (PROBABLY)

(c) May you continuously practice in the neighboring state, as long as you follow the steps discussed above, and also limit your practice to the law of the state where you are licensed to practice?

MAYBE

Analysis

ABA Model Rules

The ABA Model Rules contain two flat prohibitions on lawyers' activities in states where they are not licensed.

First, a lawyer not licensed in a jurisdiction

182

35951124_10 UPL, MDP and MJP (Defining What Lawyers Do and McGuireWoods LLP Where They Can Do It): Part II T. Spahn (1/1/19) Hypotheticals and Analyses

shall not . . . except as authorized by these Rules or other law, establish an office or other systematic and continuous presence in this jurisdiction for the practice of law.

ABA Model Rule 5.5(b)(1) (emphasis added).

Second, such a lawyer may not

hold out to the public or otherwise represent that the lawyer is admitted to practice law in this jurisdiction.

ABA Model Rule 5.5(b)(2) (emphasis added).

A single comment provides a fairly meager explanation.

Other than as authorized by law or this Rule, a lawyer who is not admitted to practice generally in this jurisdiction violates paragraph (b) if the lawyer establishes an office or other systematic and continuous presence in this jurisdiction for the practice of law. Presence may be systematic and continuous even if the lawyer is not physically present here. Such a lawyer must not hold out to the public or otherwise represent that the lawyer is admitted to practice law in this jurisdiction.

ABA Model Rule 5.5 cmt. [4].

Lawyers should not have any difficulty complying with the second of these requirements. They must always describe the limits of their ability to practice in all of their marketing, during communications with clients and prospective clients, etc.

The first requirement presents a greater challenge. It can be very difficult to draw

the line between an impermissible "systematic and continuous presence" in a state, and

the type of permissible activity that lawyers can conduct "on a temporary basis" in a state where they are not licensed.

183

35951124_10 UPL, MDP and MJP (Defining What Lawyers Do and McGuireWoods LLP Where They Can Do It): Part II T. Spahn (1/1/19) Hypotheticals and Analyses

Restatement

The Restatement similarly indicates that it "would be impermissible for a lawyer to set up an office for the general practice of non-litigation law in a jurisdiction in which the lawyer is not admitted." Restatement (Third) of Law Governing Lawyers § 3 cmt. e

(2000) (emphasis added).

The Restatement then provides an additional explanation of this restriction.

The extent to which a lawyer may practice beyond the borders of the lawyer's home state depends on the circumstances in which the lawyer acts in both the lawyer's home state and the other state. At one extreme, it is clear that a lawyer's admission to practice in one jurisdiction does not authorize the lawyer to practice generally in another jurisdiction as if the lawyer were also fully admitted there. Thus, a lawyer admitted in State A may not open an office in State B for the general practice of law there or otherwise engage in the continuous, regular, or repeated representation of clients within the other state.

Id. (emphasis added).

The Restatement also includes a condemnation of state rules that take a parochial attitude toward multijurisdictional practice.

The rules governing interstate practice by nonlocal lawyers were formed at a time when lawyers conducted very little practice of that nature. Thus, the limitation on legal services threatened by such rules imposed little actual inconvenience. However, as interstate and international commerce, transportation, and communications have expanded, clients have increasingly required a truly interstate and international range of practice by their lawyers. (To a limited extent, many states recognize such needs in the international realm by providing for limited practice in the state by foreign consultants. . . .) Applied literally, the old restrictions on practice of law in a state by a lawyer admitted elsewhere could seriously inconvenience clients who have need of such services within the state. Retaining locally admitted counsel would often cause serious

184

35951124_10 UPL, MDP and MJP (Defining What Lawyers Do and McGuireWoods LLP Where They Can Do It): Part II T. Spahn (1/1/19) Hypotheticals and Analyses

delay and expense and could require the client to deal with unfamiliar counsel.

Id. (emphasis added).

The Restatement then moves to a recognition that these archaic state rules also

fly in the face of modern developments in communications.

Modern communications, including ready electronic connection to much of the law of every state, makes concern about a competent analysis of a distant state's law unfounded. Accordingly, there is much to be said for a rule permitting a lawyer to practice in any state, except for litigation matters of law or for the purpose of establishing a permanent in-state branch office. Results approaching that rule may arguably be required under the federal interstate commerce clause and the privileges and immunities clause. The approach of the Section is more guarded. However, its primary focus is appropriately on the needs of clients.

Id. (emphases added).

(a) Perhaps the purest type of improper multijurisdictional practice of law

involves a lawyer simply moving to another state and "hanging out a shingle" to practice

there.

Of course, some states have specific statutes or regulations allowing out-of-state lawyers to set up shop in those states. Most states have adopted such regulations for in-house lawyers, government lawyers and military spouses.

Absent such permission, courts and bars throughout the United States have

forbidden and frequently punished such conduct.

• District of Columbia UPL Op. 22-17 (3/2/17) ("The Committee concludes that the terms 'associate' or 'counsel,' when used in a legal context, convey to members of the public that an individual is authorized to practice law. The Committee therefore concludes that, unless an individual is authorized to practice law in the District of Columbia as a member of the District of Columbia Bar or pursuant to one of Rule 49(c)'s exceptions, the individual, in

185

35951124_10 UPL, MDP and MJP (Defining What Lawyers Do and McGuireWoods LLP Where They Can Do It): Part II T. Spahn (1/1/19) Hypotheticals and Analyses

a legal context, cannot describe himself of herself as an 'associate' or as 'counsel' in connection with an office or location within the District of Columbia. Individuals who identify themselves, in a legal context, as associates or as counsel in the District of Columbia must satisfy all of the requirements of one of Rule 49(c)'s exceptions unless enrolled as active members of the District of Columbia Bar." (emphases added); "Rule 49(a) provides, subject to specific exceptions in Rule 49(c), that no person shall 'in any manner hold out as authorized or competent to practice law' in the District of Columbia unless that person is an active member of the District of Columbia Bar. Rule 49(b)(4) defines '[h]old out as authorized . . . to practice law' as 'to indicate in any manner to any other person that one is competent, authorized, or available to practice law from an office or location in the District of Columbia.' The Rule explains that,'[a]mong the characteristics which give such an indication' are 'Esq.,' 'lawyer,' 'attorney at law,' 'counselor at law,' 'contract lawyer,' 'trial or legal advocate,' 'legal representative,' and 'judge.'" (alteration in original) (emphasis added); "By way of example, an individual who is a member of another state's bar and who has submitted an application for admission to the District of Columbia Bar may describe himself or herself as an 'associate' or 'counsel' if he or she is authorized to practice law in the District of Columbia by Rule 49(c)(8), which sets forth certain requirements. That individual must also indicate in a prominent manner that the individual is not a member of the District of Columbia Bar and is practicing under the supervision of members of the District of Columbia Bar. See Rule 49(c)(8); D.C. Committee on Unauthorized Practice of Law Opinion 20-08, Limitations on Notice of Bar Status Under Rule 49(c)(8), issued on January 18, 2008. The Committee notes that many law firms use the title 'law clerk,' and limit employees to summer associate functions or other functions that are authorized by Rule 49(c) exceptions, until the employees are admitted to a bar. And the term 'summer associate' is well-understood in the legal community to mean a law student, not an attorney authorized to practice law." (emphasis added)).

• Gerber v. Disciplinary Bd. of N.D. Sup. Ct., 868 N.W.2d 861, 866, 868 (N.D. 2015) (issuing an admonition of a Minnesota lawyer holding himself out as a "staff attorney" in the law firm's North Dakota office despite the lawyer's failure to be admitted into practice in North Dakota; "[W]e conclude clear and convincing evidence establishes that Gerber and his law firm held him out as someone authorized to practice law in North Dakota. The Fredrikson firm's news release plainly states Gerber was hired as an attorney in the firm's Bismarck office, identifying him as a 'government relations specialist' and a 'staff attorney.' The release also states Gerber's 'energy practice focuses on title examination and oil and gas law.' The news release contained no disclaimers alerting the public to the fact that Gerber was not admitted to practice in North Dakota."; "[I]n addition to the law firm's news release, evidence establishes that Gerber identified himself as a 'staff attorney' while

186

35951124_10 UPL, MDP and MJP (Defining What Lawyers Do and McGuireWoods LLP Where They Can Do It): Part II T. Spahn (1/1/19) Hypotheticals and Analyses

working in Bismarck. Gerber self-identified as a 'staff attorney' on his application for admission to the North Dakota Bar. He also identified himself as a 'staff attorney' and a 'government relations attorney' in an affidavit clarifying his duties to the State Board of Law Examiners. Gerber admittedly worked as a 'staff attorney' or 'government relations attorney' in Fredrikson's Bismarck office for over a year, logging 2,476.40 billable hours, of which 1,686.34 hours were billed to clients. Based on this record, we conclude clear and convincing evidence establishes that Gerber violated N.D.R. Prof. Conduct 5.5(d).").

• Stewart Bishop, Md. Appeals Court Disbars Unlicensed Atty Who Helmed Firm, Law360, August 7, 2015 ("A Maryland appeals court on Thursday preemptively disbarred a lawyer who acted as managing attorney for a Maryland law firm despite being unlicensed to practice law in the state, saying the attorney failed to ensure client matters were correctly handled."; "The divided Court of Appeals of Maryland said Tawana D. Shephard should be disbarred in the state for her work at Glenmore Law Firm LLC in the Washington, D.C., suburb of Beltsville, Maryland."; "While Shephard is admitted to the Virginia and D.C. bars and is admitted to the bar of the U.S. District Court for the District of Maryland, she was never licensed in the state of Maryland, according to the opinion."; "The court said by acting as managing attorney for the Glenmore firm, meeting with clients in Maryland and representing clients in the state, she misled clients and the general public since she never disclosed that she wasn't licensed to practice law in the state."; "'Further, during [Shephard's] tenure as 'managing attorney,' several clients paid fees to [Glenmore] and did not receive the services that they were promised,' the judges wrote. 'As an attorney with 18 years of experience, albeit practicing in other states, we would expect [Shephard] to understand the nature of her actions and the responsibilities related thereto.'").

• Zoe Tillman, District of Columbia Man Convicted of Unauthorized Practice of Law for Two Decades, Nat'l L.J., May 10, 2012 ("District of Columbia Superior Court Judge Jóse López today convicted Benoit Brookens, 62, of contempt of court for engaging in the unauthorized practice of law."; "Brookens has never been a member of the District of Columbia Bar [a press release indicated that he had been admitted to practice in Wisconsin and Pennsylvania], but, according to court filings, maintained an office in Washington starting in the late 1970s. In a 1988 ruling, the District of Columbia Court of Appeals ordered Brookens to stop practicing law and advertising himself publicly as an attorney licensed to practice in the District."; "Brookens, who could face up to four years in prison and a $4,000 fine, is scheduled for sentencing on June 22." (emphasis added)).

187

35951124_10 UPL, MDP and MJP (Defining What Lawyers Do and McGuireWoods LLP Where They Can Do It): Part II T. Spahn (1/1/19) Hypotheticals and Analyses

• Ky. Bar Ass'n v. Moeves, 297 S.W.3d 552 (Ky. 2009) (punishing a Kentucky lawyer for the unauthorized practice of law in Ohio).

• Ramirez v. England, 320 F. Supp. 2d 368 (D. Md. 2004) (explaining that a lawyer who is a member of the Mississippi Bar but not the Maryland Bar could not practice out of a home office in Maryland; noting that the lawyer included a Maryland address on her letterhead, without an indication that she was not a member of the Maryland Bar).

• Servidone Constr. Corp. v. St. Paul Fire & Marine Ins. Co., 911 F. Supp. 560, 569, 570, 572 (N.D.N.Y. 1995) (Maryland lawyer could not recover fees for work in federal contract action pending in New York; "[A]n attorney who is not licensed to practice in New York may not establish an office in New York from which he advises clients about legal matters, whether this advice concerns New York law or the law of another jurisdiction."; "Mr. Goddard maintains an office in New York City, which is the only office from which he engages in the practice of law. Moreover, although Mr. Gemayel's phone calls to his New York client apparently were made only to inform her of the progress of the case, Mr. Goddard met with representatives of Servidone in his New York office to discuss issues which, arguably, were only tangentially related to the prosecution of the Servidone/Texas matter; i.e., the effects of a power of attorney and the drafting of a confession of judgment -- both of which concerned his fee rather than the merits of the litigation itself. "; "[A]n attorney who is not licensed to practice law in New York, even if he is associated with a properly constituted law firm in New York, may not 'practice' law in this state." (emphases added)).

• Unauthorized Practice of Law Comm. v. Bodhaine, 738 P.2d 376, 377 (Colo. 1987) (lawyer not licensed in Colorado could not practice law there; "An attorney licensed to practice law in another state may not engage in the practice of law in Colorado without obtaining a license or authorization from the Colorado Supreme Court.").

Lawyers' accurate marketing of their license status does not justify multijurisdictional violations. Describing the limitation on his or her license does not allow a lawyer to move to another state and "hang out a shingle." It is the practice of law that violates the UPL statute.

• Illinois LEO 12-09 (3/2012) (explaining that a non-Illinois lawyer may not practice physically or "virtually" in Illinois even if the law firm's co-owner is licensed in Illinois and directly supervises the non-Illinois lawyer on matters involving Illinois clients; "Two attorneys wish to establish a law practice

188

35951124_10 UPL, MDP and MJP (Defining What Lawyers Do and McGuireWoods LLP Where They Can Do It): Part II T. Spahn (1/1/19) Hypotheticals and Analyses

owned 50/50 between them. One is licensed only in Illinois, one is licensed only in State X."; "Both live and primarily work in Illinois. However, the attorney licensed in State X makes frequent visits to State X for networking and to cultivate a client base there. The attorneys agree that the Illinois-licensed attorney will have direct supervision and ultimate authority over matters involving Illinois clients, although the State X-licensed attorney will interact with Illinois clients and dispense legal advice to them from time to time."; "The Illinois-licensed attorney will sign all pleadings in Illinois courts, make all Illinois court appearances, and conduct any Illinois real estate closings personally. The State X-licensed attorney will engage in networking and market himself in Illinois as an attorney, but will take precautions to ensure that potential clients do not get the impression that he is licensed in Illinois. All letterheads and business cards will clearly and correctly indicate the jurisdictions in which each attorney is licensed to practice. Both attorneys agree to make sure, at the time any client is acquired, that the client understands that the State X-licensed attorney is not licensed in Illinois. Retainer agreements will contain bold-type disclosures to this effect."; "[T]he State X lawyer would work primarily in Illinois, which means that he would have a systematic and continuous presence (presumably including an office) in Illinois for the practice of law, in violation of paragraph (b)(1). The fact that the state of admission is accurately displayed does not vitiate that violation, as Rule 5.5(b)(1) prohibits the systematic and continuous presence, independent of the lawyer's representation as to his bar admission.").

(b) The general prohibition on lawyers' continuous practice of law in a state where they are not licensed prohibits such activity even if a lawyer practices law under the supervision of a fully-licensed lawyer.

• Illinois LEO 12-09 (3/2012) (explaining that a non-Illinois lawyer may not practice physically or "virtually" in Illinois even if the law firm's co-owner is licensed in Illinois and directly supervises the non-Illinois lawyer on matters involving Illinois clients; "Two attorneys wish to establish a law practice owned 50/50 between them. One is licensed only in Illinois, one is licensed only in State X."; "Both live and primarily work in Illinois. However, the attorney licensed in State X makes frequent visits to State X for networking and to cultivate a client base there. The attorneys agree that the Illinois-licensed attorney will have direct supervision and ultimate authority over matters involving Illinois clients, although the State X-licensed attorney will interact with Illinois clients and dispense legal advice to them from time to time." (emphasis added); "The Illinois-licensed attorney will sign all pleadings in Illinois courts, make all Illinois court appearances, and conduct any Illinois real estate closings personally. The State X-licensed attorney will engage in networking and market himself in Illinois as an attorney, but will take

189

35951124_10 UPL, MDP and MJP (Defining What Lawyers Do and McGuireWoods LLP Where They Can Do It): Part II T. Spahn (1/1/19) Hypotheticals and Analyses

precautions to ensure that potential clients do not get the impression that he is licensed in Illinois. All letterheads and business cards will clearly and correctly indicate the jurisdictions in which each attorney is licensed to practice. Both attorneys agree to make sure, at the time any client is acquired, that the client understands that the State X-licensed attorney is not licensed in Illinois. Retainer agreements will contain bold-type disclosures to this effect." (emphasis added); "[T]he State X lawyer would work primarily in Illinois, which means that he would have a systematic and continuous presence (presumably including an office) in Illinois for the practice of law, in violation of paragraph (b)(1). The fact that the state of admission is accurately displayed does not vitiate that violation, as Rule 5.5(b)(1) prohibits the systematic and continuous presence, independent of the lawyer's representation as to his bar admission.").

• Servidone Constr. Corp. v. St. Paul Fire & Marine Ins. Co., 911 F. Supp. 560, 569, 570, 572 (N.D.N.Y. 1995) (Maryland lawyer could not recover fees for work in federal contract action pending in New York; "[A]n attorney who is not licensed to practice in New York may not establish an office in New York from which he advises clients about legal matters, whether this advice concerns New York law or the law of another jurisdiction. "; "Mr. Goddard maintains an office in New York City, which is the only office from which he engages in the practice of law. Moreover, although Mr. Gemayel's phone calls to his New York client apparently were made only to inform her of the progress of the case, Mr. Goddard met with representatives of Servidone in his New York office to discuss issues which, arguably, were only tangentially related to the prosecution of the Servidone/Texas matter; i.e., the effects of a power of attorney and the drafting of a confession of judgment -- both of which concerned his fee rather than the merits of the litigation itself. "; "[A]n attorney who is not licensed to practice law in New York, even if he is associated with a properly constituted law firm in New York, may not 'practice' law in this state." (emphasis added)).

Significantly, locally licensed lawyers assisting in such an arrangement put themselves at risk of assisting another lawyer in the unauthorized practiced of law.

• ABA Model Rule 5.5(a) ("A lawyer shall not practice law in a jurisdiction in violation of the regulation of the legal profession in that jurisdiction, or assist another in doing so." (emphasis added)).

(c) Lawyers might seek to live in a state where they are not licensed, while continuously practicing -- "virtually" -- in a state where they are licensed. There are several scenarios in which such a arrangement might be attractive. For instance,

190

35951124_10 UPL, MDP and MJP (Defining What Lawyers Do and McGuireWoods LLP Where They Can Do It): Part II T. Spahn (1/1/19) Hypotheticals and Analyses

lawyers might want to continue practicing "big city" law while living in more attractive or less expensive rural settings. They might want to be near aging parents, or follow a spouse who will be attending graduate school for several years, etc.

One might wonder why the state where such lawyers will be physically present would care about any multijurisdictional implications. Presumably, that state has an interest in protecting its own citizens from lawyers representing them without local knowledge, without any supervision from that state's bar, etc. So why would that state be concerned, as long as those lawyers do not hold themselves out to practice in the state, do not represent any citizens of that state, etc.?

In this scenario, the lawyers rather than the bar would have an interest in rejecting the old "physical presence" standard, and instead focus on the "virtual" practice factors.

A February 2013 release of the ABA Ethics 20/20 Commission noted this issue, but without reaching any conclusions.

Conversely, a lawyer may be licensed in one jurisdiction, but live in a jurisdiction where the lawyer is not licensed. If the lawyer conducts a virtual practice from the latter jurisdiction and serves clients only in the jurisdiction where the lawyer is actually licensed, there is a question of whether the lawyer has a "systematic and continuous" presence in the jurisdiction where the lawyer is living and thus violates Rule 5.5(b) in that jurisdiction. The Rule is unclear in this regard as well.

Am. Bar Ass'n Comm'n on Ethics 20/20, Introduction and Overview, at 10 n.27 (Feb.

2013).

191

35951124_10 UPL, MDP and MJP (Defining What Lawyers Do and McGuireWoods LLP Where They Can Do It): Part II T. Spahn (1/1/19) Hypotheticals and Analyses

Most other states continue to focus on lawyers' physical presence – generally barring lawyers domiciled in the state from practicing law in that state without a license -

- even if they do not provide advice about their host state's law.

Predictably, Florida takes such a protective approach. A Florida Rules comment prohibits lawyers from establishing "an office or regular presence" in Florida – even if they limit their practice to the law of a state where they are licensed.

• Florida Rule 4-5.5 cmt. (". . . a lawyer licensed to practice law in New York could not establish an office or regular presence in Florida to practice New York law. Such activity would constitute the unlicensed practice of law. However, for purposes of this rule, a lawyer licensed in another jurisdiction who is in Florida for vacation or for a limited period of time, may provide services to their clients in the jurisdiction where admitted as this does not constitute a regular presence. The lawyer must not hold out to the public or otherwise represent that the lawyer is admitted to practice law in Florida. Presence may be regular even if the lawyer is not physically present here. (emphasis added)

Florida is among the most jealous of all states about out-of-state lawyers practicing there (especially on a systematic basis). The reference to New York lawyers trying to horn in on Florida lawyers' practice is a blatant shot across the bow of New York lawyers seeking a warmer climate.

Some states have adopted the same approach, but in a more subtle fashion.

For instance, Colorado prevents lawyers domiciled in that state from taking advantage of the ethics provisions permitting non-Colorado lawyers to temporarily practice in the state.

• Rules Governing Admission To The Practice Of Law In Colorado; Rule 205.1 (Revised 2014) ("(1) Eligibility. An attorney who meets the following conditions is an out-of-state attorney for the purpose of this rule: (a) The attorney is licensed to practice law and is on active status in another jurisdiction in the United States; (b) The attorney is a member in good standing of the bar of all courts and jurisdictions in which he or she is

192

35951124_10 UPL, MDP and MJP (Defining What Lawyers Do and McGuireWoods LLP Where They Can Do It): Part II T. Spahn (1/1/19) Hypotheticals and Analyses

admitted to practice; (c) The attorney has not established domicile in Colorado; and (d) The attorney has not established a place for the regular practice of law in Colorado from which the attorney holds himself or herself out to the public as practicing Colorado law or solicits or accepts Colorado clients." (emphasis added)).

In 2015, some New York lawyers unsuccessfully sought to move New York in the

direction of allowing such practice.

• ABA/BNA Lawyers' Manual on Professional Conduct, Vol. 31, No. 26 (12/30/15) ("While characterizing the new temporary practice rule as a 'huge advance' for New York, Davis [Anthony Davis of Hinshaw & Culbertson LLP] pointed out that the court of appeals did not adopt a recommendation made by a group of large law firms that have offices in New York and other jurisdictions."; "That recommendation took aim at the situation where a lawyer lives in one state but practices law remotely from an office in another state where the lawyer is admitted to practice. For example, a lawyer may live in New York and commute to New Jersey or Connecticut. In this situation, Davis said, it's a stretch to say the lawyer is temporarily in New York, and the lawyer arguably may be engaging in unauthorized practice there."; "In comments on the proposed temporary practice rule, the large firms recommended additional language that would allow a lawyer authorized to practice law in a U.S. jurisdiction to provide legal services that exclusively involve federal law, the law of another jurisdiction or tribal law, provided the lawyer does not hold himself out in any way as having an office for the practice of law in New York."; "This language, the firms said, would prevent UPL accusations against lawyers who live in New York and use technology to practice remotely as if they were physically in their state of admission, provided they do not practice New York law or hold themselves out as doing so.).

A few states have allowed lawyers to systematically and continuously practice within a state without being licensed there -- as long as they do not give any clients

advice about the law of that state. This approach does not focus specifically on "virtual"

practice. For instance, a new associate might work in a large law firm's office without

being licensed in that state, and exclusively conduct research into New York and

Delaware corporate law, etc. But focusing on the type of legal advice that such lawyers

193

35951124_10 UPL, MDP and MJP (Defining What Lawyers Do and McGuireWoods LLP Where They Can Do It): Part II T. Spahn (1/1/19) Hypotheticals and Analyses

can give opens up the possibility of lawyers physically present in states where they are not licensed "virtually" representing clients in states where they are licensed.

In 2011, Virginia indicated that such practice does not run afoul of the MJP rules -- as long as the lawyer physically present in Virginia limits her legal advice to federal law or to the law of states where she is licensed.

• Virginia LEO 1856 (9/19/11) (explaining that under Virginia Rule 5.5, non- Virginia lawyers "may not practice Virginia law on a 'systematic and continuous' basis," unless they (1) limit their practice to the "law of the jurisdiction/s where they are licensed"; (2) practice "exclusively federal law" under the federal supremacy clause (such as "lawyers with practices limited to immigration or military law or who practice before the Internal Revenue Service, the United States Tax Court, or the United States Patent and Trademark Office," although lawyers such as bankruptcy, patent or federal procurement lawyers must abide by courts' possible limitation of practice before the courts to members of the Virginia Bar, and may provide advice "such as the debtor's homestead exemption and status or priority of claims or liens" or "the assignment of the patent to a third party or the organization of a corporate entity to market or franchise the invention" only under the conditions mentioned immediately below; (3) "provide advice about Virginia law or matters peripheral to federal law (described immediately above) only if they do so on a "temporary and occasional" basis and (as stated in UPL Opinion 195) "under the direct supervision of a Virginia licensed lawyer before any of the [non-Virginia] lawyer's work product is delivered to the client" or if they "associate with an active member of the Virginia State Bar."; noting that Rule 5.5 overrules an earlier UPL Opinion about which law applies to a non-Virginia lawyer's practice of another state's law while physically in Virginia; thus, "New York law should govern whether a foreign lawyer not authorized to practice in New York may advise New York clients on matters involving New York law. The [non-Virginia] lawyer's physical presence in Virginia may not be a sufficient basis to apply Virginia's rules over New York's rules governing foreign lawyer practice." Contract lawyers hired to "work on a matter involving Virginia law" must either "be licensed in Virginia or work in association with a Virginia licensed lawyer in the firm on a temporary basis" although such a lawyer's practice "could be regarded as 'continuous and systematic'" if the non-Virginia contract lawyer is hired "to work on several and various Virginia matters/cases over a period of time."; concluding that such contract lawyers need not be licensed in Virginia if the lawyer is "hired to work only on matters involving federal law or the law of the jurisdiction in which the [non-Virginia] contract lawyer is admitted.").

194

35951124_10 UPL, MDP and MJP (Defining What Lawyers Do and McGuireWoods LLP Where They Can Do It): Part II T. Spahn (1/1/19) Hypotheticals and Analyses

More recently, Arizona amended its ethics rules to permit lawyers to practice

continuously in Arizona (without a license there) as long as they give advice only about

the law of a state where they are licensed (or federal or tribal law).

A lawyer admitted in another United States jurisdiction, or a lawyer admitted in a jurisdiction outside the United States, not disbarred or suspended from practice in any jurisdiction may provide legal services in Arizona that exclusively involve federal law, the law of another jurisdiction, or tribal law.

Arizona Rule 5.5(d) (emphasIs added).

If non-Arizona lawyers can practice law under those limitations "in Arizona," they

clearly can practice "virtually" in a state where they are licensed. That other state

presumably would not object to such a practice, because the lawyer is licensed there

(although that other state might have some archaic requirements that the lawyer have a

physical office in that state, etc.).

The Arizona rule change is more significant than Virginia's 2011 legal ethics

opinion -- both because it is a formal rule, and because Arizona made the change in the midst of the nationwide debate about the multijurisdictional practice implications of a

"virtual" practice.

195

35951124_10 UPL, MDP and MJP (Defining What Lawyers Do and McGuireWoods LLP Where They Can Do It): Part II T. Spahn (1/1/19) Hypotheticals and Analyses

Best Answer

The best answer to (a) is NO (PROBABLY); the best answer to (b) is NO

(PROBABLY); the best answer to (c) is MAYBE. B 2/13, 3/17

196

35951124_10 UPL, MDP and MJP (Defining What Lawyers Do and McGuireWoods LLP Where They Can Do It): Part II T. Spahn (1/1/19) Hypotheticals and Analyses

Lawyers "Waiving in" to Another State's Bar

Hypothetical 18

You have practiced law for several years in the city where you and your wife met while attending college. She just obtained her master's degree, and earned a spot in a very prestigious Ph.D. program at a university in a neighboring state (where you are not licensed). One of your best friends from law school works at a large firm located within blocks of where your wife will be earning her doctorate, and has urged you to apply for a job there.

Will you be able to "waive in" to the neighboring state's bar?

MAYBE

Analysis

Because states jealously guard their parochial power over the legal profession,

they could insist that any lawyer wanting to continuously practice in the state take the

bar exam.

California requires all lawyers who practice there to take the California Bar exam.

In 2016, Florida similarly rejected a proposal to allow out-of-state lawyers to be admitted to that state without taking the bar exam.

• Shayna Posses, Florida Bar Governors Reject Out-Of-State Lawyer Reciprocity, Law360, Oct. 16, 2015 ("The Florida Bar's governing board on Friday shot down a controversial proposal to allow admission to out-of-state attorneys without taking the state bar exam, ending months of backlash from groups concerned about how the proposition would affect legal representation in Florida."; "The bar's Board of Governors voted unanimously to reject admission by motion following the multijurisdictional practice-state focus committee's Oct. 1 final report. The board recommended further study and evaluation before an informed decision can be reached.").

197

35951124_10 UPL, MDP and MJP (Defining What Lawyers Do and McGuireWoods LLP Where They Can Do It): Part II T. Spahn (1/1/19) Hypotheticals and Analyses

However, most states permit lawyers from other states to move there and begin continuously practicing law -- without taking the bar exam. This is popularly called

"waiving in" to the bar, but is technically called "admission by motion."

An August 2012 report by the ABA Ethics 20/20 Commission described the history and result of this procedure.

In August 2002, the ABA House of Delegates adopted the Model Rule on Admission by Motion. The Model Rule permits a lawyer admitted in one U.S. jurisdiction to gain full admission in another U.S. jurisdiction without having to pass that jurisdiction's bar examination. The lawyer, however, must satisfy several requirements, one of which is to have engaged in the active practice of law for five of the last seven years.

Admission by motion procedures now exist in forty jurisdictions. The Commission's research revealed that more than 65,000 lawyers have used the procedure in the last ten years. Approximately half of these lawyers were admitted in the District of Columbia. The Commission found that there is no evidence that lawyers admitted by motion -- either in the District of Columbia or elsewhere -- are more likely to be subject to discipline, disciplinary complaints, or malpractice suits than lawyers admitted through more traditional procedures.

Am. Bar Ass'n Comm'n on Ethics 20/20, Report to the House of Delegates, at 2 (Aug.

2012).

Not every state follows the ABA Model Rule on Admission by Motion, but many do.

In the same year (2016), that the Florida Bar rejected admission by motion, New

Jersey allowed such admission for the first time.

• New Jersey Supreme Court, Administrative Determinations on the Report and Recommendations of the Special Committee on Attorney Ethics and Admissions (4/14/16) ("After careful review, the Supreme Court decided to

198

35951124_10 UPL, MDP and MJP (Defining What Lawyers Do and McGuireWoods LLP Where They Can Do It): Part II T. Spahn (1/1/19) Hypotheticals and Analyses

adopt admission by motion with the recommended additional restrictions. The Court has studied this proposal in various forms since 1983, when the Supreme Court Advisory Committee on Bar Admissions issued its report recommending admission by motion. The Ad Hoc Committee on Bar Admissions (Wallace Committee) similarly recommended admission by motion in its report issued in 2002. The time has come for New Jersey to join the vast majority of United States jurisdictions that offer admission by motion. The Court acknowledges the legitimate concern that its decision may have adverse consequences for some New Jersey lawyers, particularly given today’s economic climate. It finds, however, that the current rule, requiring out-of-state lawyers to retake a multi-state bar exam that does not test New Jersey law, is a barrier to the practice of law in New Jersey that is not grounded in the public interest.").

Several bars groups have been pushing states in that direction.

• Sue Reisinger, Association of Corporate Counsel Challenges New Mexico In- House Counsel Rules, Corporate Counsel, July 31, 2013 ("The New Mexico Supreme Court shouldn’t be gambling with a legal client’s services, said the Association of Corporate Counsel (ACC) in a comment letter sent to Chief Justice Petra Jimenez Maes last week."; "The letter urges the court to pass new rules removing restrictive bar admission standards for experienced lawyers from outside the state, if the other state reciprocates. But the letter goes further than the proposed rules, arguing for an end to reciprocity requirements."; "Amar Sarwal, ACC’s vice president and chief legal strategist, issued a statement criticizing the current New Mexico rules."; "'Requiring experienced lawyers from other states to retake the bar exam in order to practice in New Mexico privileges the parochial interests of local lawyers, rather than those of the clients they ostensibly serve,' Sarwal said."; "'In fact, the proposal's reciprocity requirement also needlessly benefits lawyers alone, while depriving clients of their experienced counsel of choice,' he added."; "Currently in New Mexico, unlike a majority of United States jurisdictions, attorneys cannot gain admission to practice law by motion or under any form of reciprocity with other states. Likewise, New Mexico attorneys applying in other states are not allowed similar access to admission without passing the bar exam."; "'Choice of counsel of clients and prospective clients is unnecessarily restricted,' states the New Mexico [proposed revision]. 'The reality of transboundary law practice is not served by New Mexico’s lack of admission by motion.'"; "The proposal goes on to state that requiring all out-of-state attorneys to sit for the bar exam is 'a relic and outdated.'"; "But the ACC letter contends that reciprocity doesn’t fix the problem. 'Instead, reciprocity subjects the needs of clients to the whims of state-to-state diplomacy. A more open admission system serves clients better,' it says.").

199

35951124_10 UPL, MDP and MJP (Defining What Lawyers Do and McGuireWoods LLP Where They Can Do It): Part II T. Spahn (1/1/19) Hypotheticals and Analyses

Under most states' approach, a lawyer seeking admission by motion must have been admitted to practice in another U.S. jurisdiction; hold a degree from an ABA- approved law school; have been primarily engaged in the active practice of law for five out of the previous seven years in the state where the lawyer was fully licensed to practice law; demonstrate that the lawyer is in good standing in any such jurisdiction; not currently be subject to lawyer discipline; and meet the new state's character and fitness standards. States disagree about the meaning of the term "active practice of law," but most states count private practice, in-house or governmental practice, or service as a professor or a judge. Many states do not permit admission by motion if the lawyer has recently failed the bar exam anywhere.

States allowing admission by motion generally extend that privilege only to lawyers from states which grant its lawyers the same opportunity -- called "reciprocity."

Lawyers hoping to gain admission by motion to another state's bar therefore must first determine if the state where they are practicing and the state where they hope to move grant such reciprocal rights. There are actually some surprising situations in which states do not grant each other's lawyers such reciprocal rights.

States' reciprocity arrangements change from time to time.

Not surprisingly, this reciprocity requirement has been challenged on constitutional grounds. Most challenges have failed.

• Nat'l Ass'n for Advancement of Multijurisdiction Practice v. Howell, 851 F.3d 12, 16, 20 (D.C. Cir. 2017) (upholding the admission rules for the United States District Court for the District of Columbia, which required that lawyers be members in good standing of the D.C. Bar, or of the bar in any state where they "maintain their principal law office" (citation omitted); "The Court does not doubt the sincerity of NAAMJP's convictions or its eagerness to reduce barriers to legal practice in the various state and federal courts across 200

35951124_10 UPL, MDP and MJP (Defining What Lawyers Do and McGuireWoods LLP Where They Can Do It): Part II T. Spahn (1/1/19) Hypotheticals and Analyses

the country. Indeed, there may be good policy reasons for the outcomes NAAMJP urges. But, as has been amply demonstrated in dozens of legal opinions penned by judges across the country, NAAMJP has identified no legal basis upon which to compel federal or state courts to adopt the rules it desires. Accordingly, the judgment of the district court is Affirmed."), cert. denied, 138 S. Ct. 420 (2017).

• Nat'l Ass'n for Advancement of Multijurisdictional Practice v. Castille, 66 F. Supp. 3d 633, 638, 661 (E.D. Pa. 2014) (upholding Pennsylvania's reciprocity requirement for lawyers admitted by motion; "This lawsuit is a constitutional challenge to Pennsylvania's reciprocal bar admissions rule. The rule in question, Rule 204, Pennsylvania Bar Admission Rules, provides that the Pennsylvania bar will allow experienced lawyers admitted in other states to join the Pennsylvania bar without taking the Pennsylvania bar exam, subject to certain additional requirements. The particular additional requirement at issue here limits admission by motion to lawyers practicing in states that also allow Pennsylvania lawyers to gain admission by motion. In other words, Rule 204 only allows admission by motion for lawyers admitted in states with reciprocal admission-by-motion policies. Plaintiffs contend that this reciprocity policy infringes the rights of lawyers who wish to practice in Pennsylvania but now practice only in a state that does not have a reciprocal admission policy."; "Plaintiffs are two individuals and an organization. Mr. Rosario, an attorney, graduated from an accredited law school in Maryland and is admitted to practice law in Maryland and Washington, D.C. He applied for admission to the Pennsylvania bar, but Pennsylvania rejected his application because Maryland is not a reciprocal state, and he had gained admission to the D.C. bar by motion rather than exam. Mr. Rosario asserts that he would apply for reciprocal admission in Pennsylvania again if the rules changed."; "This decision only examines the constitutionality of Rule 204, not the wisdom of the policy. Plaintiffs marshal substantial evidence that preventing experienced lawyers from gaining admission to additional state bars is an outdated policy that ignores the modern realities of legal practice and limits the choices of consumers of legal services. For example, Plaintiffs point out that the American Bar Association recommends that states grant admission by motion for experienced lawyers without regard to whether other states grant reciprocal admission. Plaintiffs' Motion for Summary Judgment 8 (citing ABA Multijurisdictional Practice Commission (2002) and ABA Commission on Ethics 20/20 (2012))."; "However, the Constitution does not mandate that states adopt the best policies. It requires only that states do not stray too far from fundamental principles that define the proper bounds of governmental power. In this case Pennsylvania has not exceeded its authority.").

• Maria Chutchian, Out-of-State Attorneys Challenge Arizona Bar Admissions Rules, Law360, August 21, 2012 ("The National Association for the

201

35951124_10 UPL, MDP and MJP (Defining What Lawyers Do and McGuireWoods LLP Where They Can Do It): Part II T. Spahn (1/1/19) Hypotheticals and Analyses

Advancement of Multijurisdiction Practice (NAAMJP) last week challenged the Arizona Supreme Court's rules forcing attorneys already licensed in certain states to take the state's bar exam in order to practice law within its borders."; "In a complaint filed Aug. 13 in Arizona federal court, the association and two attorneys sued the Arizona Supreme Court and three of its justices, questioning the constitutionality of the state's bar admission rules. Though it is one of nine states to have adopted the uniform bar exam [UBE], Arizona only allows attorneys from other states admission on motion if Arizona lawyers are given reciprocity in their home states, known as 'tit-for-tat' rules."; "The suit seeks an order invalidating the Arizona Supreme Court's rule, finding tit-for-tat admission rules unconstitutional, and admitting plaintiffs Allison Girvin and Mark Anderson -- who have licenses to practice in California and Montana, respectively -- to the state bar."; "'The main thrust of our argument is that there's no reason you should be discriminated against based on your prior place of residence,' Grant Savoy of the NAAMJP said.").

In August 2012, the ABA changed the ABA Model Rule for Admission by Motion.

Under the revised approach, a lawyer can move to another state after practicing only three out of the previous five years elsewhere.

In its supporting report, the ABA Ethics 20/20 Commission noted that five jurisdiction have already moved to this shorter duration-of-practice requirement. The

Commission explained its reasoning.

The Commission believes this change responds to client needs and market demands in an increasingly borderless world, where lawyers frequently need to gain admission in other U.S. jurisdictions. For example, lawyers regularly need to move to, or establish a regular practice in, another jurisdiction in order to serve clients who are relocating or who regularly do business in the jurisdiction in which motion admission is sought. The Commission's proposal would address this need, thus benefitting both lawyers and their clients.

The proposal also recognizes that lawyers often need to move to new jurisdictions for a wide range of personal reasons, including the need to find employment. The Commission determined that a reduction of the active practice requirement from five to three years would have particularly salutary effects for less senior lawyers, who are

202

35951124_10 UPL, MDP and MJP (Defining What Lawyers Do and McGuireWoods LLP Where They Can Do It): Part II T. Spahn (1/1/19) Hypotheticals and Analyses

most likely to need to move from one jurisdiction to another. The challenging legal employment marketplace only increases the likelihood that relatively junior lawyers will need to move to a new jurisdiction in search of employment.

Am. Bar Ass'n Comm'n on Ethics 20/20, Report to the House of Delegates, at 3 (Aug.

2012).

The ABA maintains current publicly available charts of states' requirements and process for admission by motion. The National Conference of Bar Examiners also has useful current information about states' differing requirements.

Best Answer

The best answer to this hypothetical is MAYBE.

B 2/13, 3/17

203

35951124_10 UPL, MDP and MJP (Defining What Lawyers Do and McGuireWoods LLP Where They Can Do It): Part II T. Spahn (1/1/19) Hypotheticals and Analyses

Permissible Activities While Waiting to be Admitted by Motion in a New State

Hypothetical 19

After your wife was admitted to a prestigious Ph.D. program in a neighboring state, you accepted a job offer at a firm near the university where your wife will be studying. Fortunately, you have been practicing for five years in your home state, so you are eligible to apply for admission by motion to the bar of the neighboring state. You started work this morning, and you want to make sure that you do not engage in any improper activities until you are a full member of your new state's bar.

(a) Can you use business cards with your new firm's name and address?

YES (IF IT ACCURATELY DESCRIBES YOUR BAR MEMBERSHIP)

(b) Can you negotiate business transactions with opposing counsel on your own?

YES (IF YOU WORK WITH A LICENSED LAWYER)

(c) May you appear in your new state's courts?

YES (IF YOU ARE ADMITTED PRO HAC VICE)

(d) Will you face a filing deadline for your application to be admitted by motion in your new state?

YES

(e) Once you are admitted by motion in the new state, will you have all the rights of lawyers who were admitted after taking the bar exam?

YES (PROBABLY)

Analysis

All or most jurisdictions allowing admission by motion have all adopted their own

specific rules for what lawyers moving into the state and continuously practicing there

204

35951124_10 UPL, MDP and MJP (Defining What Lawyers Do and McGuireWoods LLP Where They Can Do It): Part II T. Spahn (1/1/19) Hypotheticals and Analyses

may and may not do before they are admitted. In 2012, the ABA for the first time

adopted a Model Rule on Practice Pending Admission.

Most states have been moving in the direction of liberalizing their admission by motion standards, as well as their rules governing what lawyers may do while they wait.

(a) Anyone working at a law firm or in another place which might be perceived

as involving the practice of law may use business cards, have website bio pages, be

listed in the lobby of their building, etc. -- but only if the business card or other public

description of the person's employment accurately describes his or her role.

Lawyers who are admitted in some jurisdiction but now continuously practicing in

another jurisdiction pending admission in that jurisdiction (either by motion or by taking

the bar) must follow this general common-sense rule. Some lawyers in this situation

use a phrase such as: "Not admitted in North Carolina. Admitted in Utah only." Most

bars seem to accept a more positive "spin," such as "Admitted in Utah only."

(b) Most states require lawyers continuously practicing in the state but not yet

admitted there to be supervised by a fully licensed lawyer in that state. There might be

a question about the degree of supervision required. Most states seem to allow such

lawyers to essentially act on their own, as long as they receive direction from and report

back to a fully licensed lawyer.

The ABA Model Rule on Practice Pending Admission (adopted 8/6/12) requires

less than the currently popular standard -- instead allowing such lawyers to continuously

practice in their new state (pending admission) as long as the lawyer "associates with a

lawyer who is admitted to practice in this jurisdiction." ABA Model Rule on Practice

Pending Admission 1(f).

205

35951124_10 UPL, MDP and MJP (Defining What Lawyers Do and McGuireWoods LLP Where They Can Do It): Part II T. Spahn (1/1/19) Hypotheticals and Analyses

The Commission explained its reasoning.

[T]he lawyer must associate with a lawyer who is licensed to practice in the jurisdiction. This requirement is designed to ensure that the incoming lawyer has the ability to consult with a lawyer who is licensed in the jurisdiction regarding any issues that may require knowledge of distinctly local laws or procedures. This requirement is similar to the requirement in Rule 5.5(c)(1), which permits an out-of-state lawyer to practice temporarily in the jurisdiction if the lawyer associates with a lawyer who is admitted in the jurisdiction. The Commission was reluctant to impose a stricter requirement, such as a requirement to be directly supervised by a lawyer who is admitted in the jurisdiction, because of the particular obstacles such a requirement would impose on solo practitioners. In particular, in-state lawyers may be reluctant to directly supervise a lawyer from another jurisdiction who is not in the same office, because of the administrative difficulties associated with supervising a lawyer in a different law office. For these reasons, the Commission concluded that the "association" requirement, which has worked well in the context of temporary practice under Rule 5.5(c)(1), is an adequate safeguard in the context of the Model Rule on Practice Pending Admission as well.

Am. Bar Ass'n Comm'n on Ethics 20/20, Report to the House of Delegates, at 4 (Aug. 6,

2012) (emphases added).

In 2017, the Ohio Supreme Court rejected the Ohio Bar's narrow approach to what a Kentucky lawyer could do while physically practicing in Ohio awaiting her admission by motion into the Ohio Bar.

• In re Jones, 2018-Ohio-4182, --- N.E.3d ---, at ¶¶ 4, 20, 1, 2 (Ohio 2018) (over a strong dissent (concurring in the result but for different reasons) and a contrary position taken by the Ohio Bar, approving a Kentucky lawyer's application for admission without examination for the Ohio Bar, despite having moved to Ohio one month after filing her application and having since then "practiced law exclusively in matters relating to pending or potential proceedings before tribunals in Kentucky"; finding that her practice in Ohio was "temporary" because "those services are transitory and will continue only until the application is resolved"; "Alice Auclair Jones is licensed to

206

35951124_10 UPL, MDP and MJP (Defining What Lawyers Do and McGuireWoods LLP Where They Can Do It): Part II T. Spahn (1/1/19) Hypotheticals and Analyses

practice law in Kentucky and applied for admission to the practice of law in Ohio without examination. The Board of Commissioners on Character and Fitness found that after Jones filed her application, she engaged in the unauthorized practice of law by practicing Kentucky law from an Ohio office and failed to prove she has the requisite character, fitness, and moral qualifications to practice law in Ohio; therefore, it recommends that we disapprove her application. Jones objects to the findings and the recommendation."; "The issues presented are whether Jones violated Prof. Cond. R. 5.5(c), which permits a lawyer to 'provide legal services on a temporary basis in this jurisdiction' under certain circumstances, and whether this court should approve her application. We conclude that a lawyer admitted to practice law in another jurisdiction who provides legal services exclusively in that jurisdiction from an office in Ohio pending resolution of an application for admission to the Ohio bar withiout examination and who otherwise complies with the provisions of Prof. Cond. R. 5.5(c) is providing legal services on a temporary basis and therefore has not engaged in the unauthorized practice of law. Because Jones complied with the rule, we conclude she did not engage in the unauthorized practice of law and has the requisite character, fitness, and moral qualifications to practice law in Ohio, and we approve her application pursuant to Gov. Bar R.I(9)(F)(1).").

(c) States permitting lawyers to continuously practice in the state while waiting to be admitted by motion or to take the bar exam generally prohibit such lawyers from appearing in the courts of that state without being admitted pro hac -- just as if the lawyer were practicing in another state. See, e.g., ABA Model Rule on Practice

Pending Admission 3.

(d) The jurisdictions that currently allow admission by motion have adopted their own deadlines for lawyers moving into the state and planning to seek such admission by motion.

States generally have adopted one or both of two deadlines. One deadline sets the time for the lawyer to apply for admission by motion or to take the bar exam. The other deadline sometimes sets a date by which such lawyers must be fully admitted to practice in the state, or (presumably) leave the state.

207

35951124_10 UPL, MDP and MJP (Defining What Lawyers Do and McGuireWoods LLP Where They Can Do It): Part II T. Spahn (1/1/19) Hypotheticals and Analyses

The ABA Model Rule for Practice Pending Admission requires a lawyer planning to move to another state and apply for admission by motion in that state to notify the new state's Disciplinary Council and the "Admissions Authority" before moving to the new jurisdiction, and submit a complete application for admission by motion or by examination within 45 days of moving to the new state. Such lawyers may engage in the continuous practice of law in the new state for no more than 365 days, as long as the lawyer complies with the restrictions contained elsewhere in the Model Rule. ABA

Model Rule on Practice Pending Admission.

Some states take a fairly liberal approach, and insist only that such lawyers move promptly.

• North Carolina State Bar Authorized Practice Comm., Guidelines for Attorneys Licensed in other Jurisdictions (July 2003) (explaining that a lawyer licensed in some other state should seek admission to the North Carolina Bar quickly after moving to North Carolina; "Ideally, such an unlicensed attorney should apply for admission to the North Carolina State Bar before her [sic] or she relocates to the state, begins work at a law firm located in the state, or engages in any activities constituting the practice of law in North Carolina. If the unlicensed attorney has already relocated to North Carolina or begun work at a firm located in the state under the supervision of a licensed North Carolina attorney, he or she is encouraged to apply for admission promptly. The Board of Law Examiners may question the reasons for any significant delay in submitting an application for admission to the North Carolina State Bar after the unlicensed attorney relocates to North Carolina." (emphasis added)).

Other jurisdictions have very specific deadlines.

• D.C. Ct. App. R. 49(c)(8) (allowing the following authority "[p]racticing law from a principal office located in the District of Columbia, while an active member in good standing of the highest court of a state or territory, and while not disbarred or suspended for disciplinary reasons or after resignation with charges pending in any jurisdiction or court, under the direct supervision of an enrolled, active member of the District of Columbia Bar, for one period not to exceed 360 days from the commencement of such practice, during pendency of a person's first application for admission to the District of

208

35951124_10 UPL, MDP and MJP (Defining What Lawyers Do and McGuireWoods LLP Where They Can Do It): Part II T. Spahn (1/1/19) Hypotheticals and Analyses

Columbia Bar; provided that the practitioner has submitted the application for admission within ninety (90) days of commencing practice in the District of Columbia, that the District of Columbia Bar member takes responsibility for the quality of the work and complaints concerning the services, that the practitioner or the District of Columbia Bar member gives notice to the public of the member's supervision and the practitioner's bar status, and that the practitioner is admitted pro hac vice to the extent he or she provides legal services in the courts of the District of Columbia." (emphases added)).

(e) Most states treat lawyers admitted by motion to their bar exactly the same

as lawyers who have taken the bar exam. However, as in so many other areas

involving multijurisdictional practice, some states have taken a different approach.

Virginia repeatedly wrestled with the status of lawyers who were admitted by

motion rather than by examination. Although Virginia ultimately determined to treat

them exactly the same, the many twists and turns on the way to that conclusion

highlight the inherit hostility some states exhibit toward lawyers who have not taken the

bar exam to gain admission to their bars.

Virginia traditionally only allowed admission by motion to lawyers intending to practice full time as a member of the Virginia State Bar. Regulations defined this as thirty-five hours per week. Former Supreme Court Rule 1A:1 and 1A:3.

In 2012, the Virginia Supreme Court triggered anxiety in many Virginia lawyers

admitted by motion by indicating that such lawyers who stopped practicing full time in

Virginia might lose their ability to practice in the state. Over 2,000 of Virginia's nearly

30,000 lawyers had been admitted by motion, and were theoretically at risk of losing

their right to practice.

This controversy generated substantial publicity in Virginia.

• Peter Vieth, Waiving In: Supreme Court floats new rules on admission of out-of-state lawyers, Va. Law. Wkly., Dec. 17, 2012 ("Debate continues in

209

35951124_10 UPL, MDP and MJP (Defining What Lawyers Do and McGuireWoods LLP Where They Can Do It): Part II T. Spahn (1/1/19) Hypotheticals and Analyses

Virginia legal circles over how the state should regulate lawyers who 'waive in' to Virginia practice without taking the bar exam."; "Lawyers who take another state's bar exam can gain admission to the Virginia State Bar (VSB) 'on motion' after satisfying a number of procedural requirements, a system generally referred to as 'waiving in' to the VSB."; "Voices of dissent rose to a full howl in 2011 when many of the lawyers admitted in this fashion discovered they risked losing their licenses if they failed to maintain full-time law offices in the state. The Supreme Court of Virginia has since proposed relaxed requirements for waived-in lawyers, but lawyer comments on the proposal have been all over the map. Some say the proposed rule revisions still don't go far enough. Many others suggest tweaks."; "Comments received by the Supreme Court indicate some lawyers -- and an American Bar Association (ABA) panel -- believe the court should open up admissions by motion without regard to whether an applicant is licensed by a state with reciprocal privileges or whether the lawyer practices predominantly in Virginia."; "Other comments suggest the court should adjust the rules to avoid unfairly excluding lawyers with backgrounds as corporate counsel, military lawyers and federal practitioners."; "The issue affects about 7 percent of the lawyers in Virginia. According to figures from the Virginia State Bar, 2,114 of 29,941 active VSB members were admitted by reciprocity as of November 20."; "The uproar of 2011 was sparked by a planned administrative change. The Supreme Court wanted to hand off to the VSB the job of policing the full-time practice rule. When the court's rule change proposal was circulated, many lawyers believed a new restriction was in the offing."; "The object of the ire was the requirement -- spelled out in regulations -- that bar admission can be revoked if a lawyer admitted on motion ever fails to 'practice full time as a member of the Virginia State Bar.' The regulations specified at least 35 hours a week of practice with a regular in-state office location. Lawyers admitted on motion would have been barred from splitting time between a Virginia office and any outside office location."; "Many waived-in lawyers had not realized their practice was subject to the full-time rule. They complained the rule unfairly hampered attorneys taking time off for child birth or child care, older lawyers hoping to limit their work hours and retired lawyers who wanted to offer pro bono services."; "Under the reciprocity rule, a lawyer licensed in another state is eligible to practice in Virginia without examination only if Virginia lawyers may be admitted to practice in that state without examination. Of the states allowing admission on motion, 25 require reciprocity, according to figures compiled by the ABA."; "Eleven states, including Maryland and South Carolina, do not allow admission on motion at all.").

210

35951124_10 UPL, MDP and MJP (Defining What Lawyers Do and McGuireWoods LLP Where They Can Do It): Part II T. Spahn (1/1/19) Hypotheticals and Analyses

On October 22, 2012, the Virginia Supreme Court issued proposed amendments to these rules. One of the most important amendments would permit lawyers to apply for admission by motion in Virginia if they intend

for at least five years immediately thereafter to practice predominantly in this Commonwealth as a member of the Virginia State Bar. "Predominantly" means that during each year the applicant's practice in Virginia must exceed, or be equal to, his or her practice in all other jurisdictions combined.

Proposed Amendment to Virginia Supreme Court Rule 1A:1 (adopted on October 22,

2012).

The second important change would loosen somewhat the requirement that such lawyers continue to practice in that fashion for the rest of their legal careers.

Until a person who has been admitted under Rule 1A:1 has practiced law predominately in Virginia for a total of five years following his or her admission, which are not required to be five consecutive years, such person shall certify annually to the Virginia State Bar whether he or she currently satisfies the requirements of clause (c) of that Rule and furnish such supporting evidence as the Bar may require. Following receipt of evidence satisfactory to the Supreme Court that a person who has been admitted to practice pursuant to Rule 1A:1 no longer satisfies the requirement of clause (c) of that Rule, the Supreme Court may revoke the certificate issued to that person; provided, however, that a person who has practice law predominately in Virginia for at least five years after having been admitted pursuant to Rule 1A:2 shall thereafter be deemed to have satisfied the requirement of clause (c) of that Rule.

If a person admitted under Rule 1A:1 who has practiced law predominately in Virginia for less than five years elects to stop practicing law predominately in Virginia, he or she shall immediately notify the Virginia State Bar, at which time such person shall be transferred to Associate status. When such person thereafter notifies the Virginia State Bar in writing of his or her intent to resume practicing

211

35951124_10 UPL, MDP and MJP (Defining What Lawyers Do and McGuireWoods LLP Where They Can Do It): Part II T. Spahn (1/1/19) Hypotheticals and Analyses

law predominately in Virginia, he or she shall be returned to Active status upon satisfying the requirements then in effect.

Following receipt of evidence that a person who has been admitted to practice pursuant to Rule 1A:1, or pursuant to Rule 1A:2 prior to July 1, 2000, has been disbarred pursuant to Part Six of the Rules, the Supreme Court will revoke the certificate issued to that person.

Proposed Amendment to Virginia Supreme Court Rule 1A:3.

Although this new proposed rule would have made it easier for lawyers admitted

by motion in Virginia to keep practicing after an initial period of full-time practice, it predictably drew severe criticism from Virginia lawyers. So the Virginia Supreme Court tried again.

On April 15, 2013, the Virginia Supreme Court invited comments on a new proposal that liberalized the admission by motion process in several ways.

First, the proposal followed the ABA 2020s Commission's recommendation --

shortening the required practice in another state from five out of the seven preceding

years to three out of the five preceding years. Proposed Amendments to Virginia

Supreme Court Rules 1A:1 and 1A:3 (issued on April 15, 2013).

Second, the proposal did not limit such lawyers admitted by motion from

practicing like every other Virginia lawyer. This explicitly acknowledged that such

lawyers would become full Virginia lawyers in all respects.

This new proposed rule therefore seemed to satisfy those worried about some

later practice limitation on lawyers admitted by motion to the Virginia Bar.

A Virginia Lawyers Weekly article even predicted that the proposed new rule

would yield a "flood of new applications" to be admitted by motion to Virginia.

212

35951124_10 UPL, MDP and MJP (Defining What Lawyers Do and McGuireWoods LLP Where They Can Do It): Part II T. Spahn (1/1/19) Hypotheticals and Analyses

• Peter Vieth, New Rules Could Boost Lawyer Applications, Va. Law. Wkly., June 21, 2013 ("State officials are bracing for a flood of new applications if the Supreme Court of Virginia approves relaxed standards for allowing out- of-state lawyers to practice in Virginia."; "Northern Virginia leaders say the rule changes proposed by the court would throw out a welcome mat for large firms looking for a lower cost location close to downtown Washington, D.C."; "Law firms have balked at moving across the Potomac from downtown Washington because many of the lawyers would have to sit for the Virginia bar exam in order to practice in Virginia. The new rules - if approved by the court - would eliminate many of the barriers to waiving in to Virginia practice without taking the bar exam."; "The state Board of Bar Examiners said it likely would need an additional employee to process applications, mainly from Washington lawyers hoping to practice in Virginia."; "The Supreme Court received 20 comments supporting the relaxed rules, with only two voices of opposition. Other comments offered suggestions or questions about the proposed changes."; "The court's proposal for admitting 'foreign lawyers' to practice in Virginia would ditch restrictions - actual and proposed - that have sparked controversy over the past three years.'"; "The plan would eliminate the current full-time practice rule. Waived-in lawyers - about seven percent of active bar members - now risk losing their Virginia certification if they fail to practice at least 35 hours a week as a member of the Virginia State Bar."; "The regulations require practice from a Virginia office. Virtual offices, shared office space and dividing time with an out-of-state office are not permitted."; "Publicity about the existing full-time practice requirements led to an outcry in 2011. Many waived-in lawyers had not realized they could lose their right to practice in Virginia if they took time off, slowed down their practice, or spent time at an out-of-state office."; "In its first attempt to address the complaints, the Supreme Court proposed last October to replace the full-time practice rule with a minimum requirement of five years of practice with a "predominantly" in Virginia, with annual certification." "After a variety of comments, the court in April dropped the proposed requirement for practice predominantly in Virginia. The proposal now under consideration contains no restrictions on a lawyer's ongoing practice once the lawyer is certified to practice in Virginia."; "The latest proposal does preserve Virginia's reciprocity rule. Out-of-state lawyers can be admitted to Virginia practice without examination only if their home state would extend similar privileges to Virginia-licensed lawyers.").

The Virginia Supreme Court adopted its new rule on November 1, 2013. There was good news and surprising new bad news in the final rule -- which created another flurry of controversy.

213

35951124_10 UPL, MDP and MJP (Defining What Lawyers Do and McGuireWoods LLP Where They Can Do It): Part II T. Spahn (1/1/19) Hypotheticals and Analyses

The good news was the Supreme Court's explicit elimination of a "second class

citizen" status of any lawyer previously admitted or to be admitted by motion in Virginia.

An attorney admitted to practice law in this Commonwealth without examination under prior versions of this Rule is no longer subject to the requirement that he or she intends to practice law full time as a member of the Virginia State Bar.

Amended Rule 1A:1(f) (adopted by Order entered November 1, 2013, effective as of

February 1, 2014).

The bad news came from a two-word addition to the rule that the Virginia

Supreme Court had proposed on April 15, 2013.

Any person who has been admitted by examination to practice law before the court of last resort of any state or territory of the United States or of the District of Columbia may file an application to be admitted to practice law in this Commonwealth without examination, if counsel licensed to practice law in this Commonwealth may be admitted in that jurisdiction without examination.

Id. at (a) (emphasis added). Coupled with a continuing reciprocity requirement, this new

provision prevented lawyers from being admitted by motion into Virginia if they wished

to rely on their membership in the D.C. Bar (with which Virginia has reciprocity) -- which

in turn they had arranged by relying on admission by examination in a state with which

Virginia does not have reciprocity. In other words, lawyers could not use the easily

obtained D.C. Bar membership as a springboard into Virginia from a state from which

they could not be admitted by motion directly into Virginia because there was no

reciprocity.

A Virginia Lawyers Weekly article quoted this added restriction and its possible

effect on lawyers wishing to move into Virginia.

214

35951124_10 UPL, MDP and MJP (Defining What Lawyers Do and McGuireWoods LLP Where They Can Do It): Part II T. Spahn (1/1/19) Hypotheticals and Analyses

• Peter Vieth, Supreme Court of Virginia eases restrictions on waived-in lawyers, Va. Law. Wkly., Nov. 8, 2013 ("After a three-year process marked by protests from waived-in lawyers and several proposed rule rewrites, the Supreme Court of Virginia has dropped most special practice requirements for lawyers admitted in Virginia without examination."; "The rule for lawyers admitted 'on motion' was amended on Nov. 1, with the changes to take effect on Feb. 1."; "Street had written that the changes could bring a flood of applications, mainly from Washington lawyers hoping to practice in Virginia."; "The flood might not be as big an inundation as it could have been, one observer said."; "Developers hoping to lure Washington law firms to cheaper real estate on the other side of the Potomac did not get all the help they hoped for in the new rules."; "Arlington County interests sought a provision allowing young lawyers without experience to practice under supervision of Virginia lawyers while pursuing Virginia bar admission. Developers also hoped the Supreme Court would allow high level government service to count as part of the required three years of prior practice."; "Those changes would have made it feasible for large Washington firms to move wholesale to a Virginia office with few lawyers having to sit for the Virginia bar exam, boosters said."; "'They simply didn't choose to address head-on the issue we were raising,' said John G. Milliken, counsel for three developers hoping to attract Washington firms to Virginia properties."; "Others who had urged a more expansive admission rule included former Attorney General Jerry W. Kilgore, representing the Arlington County Department of Economic Development, and Senator Richard L. Saslaw, D-Springfield."; "Nonetheless, the rule changes remove restrictions that have chafed lawyers holding certificates to practice in Virginia by reciprocity. Those lawyers will no longer be subject to the requirement that they practice full time."; "Currently, waived- in lawyers risk the loss of certification if their Virginia practice - in a bricks- and-mortar Virginia office -- falls short of 35 hours a week."; "An earlier reform proposal, to replace the full time practice requirement with a probation period of practice 'predominantly' in Virginia, failed to win support."; "The new rule adds language to prevent so-called 'leapfrog' candidates who avoid the reciprocity requirement by first waiving in to the Washington bar and then jumping to Virginia."; "The reciprocity requirement normally bars certification without examination if an applicant's home state does not allow similar privileges for Virginia lawyers." (emphasis added)).

Apparently having second thoughts about this restriction which had been added to its April 2013 proposed rules change, the Supreme Court eventually dropped those two words.

215

35951124_10 UPL, MDP and MJP (Defining What Lawyers Do and McGuireWoods LLP Where They Can Do It): Part II T. Spahn (1/1/19) Hypotheticals and Analyses

On October 31, 2014, the Supreme Court adopted a new Rule 1A:1(a) rule

without the "by examination" limitation.

Any person who has been admitted to practice law before the court of last resort of any state or territory of the United States or of the District of Columbia may file an application to be admitted to practice law in this Commonwealth without examination, if counsel licensed to practice law in this Commonwealth may be admitted in that jurisdiction without examination.

Virginia Rule 1A:1(a) (adopted October 31, 2014).

A Virginia Lawyers Weekly article described the change.

• Peter Vieth, Virginia lowers a barrier for out-of-state lawyers, Va. Law. Wkly., Nov. 24, 2014 ("The Supreme Court of Virginia quietly has opened the door a bit wider for out-of-state attorneys seeking to practice in Virginia without taking the bar examination."; "In a rule change last month, the court now allows lawyers from states that do not permit admission-without-examination to practice in Virginia by first becoming admitted in a more permissive jurisdiction, such as Washington, DC."; "The move could be a boon to some Maryland lawyers -- now unable to practice in Virginia without taking the bar exam -- and possibly to some Washington law firms considering a presence in Virginia."; "The Oct. 31 rule change follows reforms last year to Virginia's admission-on-motion rules relaxing some requirements for outside lawyers seeking to waive the bar exam and practice here."; "The change also comes amid legal challenges around the country to restrictions on lawyers' ability to practice throughout the states and the call for liberalized cross-state opportunities championed by the American Bar Association and other groups."; "Leaders at the Virginia State Bar said they had no prior knowledge of the court's plan to liberalize the reciprocity rule."; "Both Chief Justice Cynthia D. Kinser and Chief Justice-elect Donald W. Lemons declined comment through a court spokesperson in response to a request for further information."; "The basic rule change was a simple one. The court removed just two words from the language on reciprocity in Rule 1A:1."; "Instead of opening admission on motion to "Any person who has been admitted by examination to practice law before the court of last resort of any state or territory. . . ,' the court removed the words 'by examination.'"; "The effect is to allow admission-on-motion for any lawyer licensed in a jurisdiction where Virginia lawyers could apply for admission-on-motion, regardless of how the lawyer became licensed in that foreign jurisdiction."; "The court made corresponding changes in the regulations for applying to Virginia law

216

35951124_10 UPL, MDP and MJP (Defining What Lawyers Do and McGuireWoods LLP Where They Can Do It): Part II T. Spahn (1/1/19) Hypotheticals and Analyses

practice, but retained a policy statement regarding reciprocity." (emphases added)).

The Virginia Supreme Court has not tweaked the Rule since then, so it looks as if

Virginia has settled into a very welcoming approach to lawyers being admitted by

motion to the Virginia Bar.

And Virginia continues to liberalize its reciprocity provisions. North Carolina and

Virginia established reciprocity in August 2015. Maryland and Virginia still have not

done so. But under the new Virginia approach, a Maryland lawyer can first join the D.C.

Bar, and then use that membership to be admitted by motion to the Virginia Bar.

The Virginia experience highlights both the push for, and the counter-pressure against, allowing lawyers to move freely from state to state without taking their new state's bar examination -- which is an understandably frightening impediment for many lawyers.

The ABA keeps current publicly available charts of states' requirements and processes for admission by motion. The National Conference of Bar Examiners also has useful current information about states' differing requirements.

Best Answer

The best answer to (a) is YES (IF IT ACCURATELY DESCRIBES YOUR BAR

MEMBERSHIP); the best answer to (b) is YES (IF YOU WORK WITH A LICENSED

LAWYER); the best answer to (c) is YES (IF YOU ARE ADMITTED PRO HAC VICE); the best answer to (d) is YES; the best answer to (e) is YES (PROBABLY).

B 2/13, 9/16

217

35951124_10 UPL, MDP and MJP (Defining What Lawyers Do and McGuireWoods LLP Where They Can Do It): Part II T. Spahn (1/1/19) Hypotheticals and Analyses

Practicing Before Federal Agencies and Specialized Tribunals

Hypothetical 20

You started practicing in Minnesota immediately after graduating from law school two years ago, but now have the opportunity to join a firm in another state. You are not able to waive into that state's bar, and for obvious reasons would like to avoid taking another bar exam. You decided from the beginning to limit your practice to trust and estate matters, usually involving federal tax issues. You wonder whether you can avoid taking the bar exam in your new state if you limit your practice even further -- working only on matters before the U.S. Tax Court in Washington, D.C.

May you continuously practice in a state if you limit your work to matters before a federal agency or specialized tribunal?

YES (PROBABLY)

Analysis

The Supremacy Clause,1 prevents states from prohibiting or restricting a lawyer's practice of law before a federal agency or specialized tribunal.

The ABA Model Rules indicate that a lawyer admitted to some United States jurisdictions (and not "disbarred or suspended from practice in any [other] jurisdiction") may provide legal services that

are services that the lawyer is authorized to provide by federal law or other law of this jurisdiction.

ABA Model Rule 5.5(d)(2) (emphasis added). A comment provides additional explanation.

Paragraph (d)(2) recognizes that a lawyer may provide legal services in a jurisdiction in which the lawyer is not licensed when authorized to do so by federal or other law, which

1 U.S. Const. art. VI, § 2. 218

35951124_10 UPL, MDP and MJP (Defining What Lawyers Do and McGuireWoods LLP Where They Can Do It): Part II T. Spahn (1/1/19) Hypotheticals and Analyses

includes statute, court rule, executive regulation or judicial precedent.

ABA Model Rule 5.5 cmt. [18] (emphasis added).

The ABA Model Rules explain that lawyers engaging in such practice must subject themselves to the disciplinary authority of the state where they are located,2 and must accurately describe the limits of their license in all marketing material and in their disclosures to clients.3

The Restatement provides a more detailed explanation of what lawyers can and cannot do.

A lawyer properly admitted to practice before a federal agency or in a federal court . . . may practice federal law for a client either at the physical location of the agency or court or in an office in any state, so long as the lawyer's practice arises out of or is reasonably related to the agency's or court's business. Such a basis for authorized practice is recognized in Subsection (2). Thus, a lawyer registered with the United States Patent and Trademark Office could counsel a client from an office anywhere about filing a patent or about assigning the ensuing patent right, matters reasonably related to the lawyer's admission to the agency. (The permissible scope of practice of a nonlawyer patent agent may be less, since admission to the agency does not suggest competence to deal with matters, such as the assignment of patents, beyond the jurisdiction of the agency).

Restatement (Third) of Law Governing Lawyers § 3 cmt. g (2000) (emphasis added).

A Restatement illustration explains how these principles work in allowing a lawyer to practice before a federal agency (rather than a federal court).

Lawyer, who practices with a law firm in California, is a nationally known expert in corporate mergers and

2 ABA Model Rule 5.5 cmt. [19]. 3 ABA Model Rule 5.5 cmt. [20]. 219

35951124_10 UPL, MDP and MJP (Defining What Lawyers Do and McGuireWoods LLP Where They Can Do It): Part II T. Spahn (1/1/19) Hypotheticals and Analyses

acquisitions. Utility is a major electricity generator and distributor in the southeastern United States. Under the new legislation referred to in Illustration 3, Utility is considering a hostile takeover of Old Company, an established regional electricity generator and distributor in the northeastern United States. Legal work on the acquisition would require the physical presence of Utility's mergers-and-acquisitions counsel in a number of states in addition to the West Coast state in which Lawyer is admitted, in addition to representation before at least one federal agency in Washington, D.C. Given the multistate and federal nature of the legal work, Lawyer and other members of Lawyer's firm may represent Utility as requested.

Restatement (Third) of Law Governing Lawyers § 3 cmt. e, illus. 4 (2000) (emphases added).

The ACTEC Commentaries explicitly take the position that lawyers licensed in any state should be able to practice before the Tax Court,4 the IRS,5 and other similar administrative tribunals6 -- even if they are doing so while physically located in a state where they are not licensed.

4 American College of Trust & Estate Counsel, Commentaries on the Model Rules of Professional Conduct, Commentary on MRPC 5.5, at 205 (5th ed. 2016) ("While the text of MRPC 5.5(d)(2) appears expressly to permit multijurisdictional practice in these circumstances, given the ease with which a lawyer can qualify to practice before the Tax Court or the IRS, the lawyer should consider seeking an opinion of the non-admitted jurisdiction's bar counsel."). 5 American College of Trust & Estate Counsel, Commentaries on the Model Rules of Professional Conduct, Commentary on MRPC 5.5, at 205 (5th ed. 2016) ("A lawyer providing legal services regarding estate planning and administration often represents clients in disputes with the IRS. A lawyer 'may practice before the Internal Revenue Service by filing with the Internal Revenue Service a written declaration that he or she is currently qualified as an attorney and is authorized to represent the party or parties on whose behalf he or she acts.' [31 CFR §10.3; see generally 31 CFR Part 10, §10.0 et seq. (published as a pamphlet as Treasury Department Circular No. 230)]. In addition, a lawyer may practice before the United States Tax Court by complying with its requirements for admission (Tax Court Rule 24). Pursuant to paragraph (d)(2) of MRPC 5.5, a lawyer who is authorized to practice before the IRS or the Tax Court would be able to practice in any non-admitted jurisdiction adopting MRPC 5.5(d)(2). Moreover, unlike MRPC 5.5(c), there is no requirement that the practice in the non-admitted jurisdiction be on a 'temporary basis.'"). 6 See, e.g., American College of Trust & Estate Counsel, Commentaries on the Model Rules of Professional Conduct, Commentary on MRPC 5.5, at 205-06 (5th ed. 2016) ("In addition, states adopting MRPC 5.5(d)(2) may have state rules regulating practice before a state administrative tribunal, such as a tax commission, or an administrative law judge, that would authorize a lawyer admitted in another 220

35951124_10 UPL, MDP and MJP (Defining What Lawyers Do and McGuireWoods LLP Where They Can Do It): Part II T. Spahn (1/1/19) Hypotheticals and Analyses

This general principle allows lawyers to practice before the United States Patent

and Trademark Office, military courts, etc. -- even if the lawyers establish a "continuous

and systematic" presence in a state where they are not licensed.

Not surprisingly, the Virginia Bar has dealt with this issue -- because several federal agencies are located in Northern Virginia. For instance, the Virginia Bar has explicitly explained that out-of-state lawyers can practice in Virginia without being licensed in Virginia -- as long as they limit the practice to matters before the U.S. Patent

Office.

Based on this authority, an attorney who is licensed other than in Virginia, who is registered and authorized to practice before the U.S. Patent Office and who is a member of a Virginia law firm can provide all legal services and representation related to a patent law practice to all clients needing such services and representation regardless of where the clients are located. These services and representation may include rendering legal advice and/or written opinions for clients on issues such a patent infringement, patent claim construction, patent validity, or enforceability of a patent. The patent attorney may provide such advice and opinions to a client whether related to a matter the patent attorney is actually handling for the client before the USPTO or not. The patent attorney can conduct this practice and provide these services while physically in Virginia and without the supervision or association of a Virginia licensed attorney, so long as the patent attorney limits his/her activity to the practice of patent law and is not

jurisdiction to practice before the commission or administrative law tribunal in the non-admitted state."; "While the text of MRPC 5.5(d)(2) appears expressly to permit multijurisdictional practice in these circumstances, given the ease with which a lawyer can qualify to practice before the Tax Court or the IRS, the lawyer should consider seeking an opinion of the non-admitted jurisdiction's bar counsel."; "When authorized by federal or state law, including authorization by 'statute, court rule, executive regulation or judicial precedent,' the lawyer 'may establish an office or other systematic and continuous presence in [the non-admitted] jurisdiction for the practice of law . . .' (MRPC 5.5, Comments 18 and 15). For example, a lawyer in South Carolina might be able to practice full-time in Georgia (Georgia having adopted MRPC 5.5(d)(2) if the practice were limited to handling tax appeals with the IRS and tax court litigation. However, the lawyer must take steps not to mislead potential clients about the lawyer's right to practice generally in Georgia [MRPC 5.5(b)(2) . . .]." (alterations in original)). 221

35951124_10 UPL, MDP and MJP (Defining What Lawyers Do and McGuireWoods LLP Where They Can Do It): Part II T. Spahn (1/1/19) Hypotheticals and Analyses

in any manner attempting to practice Virginia law. Provided the patent attorney's practice is limited as described herein, he or she may also maintain an office in Virginia to conduct that limited practice. If the patent attorney is a member of a law firm with offices in Virginia and elsewhere, the extent to which the patent attorney can conduct his/her practice outside of Virginia will depend on the unauthorized practice rules and/or rules of professional conduct in those other jurisdictions. If the patent attorney provides advice and counsel regarding patent law to a Virginia client from a location outside of Virginia, this would not be the unauthorized practice of law in Virginia because the attorney is not physically in Virginia and because he/she is otherwise authorized to practice patent law.

Virginia UPL Op. 210 (8/8/06) (footnote omitted; emphases added). The Virginia Bar

warned the lawyer to explain in any marketing material the limits of the lawyer's license.

In an earlier legal ethics opinion, the Virginia Bar took the same approach to out-

of-state lawyers practicing before military courts and boards.

• Virginia UPL Op. 89 (2/19/86) ("It is not the unauthorized practice of law for an attorney not licensed, but living, in Virginia to represent individuals before military courts and boards on military reservations.").

Courts also take this position.

• Augustine v. Dep't of Veterans Affairs, 429 F.3d 1334 (Fed. Cir. 2005) (holding that federal rather than state law governed a lawyer's ability to practice before the Merit Systems Protection Board and other federal agencies).

Lawyers relying on this principle must be very careful to limit their practice so

they fit within this narrow exception. For instance, a client with a case before the Tax

Court might want advice about such peripheral matters as state creditors' rights laws

and regulations. A lawyer limiting his or her practice to Tax Court cases could not provide such advice.

222

35951124_10 UPL, MDP and MJP (Defining What Lawyers Do and McGuireWoods LLP Where They Can Do It): Part II T. Spahn (1/1/19) Hypotheticals and Analyses

Lawyers not carefully complying with these limitations can face serious

consequences. For example, in 2010 a Virginia grand jury indicted a lawyer who was

eligible to practice before the U.S. Patent and Trademark Office -- but who held himself out as authorized to practice generally in Virginia, and actually did so.

• Peter Vieth, Williamsburg Patent Lawyer Charged With UPL, VLW Blog (Nov. 22, 2010), http://valawyersweekly.com/vlwblog/2010/11/22/ williamsburg-patent-lawyer-charged-with-upl/ ("A Williamsburg grand jury has returned an indictment charging patent attorney Bambi F. Walters with practicing law without a license, according to the local prosecutor."; "Williamsburg/James City County Commonwealth's Attorney Nate Green said the charge resulted from a Virginia State Bar [VSB] investigation. VSB Counsel Ned Davis confirmed his office had turned over results of its investigation to the local authorities."; "Walters is an attorney registered with the U.S. Patent and Trademark Office, but she does not hold a Virginia law license, according to officials at the VSB. Her website indicates she practices in patent, trademark and related areas. 'We have worked with many nonprofit organizations and associations,' her website also reads."; "Green declined to provide details of the UPL charge, but he said the indictment covers an entire year, from Nov. 17, 2009, to Nov. 16, 2010. 'We believe she was practicing law during that time,' he said."; "Virginia law bars federally licensed patent attorneys without Virginia licenses from appearing 'in any court or tribunal' other than those of the United States Patent Office. A Virginia Supreme Court rule bars patent lawyers from holding themselves out as authorized to practice law generally." (emphasis added); "The charge of unauthorized practice of law is a class 1 misdemeanor. 'Once she is served, a trial date will be set,' Green said." (emphasis added); [Editor's note: Other reports indicated that Ms. Walters was licensed in North Carolina during some of the pertinent time period, but was eventually disbarred there.]).

As a practical matter, lawyers find it very difficult to rely on this exception -- and

most ultimately join the bar of the state where they are continuously practicing. Most

wise lawyers would not dare risk an unauthorized practice of law charge by attempting

to "thread the needle" in a state where they are not licensed. In addition to the

223

35951124_10 UPL, MDP and MJP (Defining What Lawyers Do and McGuireWoods LLP Where They Can Do It): Part II T. Spahn (1/1/19) Hypotheticals and Analyses

possibility of criminal prosecution or bar discipline,7 such lawyers would have to worry

about disgruntled clients or former clients using the explicit or implicit threat of charging

a UPL violation as leverage in a dispute with the lawyer.

Significantly, lawyers who do not move quickly in joining in their state's bar in

these circumstances might forfeit the chance to join the bar without taking the bar exam.

This is because some states' rules allowing out-of-state lawyers to be admitted by motion to their bar usually require that such applicants have practiced law for a certain number of years in a state where they are licensed. This often excludes from the calculation any time the lawyer spends in the state where he or she is not licensed -- even though the lawyer may permissibly engage in limited activities in that state.

Best Answer

The best answer to this hypothetical is YES (PROBABLY).

B 2/13

7 Although it seems counter-intuitive, lawyers practicing in a state without being licensed there generally can be punished by the bar of that state -- even though they are not members of that bar. Most bars have arranged for such power (by statute, court rule or other regulation) so that they can enforce the unauthorized practice of law rules rather than rely on overworked law enforcement officials. 224

35951124_10 UPL, MDP and MJP (Defining What Lawyers Do and McGuireWoods LLP Where They Can Do It): Part II T. Spahn (1/1/19) Hypotheticals and Analyses

Practicing in Federal Courts

Hypothetical 21

You have only been practicing for a few years, but you want to move to another state (where you are not licensed) to be near the school where your young child will be enrolled. You hope to avoid taking the bar exam in that other state, and you are not yet eligible to be admitted by motion to the other state's bar. So far you have been primarily a federal court litigator, and you wonder whether you can move to the other state and practice only in federal court.

May you move to another state and continuously practice only in the federal court located there, without being licensed in that other state?

NO (PROBABLY)

Analysis

This hypothetical deals with the impact of the Supremacy Clause1 on lawyers'

ability to practice in federal court without going through the pro hac vice process -- and without being licensed in the federal court's host state.

Of course, even lawyers licensed in a state's supreme court must comply with the admission process promulgated by federal courts in that state.

• In re Brandt, Case No. 09-J-16022, slip op. at 1 (Cal. Bar Ct. June 27, 2017) (not for publication) (suspending for one year a lawyer who filed approximately one hundred cases in bankruptcy court despite not being admitted there; "This proceeding is based on the final determination of a three-judge panel of the United States Bankruptcy Court for the Central District of California that Benjamin Tyler Brandt committed misconduct in the bankruptcy court. The panel found that he filed bankruptcy petitions in 'about 100' cases while not admitted to practice before the United States District Court for the Central District of California, in violation of a local bankruptcy rule. These actions constituted the unauthorized practice of law (UPL). The panel suspended Brandt from practicing before the bankruptcy court for one year and ordered him to complete 10 hours of continuing legal education before he could be reinstated to practice.").

1 U.S. Const. art. VI, § 2. 225

35951124_10 UPL, MDP and MJP (Defining What Lawyers Do and McGuireWoods LLP Where They Can Do It): Part II T. Spahn (1/1/19) Hypotheticals and Analyses

• Django Gold, Samsung Avoid Sanctions After Quinn Emanuel Attorney's Slipup, Law360, Aug. 17, 2012 ("Samsung Electronics Company Limited will not be penalized for a Quinn Emanuel Urquhart & Sullivan LLP attorney's failure to secure admission to the Northern District of California when she appeared in court last week, a judge presiding over Samsung's patent dispute with Apple Inc. ruled Thursday."; "United States Magistrate Judge Paul S. Grewal said that though Samsung counsel Susan R. Estrich was in fact not admitted to the court when she presented arguments for Samsung on August 7, levying penalties for the oversight 'would serve no meaningful purpose.'"; "'The court has previously alluded to the importance of bar admission in this district,' Judge Grewal said. 'And yet it is not lost on the court that sometimes even highly skilled advocates simply make mistakes without any nefarious intent. Counsel’s declaration, with its explanation and apology, persuades the court that this is one of those times.'"; "Estrich -- a former Harvard Law School professor, current Fox News legal and political analyst, and partner in Quinn Emanuel's Los Angeles office -- last week took heat from the California court after it came to light that, prior to presenting arguments for a Samsung jury instructions motion, she had neither entered an appearance nor even been admitted to practice before the court in which Samsung and Apple are currently waging battle over patents related to Apple's smartphones and tablets."; "'Not only did counsel present arguments without first entering an appearance, there is no record in the court’s database that she is admitted to practice in this court,' Judge Grewal said the day after Estrich appeared before his court. 'This is potentially a more serious breach. While the requirements for admission to this district's bar may not be particularly onerous for one licensed to practice law in the state of California, they are no mere formality.'"; "In response to the judge's request for a declaration explaining her conduct, Estrich on August 9 professed that she believed she had been admitted to the court in 1986, when she moved to California and her previous firm helped her comply with the administrative requirements for appearing before the state's federal courts."; "As soon as she learned of a problem in her admission status, she applied for and has since been admitted to practice before the district and filed an appearance in the case, she said."; "'I respectfully request that the court not hold these inadvertent omissions on my part against the merits of my client's case, and further respectfully ask that the court determine this matter as concluded now that I have obtained admission to the bar of this court and have filed a notice of appearance,' Estrich said in her declaration.").

226

35951124_10 UPL, MDP and MJP (Defining What Lawyers Do and McGuireWoods LLP Where They Can Do It): Part II T. Spahn (1/1/19) Hypotheticals and Analyses

The ABA Model Rules indicate that a lawyer admitted in some United States jurisdiction (and not "disbarred or suspended from practice in any [other] jurisdiction") may provide legal services that

the lawyer is authorized to provide by federal or other law or rule to provide in this jurisdiction.

ABA Model Rule 5.5(d)(2). A comment provides additional explanation.

Paragraph (d)(2) recognizes that a lawyer may provide legal services in a jurisdiction in which the lawyer is not licensed when authorized to do so by federal or other law, which includes statute, court rule, executive regulation or judicial precedent. See, e.g., The ABA Model Rule on Practice Pending Admission.

ABA Model Rule 5.5 cmt. [18]. This provision acknowledges the supremacy of federal law, although it does not specifically explain that lawyers may practice before a federal court in states where they are not licensed.

The Restatement provides a more explicit explanation.

A lawyer admitted in one state who is admitted to practice in a United States district court located in another state, but who is not otherwise admitted in the second state, can practice law in the state so long as the practice is limited to cases filed in that federal court.

Restatement (Third) of Law Governing Lawyers § 3 cmt. g (2000) (emphasis added).

Interestingly, the Restatement criticizes federal court rules requiring lawyers practicing before the court to be licensed by the state in which the federal court sits.

Local rules in some few federal district courts additionally require admission to the bar of the sitting state as a condition of admission to the federal court. The requirement is inconsistent with the federal nature of the court's business.

Restatement (Third) of Law Governing Lawyers § 3 cmt. g (2000) (emphasis added).

227

35951124_10 UPL, MDP and MJP (Defining What Lawyers Do and McGuireWoods LLP Where They Can Do It): Part II T. Spahn (1/1/19) Hypotheticals and Analyses

Practicing law in a federal court located in a jurisdiction where a lawyer is not licensed (and without being admitted pro hac vice) carries a number of implications.

First, ABA Model Rule 5.5 cmt. [19] indicates that such a lawyer "is subject to the disciplinary authority of this jurisdiction." Thus, even a lawyer not licensed in the jurisdiction where a federal court or agency sits normally may be punished by that jurisdiction for any ethical misconduct in which the lawyer engages while practicing before the federal court located in that state. Some courts take the opposite approach.

• Disciplinary Counsel v. Harris, 996 N.E.2d 921, 923, 924, 925 (Ohio 2013) (finding that the Ohio Bar Disciplinary Authority had no power over an out-of- state lawyer practicing before a bankruptcy court in Ohio; explaining that the unauthorized practice of law authority could pursue punishment for improper conduct not described in the opinion; "This issue in this case is whether Donald Harris, an attorney who is admitted to the pratice of law in the District of Columbia and the Northern and Southern Districts of Ohio, but who is not admitted to the practice of law in the state of Ohio, is subject to the disciplinary authority of this court. Because Harris is not a member of the Ohio bar and has not taken an oath to be bound by the Ohio Rules of Professional Conduct, these rules do not apply to him; rather, his conduct is subject to review by the Board on the Unauthorized Practice of Law ('UPL Board')."; "Disciplinary counsel maintains that since Harris is an out-of-state attorney practicing federal law within Ohio's boundaries, he is subject to the disciplinary authority of this state pursuant to Prof. Cond. R. 8.5."; "Although Harris is licensed to practice law in another jurisdiction, because he is not admitted to the Ohio bar, our Rules of Professional Conduct, designed to regulate conduct of attorneys admitted to practice law in Ohio, do not apply to him. He never subjected himself to them because he has never been admitted to practice law in this state."; "In this regard, Harris is no different from an accountant, a real estate agent, or a financial planner who undertakes activity that constitutes the practice of law and who becomes subject to discipline pursuant to the unauthorized practice of law framework. It is inconsistent to conclude that an attorney admitted in another jurisdiction who engages in the unauthorized practice of law in Ohio becomes subject to the Board of Commissioners on Grievances and Discipline when another professional, such as a real estate agent, who engages in the unauthorized practice of law becomes subject to the UPL Board."; "Additionally, our sanctions for serious violations of the Rules of Professional conduct, suspension and disbarment, are ineffective and meaningless to Harris because he is not a member of the Ohio bar. We cannot suspend or disbar 228

35951124_10 UPL, MDP and MJP (Defining What Lawyers Do and McGuireWoods LLP Where They Can Do It): Part II T. Spahn (1/1/19) Hypotheticals and Analyses

an attorney who is not a member of the Ohio bar. Thus, we consider these matters as alleged unauthorized practice of law violations."; "Harris represented Aimee Skeel in two bankruptcy petitions filed in the United States Bankruptcy Court for the Northern District of Ohio. We determine that Harris did not engage in the unauthorized practice of law when he represented Skeel because, as a member of the District of Columbia bar, and having been admitted to practice in the Northern District of Ohio, he was authorized to practice before the United States Bankruptcy Court for the Northern District of Ohio. As such, he becomes subject to the disciplinary authority of those federal courts.").

Second, lawyers cannot market themselves to others as licensed in a jurisdiction

where they are not actually licensed. ABA Model Rule 5.5(b)(2). Lawyer practicing only

before federal courts can avoid this violation by correctly describing in any public

communication the limit of what they can do and where they can do it.

Third, lawyers normally must affirmatively tell their clients about the scope of their

ability to practice law.

In some circumstances, a lawyer who practices law in this jurisdiction pursuant to paragraphs (c) or (d) may have to inform the client that the lawyer is not licensed to practice law in this jurisdiction. For example, that may be required when the representation occurs primarily in this jurisdiction and requires knowledge of the law of this jurisdiction. See Rule 1.4(b).

ABA Model Rule 5.5 cmt. [20] (emphasis added). At least one state explicitly requires

such disclosure to clients and others.2

Fourth, lawyers obviously must satisfy the federal courts' requirements. In many

federal courts, this obligation brings lawyers full circle back to the host state's MJP

rules.

2 Virginia Rule 5.5(a)(3) ("A Foreign Lawyer [non-Virginia or non-US lawyer] shall inform the client and interest third parties in writing: (i) that the lawyer is not admitted to practice in Virginia; (ii) the jurisdiction(s) in which the lawyer is licensed to practice; and (3) the lawyer's office address in the foreign jurisdiction."). 229

35951124_10 UPL, MDP and MJP (Defining What Lawyers Do and McGuireWoods LLP Where They Can Do It): Part II T. Spahn (1/1/19) Hypotheticals and Analyses

The analysis starts with a basic principle -- federal courts have the power to decide who can practice before them, regardless of their host state's bar requirements.

In Surrick v. Killion, 449 F.3d 520 (3d Cir. 2006), for instance, the Third Circuit held that the Eastern District of Pennsylvania federal court could decide the ability of lawyers to practice before it -- and that its power preempted a Pennsylvania rule prohibiting such action by a lawyer not licensed in Pennsylvania. The Third Circuit also explained that this federal court power also covered lawyers participating in diversity

(rather than federal question) cases -- noting that there is no authority "that a federal court's power to determine who may practice law before it depends on the type of cases a lawyer intends to practice." Id. at 534. Theoretically, this presumably would permit a federal court to allow lawyers to practice state law (being litigated in diversity cases) without being admitted to the state where the federal court sits.

Other courts have taken the same basic approach:

• Spanos v. Skouras Theatres Corp., 364 F.2d 161 (2d Cir.) (holding that a California lawyer not admitted to practice in New York could recover fees for work on a New York federal court case although he was not admitted pro hac vice, noting that New York could not interfere with lawyers' right to practice before a federal court), cert. denied, 385 U.S. 987 (1966).

• Cowen v. Calabrese, 41 Cal. Rptr. 441, 443 (Dist. Ct. App. 1964) (holding that a lawyer not admitted to the California bar could recover fees for work in the federal bankruptcy court located in California; "The State Bar Act of California does not purport to regulate the practice of lawyers before the United States Courts, or their referees.").

Thus, federal courts' ability to determine who can practice before them does not depend on the type of law, but only on the forum. In 2010, the Oklahoma Supreme

Court recognized that

230

35951124_10 UPL, MDP and MJP (Defining What Lawyers Do and McGuireWoods LLP Where They Can Do It): Part II T. Spahn (1/1/19) Hypotheticals and Analyses

[t]he federal government is the conferring authority on the right to practice in its courts . . . which of course, is beyond the reach of any state. Clearly, that court controls whom it will authorize to practice before it and limits a practitioner's legal services to those incidental to a specific or pending case in that court.

In re Mooreland-Rucker, 237 P.3d 784, 788 (Okla. 2010) (footnote omitted) (granting a petition for reinstatement to an Oklahoma lawyer, but warning her that her actions in handling bankruptcy matters in Texas (without being licensed there) could violate the

UPL rules).

Although federal courts understandably possess the power to decide what license lawyers must have to practice before the court, most federal courts implicitly defer to their host states -- requiring lawyers to be admitted in the host state before practicing before federal courts located in the host state.

For instance, the Ninth Circuit rejected a lawyer's argument that a federal district court could not insist that he be licensed in Arizona before practicing before the Arizona district court.

The District of Arizona's requirement that members of its bar be active members of the State Bar of Arizona ensures that practitioners before the district court have received the ongoing training mandated in the continuing legal education requirement.

In re North, 383 F.3d 871, 878 (9th Cir. 2004). Although this approach sounds deferential, it still represents federal courts' exercise of the power to decide who can practice before them.

Thus, many if not most federal courts decide for themselves that they will require lawyers appearing before them to be licensed in the court's host state.

231

35951124_10 UPL, MDP and MJP (Defining What Lawyers Do and McGuireWoods LLP Where They Can Do It): Part II T. Spahn (1/1/19) Hypotheticals and Analyses

• Maryland LEO 2007-06 (10/10/06) ("You state that you will practice from a home office and the practice will be strictly limited to the area of employment- and family-based immigration petitions. Moreover, you state that those areas do not involve state law."; explaining that "The Ethics Committee may only provide opinions regarding whether a practice is ethical, and may not provide legal opinions. Whether a certain practice, such as your limited immigration practice, constitutes the unauthorized practice of law is a legal issue that can only be determined by the Court of Appeals."; "In addition, you are referred to Rule 701 of the Rules of the U.S. District Court of the District of Maryland, should your limited immigration practice include activity in that court. Rule 701 provides that: 'Except as provided in subsection c of this rule, an attorney is qualified for admission to the bar of this District if the attorney is and continuously remains a member in good standing of the highest court of any State in which the attorney maintains his or her principal law office.' Subsection c is not applicable because your principal law office will be in your home office in Maryland. Subsection d, however, is applicable and states that 'An attorney who is not a member of the Maryland Bar is not qualified for admission to the bar of this District if the attorney maintains any law office in Maryland.'" (emphases added)).

• Ramirez v. England, 320 F. Supp. 2d 368, 371 n.5 (D. Md. 2004) (explaining that a lawyer who is a member of the Mississippi Bar but not the Maryland Bar could not practice out of a home office in Maryland; noting that the lawyer included a Maryland address on her letterhead, without an indication that she was not a member of the Maryland Bar (although the letterhead contained the phrase "Admitted, U.S. District Court, District of Columbia"); holding that the lawyer could not practice before the D.C. federal court, because its rules require that the lawyer appearing before the federal court have her principal office in a state in which she is licensed; rejecting the lawyer's motion to appear pro hac vice in the Maryland federal court).

• Maryland LEO 2002-09 (2002) ("the Committee refers you to Rule 701 of the Rules of the United States District Court for the District of Maryland which states that: 'an attorney is qualified for admission to the bar of this district if the attorney is a member in good standing of the highest court of any State (or the District of Columbia) in which the attorney maintains his or her principal law office, or of the Court of Appeals of Maryland. . . .' In order to practice in Federal District Court in Maryland, you would have to either (a) be a member in good standing of the Maryland Court of Appeals or (b) be a member of the highest court of the District of Columbia (or any other State) AND maintain your principal law office in D.C. (or in the jurisdiction in which you are licensed). If you did not maintain your principal office in D.C., the only way you could be admitted to practice in Federal District Court in Maryland would be to be admitted to the Bar in Maryland.").

232

35951124_10 UPL, MDP and MJP (Defining What Lawyers Do and McGuireWoods LLP Where They Can Do It): Part II T. Spahn (1/1/19) Hypotheticals and Analyses

• Virginia UPL Opinion 201 (1/22/01) (warning that "[a] foreign attorney must be careful, however, to observe the local rules of the federal courts sitting in Virginia, which require that an attorney either be a member of the Virginia State Bar, or be admitted on motion, pro hac vice, in association with an attorney admitted to practice in the Virginia federal courts."; explaining that "[n]ot all federal courts require attorneys to be admitted to practice in the state in which they sit. Therefore, it is possible for a foreign attorney in Virginia to advise a Virginia client on matters pending in a federal court outside of Virginia, if the foreign attorney is admitted to practice in that federal court." (emphasis added)).

The Washington, D.C., federal courts have struggled with such membership issues for years.

• Nat'l Ass'n for Advancement of Multijurisdiction Practice v. Howell, 851 F.3d 12, 16, 20 (D.C. Cir.) (rejecting plaintiff's challenge to the District of Columbia District Court admission rule; "In the present case, NAAMJP and two of its members allege bar admission conditions for the United States District Court for the District of Columbia, established in the identical text of Local Civil Rule 83.8 and Local Criminal Rule 57.21 (collectively, the 'Local Rule'), violate statutory and constitutional legal standards. Specifically, the Local Rule provides: 'Admission to and continuing membership in the Bar of this Court are limited to: (1) attorneys who are active members in good standing in the District of Columbia Bar; or (2) attorneys who are active members in good standing of the Bar of any state in which they maintain their principal law office; or (3) in-house attorneys who are active members in good standing of the Bar of any state and who are authorized to provide legal advice in the state in which they are employed by their organization client.' D.D.C. Local Civ. R. 83.8(a); D.D.C. Local Crim. R. 57.21(a). NAAMJP focuses its challenge on the second option, the Primary Office Provision."; "The Court does not doubt the sincerity of NAAMJP's convictions or its eagerness to reduce barriers to legal practice in the various state and federal courts across the country. Indeed, there may be good policy reasons for the outcomes NAAMJP urges. But, as has been amply demonstrated in dozens of legal opinions penned by judges across the country, NAAMJP has identified no legal basis upon which to compel federal or state courts to adopt the rules it desires. Accordingly, the judgment of the district court is Affirmed."), cert. denied, 138 S. Ct. 420 (2017).).

• Sue Reisinger, ACC Fighting In-House Rules in Pa., DC, Corporate Counsel, May 5, 2014 ("The Association of Corporate Counsel [ACC] has taken two recent legal stands to promote in-house practice: one opposes a Pennsylvania trial court's ruling that attorney-client privilege ends when a corporation is dissolved, and the other urges the United States District Court

233

35951124_10 UPL, MDP and MJP (Defining What Lawyers Do and McGuireWoods LLP Where They Can Do It): Part II T. Spahn (1/1/19) Hypotheticals and Analyses

in Washington, D.C., to expand its bar admission rules."; "In its April 28 amicus brief in the Superior Court of Pennsylvania, ACC argued against chilling corporate communication and cited the importance of attorney-client privilege in today's complex regulatory world. 'Contrary to the trial court's holding, that undeniable need does not diminish when a business faces dissolution,' said ACC chief legal strategist Amar Sarwal in a statement."; "The brief states, 'ACC is deeply concerned about the precedent in this case for both ACC's members and their clients in Pennsylvania, as well as its national and international membership.' If the lower court's ruling is allowed to stand, it could harm company stakeholders and 'will significantly impair the ability of [other] in-house counsel to provide the legal advice necessary to guide their clients' behavior and promote corporate compliance with the law,' the brief adds."; "Burt Rublin and Andrew Kampf of Ballard Spahr in Philadelphia are representing ACC in the case."; "In its other legal stand, ACC sent a letter last week to the advisory committee on local rules for the federal district court in Washington, D.C., praising a proposal to eliminate reciprocity requirements for attorneys in cross-border practices. The letter said dropping the requirement aids in-house and corporate lawyers 'in their efforts to more effectively represent their organization-clients.'"; "But the letter also suggests modifying the proposed rules to aid cross-border activities of in-house counsel. It was signed by Sarwal and David Kessler, who is senior director of legal operations at McAfee Inc. in Washington, as well as vice president of public policy for ACC's D.C. metro area chapter."; "The current rule limits admission to the Washington, D.C., bar to an active member in good standing of the highest court in the state where 'the attorney maintains his/her principal law office.'").

• Zoe Tillman, D.C. Federal Judges Vote to Change Admission Rules, Legal Times, Mar. 20, 2014 ("The judges of the United States District Court for the District of Columbia recently voted to end rules that limit certain out-of-state lawyers from waiving in to the court's bar."; "Under the court's original admission rules, lawyers from other states could only waive in if they belonged to another federal court bar that allowed members of the Washington federal district court to waive in. The D.C. court was sued over the reciprocity rules last year by the National Association for the Advancement of Multijurisdiction Practice, which claimed it was discriminatory."; "The judges voted March 4 to eliminate the reciprocity provisions. The changes will go into effect following a 45-day public comment period, assuming the court doesn't decide to modify or withdraw them."; "According to the court's website, 25 of the 94 United States district courts nationwide offer reciprocal admission to members of the District of Columbia court's bar. The 25 courts are located in 15 states."; "The association, a public benefit corporation based in Los Angeles, claimed the reciprocity rules violated the First Amendment by restricting the speech and petition rights of lawyers admitted to other federal courts. The group also

234

35951124_10 UPL, MDP and MJP (Defining What Lawyers Do and McGuireWoods LLP Where They Can Do It): Part II T. Spahn (1/1/19) Hypotheticals and Analyses

said the rules violated the Fifth Amendment by creating different admission standards for equally qualified attorneys.").

• District of Columbia UPL Op. 17-06 (7/21/06) (assessing the rules governing lawyers working in Washington, D.C., who practice "only before federal courts to which they are admitted," although they are not active members of the D.C. Bar; "Such lawyers qualify for the exception in Rule 49(c)(3) [the Washington, D.C. UPL Rules], but only if they make clear in business documents both that (1) they are not admitted in the District of Columbia and (2) their practice is limited to practice before federal courts (and to other matters within the scope of other exceptions in section (c))" emphasizing that the exception "applies only if the entire practice of a D.C.-based lawyer falls within section (c). If any part of the practice -- no matter how small a percentage -- is not covered by an exception, Rule 49 requires a practitioner with an office in the District of Columbia to be an active member of the D.C. Bar."; "The Committee notes that lawyers based in the District of Columbia may not be eligible for admission to certain federal courts located in the District. The local rules of the U.S. District Court for the District of Columbia do 'not permit a person who holds himself out as a lawyer in the District of Columbia or who practices routinely from an office in the District of Columbia to rely on a bar admission from another jurisdiction to practice law in this Court,' and any such person who practices in that Court 'is engaged in the unauthorized practice of law in the District of Columbia.' . . . For the reasons explained above, even if lawyers based in the District are admitted to practice in a federal court and otherwise qualify for the federal court practice exception in Rule 49(c)(3), they violate the 'holding out' provisions of Rule 49 unless they affirmatively disclose in business documents their bar status and the limits on their practice." (emphasis added)).

• Blackman v. District of Columbia, 355 F. Supp. 2d 171, 172-73, 173 (D. D.C. 2005) (finding that a lawyer had engaged in the unauthorized practice of law in Washington, D.C.; "When the Court received and reviewed the motion for preliminary injunction, it noticed that Mr. Gates had included a Bar number but not a District of Columbia Bar number. It further noted that Mr. Gates listed his office address as 1806 11th Street, N.W., in Washington, D.C., not in Louisiana. It appears that Mr. Gates is a member of the Bar of this Court, having been admitted reciprocally on the basis of his Texas Bar membership, and therefore he would be permitted to practice occasionally in this Court if his primary office were not in the District of Columbia. Without a District of Columbia Bar license, however, he may not practice law in the District of Columbia, including in this Court. Indeed, it appears to the undersigned that Mr. Gates is engaged in the unauthorized practice of law in the District of Columbia in violation of the Rules of the District of Columbia Court of Appeals."; "Practice in this Court by a lawyer whose primary office is in the District of Columbia but who is not a member of the District of

235

35951124_10 UPL, MDP and MJP (Defining What Lawyers Do and McGuireWoods LLP Where They Can Do It): Part II T. Spahn (1/1/19) Hypotheticals and Analyses

Columbia Bar is engaged in the unauthorized practice of law in the District of Columbia."; striking from the docket a pleading filed by the lawyer, and referring to the matter to the District of Columbia Committee on Unauthorized Practice of Law (emphasis added)).

A lawyer appearing in federal court without meeting that court's requirement that the lawyer must be a member in good standing of the host state's bar might face some sanction, and even find that his or her pleading is a nullity.

• Order for Dismissal at 10, 11, 12, 13, Third Degree Films, Inc. v. Does 1-259, Consolidated Case No. 4:11cv570-RH/WCS (N.D. Fla. Apr. 2, 2012), ECF No. 70 ("The complaints in these cases assert that a large number of unknown defendants have infringed the plaintiffs' copyrights by sharing digital files. The cases have been consolidated for pretrial purposes."; "The plaintiffs filed the cases through the attorney Terik Hashmi, also known as Teryk Hashmi. As it turns out, Mr. Hashmi lives in Florida but is not a member of the Florida Bar. To the contrary, prior to filing these cases, Mr. Hashmi executed a cease-and-desist affidavit making clear that he has no authority to represent clients in Florida except in immigration matters. . . . Mr. Hashmi's filing of these cases was a clear violation of the undertakings set out in the affidavit."; "The plaintiffs filed a response asserting that under Florida state law an attorney's lack of authority to practice in the state is not a basis for dismissing a lawsuit."; "The response fails for three reasons."; "First, the issue of whether a federal lawsuit should be dismissed based on the plaintiff's attorney's lack of authority to file the case is an issue of federal law, not state law." (emphasis added); "He swore that he understood he could not provide legal advice or otherwise practice law in Florida, except on immigration matters. He swore that he understood that it would constitute the unlicensed practice of law to hold himself out as an attorney or as able to render legal advice or services; to offer legal services to others; or to collect fees for legal services. The assertion that Mr. Hashmi thought he could properly undertake to represent these plaintiffs in these cases -- cases arising in Florida and having nothing to do with immigration -- is plainly unfounded." (emphasis added); "Third, the response to the order to show cause asserts—but is unsupported by any evidence indicating—that the plaintiffs themselves were unaware of the violation." (emphasis added); "If the plaintiffs indeed filed these cases in good faith and without knowledge of Mr. Hashmi's status, it would have been an easy matter to file a declaration saying so. The plaintiffs did not do that. Nor have they denied demanding and receiving settlement proceeds through Mr. Hashmi, or offered to give them back."; "Under these circumstances, requiring the plaintiffs to start over and do it right is not too harsh a sanction." (emphasis added)).

236

35951124_10 UPL, MDP and MJP (Defining What Lawyers Do and McGuireWoods LLP Where They Can Do It): Part II T. Spahn (1/1/19) Hypotheticals and Analyses

Most states acknowledge federal courts' power. For instance, in 2012 the Florida

Supreme Court adopted a rule prohibiting non-Florida lawyers from making an initial or first appearance in more 3 cases during any 365-day period -- but explicitly excluding appearances in Florida federal courts.

• Florida Rule 1-3.10. Appearance by Non-Florida Lawyer in a Florida Court ("This rule is not applicable to appearances in federal courts sitting in Florida, as appearances before each of those courts are regulated by the rules applicable to those courts. Further, an appearance in a federal court sitting in Florida does not constitute an 'appearance' as contemplated by subdivision (a)(2), because subdivision (a)(2) applies only to appearances before Florida state courts.").

Some states meekly push back at federal courts' exercise of their discretion. For instance, the Pennsylvania Supreme Court indicated that a Pennsylvania lawyer suspended from practicing in that state had violated the suspension order despite having only practiced in federal court. The Pennsylvania Supreme Court took a very narrow view of the Supremacy Clause.

Simply because federal statutes allow for the promulgation of rules, and permit one to conduct case by counsel in federal courts, certainly does not evidence a Congressional intent to divest state courts of their authority to regulate the practice of law by suspended attorneys within their borders. Likewise, while under the local rules for the United States District Court for the Eastern District of Pennsylvania an attorney may be reinstated to "practice before this court notwithstanding the refusal of a state court to reinstate the attorney to practice," E.D. Pa. R. Civ. P. 83.6 VII (I), this does not evidence an intent to prohibit our Court from regulating the maintenance of a law office by a suspended attorney for the proffering of himself to the public or for client consultation and advice. In light of the historical licensing and regulation of attorneys by the states within their respective jurisdictions, Leis v. Flynt, 439 U.S. 438, 58 L. Ed. 2d 717, 99 S. Ct. 698 (1979), and the failure of Mr. Marcone to offer a federal statute or rule that even minimally suggests an intention by Congress to limit the breadth of our Rules in

237

35951124_10 UPL, MDP and MJP (Defining What Lawyers Do and McGuireWoods LLP Where They Can Do It): Part II T. Spahn (1/1/19) Hypotheticals and Analyses

this regard, we simply cannot find support for the proposition that Congress has expressed the sweeping intention of occupying the field to the exclusion of the states.

Office of Disciplinary Counsel v. Marcone, 855 A.2d 654, 665 (Pa. 2004) (emphases added). As explained above, one year later the Third Circuit explicitly asserted federal courts' power to decide the ability of non-Pennsylvania lawyers to practice in a federal court located in Pennsylvania.

Constitutional challenges to federal courts' bar membership requirements usually face.

• Nat'l Ass'n for Advancement of Multijurisdiction Practice v. Simandle, 658 F. App'x 127, 130 (3d Cir. 2016) (holding a lower court's dismissal of a lawsuit challenging the New Jersey district court's rule on allowing New Jersey lawyers to join that court's bar; "In the present case, the NAAMJP and two of its members challenge the conditions placed on admission to the bar of the United States District Court for the District of New Jersey (the 'District Court'). They allege that the District Court's local rules, which generally incorporate New Jersey state admission rules by limiting federal admission to those licensed to practice by the Supreme Court of New Jersey, violate federal statutory and constitutional standards. The defendants -- judges of the District Court and former Attorney General Eric Holder -- moved to dismiss the NAAMJP's complaint. In a thorough and thoughtful opinion, the District Court granted the motion to dismiss. We agree with the reasoning of the District Court in all respects, and will affirm.").

• Nat'l Ass'n for Advancement of Multijurisdictional Practice v. Lynch, 824 F.3d 191, 194, 194-95 (4th Cir. 2016) (upholding the constitutionality of the District of Maryland's local rule which does not allow a non-Maryland lawyer to join the court's bar if the lawyer maintains a Maryland law office; "The National Association for the Advancement of Multijurisdictional Practice and four attorneys (together, 'NAAMJP') challenge the conditions placed on the privilege of admission to the Bar of the United States District Court for the District of Maryland (the 'District' or the 'District Court') in Local Rule 701 ('Rule 701' or the 'Rule'). Because Rule 701 violates neither the Constitution nor federal law, we affirm the district court's decision." (footnote omitted); "Rule 701 governs attorney admission to practice in the District Court. The Rule contains requirements based on the state of licensure and, in some instances, the location of the attorney's law office. The Rule allows for admission of attorneys licensed in the State of Maryland. The Rule also

238

35951124_10 UPL, MDP and MJP (Defining What Lawyers Do and McGuireWoods LLP Where They Can Do It): Part II T. Spahn (1/1/19) Hypotheticals and Analyses

allows for admission of non-Maryland attorneys if the applying attorney maintains his or her principal law office in the state in which he or she is licensed to practice law, as long as the relevant federal district court in the state of licensure does not deny admission to Maryland attorneys. In other words, admission to non-Maryland attorneys extends only to attorneys in states whose district courts observe reciprocity with the District Court. Regardless of reciprocity, however, the District will not admit a non-Maryland attorney if that attorney maintains a law office in Maryland." (footnotes omitted)).

Although theoretically some federal courts allow lawyers to practice before them

despite not being licensed by the courts' host state, as a practical matter lawyers should

never count on doing so.

Not surprisingly, states do not permit out-of-state lawyers to practice in state courts based on the lawyer's admission to a federal court in the state.

• In re Marcone, 395 F. App'x 807, 808, 809 (3d Cir. 2010) (unpublished opinion) (suspending for two years a lawyer who continued to practice in Pennsylvania despite being suspended by the Pennsylvania Supreme Court from practicing law; concluding that the lawyer could not rely on his membership in the bar of a federal district allowed him to practice in Pennsylvania; "Prior to Marcone's suspension by the District Court, the Pennsylvania Supreme Court had suspended him from the practice of law."; "Marcone claimed that he was entitled to render legal services to the broker because he had a 'federal license' by virtue of his admission to practice law in the Eastern District of Pennsylvania."; "Marcone contends that the District Court erred because his 'federal license' to practice in the Eastern District of Pennsylvania provided him with a 'safe pass' to practice law even though his Pennsylvania license had been suspended."; "[F]ederal law authorized Marcone to practice law only before the District Court for the Eastern District of Pennsylvania. Yet Marcone relied on his admission to the Eastern District bar to provide legal services in a matter that had no connection with litigation either pending in the Eastern District or intended to be prosecuted before that Court. Accordingly, we conclude that the District Court did not err in applying Sperry [Sperry v. Florida, 373 U.S. 379 (1963)] and in determining that Marcone engaged in the unauthorized practice of law when he rendered the legal services to the real estate broker.").

Best Answer

The best answer to this hypothetical is NO (PROBABLY). B 2/13 239

35951124_10 UPL, MDP and MJP (Defining What Lawyers Do and McGuireWoods LLP Where They Can Do It): Part II T. Spahn (1/1/19) Hypotheticals and Analyses

Practicing "Federal" Law

Hypothetical 22

After practicing for many years as a labor lawyer in a northeastern state, you want to move to a southern state to be near your aging parents. The thought of taking another bar exam frightens you, and you are not eligible to be admitted by motion to the bar of that southern state (because you recently have been serving as an in-house lawyer in a state where you are not fully licensed). You wonder whether you will be able to move to the southern state and practice a certain type of law, without obtaining a license there.

May you move to another state and practice "federal labor law" without a license in that state?

NO (PROBABLY)

Analysis

This question deals with the application of the Supremacy Clause1 to the type of law that a lawyer practices -- rather than to the forum (such as a federal agency or a federal court) in which the lawyer practices.

The ABA Model Rules indicate that a lawyer admitted in some United States jurisdiction (and not "disbarred or suspended from practice in any [other] jurisdiction") may provide legal services that

are services that the lawyer is authorized to provide by federal law or other law of this jurisdiction.

ABA Model Rule 5.5(d)(2). A comment provides additional explanation.

Paragraph (d)(2) recognizes that a lawyer may provide legal services in a jurisdiction in which the lawyer is not licensed when authorized to do so by federal or other law, which includes statute, court rule, executive regulation or judicial precedent.

1 U.S. Const. art. VI, § 2. 240

35951124_10 UPL, MDP and MJP (Defining What Lawyers Do and McGuireWoods LLP Where They Can Do It): Part II T. Spahn (1/1/19) Hypotheticals and Analyses

ABA Model Rule 5.5 cmt. [18].

The ABA Model Rules indicate that lawyers engaging in such practice must subject themselves to the disciplinary authority of the state where they are located,2 and must accurately describe the limits of their license in all marketing material and in their disclosure to clients.3

Not surprisingly, courts and bars dealing with this issue focus on several key issues.

First, any lawyer engaging in such conduct must avoid "holding himself out" as a lawyer licensed in that state.

• In re Defillo, 762 S.E.2d 552, 553, 555 (S.C. 2014) (disbarring a Florida lawyer who held herself out as a South Carolina lawyer and represented immigrant clients in South Carolina, among other ethics violations; "In 2012, respondent opened an office in Greenville, ostensibly to handle federal immigration matters. Respondent had no law partners or associates who were licensed in South Carolina except for a period of approximately fourteen days in August 2012. Respondent offered to provide legal services in South Carolina using methods specifically targeted at potential clients in South Carolina, including a law firm website, business cards, print advertisements, and radio commercials as discussed below."; "In connection with her representation of two clients in federal immigration matters, respondent sent letters to judges for the state circuit court in Greenville, requesting certification that the clients were crime victims. The letterhead contained in the phrase 'Attorneys and Counselers at Law' when, in fact, respondent had no partners or associates at the letters were written. Respondent's letterhead included her Greenville office address without indicating the jurisdictional limitations on her ability to practice law."; "Respondent advertised her law firm through the use of a website available to residents of South Carolina. Included on the website are references to respondent's Greenville office. Respondent's website contains material misrepresentations and omits facts necessary to make the contents considered as a whole not materially misleading. On her website, respondent advertises her office in Greenville but fails to state that she is not

2 ABA Model Rule 5.5 cmt. [19]. 3 ABA Model Rule 5.5 cmt. [20]. 241

35951124_10 UPL, MDP and MJP (Defining What Lawyers Do and McGuireWoods LLP Where They Can Do It): Part II T. Spahn (1/1/19) Hypotheticals and Analyses

licensed to practice law in South Carolina or to otherwise set forth the jurisdictional limitations on her practice in this state. Further, respondent's website is not limited to the promotion of her federal immigration practice as she advertises her experience in both criminal and family law and offers to 'analyze the facts of [her prospective client's] case by applying current . . . State Laws.'"; "In the current matter, not only did respondent target residents of South Carolina through various forms of advertising including radio communications and print media, but she also held herself out as licensed to practice law in this state, welcomed clients with criminal and family law concerns, and sent letters on behalf of clients addressed to state court judges. Further, when she did participate in the disciplinary investigation, respondent made false statements of material fact concerning the extent of her practice and the extent of her advertising in South Carolina to [Office of Disciplinary Counsel].").

• Alaska LEO 2010-1 (2010) ("Assuming that the lawyer clearly advises his clients that he is not an Alaska lawyer and avoids advising regarding legal issues outside of immigration law, the lawyer may maintain a physical office in Alaska."; "Under Rule 7.1, the lawyer is prohibited from making false or misleading communications regarding the lawyer or his services. Affirmative steps should be taken to avoid the potential for violating these rules. Examples of steps the lawyer might consider include expressly addressing the issue in the fee agreement, noting the limits of the practice on letterhead or in office signage, and affirmatively stating to clients that the lawyer is not a member of the Alaska Bar Association.").

• Gould v. Harkness, 470 F. Supp. 2d 1357, 1358, 1361, 1362, 1363 (S.D. Fla. 2006) (holding that a New York lawyer residing in Florida cannot advertise for clients who need help with "New York Legal Matters Only" or need the services of his "Federal Administrative Practice"; holding that the advertising does not deserve protection as commercial speech; "Plaintiff seeks to advertise his availability as an attorney in Florida even though he is only licensed in New York. Although he seems to limit his practice to New York legal matters, he does not state that he is not a member of the Florida Bar or that he is not authorized to practice in Florida. In other words, like the defendant in Tate, Plaintiff fails to state that he is only licensed to practice in New York. By stating 'Free Phone Consultation' and listing a Miami phone number along with a Miami address for his office, Plaintiff creates the impression that he is authorized to practice law in Florida."; also noting that the lawyer's establishment of a Florida office amounts to the unauthorized practice of law; "In this case, there is no evidence that anyone other than Plaintiff would be responsible for his office's operations and because Plaintiff is not a member of the Florida bar, his establishment of a Florida law office constitutes UPL."; noting that "[p]laintiff cannot point to federal legislation or regulations that specifically authorize him to engage in general federal

242

35951124_10 UPL, MDP and MJP (Defining What Lawyers Do and McGuireWoods LLP Where They Can Do It): Part II T. Spahn (1/1/19) Hypotheticals and Analyses

administrative practice" (emphasis added); finding that the Florida MJP rule (effective in 2005) does not allow the lawyer to conduct his practice in Florida, because it is not a "temporary" practice), aff'd sub nom. Gould v. Florida Bar, 259 F. App'x 208 (11th Cir. 2007) (unpublished opinion), cert. denied, 128 S. Ct. 2433 (2008).

• Attorney Grievance Comm'n of Md. v. Harris-Smith, 737 A.2d 567, 568 (Md. 1999) (suspending for thirty days a Pennsylvania lawyer who claimed to be engaging "exclusively" in bankruptcy law in Maryland, but whose letterhead listed the Maryland office without any explanation of the limitation on her license).

• In re Peterson, 163 B.R. 665, 675 (Bankr. D. Conn. 1994) ("Put another way, an attorney who is not licensed by the State of Connecticut but who is authorized to practice before the bankruptcy court may . . . practice law in this state and even maintain an office here so long as the services rendered are limited to those reasonably necessary and incident to the specific matter pending in this court. There is a difference, however, between maintaining an office in this state for the convenience of litigating a matter in this court and maintaining such an office for the purpose of giving legal advice on bankruptcy matters to all clients who seek it and accepting all cases which can be filed here. In the instant case, the evidence disclosed that Betsos held himself out to give advice to 'all comers.'" (citation omitted)).

• Florida Bar v. Savitt, 363 So. 2d 559, 560-61 (Fla. 1978) (enjoining the law firm of Stroock & Stroock & Lavan from allowing any of its lawyers to practice law out of its Miami office unless the lawyer was admitted to practice in Florida; operating a Miami office without a member of the Florida Bar "assuming on a continuing basis responsibility for the supervision of the operations"; allowing any of its Miami lawyers who are not members of the Florida Bar to supervise associates; implying to any clients that its lawyers are members of the Florida Bar if they are not; using letterhead in its Miami office that gives the false impression that the lawyers practicing there are members of the Florida Bar if they are not; holding that the law firm could "give legal advice concerning a right or obligation governed by federal law . . . provided that, if the lawyer giving the legal advice is not a member of The Florida Bar, the lawyer is in Florida on a transitory basis and it has initially been made clear to the client and immediately confirmed in writing that the lawyer is not a member of the Florida Bar.").

Second, lawyers properly practicing purely federal law in a state without being licensed there can be punished for ethical misconduct in the state.

243

35951124_10 UPL, MDP and MJP (Defining What Lawyers Do and McGuireWoods LLP Where They Can Do It): Part II T. Spahn (1/1/19) Hypotheticals and Analyses

• Peter Vieth, Virginia State Bar reprimands New York lawyer practicing in Virginia, Va. Law. Wkly., Jan. 7, 2014 ("A New York-licensed lawyer practicing immigration law in Virginia has been disciplined for misconduct by the Virginia State Bar."; "The bar's action was based on rules that subject foreign lawyers practicing in Virginia to this state's discipline system. It's the second time in recent years the bar has imposed discipline on a lawyer who did not hold a Virginia license."; "Bibi Bahizi Musafiri was licensed in New York in 2006, but has never been licensed in Virginia, according to findings of a Virginia State Bar district committee. She practiced immigration law in Fredericksburg, Arlington and Falls Church."; "A client complained Musafiri failed to file an immigration form as promised. The client paid a total of $3,000 in installments, but Musafiri did not place the money in any trust account. In fact, the bar said, Musafiri did not have a trust account."; "The client later paid another lawyer $2,000 to handle her immigration issues. Musafiri did not return the client's fees, the Virginia State Bar said."; "Musafiri took part in two days of hearings with the Virginia State Bar district committee. The panel imposed a public reprimand with terms, including repayment of the client's $3,000 and taking 30 hours of legal education on ethics, trust accounts and immigration law.").

• State ex rel. York v. W. Va. Office of Disciplinary Counsel, 744 S.E.2d 293, 296, 302, 304 (W.Va. 2013) (holding that West Virginia could discipline a lawyer who was not licensed in West Virginia but practicing patent law in West Virginia; "[T]he petitioner is not a member of the West Virginia State Bar, and his practice was limited to patent and trademark issues before the PTO while associated with the Waters Law Group. The petitioner represented clients from West Virginia, but he did not appear in the state courts of West Virginia."; "Although this Court has not yet adopted those rules, we find that the rationale of Model Rules 5.5 and 8.5 comports with the express intention of this state to authorize disciplinary action against 'any individual admitted to the practice of law in another jurisdiction who engages in the practice of law in West Virginia.' W.Va. R. Law. Disc. P. 1."; finding that federal preemption did not prevent the West Virginia Bar from investigating and punishing the lawyer; "The ODC [Office of Disciplinary Counsel] and LDB [Lawyer Discipline Board] argue that because they will not seek to suspend or expel the petitioner from his federal practice, there is no conflict presently existing in the simultaneous federal and state disciplinary investigations. Based upon the foregoing, this Court agrees and finds that federal law does not preempt this state's disciplinary proceedings in this matter." (footnote omitted)).

244

35951124_10 UPL, MDP and MJP (Defining What Lawyers Do and McGuireWoods LLP Where They Can Do It): Part II T. Spahn (1/1/19) Hypotheticals and Analyses

Not surprisingly, such discipline could trigger reciprocal discipline by the lawyers' home

state.4

Third, lawyers attempting to limit their practice in this way must scrupulously

avoid providing advice outside the narrow range of federal law.

Although there is no consensus list of purely "federal" law that out-of-state lawyers may safely practice, a 2003 North Carolina guideline provides a useful if not universally accepted list.

• North Carolina State Bar Authorized Practice Comm., Guidelines for Attorneys Licensed in other Jurisdictions (July 2003) (allowing an unlicensed lawyer residing in North Carolina to limit his or her practice to federal law; however explaining that "the law firm and unlicensed attorney must clearly indicate on any public communication that the attorney is not licensed in North Carolina, specify unlicensed attorney's licensed jurisdiction(s), and note that the unlicensed attorney's practice is limited to specific area(s) of federal law. . . . Areas of exclusive federal law include, but are not limited to, immigration, patent, federal tax, federal tort claims, military, maritime, and claims before the Society Security Administration. Other areas of federal law

4 Lawyers' home states normally do not impose their logistical requirements on these lawyers practicing purely federal law elsewhere. New York LEO 1058 (6/10/15) (analyzing a New York lawyer's requirement to keep a New York trust account if she will practice in Illinois as an immigration lawyer; "The inquirer was recently admitted to practice in New York but plans to reside and practice solely in Illinois and will engage in a practice limited to immigration law. She inquires about her obligations under Rule 1.15(b) of the New York Rules of Professional Conduct (the 'Rules') regarding client trust accounts."; "Federal immigration regulations provide that a person entitled to representation may be represented by a member in good standing of the bar of the highest court of any state, who is not under suspension or otherwise restricted in his or her practice of law, as long as the lawyer is registered to practice with the Executive Office for Immigration Review. See 8 C.F.R. §§ 1001.1(f), 1292(a)(1); see also Anna Marie Gallagher, 'A primer on immigration court practice,' 08-12 Immigration Briefings 1 (2008) (hereinafter 'Gallagher') (noting six categories of persons who are permitted to represent parties in the immigration courts); N.Y. State 863 (2011) (discussing lawyer licensed only in Texas who works at law firm in New York State that 'exclusively practices immigration law). The cited federal regulations apparently apply to appearances before the U.S. Citizenship and Immigration Service, the Board of Immigration Appeals, and an Immigration Court or Judge, including any related application, proceeding, practice, and preparation. The full scope of federal law permitted lawyers to practice immigration law is a question of law beyond this Committee's jurisdiction. See N.Y. State 863 (2011)."; "If the inquirer receives client funds in connection with a proceeding in a court before which the inquirer has been admitted to practice (either generally or for purposes of that proceeding), the rules to be applied will be the rules of the jurisdiction in which the court sits, unless the rules of the court provide otherwise."; "For purposes of applying New York's Rule 8.5(b), the Board of Immigration Appeals is not a 'court.'"; ultimately finding that the New York lawyer did not have to establish a New York trust account). 245

35951124_10 UPL, MDP and MJP (Defining What Lawyers Do and McGuireWoods LLP Where They Can Do It): Part II T. Spahn (1/1/19) Hypotheticals and Analyses

including, but not limited to, bankruptcy and trademark law, may involve issues of state law and, consequently, cannot be practiced in the North Carolina without being admitted to the North Carolina State Bar." (emphasis added)).

A better way to identify the acceptable practice areas involves a review of states' ethics rules, opinions and occasional case law approving such an arrangement.

• Virginia Rule 5.5 cmt. [4] ("Despite the foregoing general prohibition, a Foreign Lawyer [non-Virginia and non-US lawyers] may establish an office or other systematic and continuous presence in Virginia if the Foreign Lawyer's practice is limited to areas which by state or federal law do not require admission to the Virginia State Bar. Examples of lawyers admitted in another United States jurisdiction include those lawyers whose practices are limited to federal tax practice before the IRS and Tax Court, patent law before the Patent and Trademark Office, or immigration law.").

• Alaska LEO 2010-1 (2010) ("Assuming that the lawyer clearly advises his clients that he is not an Alaska lawyer and avoids advising regarding legal issues outside of immigration law, the lawyer may maintain a physical office in Alaska."; "Under Rule 7.1, the lawyer is prohibited from making false or misleading communications regarding the lawyer or his services. Affirmative steps should be taken to avoid the potential for violating these rules. Examples of steps the lawyer might consider include expressly addressing the issue in the fee agreement, noting the limits of the practice on letterhead or in office signage, and affirmatively stating to clients that the lawyer is not a member of the Alaska Bar Association." (emphasis added)).

• North Carolina Rule 7.5 cmt. [2] ("This rule does not prohibit the employment by a law firm of a lawyer who is licensed to practice in another jurisdiction, but not in North Carolina, provided the lawyer's practice is limited to areas that do not require a North Carolina law license such as immigration law, federal tort claims, military law, and the like. The lawyer's name may be included in the firm letterhead, provided all communications by such lawyer on behalf of the firm indicate the jurisdiction in which the lawyer is licensed as well as the fact that the lawyer is not licensed in North Carolina. If law offices are maintained in another jurisdiction, the law firm is an interstate law firm and must register with the North Carolina State Bar as required by 27 N.C.A.C. 1E, Section .0200." (emphasis added)).

• New Jersey UPL Op. 44 (2008) (holding that lawyer not licensed in New Jersey may practice in a New Jersey law firm if he "solely engages in immigration law" (emphasis added); also holding that the lawyer can advertise his availability to practice immigration law, as long as all his

246

35951124_10 UPL, MDP and MJP (Defining What Lawyers Do and McGuireWoods LLP Where They Can Do It): Part II T. Spahn (1/1/19) Hypotheticals and Analyses

advertising states that the lawyer is not licensed in New Jersey and limits his practice to immigration matters).

• Philadelphia LEO 2005-14 (8/2005) ("Since the inquirer's situation clearly fits within 5.5d(2) it becomes clear that he is not required to be admitted to the Pennsylvania Bar in order to maintain an office here provided he limits his practice to immigration work. This is true whether or not he is in a partnership with a Pennsylvania admitted attorney." (emphasis added); "The Committee notes that oftentimes state law issues, for example domestic relations law, will have an impact on representation in an immigration matter. The inquirer is required by Rule 1.1 (Competence), if dealing with any of these questions to have sufficient knowledge of such law in order to provide competent advice. However, the inquirer's involvement in such areas must be limited to advice and discussion on such matters as they impact the client's immigration matter and nothing further. Should the client request that the inquirer become more involved, to do so would place the inquirer in violation of Rule 5.5." (emphasis added)).

• Philadelphia LEO 2004-6 (8/2004) (analyzing the following fact pattern: "The inquirer is an immigration attorney practicing in an LLP with one other attorney who is admitted to the Pennsylvania Bar. The firm's office is located in Pennsylvania. The inquirer practices only federal administrative law exclusively before the immigration courts in Philadelphia. He does not give advice or handle any matters involving Pennsylvania law nor does he hold himself out as a member of the Pennsylvania Bar."; explaining that the lawyer had earlier been advised that his practice was acceptable, but that he wondered whether Pennsylvania's recently revised Rule 5.5 changed the analysis; explaining that he could continue practicing even if he did not partner with a Pennsylvania lawyer who is actively participating in the matters; "As the inquirer is in partnership with a Pennsylvania-admitted attorney, his present practice of immigration law within his Pennsylvania office would be authorized by Rule 5.5(c)(1), provided his partner actively participates in the matter. However, even if his partner does not, Rule 5.5(d)(2) makes it clear that the inquirer is authorized to provide his immigration services by virtue of his admission to the Immigration Court." (emphases added); ultimately concluding that the lawyer could establish a continuous presence in Pennsylvania even if he was not admitted there; "Since the inquirer's situation clearly fits within 5.5d(2) it becomes clear that he is not required to be admitted to the Pennsylvania Bar in order to maintain an office here provided he limits his practice to immigration work. This is true whether or not he is in a partnership with a Pennsylvania admitted attorney. The answer to the second question renders the third question moot." (emphasis added); "The Committee does remind the inquirer that since he is not admitted to practice generally in Pennsylvania and is only able to practice

247

35951124_10 UPL, MDP and MJP (Defining What Lawyers Do and McGuireWoods LLP Where They Can Do It): Part II T. Spahn (1/1/19) Hypotheticals and Analyses

in immigration matters that any advertisements, stationary, cards, etc. must so note in accordance with Rules of Professional Conduct 7.1a and 7.5a.").

• North Carolina LEO 2000-3 (7/21/00) (explaining that a lawyer does not engage in improper solicitation by posting messages on a message board that is not interactive; also explaining that "[l]imiting responses to inquiries involving federal law should avoid the unauthorized practice of law in jurisdictions where the P Law Firm lawyers are not licensed to practice law. It is assumed a lawyer with an active law license from any state may practice federal telecommunications law. However, to avoid the possibility of misleading a user of the message board, a lawyer responding to an inquiry should state the jurisdictions where he or she is licensed to practice law." (emphasis added)).

Not surprisingly, lawyers face discipline if they engage in practice outside one of these narrow topics.

• Attorney Grievance Comm'n v. Ambe, 38 A.3d 390, 393, 393-94, 398, 414, 415 (Md. 2012) (reprimanding a New York lawyer for representing immigration clients in Maryland on non-immigration personal injury cases; noting that the lawyer had continued to hold himself out as a Maryland lawyer after being warned not to do so; explaining the background: "The Respondent, Jude Ambe, received a LLM degree from the American University School of Law in 2008. Respondent was not eligible to take the Maryland Bar Exam in 2009 because he does not have a JD degree. Instead, Respondent took the New York Bar Exam, passed, and was admitted to practice in the State of New York in February 2009. Respondent is not, and has never been, a member of the Bar of the Court of Appeals of Maryland."; "Since his admission to the New York Bar, Respondent has maintained a law office at 8121 Georgia Avenue, Silver Spring, Montgomery County, Maryland. According to Respondent, the office is maintained solely for the practice of immigration law. Respondent does not maintain a law office in New York or any other jurisdiction."; "On February 23, 2010, Bar Counsel wrote to Respondent and, among other things, stated that: 'since you are not a member of the Maryland Bar, if your practice is limited only to immigration matters then your letterhead and any signs must indicate that you are a member of the New York Bar, specifically state that you are not a member of the Maryland Bar, and that your practice is limited only to federal immigration matters. Therefore it may be that you are holding yourself out as able to practice law in Maryland without restrictions, and therefore, it may be necessary to investigate whether you have been engaged in the unauthorized practice of law, and seek an injunction.' Bar Counsel also asked Respondent to respond further."; "There is clear and convincing evidence that the Respondent violated Rule 5.5(a) by representing clients in

248

35951124_10 UPL, MDP and MJP (Defining What Lawyers Do and McGuireWoods LLP Where They Can Do It): Part II T. Spahn (1/1/19) Hypotheticals and Analyses

Maryland state tort law cases while not licensed to practice law in Maryland."; "[A]fter Respondent was informed that his practice in Maryland had to be limited to federal immigration matters he continued to work on at least one of the Maryland tort law cases." (emphasis added); "The situation in the case sub judice appears to be that of an inexperienced attorney who became licensed in 2009 (in New York), and without any prior experience with lawyering in any jurisdiction, wanted to open a practice dedicated solely to the practice of immigration law. He got in trouble when he attempted to assist immigration clients in areas of the law he was not permitted to practice in this state. There is absolutely no evidence that he was attempting to solicit clients for representation in matters of Maryland law."; "Respondent did not violate Rule 5.5(a) by representing clients in Maryland state tort law matters while not licensed to practice law in Maryland. But, as we have indicated, it does not appear he actively solicited any clients in respect to Maryland issues . . . ."; finding a public reprimand sufficient).

• Gould v. Harkness, 470 F. Supp. 2d 1357, 1358, 1361, 1362, 1363 (S.D. Fla. 2006) (holding that a New York lawyer residing in Florida cannot advertise for clients who need help with "New York Legal Matters Only" or need the services of his "Federal Administrative Practice"; holding that the advertising does not deserve protection as commercial speech; "Plaintiff seeks to advertise his availability as an attorney in Florida even though he is only licensed in New York. Although he seems to limit his practice to New York legal matters, he does not state that he is not a member of the Florida Bar or that he is not authorized to practice in Florida. In other words, like the defendant in Tate, Plaintiff fails to state that he is only licensed to practice in New York. By stating 'Free Phone Consultation' and listing a Miami phone number along with a Miami address for his office, Plaintiff creates the impression that he is authorized to practice law in Florida."; also noting that the lawyer's establishment of a Florida office amounts to the unauthorized practice of law; "In this case, there is no evidence that anyone other than Plaintiff would be responsible for his office's operations and because Plaintiff is not a member of the Florida bar, his establishment of a Florida law office constitutes UPL."; noting that "[p]laintiff cannot point to federal legislation or regulations that specifically authorize him to engage in general federal administrative practice"; finding that the Florida MJP rule (effective in 2005) does not allow the lawyer to conduct his practice in Florida, because it is not a "temporary" practice (emphasis added)), aff'd sub nom. Gould v. Florida Bar, 259 F. App'x 208 (11th Cir. 2007) (unpublished opinion), cert. denied, 128 S. Ct. 2433 (2008).

• Maryland LEO 2005-07 (2005) ("You contemplate the leasing of office space in Maryland, for use by attorneys from the District of Columbia office. You propose that the firm's letterhead, business cards and the door to the leased space would prominently display that that attorneys are not licensed to

249

35951124_10 UPL, MDP and MJP (Defining What Lawyers Do and McGuireWoods LLP Where They Can Do It): Part II T. Spahn (1/1/19) Hypotheticals and Analyses

practice law in Maryland. You state that, as is the nature of your work in the District of Columbia, all matters handled out of the leased space would be limited to federal tax law. The firm would not actively seek or solicit clients in Maryland, and Maryland-based clients would rarely be seen there. Finally, you state that none of the attorneys utilizing the leased space would represent clients in Maryland courts." (emphasis added); "[O]pening a law office in Maryland, where no one is admitted to the practice of law in Maryland, will result in your firm's unauthorized practice of law in this jurisdiction." (emphasis added)).

• In re Peterson, 163 B.R. 665, 675 (Bankr. D. Conn. 1994) ("Put another way, an attorney who is not licensed by the State of Connecticut but who is authorized to practice before the bankruptcy court may . . . practice law in this state and even maintain an office here so long as the services rendered are limited to those reasonably necessary and incident to the specific matter pending in this court. There is a difference, however, between maintaining an office in this state for the convenience of litigating a matter in this court and maintaining such an office for the purpose of giving legal advice on bankruptcy matters to all clients who seek it and accepting all cases which can be filed here. In the instant case, the evidence disclosed that Betsos held himself out to give advice to 'all comers.'" (citation omitted)).

Some bars defer to courts in analyzing this issue.

• Maryland LEO 2007-06 (10/10/06) ("You state that you will practice from a home office and the practice will be strictly limited to the area of employment- and family-based immigration petitions. Moreover, you state that those areas do not involve state law." (emphasis added); explaining that "the Ethics Committee may only provide opinions regarding whether a practice is ethical, and may not provide legal opinions. Whether a certain practice, such as your limited immigration practice, constitutes the unauthorized practice of law is a legal issue that can only be determined by the Court of Appeals." (emphasis added); "In addition, you are referred to Rule 701 of the Rules of the U.S. District Court of the District of Maryland, should your limited immigration practice include activity in that court. Rule 701 provides that: 'Except as provided in subsection c of this rule, an attorney is qualified for admission to the bar of this District if the attorney is and continuously remains a member in good standing of the highest court of any State in which the attorney maintains his or her principal law office.' Subsection c is not applicable because your principal law office will be in your home office in Maryland. Subsection d, however, is applicable and states that 'An attorney who is not a member of the Maryland Bar is not qualified for admission to the bar of this District if the attorney maintains any law office in Maryland.'").

250

35951124_10 UPL, MDP and MJP (Defining What Lawyers Do and McGuireWoods LLP Where They Can Do It): Part II T. Spahn (1/1/19) Hypotheticals and Analyses

Although it is theoretically possible for a lawyer to continuously practice in some area of "federal law" in a state where the lawyer is not licensed, the risk of crossing the line into unauthorized practice of law is very high. The less precise the area of federal law, the higher the risk.

For instance in Kennedy v. Bar Ass'n, 561 A.2d 200 (Md. 1989), the court acknowledged the possibility that a lawyer could properly draw the line when advising clients -- but found as a practical matter that the lawyer could not adequately serve clients by trying to do so.

We will not go so far as to say that it is theoretically impossible for Kennedy to maintain a principal office in Maryland exclusively for engaging in a practice before the federal court in Maryland and the courts in the District of Columbia. It seems, however, that it would be practically impossible to do so. Nevertheless, we shall not foreclose the possibility of Kennedy's presenting to the Circuit Court of Montgomery County, in the exercise of its continuing jurisdiction over the injunction, any proposal whereby Kennedy, without holding himself out as practicing law in Maryland, could first pinpoint clients whose specific matters actually required counsel before those courts where Kennedy is currently admitted to practice, and thereby could limit his legal representation in Maryland to those specific matters.

Id. at 211 (emphases added).

Significantly, the court did not focus on what the lawyer would do for the clients he represented. Instead, the court noted that the lawyer would be engaging in the practice of law (in Maryland) when deciding whether he could represent them. The court explained that

advising clients by applying legal principles to the client's problem is practicing law. When Kennedy, who is unadmitted in Maryland, set up his principal office for the

251

35951124_10 UPL, MDP and MJP (Defining What Lawyers Do and McGuireWoods LLP Where They Can Do It): Part II T. Spahn (1/1/19) Hypotheticals and Analyses

practice of law in Maryland and began advising clients and preparing legal documents for them from that office, he engaged in the unauthorized practice of law. This is so whether the legal principles he was applying were established by the law of Montgomery County, the State of Maryland, some other state of the United States, the United States of America, or a foreign nation. . . . He is not permitted to sort through clients who may present themselves at his Maryland office and represent only those whose legal matters would require suit or defense in a Washington, D.C. court or in the federal court in Maryland because the very acts of interview, analysis and explanation of legal rights constitute practicing law in Maryland. For an unadmitted person to do so on a regular basis from a Maryland principal office is the unauthorized practice of law in Maryland.

Id. at 208-10 (emphasis added). Of course, this analysis created an insoluble dilemma

for such a lawyer. The District of Columbia Bar later suspended Kennedy for nine

months because of this infraction in Maryland. In re Kennedy, 605 A.2d 600 (D.C.

1992).

More recently, the Virgin Island Supreme Court took the same narrow approach.

In a frightening 2018 case, the Supreme Court of the Virgin Islands found that

three Akin Gump lawyers had committed the unauthorized practice of law in the Virgin

Islands, and referred them, two Virginia-based colleagues and local counsel to bar

disciplinary authorities and the Virgin Islands Attorney General (mentioning the

possibility of prosecution).

In In re Jindal, S. Ct. BA Nos. 2018-0018, 2018-0019, 2018-0020, 2018 V.I.

Supreme LEXIS 38 (V.I. Nov. 29, 2018),5 three Akin Gump lawyers sought admission

5 In re Jindal, S. Ct. BA. Nos. 2018-0018 to 2018-0020, 2018 V.I. Supreme LEXIS 38, at *10 n.3, *13, *4-5, *2-3 (V.I. Nov. 29, 2018) (per curiam) (denying pro hac admission petitions of three Akin Gump lawyers after they sought fees for successfully representing their client in the Virgin Islands courts; acknowledging that their pro hac vice motions had been granted, but noting that they had never taken the 252

35951124_10 UPL, MDP and MJP (Defining What Lawyers Do and McGuireWoods LLP Where They Can Do It): Part II T. Spahn (1/1/19) Hypotheticals and Analyses

pro hac vice in the Virgin Island courts to represent a client in an antitrust case. Those

lawyers had unwisely neglected to fulfill all the pro hac vice admissions requirements.

Although this Court granted those motions, it advised Attorney Eckard and the applicants that their authorization to practice on a pro hac vice basis would not commence until and unless they were administered the pro hac vice attorney oath. However, neither Jindal, Roush, nor Schmitten ever took the pro hac vice attorney oath.

Id. at *2.

Just a few months later, the lawyer's adversaries voluntarily dismissed all claims

against Akin Gump's client. Akin Gump's client soon sought approximately $75,000 in

attorneys fees. In addition to the time spent by the three lawyers, two other Virginia-

based Akin Gump lawyers' "legal services" appeared on the fee petition.

[T]he motion included affidavits from Amanda B. Lowe and Patrick O'Brien – two Virginia attorneys employed by Akin Gump, who are not admitted to practice in the Virgin Islands and who never applied to this Court for pro hac vice admission – in which they also stated that they represented U.S. Concrete and included itemized lists of "legal services" . . . .

Id. at *2-3.

necessary attorney oath and therefore were not authorized to practice in the Virgin Islands; explaining that their litigation adversary had asked for a stay of their request for attorneys fees because they had violated the Virgin Islands unauthorized practice of law regulations – but that the three lawyers had never responded to the unauthorized practice of law complaint; rejecting the lawyers' argument that they were practicing federal law (noting that the antitrust case involved Virgin Islands law, and that they could not ethically have practiced in the Virgin Island courts even if the case had involved federal law); also rejecting their argument that requiring pro hac admission in the Virgin Islands by "every attorney who works on a matter that 'touches' the Virgin Islands" would "'cripple cross-border transactions and litigation'"; warning that "[s]ince the underlying conduct may potentially warrant action beyond the denial of pro hac vice admission" (having earlier issued a show cause order demanding that the lawyers show cause why they should not be referred "for further investigation and prosecution"), referring to two Virgin Islands bar disciplinary entities and the Virgin Islands Attorney General): (1) the three Akin Gump lawyers; (2) two other Virginia-based Akin Gump lawyers who were listed as having provided "legal services," but who were never admitted pro hac; (3) the Akin Gump law firm; and (4) Akin Gump's local counsel). 253

35951124_10 UPL, MDP and MJP (Defining What Lawyers Do and McGuireWoods LLP Where They Can Do It): Part II T. Spahn (1/1/19) Hypotheticals and Analyses

Plaintiffs' lawyer sought a stay of the fee issue, along with filing a "Notice to Court of Possible Violation of Pro Hac Vice Order." That Notice mentioned the Akin Gump lawyers' violation of "the prohibition on the unauthorized practice of law" codified in

Virgin Islands law and the Virgin Islands Rules of Professional Conduct. Id. at *4.

The three Akin Gump lawyers' second unwise move was to ignore what the court called plaintiffs' "unauthorized practice of law complaint."

Neither Attorney Eckard [Akin Gump's local counsel, who had filed the pro hac admission motions] nor Applicants Jindal, Roush or Schmitten responded to the May 23, 2018 unauthorized practice of law complaint. This Court, in a June 15, 2018 order, noted the seriousness of the allegations and directed Attorney Eckard and Applicants Jindal, Roush, and Schmitten to show cause, in writing, as to why their conduct (1) should not constitute the unauthorized practice of law or the aiding and abetting of the unauthorized practice of law; (2) should not result in denial of the applications byJindal, Roush, and Schmitten for pro hac vice admission; and (3) should not be referred to the Board on Unauthorized Practice of Law, the Board on Professional Responsibility, the Office of Disciplinary Counsel, the Virgin Islands Attorney General, or other authorities for further investigation and prosecution.

Id. at *4-5 (emphasis added). The three lawyers responded to the show cause order.

The court rejected several arguments advanced by the Akin Gump lawyers.

Among other things, the court rejected the Akin Gump lawyers' argument that they were antitrust counsel for their client and therefore somehow authorized to practice in the Virgin Islands. The court noted that the case "did not raise a claim under federal antitrust law, or assert any federal claim for that matter." Id. at *12. The court then indicated that even if the case had involved federal antitrust law, the Akin Gump lawyers would still have had to follow the Virgin Islands pro hac process.

254

35951124_10 UPL, MDP and MJP (Defining What Lawyers Do and McGuireWoods LLP Where They Can Do It): Part II T. Spahn (1/1/19) Hypotheticals and Analyses

But even if the Gilbert lawsuit involved a federal claim, the Supreme Court of the United States has held that "[t]here is no right of federal origin that permits . . . lawyers to appear in state courts without meeting that State's bar admission requirements." Leis v. Flynt, 439 U.S. 438, 443, 99 S. Ct. 698, 58 L. Ed. 2d 717 (1979). Neither Supreme Court Rule 201, Virgin Islands Rule of Professional Conduct 211.5.5, section 443 of title 4 of the Virgin Islands Code, nor any other applicable Virgin Islands rule or statute codifies a federal practice exception to either the prohibition on the unauthorized practice of law or the requirement that one cannot practice law in the Virgin Islands without being a member of the Virgin Islands Bar, and we agree with the courts that have declined to recognize such an exception. See Chandris, S.A. v. Yanakakis, 668 So.2d 180, 184 (Fla. 1995) ("We find no merit to [the] argument that there is a general federal law exception to Florida's bar admission requirement."); Kennedy v. Bar Ass'n of Montgomery County, Inc., 561 A.2d 200, 208 (Md. 1989) ("[A]dvising clients by applying legal principles to the client's problem is practicing law . . . . This is so whether the legal principles [the attorney] was applying were established by the law of Montgomery County, the State of Maryland, some other state of the United States, the United States of America, or a foreign nation.").

Id. at *12-13 (alterations in original).

The court ultimately denied the three Akin Gump lawyers' pro hac vice petitions.

The court then ended with an ominious paragraph.

Since the underlying conduct may potentially warrant action beyond the denial of pro hac vice admission, we also refer this matter to the Office of Disciplinary Counsel, the Board on Professional Responsibility, the Board on Unauthorized Practice of Law, and the Virgin Islands Attorney General for the purpose of taking any additional action which they may find appropriate with respect to the conduct of Eckard [Virgin Islands local counsel], Jindal, Roush, Schmitten [the three Akin Gump lawyers who had been admitted pro hac, but who had never taken the oath], Lowe, O'Brien [the two Virginia- based Akin Gump lawyers who had never applied for pro hac admission, but who had sought reimbursement for the

255

35951124_10 UPL, MDP and MJP (Defining What Lawyers Do and McGuireWoods LLP Where They Can Do It): Part II T. Spahn (1/1/19) Hypotheticals and Analyses

"legal services" they had performed on the case], and the Akin Gump law firm.

Id. at *13-14 (emphases added). The court added a footnote describing those entities'

possible steps, and the limited nature of their future role.

As this Court has previously explained, multiple entities may exercise concurrent jurisdiction to investigate and adjudicate unauthorized practice of law complaints, with different remedies beng available to those entities. For instance, the Board on Professional Responsibility may impose attorney discipline on Eckard and the three applicants for pro hac vice admission, whereas the Board on Unauthorized Practice of Law may obtain an order, enforceable by contempt, to direct that the unauthorized practice of law be ceased immediately. However, ultimately it is this Court that possesses the final word on what constitutes the unauthorized practice of law in the Virgin Islands, and whether the prohibition on unauthorized practice has been violated. Because we find that the prohibition on unauthorized practice has occurred in this case, any proceeding before the Board on Professional Responsibility or the Board on Unauthorized Practice of Law will be limited to the issue of remedy, and not the question of whether the prohibition on the unauthorized practice of law has been violated.

Id. at *14 n.6 (emphases added).

The more narrow the area of federal law, the more likely a court or bar is to

approve such practice by an out-of-state lawyer. Even with these precautions, however,

as a practical matter such a practice limited to certain topics (as opposed to certain

courts or boards) usually cannot work.

Fourth, most courts and bars analyzing this issue do not address the obvious issue facing a lawyer establishing a continuous presence in another state, but attempting to limit his or her practice to "federal" law. During any initial interview with a client, such a lawyer will have to essentially engage in a "triage" process -- separating

256

35951124_10 UPL, MDP and MJP (Defining What Lawyers Do and McGuireWoods LLP Where They Can Do It): Part II T. Spahn (1/1/19) Hypotheticals and Analyses

"federal" from non-"federal" law, and advising the new client of the lawyer's inability to advise on the second type of legal questions. Even that process is the "practice of law" under nearly every state's definition, and obviously does not involve purely "federal" law.

In 2010, the Oklahoma Supreme Court warned an Oklahoma lawyer that her practice of law in Texas involving bankruptcy law might violate the unauthorized practice of law rules in undertaking such a preliminary analysis of a client's position.

• In re Mooreland-Rucker, 237 P.3d 784, 790, 791 (Okla. 2010) (granting a petition for reinstatement to an Oklahoma lawyer, but warning her that her actions in handling bankruptcy matters in Texas (without being licensed there) could violate the UPL rules; "[T]he legal services which Petitioner rendered can hardly be deemed limited to federal law given the fact that bankruptcy law is inextricably intertwined with state law. . . . [T]he act of assessing the legal position of Texas debtors through analysis, interview, or explanation of legal rights and then representing only those clients whose cases squarely fit within the federal court's jurisdiction is no shield against Petitioner's unauthorized activities within Texas."; "When a debtor first seeks financial triage, a practitioner must explore all viable options since bankruptcy may not be in the best interest of the debtor. That assessment implicates numerous state law issues, including the strength of the creditors' claims and any defenses, other forms of debt relief, state foreclosure proceedings, and resolving the ultimate issue: to file or not to file, and the accompanying consequences. . . . Absent the knowledge of state law issues, a lawyer cannot competently advise the debtor and the lawyer may be influenced to tilt his advice toward filing for bankruptcy." (emphasis added); reinstating the lawyer's license, but providing a warning: "The Petitioner has advised, under oath, that she intends to resume the same activities. The Petitioner is warned that this conduct in the future would constitute the unauthorized practice of law and would warrant disciplinary action.").

Best Answer

The best answer to this hypothetical is NO (PROBABLY).

B 2/13

257

35951124_10 UPL, MDP and MJP (Defining What Lawyers Do and McGuireWoods LLP Where They Can Do It): Part II T. Spahn (1/1/19) Hypotheticals and Analyses

Lawyers Acting as Paralegals in a State Where They Are Not Licensed

Hypothetical 23

You have practiced for about 35 years in a midwestern state, but have spent an increasing amount of time each winter in a southern state with a friendlier winter climate. You realize that you cannot simply "hang out a shingle" in the southern state, but you wonder if you can stay busy (and earn some money) during the winter months by working part time as a paralegal at a law firm around the corner from your condominium.

Without violating the southern state's unauthorized practice of law rules, may you act as a paralegal in that state?

YES (PROBABLY)

Analysis

Lawyers hoping to engage in some law-related activity in another state clearly

face the temptation to simply call themselves paralegals rather than lawyers. The most

obvious reason why a lawyer might wish to engage only in paralegal services in another

state is to avoid having to join the bar of the state, and also avoid criminal prosecution

for the unauthorized practice of law.

Nothing in the ABA Model Rules would seem to prohibit such conduct, although

such lawyers have to carefully avoid holding themselves out as lawyers, and also have

to avoid actually practicing law.

Some states explicitly approve such an arrangement.

• Virginia UPL Op. 201 (1/22/01) ("Except as permitted under the cited rule, a foreign attorney, although admitted to and in good standing in the bar of his home jurisdiction, may not advise or prepare legal documents for a Virginia client in Virginia on matters involving Virginia law. The foreign attorney may give advice to and prepare legal instruments for a Virginia lawyer who may then decide whether such work product is acceptable for the client. UPL Op. 258

35951124_10 UPL, MDP and MJP (Defining What Lawyers Do and McGuireWoods LLP Where They Can Do It): Part II T. Spahn (1/1/19) Hypotheticals and Analyses

107 (1987). Therefore, a non-Virginia licensed attorney may provide legal services concerning Virginia law when directly supervised by a Virginia- licensed attorney if the attorney-client relationship remains between the Virginia attorney and the client." (footnote omitted) (emphasis added)).

Other jurisdictions are not as clear about the permissibility of such conduct.

Given the transitory nature of its lawyer population, it should come as no surprise that

the D.C. Bar has dealt most extensively with this issue. In one legal ethics opinion, the

D.C. Bar did not establish a per se prohibition on out-of-state lawyers acting as

paralegals in D.C., but recognized that "[w]hen a person is hired and billed as a

lawyer . . . the person is generally engaged in the practice of law, and is certainly being

held out as authorized or competent to practice law."1

1 District of Columbia UPL Op. 16-05 (6/17/05) (holding that "practicing law in the District of Columbia as a contract lawyer is no different than practicing law as a non-contract partner, associate or other employee. Unless the contract lawyer independently qualifies for one of the exceptions to Rule 49, the lawyer must be a member of the D.C. Bar."; explaining that a lawyer from other jurisdiction seeking to waive into the D.C. Bar must act within 90 days "after commencing practice in the District"; explaining that the lawyer rules do not apply to "paralegal work" such as document or privilege review; explaining that determining whether a contract lawyer performing such paralegal work must comply with the D.C. UPL Rules "depends on whether the person is being held out, and billed out, as a lawyer or as a paralegal. Rule 49 does not regulate the hiring of a person as a paralegal or a law clerk, even though the person may be admitted to the practice of law in another jurisdiction. When a person is hired and billed as a lawyer, however, the person is generally engaged in the practice of law, and is certainly being held out as authorized or competent to practice law. Clients would reasonably assume that the person held out as a contract lawyer performs functions that are different in degree, if not in kind, from those performed by paralegals or law clerks, and that the cost of services performed by contract lawyers reflects the legal training and judgment that they bring to the work they perform. When a client is paying for the services of a lawyer, and not a paralegal or a law clerk, the person providing the services and the person's employer must comply with Rule 49."; also explaining that "if a contract lawyer is supervised not as a paralegal or law clerk but as a subordinate attorney would be supervised, the contract lawyer is engaged in the practice of law"; also explaining that "Rule 49 also applies if a contract lawyer regularly takes short-term assignments in the District of Columbia. A contract lawyer, as defined for purposes of this Opinion, is employed for a limited period or project, and a contract lawyer may accept assignments outside the District of Columbia or have periods of non-employment between assignments. However, a contract lawyer whose presence in the District of Columbia as a contract lawyer is not occasional or incidental within the meaning of Rule 49(b)(3) is subject to Rule 49, even if each assignment, considered in isolation, might constitute only incidental or occasional presence here. For example, a contract lawyer who regularly practices in the District despite occasional projects in other jurisdictions is engaged in the practice of law, and holding out as authorized to practice law, 'in' the District of Columbia within the meaning of Rule 49(b)(3). Conversely, a contract lawyer who practices primarily in another jurisdiction generally is not engaging in the practice of law 'in' the District of Columbia simply by taking on one discrete project here on an incidental basis."). 259

35951124_10 UPL, MDP and MJP (Defining What Lawyers Do and McGuireWoods LLP Where They Can Do It): Part II T. Spahn (1/1/19) Hypotheticals and Analyses

Interestingly, the District of Columbia Bar pointed to the lawyer's billing rate as one factor.

Clients would reasonably assume that the person held out as a contract lawyer performs functions that are different in degree, if not in kind, from those performed by paralegals or law clerks, and that the cost of services performed by contract lawyers reflects the legal training and judgment that they bring to the work they perform. When a client is paying for the services of a lawyer, and not a paralegal or a law clerk, the person providing the services and the person's employer must comply with Rule 49.

District of Columbia UPL Op. 16-05 (6/17/05). The District of Columbia Bar also noted that the nature of the organizational structure might play a role in the analysis.

[I]f a contract lawyer is supervised not as a paralegal or law clerk but as a subordinate attorney would be supervised, the contract lawyer is engaged in the practice of law

Id.

If the lawyer might want to eventually be admitted by motion to another state's bar, the lawyer should be very careful before trying this approach. The lawyer will have to identify the time period in which he or she was acting purely as a paralegal. That time would not count toward the required duration of practice justifying the lawyer's admission by motion. The bar might also carefully scrutinize the lawyer's activities during that time, to assure the lawyer was not engaging in improper multijurisdictional practice.

Despite these other issues, lawyers who carefully limit their out-of-state activities to paralegal duties generally should not face any problems.

260

35951124_10 UPL, MDP and MJP (Defining What Lawyers Do and McGuireWoods LLP Where They Can Do It): Part II T. Spahn (1/1/19) Hypotheticals and Analyses

Best Answer

The best answer to this hypothetical is YES (PROBABLY).

B 2/13

261

35951124_10 UPL, MDP and MJP (Defining What Lawyers Do and McGuireWoods LLP Where They Can Do It): Part II T. Spahn (1/1/19) Hypotheticals and Analyses

In-House Lawyers Practicing in States Where They Are Not Licensed: General Rule

Hypothetical 24

You have spent most of your legal career practicing in Minnesota as an in-house lawyer for a company headquartered in Minneapolis. You are nearing retirement age, and want to spend the last few years of your career in a warmer climate. Your client has major operations in several southern states.

May you move to a southern state where you are not licensed, and continuously practice law without joining that state's bar -- if you limit your practice to representing your corporate client/employer?

YES

Analysis

Not surprisingly, the unauthorized practice of law analysis for in-house lawyers

differs dramatically from lawyers who draw their clients from the public.

Of course, states are free to insist that any lawyer practicing within its borders

pass the state's bar exam. As explained below, most states do not take such a narrow

view, but nothing stops in-house lawyers moving to another state from taking that state's

bar. That step frees the lawyers from the risk that they could no longer practice in the

state if their employment was terminated. It might also free them to engage in a wider practice.

The ABA Model Rules contain an explicit provision essentially permitting in-

house lawyers to establish a "systematic and continuous presence" in a jurisdiction

where they are not licensed.

A lawyer admitted in another United States jurisdiction, and not disbarred or suspended from practice in any jurisdiction, may provide legal services in this jurisdiction that . . . are 262

35951124_10 UPL, MDP and MJP (Defining What Lawyers Do and McGuireWoods LLP Where They Can Do It): Part II T. Spahn (1/1/19) Hypotheticals and Analyses

provided to the lawyer's employer or its organizational affiliates and are not services or which the forum requires pro hac vice admission.

ABA Model Rule 5.5(d)(1) (emphasis added). A comment provides additional guidance

Paragraph (d)(1) applies to a U.S. or foreign lawyer who is employed by a client to provide legal services to the client or its organizational affiliates, i.e., entities that control, are controlled by, or are under common control with the employer. This paragraph does not authorize the provision of personal legal services to the employer's officers or employees. The paragraph applies to in-house corporate lawyers, government lawyers and others who are employed to render legal services to the employer. The lawyer's ability to represent the employer outside the jurisdiction in which the lawyer is licensed generally serves the interests of the employer and does not create an unreasonable risk to the client and others because the employer is well situated to access the lawyer's qualifications and the quality of the lawyer's work. To further decrease any risk to the client, when advising on the domestic law of a United States jurisdiction or on the law of the United States. The foreign lawyer authorized to practice under paragraph (d)(1) of this Rule needs to base that advice on the advice of a lawyer licensed and authorized by the jurisdiction to provide it.

ABA Model Rule 5.5(d)(1) cmt. [16] (emphasis added).

The Restatement deals with in-house lawyers in a comment rather than in a black letter rule.

States have permitted practice within the jurisdiction by inside legal counsel for a corporation or similar organization, even if the lawyer is not locally admitted and even if the lawyer's work consists entirely of in-state activities, when all of the lawyer's work is for the employer- client . . . and does not involve appearance in court. Leniency is appropriate because the only concern is with the client-employer, who is presumably in a good position to assess the quality and fitness of the lawyer's work. In the course of such work, the lawyer must deal with outsiders, such as by negotiating with others in settling litigation or

263

35951124_10 UPL, MDP and MJP (Defining What Lawyers Do and McGuireWoods LLP Where They Can Do It): Part II T. Spahn (1/1/19) Hypotheticals and Analyses

directing the activities of lawyers who do enter an appearance for the organization in litigation.

Restatement (Third) of Law Governing Lawyers § 3 cmt. f (2000) (emphasis added).

Thus, the good news for in-house lawyers is that they generally may move to another state (at least a state which has adopted the ABA Model Rules approach) and establish a "systematic and continuous presence" in that state without obtaining a license from that state.

Some states generally follow this laissez-faire approach. However, even those states usually limit the scope of advice that such in-house lawyers may provide their corporate clients. For instance, Texas apparently does not even have a rule dealing with the permissibility of out-of-state lawyers practicing as in-house lawyers in Texas.

However, a Texas rule implicitly permits such activities -- by recognizing that time lawyers spend engaging in such activities can count toward the duration of practice requirement in Texas' admission by motion rule. That duration requirement rule limits the type of advice and activities that such in-house lawyers may give and undertake if they want to use that time toward the admission by motion requirement.

The following activities in Texas, without holding a Texas law license, shall be considered for purposes of meeting the practice requirements of Rule XIII(a), Rules Governing Admission to the Bar of Texas:

A. Employment in Texas as in-house or corporate counsel for a company or business (other than a law firm or business engaged in the practice of law) having offices in Texas if:

1. the individual so employed holds a valid, active law license issued by another state (including any territory of the United States, as well as the District of Columbia) and is not prohibited by the laws and rules of the issuing jurisdiction from the activities for which the individual is employed; and

264

35951124_10 UPL, MDP and MJP (Defining What Lawyers Do and McGuireWoods LLP Where They Can Do It): Part II T. Spahn (1/1/19) Hypotheticals and Analyses

2. the individual has not taken any of the following actions, agrees not to do so until licensed in Texas, and submits an affidavit so affirming:

a. appear for the corporation in Texas courts, either in person or by signing pleadings;

b. interpret Texas law or give any advice concerning Texas law for anyone other than the corporation;

c. participate in the Texas representation of any client other than the corporation, in any manner;

d. prepare any legal instrument affecting title to real property, including a deed, deed of trust, note, mortgage, or transfer or release of lien, as proscribed by Sec. 83.001, Tex. Gov. Code Ann.; or

e. render, to anyone except the corporation, any service requiring the use of legal skill or knowledge or perform any other act constituting the practice of law under Sec. 81.101, Tex. Gov. Code Ann.

Texas Bd. of Law Examn'rs, Policy Statement on Practice Requirements for Texas Rule

XIII.

The bad news is that most states do not take a laissez-faire approach. Instead, most states require some sort of registration and (not surprisingly) payment of a fee.

The ABA Model Rules recognize that

[i]f an employed lawyer establishes an office or other systematic presence in this jurisdiction for the purpose of rendering legal services to the employer, the lawyer may be subject to registration or other requirements, including assessments for client protection funds and mandatory continuing legal education.

ABA Model Rule 5.5(d)(1) cmt. [17] (emphasis added).

In 2008, the ABA adopted for the first time a Model Rule for Registration for In-

House Counsel. The ABA amended its Model Rule in early 2016. This Model Rule

265

35951124_10 UPL, MDP and MJP (Defining What Lawyers Do and McGuireWoods LLP Where They Can Do It): Part II T. Spahn (1/1/19) Hypotheticals and Analyses

requires in-house lawyers to register in any state where they are not licensed, but in which they are continuously practicing. In essence, the rule requires in-house counsel to register with the bar of the state in which they are practicing within 180 days of beginning to work in that state. Such in-house lawyers must complete an application form, and demonstrate that they are in good standing in every jurisdiction where they are admitted to practice, among other things.

This ABA Model Rule limits what such in-house lawyers can do without being fully licensed in the state where they are continuously practicing.

1. The registered lawyer is authorized to provide legal services to the entity client or its organizational affiliates, including entities that control, are controlled by, or are under common control with the employer, and for employees, officers and directors of such entities, but only on matters directly related to their work for the entity and only to the extent consistent with Rule 1.7 of the Model Rules of Professional Conduct [or equivalent provision in the jurisdiction]; and

2. The registered lawyer shall not:

a. Except as otherwise permitted by the rules of this jurisdiction, appear before a court or any other tribunal as defined in Rule 1.0(m) of the Model Rules of Professional Conduct [or jurisdictional equivalent], or

b. Offer or provide legal services or advice to any person other than as described in paragraph B.1., or hold himself or herself out as being authorized to practice law in this jurisdiction other than as described in paragraph B.1.

Am. Bar Ass'n Section of Legal Educ. & Admissions to the Bar, Report to the House of

Delegates, § B (amended Feb. 6, 2016).

266

35951124_10 UPL, MDP and MJP (Defining What Lawyers Do and McGuireWoods LLP Where They Can Do It): Part II T. Spahn (1/1/19) Hypotheticals and Analyses

Not surprisingly, under the ABA Model Rule such lawyers are subject to discipline by the bar of the state with which they are registered. The lawyer's ability to practice in the new state under this limited registration ends if:

1. The lawyer's employment terminates;

2. The lawyer is suspended or disbarred from practice in any jurisdiction or any court or agency before which the lawyer is admitted, U.S. or foreign; or

3. The lawyer fails to maintain active status in at least one jurisdiction, U.S. or foreign.

Am. Bar Ass'n Section of Legal Educ. & Admissions to the Bar, Report to the

House of Delegates, § F (Amended, Feb. 6, 2016).

After the 2008 promulgation of this ABA Model Rule, states continued moving in the direction of requiring in-house lawyers to establish some official relationship with the state's bar. In 2009, the ABA/BNA Lawyers' Manual on Professional Conduct reported that just fourteen states had such a process in 2003, but by February 2009 the number of states had grown to thirty-two. ABA/BNA Lawyers' Manual on Professional Conduct,

Current Reports, Feb. 18, 2009, at 93. The number of states continues to grow. The

ABA keeps up-to-date several useful charts of states' current in-house lawyer requirements.

States dealt with this issue at varying times and with varying approaches. Few adopted the ABA Model Rule provisions. As states gradually rolled out their registration programs, a number of high-profile in-house lawyers had to confront the fact that they had actually let all of their active licenses lapse, usually by failing to comply with CLE or other paperwork requirements. Of course, this meant that they had been practicing

267

35951124_10 UPL, MDP and MJP (Defining What Lawyers Do and McGuireWoods LLP Where They Can Do It): Part II T. Spahn (1/1/19) Hypotheticals and Analyses

without being authorized to practice law anywhere. A number of those in-house lawyers were sanctioned for having violated unauthorized practice of law statutes. Fortunately for the corporate clients that those in-house lawyers represent, no court seems to have stripped away attorney-client privilege protection for communications to and from these nonlawyers during the time that they were not authorized to practice law anywhere.

Ironically, New York state was among the last to adopt some registration provision. In 2009, the New York State Bar admitted that it really had no useful guidance for lawyers who were continuously practicing law in New York as in-house lawyers but who were not licensed in New York state.

• New York LEO 835 (12/24/09) (explaining that the new New York ethics rules do not provide any guidance for out-of-state lawyers from acting as in-house lawyers in New York "The question whether an out-of-state lawyer may serve as in-house counsel for a New York corporation and maintain an office in New York for that purpose is a question of law, and is not answered by the New York Rules of Professional Conduct. The question is therefore beyond our jurisdiction and we offer no opinion on the question. Because the question is a recurring one, however, this Committee urges the Appellate Divisions and/or the New York State Legislature to provide further guidance regarding whether and to what extent out-of-state lawyers - especially in-house lawyers who provide services solely to a corporate employer - are authorized to practice law in New York.").

In 2011, New York state finally adopted an in-house lawyer registration rule. The news reports described the change, and noted that as of that time 43 other states and the District of Columbia had adopted such procedures for in-house lawyers.

• Joel Stashenko, Courts Offer Special Registration to Out-of-State In-House Counsel, N.Y. L.J., Apr. 12, 2011 ("In-house attorneys who are not licensed to practice law in New York will nevertheless be able to offer legal advice in the state under new rules adopted by the Court of Appeals."; "The rules approved by the state's highest court, codified as 22 New York Codes, Rules and Regulations (NYCRR) Part 522 . . . , will allow out-of-state lawyers to provide counsel to private corporations, associations or other legal entities operating in New York, as well as to their employees, directors or officers.";

268

35951124_10 UPL, MDP and MJP (Defining What Lawyers Do and McGuireWoods LLP Where They Can Do It): Part II T. Spahn (1/1/19) Hypotheticals and Analyses

"The rules take effect on April 20."; "The out-of-state lawyers must be in good standing in their home states, United States territories or the District of Columbia to win registration within their Appellate Division department."; "The attorneys will not be allowed to provide legal services to the general public, appear before agencies or other tribunals, or provide pro bono services."; "The rules are intended to solidify New York's status as a center to commercial enterprise in the United States by making it easier for national and international companies to operate here, Chief Judge Jonathan Lippman said."; "'By accommodating the growing need for the provision of in-house services, the new registration rules give New York a competitive edge in attracting corporations and other entities that in the past may have been reticent to locate here because of concerns over the unauthorized practice of law,' Judge Lippman said in a statement."; "In addition, he said the rules should lead to New York lawyers being allowed to practice in other locales that offer reciprocity."; "Forty-three other states and the District of Columbia have similar rules, the Office of Court Administration said yesterday.").

As explained above, as of 2009, only 32 states had adopted such provisions.

New York's procedure follows the standard approach that most states have taken. Among other things, such lawyers have to demonstrate that they are in good standing in states where they are licensed to practice law. Of course, a large fee is involved as well.

Lawyers registered under this provision may

(a) provide legal services in this State only to the single employer entity or its organizational affiliates, including entities that control, are controlled by, or are under common control with the employer entity, and to employees, officers and directors of such entities, but only on matters directly related to the attorney's work for the employer entity, and to the extent consistent with the New York Rules of Professional Conduct;

(b) not make appearances in this State before a tribunal, as that term is defined in the New York Rules of Professional Conduct (22 NYCRR 1200.0 Rule 1.0[w]) or engage in any activity for which pro hac vice admission would be required if engaged in by an attorney who is not admitted to the practice of law in this State[;]

269

35951124_10 UPL, MDP and MJP (Defining What Lawyers Do and McGuireWoods LLP Where They Can Do It): Part II T. Spahn (1/1/19) Hypotheticals and Analyses

(c) not provide personal or individual legal services to any customers, shareholders, owners, partners, officers, employees or agents of the identified employer; and

(d) not hold oneself out as an attorney admitted to practice in this State except on the employer's letterhead with a limiting designation.

N.Y. Comp. Codes R. & Regs. tit. 22, pt. 522.4.

New York revised its in-house lawyer process in 2015 to allow foreign lawyers to register as in-house counsel.

States have taken varying positions on how welcoming they will be to out-of-state in-house lawyers practicing permanently in the state. Some states harshly punish lawyers for not properly registering when they move to the state.

• In re Gustafson, 986 N.E.2d 377, 379 (Mass. 2013) (suspending an in-house lawyer for six months for failure to properly register in Massachusetts and for ignoring the bar's administrative process; "The respondent's failure to cooperate in the disciplinary process, including her failure to respond to the petition for discipline, did not require imposition of more than a six-month suspension.")

Other states are more welcoming. Illinois even declared an amnesty in 2013.

• Marlisse Silver Sweeney, Illinois Declares Amnesty for In-House Lawyers, Corporate Counsel, Dec. 12, 2013 ("Illinois is declaring an amnesty for in- house counsel not licensed to practice there. Since 2004, Supreme Court Rule 716 has provided for a limited law license for out-of-staters wanting to practice in the state, according to Bethany Krajelis in The Madison-St. Clair Record. But only 828 limited law licenses have been granted under the rule - - the problem being that the Association of Corporate Counsel estimates there are over 9,600 in-house counsel in Illinois."; "'Given the disparity between the number of Rule 716 licensed lawyers and the total of in-house lawyers, it is likely that there are many who are neither Illinois-licensed or have a limited corporate license,' the Association told Krajelis. In a release, the court conjectured the reason why many counsel have failed to obtain these licenses is likely due to a lack of awareness, as many jurisdictions don't have similar rules."; "To help compliance, the court announced in-house counsel have a year to take advantage of the amnesty program and can apply for licensing without paying overdue fees or having to make up

270

35951124_10 UPL, MDP and MJP (Defining What Lawyers Do and McGuireWoods LLP Where They Can Do It): Part II T. Spahn (1/1/19) Hypotheticals and Analyses

continuing legal education credits. The failure to obtain the limited law license previously won't be taken against them or be grounds for attorney discipline, says Krajelis."; "To qualify for a limited law license, the counsel must be in good disciplinary standing, pass a background check, pay the annual registration fees, as well as submit to the authority of the Illinois Attorney Registration and Disciplinary Committee and meet continuing legal education requirements.")

The influential Association of Corporate Counsel has resisted states' registration requirement efforts.

• Sue Reisinger, Association of Corporate Counsel Challenges New Mexico In- House Counsel Rules, Corporate Counsel, July 31, 2013 ("The New Mexico Supreme Court shouldn’t be gambling with a legal client’s services, said the Association of Corporate Counsel (ACC) in a comment letter sent to Chief Justice Petra Jimenez Maes last week."; "The letter urges the court to pass new rules removing restrictive bar admission standards for experienced lawyers from outside the state, if the other state reciprocates. But the letter goes further than the proposed rules, arguing for an end to reciprocity requirements."; "Amar Sarwal, ACC’s vice president and chief legal strategist, issued a statement criticizing the current New Mexico rules."; "'Requiring experienced lawyers from other states to retake the bar exam in order to practice in New Mexico privileges the parochial interests of local lawyers, rather than those of the clients they ostensibly serve,' Sarwal said."; "'In fact, the proposal's reciprocity requirement also needlessly benefits lawyers alone, while depriving clients of their experienced counsel of choice,' he added."; "Currently in New Mexico, unlike a majority of United States jurisdictions, attorneys cannot gain admission to practice law by motion or under any form of reciprocity with other states. Likewise, New Mexico attorneys applying in other states are not allowed similar access to admission without passing the bar exam."; "'Choice of counsel of clients and prospective clients is unnecessarily restricted,' states the New Mexico [proposed revision]. 'The reality of transboundary law practice is not served by New Mexico’s lack of admission by motion.'"; "The proposal goes on to state that requiring all out-of-state attorneys to sit for the bar exam is 'a relic and outdated.'"; "But the ACC letter contends that reciprocity doesn’t fix the problem. 'Instead, reciprocity subjects the needs of clients to the whims of state-to-state diplomacy. A more open admission system serves clients better,' it says.")

• Sue Reisinger, Association of Corporate Counsel Objects to Washington State In-House Rule, Corporate Counsel, May 6, 2013 ("Washington State is considering a new rule to require in-house counsel there -- already licensed to practice in another state -- to register with the state bar. The Association

271

35951124_10 UPL, MDP and MJP (Defining What Lawyers Do and McGuireWoods LLP Where They Can Do It): Part II T. Spahn (1/1/19) Hypotheticals and Analyses

of Corporate Counsel (ACC), for one, doesn't like it one bit. 'It seems like a money grab,' complained Amar Sarwal, ACC's vice president and chief legal strategist. 'There's a pile of money in registration fees involved and hassles with bureaucratic paperwork, all for no good reason.' He conceded, however, that a majority of states have adopted some sort of registration requirement. But Sarwal said there has been no evidence that in-house counsel have caused any practice or ethical problems in Washington that would support changing the rule there. 'So if it's not broke, don't fix it,' he told CorpCounsel.com. The ACC has filed a comment letter lambasting the change with the state supreme court, which must approve any proposed rule change. Besides Sarwal, the letter is signed by Bradley Toney, president of ACC's Washington Chapter and assistant general counsel to United Online/Classmates Media Corporation; and Evan Schultz, senior counsel and director of advocacy at ACC. The letter insists the state's current authorization system for in-house counsel is working well. Under the current rule, which adheres to the American Bar Association's (ABA's) Model Rules of Professional Conduct, an in-house lawyer licensed and in good standing in another state can simply show up and start working at a Washington company."; "ACC also opposes another change that would prohibit in-house lawyers, once registered, from appearing in court unless they go through a 'cumbersome and expensive pro hac procedure.' The letter sites other states, like Virginia and Colorado, that allow registered in-house counsel to appear in state court on behalf of their company. But as long as the state is tinkering with its rules of practice, ACC urges it to expand its permission for in-house counsel to do pro bono work. The current and proposed rules both restrict in-house counsel 'from working with the full range of organized programs that support services to needy clients . . . [and] limit precious legal talent,' the letter says.")

Best Answer

The best answer to this hypothetical is YES.

B 2/13, 3/17

272

35951124_10 UPL, MDP and MJP (Defining What Lawyers Do and McGuireWoods LLP Where They Can Do It): Part II T. Spahn (1/1/19) Hypotheticals and Analyses

In-House Lawyers Practicing in States Where They Are Not Licensed: Limitations

Hypothetical 25

When you moved to one of your Minnesota client/employer's divisions in a southern state to escape the cold weather, you registered with that state's bar rather than take the bar exam. Now you have a number of specific questions.

(a) May you represent one of your client/employer's partially owned subsidiaries?

YES

(b) May you represent one of your client/employer's senior executives (helping him to negotiate a divorce decree) if you are not licensed in the state where you are practicing?

NO

(c) May you represent your client/employer in a state court hearing?

NO (WITHOUT PRO HAC VICE ADMISSION)

(d) May you participate in your local bar's pro bono program?

MAYBE

(e) If you decide to join your new home state's bar, may you count the time you spend in the southern state toward the required duration under that state's admission by motion rule?

MAYBE

Analysis

ABA Model Rule 5.5(d) allows in-house lawyers to establish a "systematic and continuous presence" in a jurisdiction in which they are not licensed -- but limits the clients to whom such in-house lawyers can provide services.

273

35951124_10 UPL, MDP and MJP (Defining What Lawyers Do and McGuireWoods LLP Where They Can Do It): Part II T. Spahn (1/1/19) Hypotheticals and Analyses

(a) ABA Model Rule 5.5(d)(1) allows out-of-state in-house lawyers to continuously practice in a state where they are not licensed, as long as their legal services

are provided to the lawyer's employer or its organizational affiliates; are not services for which the forum requires pro hac vice admission; and, when preformed by a foreign lawyer and requires advice on the law of this or another U.S. jurisdiction or of the United States, such advice shall be based upon the advice of a lawyer who is duly licensed and authorized by the jurisdiction to provide such advice.

ABA Model Rule 5.5(d)(1).

Thus, such in-house lawyers must limit their representation to their

client/employer and its "organizational affiliates." Comment [16] to that rule explains

that the term "organizational affiliates" means

entities that control, are controlled by, or are under common control with the employer.

ABA Model Rule 5.5 cmt. [16].

Similarly, the Restatement indicates that such in-house lawyers may provide

legal services to "all entities within the same organizational family." Restatement (Third)

of Law Governing Lawyers § 4 cmt. e (2000) (emphasis added).

Most states have adopted this basic limitation.

• D.C. Ct. App. R. 49 ("Unauthorized Practice of Law") (allowing "[i]nternal" counsel to practice law in D.C., but "only to one's regular employer"; the Commentary to Rule 49(c)(6) explains that the exception is based on "the confinement of the lawyer's professional services to activities internal to the employer").

• Florida Bar Reg. R. 17-1.2(b) (defining a "business organization" which an "authorized house counsel" can represent without a full license as "a corporation, partnership, association or other legal entity (taken together with its respective parents, subsidiaries, and affiliates) authorized to transact

274

35951124_10 UPL, MDP and MJP (Defining What Lawyers Do and McGuireWoods LLP Where They Can Do It): Part II T. Spahn (1/1/19) Hypotheticals and Analyses

business in this state that is not itself engaged in the practice of law or the rendering of legal services outside such organization, whether for a fee or otherwise, and does not charge or collect a fee for the representation or advice other than to entities comprising such organization by the activities of the authorized house counsel" (emphasis added)).

• Kentucky Sup. Ct. R. 2.111 (allowing in-house lawyers to represent their employers and "affiliated entities").

• Missouri Sup. Ct. R. 8.105 (permitting Missouri in-house counsel a "[l]imited [a]dmission" to the Missouri Bar, allowing them to represent corporate employers -- including their "subsidiaries or affiliates" (emphasis added)).

• New York 22 NYCRR § 522.1(a) (allowing in-house lawyers to represent "a non-governmental corporation, partnership, association, or other legal entity, including its subsidiaries and organizational affiliates.").

• South Carolina Sup. Ct. R. 405(a)(9) (providing for a limited certificate for in-house lawyers who represent corporations, as long as the in-house lawyers' work is provided "solely for the business employer or the parent or subsidiary of such employer" (emphasis added)).

• Washington Admis. to Prac. R. 8(f) (recognizing a "limited license" for in-house lawyers, which limits their practice to representing their corporate employer -- "including its subsidiaries and affiliates" (emphasis added)).

The ABA keeps up-to-date several useful charts of states current provisions.

(b) ABA Model Rule 5.5(d)(1) explicitly indicates that the general freedom of

in-house lawyers to practice in a state where they are not licensed

does not authorize the provision of personal legal services to the employer's officers or employees.

ABA Model Rule 5.5 cmt. [16] (emphasis added).

The reference to "personal" legal services presumably means that in-house

lawyers practicing in another state under the ABA Model Rules' fairly liberal MJP

provision may provide company-related legal services to a client/employer's employees.

Similarly, the ABA's 2008 Model Rule for Registration of In-House Counsel explains that such in-house lawyers can represent the client/employer and its 275

35951124_10 UPL, MDP and MJP (Defining What Lawyers Do and McGuireWoods LLP Where They Can Do It): Part II T. Spahn (1/1/19) Hypotheticals and Analyses

organizational affiliates, and also provide legal services to corporate directors, officers

and employees -- "but only on matters directly related to their work for the entity and

only to the extent consistent with" the conflicts rules. Am. Bar Ass'n Section of Legal

Educ. & Admissions to the Bar, Report to the House of Delegates, § B (Aug. 2008)

(emphasis added).

Thus, in-house lawyers practicing in a state where they are not licensed may not

represent company employees in personal matters. States naturally disagree about

defining the exact contours of that limitation.

(c) The ABA Model Rule permitting in-house lawyers to practice where they

are not licensed explicitly explains that such a practice can only include services that

"are not services for which the forum requires pro hac vice admission." ABA Model

Rule 5.5(d)(1).

Similarly, the Restatement permits the provision of legal services by in-house

lawyers in states where they are not licensed, as long as the services do "not involve

appearance in court." Restatement (Third) of Law Governing Lawyers § 3 cmt. f (2000).

The 2008 ABA Model Rule for Registration of In-House Counsel restricts such

in-house lawyers from appearing in the state's courts, other than permitted by court rules. Presumably, that would permit such in-house lawyers to appear pro hac vice. It

would be odd if such in-house lawyers would be prohibited from engaging in actions that

an out-of-state lawyer may undertake. Am. Bar Ass'n Section of Legal Educ. &

Admissions to the Bar, Report to the House of Delegates, Section B.2. (Aug. 2008)

(d) In-house lawyers can provide very valuable pro bono work. The ABA

Model Rule for Registration of In-House Counsel explicitly permits pro bono activity.

276

35951124_10 UPL, MDP and MJP (Defining What Lawyers Do and McGuireWoods LLP Where They Can Do It): Part II T. Spahn (1/1/19) Hypotheticals and Analyses

Notwithstanding the provisions of paragraph B above, a lawyer registered under this section is authorized to provide pro bono legal services through an established not-for-profit bar association, pro bono program or legal services program or through such organization(s) specifically authorized in this jurisdiction.

Am. Bar Ass'n Section of Legal Education & Admissions to the Bar, Report to the House of Delegates, § C (Aug. 2008).

The influential Association of Corporate Counsel urged states to allow in-house lawyers' pro bono work even if they are not fully licensed in a state.

• Letter from Ass'n of Corporate Counsel to Clerk of Supreme Court of Ariz. (May 21, 2013), http://www.acc.com/advocacy/upload/ACC-Arizona-pro- bono-letter-May-2013.pdf (urging Arizona to allow pro bono work by in-house lawyers registered in Arizona but not fully licensed there; "Arizona has a historic opportunity to recognize more fully the sophistication, the experience, and the capacity that all of Arizona's in-house lawyers have to help the enormous number of Arizona residents who need legal services but cannot afford to pay. The Supreme Court of Arizona is considering whether to permanently amend Supreme Court Rule 38(e) to make it easier for registered Arizona in-house lawyers whose law licenses come from elsewhere to provide pro bono assistance. On behalf of the Association of Corporate Counsel and our Arizona Chapter, we are writing to support the proposal. Indeed, we urge this Court to go even further in removing rules that hinder the ability of registered in-house counsel to provide pro bono legal services." (footnote omitted); "First, the proposal would still require affected in-house lawyers to work 'in association' with an approved legal services organization that employs an Arizona-licensed lawyer. . . . By requiring registered in-house counsel to work in association with an approved legal services organization, the proposal would wrongly imply that the covered in- house lawyers -- whose employees hire them because they are smart and effective and experienced -- are second-class counsel."; "Second, while the proposal is not clear on this, it does not explicitly allow lawyers practicing under Rule 38(e)(1)(B) to do away with the pro hac vice process when representing pro bono clients in the state courts."; "[W]e request that, in addition to the proposed changes to Rule 38(e)(3), this Court take the additional steps of eliminating the association and pro hac vice requirements.")

277

35951124_10 UPL, MDP and MJP (Defining What Lawyers Do and McGuireWoods LLP Where They Can Do It): Part II T. Spahn (1/1/19) Hypotheticals and Analyses

Commentators also encouraged states to permit such in-house lawyers to handle pro bono work.

• Esther Lardent, Do Our Ethics Rules Impair Access to Justice?, Nat'l L.J., May 30, 2013 ("In an era of lawyer mobility, advances in technology and the growth of multijurisdictional clients and practice, the current state-by-state admission and regulation of lawyers seems dated. While framed in the context of client protection, these rules seem far more grounded in lessening competition for clients. The fact that admission and practice requirements in many states limit not only commercial practice but also the ability to undertake pro bono at a time when it is desperately needed is particularly troubling. In the in-house context, for example, current rules in many jurisdictions are either silent or impose unnecessary limitations on the abililty of in-house lawyers licensed and in good standing in one state but working for their corporate client in another to undertake pro bono work. The problem is not limited to in-house counsel. Regional, national and global law firms and public interest groups handling multijurisdictional pro bono matters also experience difficulties. While pro bono vice is seen as a solution, it is a cumbersome and inefficient approach if our goal is to promote and grow pro bono service.")

• Veta T. Richardson, Empower In-House Counsel to Serve States that bar pro bono work by company lawyers admitted elsewhere should amend the rules to permit it, Nat'l L.J., May 13, 2013 ("The gap between the need for legal services and the services available for low-income individuals in the United States continues to widen, with more than 80 percent of the civil legal needs of low-income people going unmet."; "ACC urged the Illinois Supreme Court to open up the state's practice laws to eliminate unnecessary restrictions on the types of pro bono work that in-house lawyers with out-of-state licenses can engage in. On April 8, the court amended the rules. The expanded opportunities will allow in-house lawyers to show the importance of pro bono work to them. Colorado and Virginia amended their practice rules in 2006 and 2011, respectively, but without the unnecessary restrictions. Colorado allows and encourages certified in-house counsel to provide voluntary pro bono services to indigent persons and organizations serving them, subject to Colorado's professional-conduct rules. Virginia also allows and encourages registered in-house counsel to provide voluntary pro bono services, subject to professional-conduct and court rules. . . . The good news is that there is a budget-neutral action that states can take to begin to answer the unmet need: States with practice rules that serve as barriers to in-house counsel pro bono (such as California, Florida, Georgia, New Jersey, Ohio and the District of Columbia) should amend their rules to allow in-house counsel to represent, pro bono, underserved individuals and nonprofit organizations, free from unnecessary restrictions. Doing so will ensure that the private bar

278

35951124_10 UPL, MDP and MJP (Defining What Lawyers Do and McGuireWoods LLP Where They Can Do It): Part II T. Spahn (1/1/19) Hypotheticals and Analyses

is fully supporting the role it can play in addressing the crisis in legal services, and will also afford in-house counsel the opportunity to efficiently provide pro bono assistance to communities in need.")

Not surprisingly, most states encourage pro bono work generally, and have tried to find a way that in-house lawyers may engage in such worthwhile activity even if they are not fully licensed in the state.

On the other hand, the concept of a limited practice for such in-house lawyers focuses, among other things, on the lack of any danger that members of the public might be harmed by the in-house lawyer's lack of familiarity with the local law. This tension is reflected in states' approach to the issue.

Some states permit in-house lawyers practicing under a limited registration scheme to provide pro bono services. Other states have limited the ability of such in-house lawyers to engage in pro bono services.

For instance, until 2011 the Virginia Rules required such in-house lawyers engaging in pro bono work to work under a fully-admitted Virginia lawyer's supervision.

Not surprisingly, that restriction caused an outrage. On April 15, 2011, the Virginia

Supreme Court amended the provision, deleting the requirement that such in-house lawyers act under the direct supervision of a Virginia lawyer. Virginia Sup. Ct. R.

1A:5(g), (h).

New York state's approach to this issue also provides an interesting insight into some of these tensions. The New York State Bar Association recommended that such in-house lawyers be permitted to engage in pro bono services. However, the New York courts rejected that recommendation, so in-house lawyers practicing in New York under

279

35951124_10 UPL, MDP and MJP (Defining What Lawyers Do and McGuireWoods LLP Where They Can Do It): Part II T. Spahn (1/1/19) Hypotheticals and Analyses

the limited registration process may not provide pro bono services. N.Y.C.R.R. § 522.1

(3/25/11).

Not surprisingly, lawyers quickly but gently pushed back, encouraging the New

York Court of Appeals to allow such pro bono work.

• Christine Simmons, Pro Bono Rule on In-House Counsel Gains Support, N.Y. L.J., Nov. 5, 2013 ("In-house counsel groups and nearly 40 chief legal officers from corporate law departments are endorsing a proposed rule that would allow in-house counsel who are not licensed in New York to represent pro bono clients."; "Under the proposed rule, registered in-house counsel could appear before any tribunal or court in the state, without the need to seek pro hac vice admission, associate with a legal services provider or work under the supervision of a New York-licensed attorney. They would remain prohibited from making appearances other than in pro bono matters."; "The Association of Corporate Counsel [ACC], the ACC's three chapters in the state and 38 New York chief legal officers wrote a combined letter in support of the rule, which was proposed by the Advisory Committee on Pro Bono Service by In-House Counsel. Chief legal officers from Anheuser-Busch InBev; Cushman & Wakefield; JPMorgan Chase & Co.; MetLife Inc.; and Viacom were among those endorsing the change."; "In their public comment, the groups said current state rules prevent many corporate counsel from fully engaging in pro bono services."; "'New York's in-house attorneys are smart, experienced, responsible, and zealous, no matter where they received their law licenses,' the letter said. 'The pending amendment simply recognizes that all of New York's in-house lawyers should be able to serve pro bono clients with the same excellence that they already serve their employers.'")

Their efforts eventually bore fruit, and the New York Court of Appeals amended the in-house registration rule to allow such pro bono work.

• Tania Karas, New Rule to Expand Roster of In-House Pro Bono Lawyers, N.Y. L.J., Dec. 3, 2013, at 1 ("In-house counsel for New York corporations who are not licensed to practice in the state will now be permitted to represent clients pro bono, Chief Judge Jonathan Lippman announced Monday. The rule change will take effect tomorrow."; "The addition of in- house lawyers to the pro bono roster is the latest in a series of initiatives by the chief judge to help bridge the 'justice gap' in New York by delivering lowcost [sic] or free civil legal services to those who can't afford an attorney. Only about 20 percent of New York residents' civil legal services needs were met last year, Lippman said."; "An advisory committee on pro bono services by in-house counsel, which is chaired by Court of Appeals Judge Victoria

280

35951124_10 UPL, MDP and MJP (Defining What Lawyers Do and McGuireWoods LLP Where They Can Do It): Part II T. Spahn (1/1/19) Hypotheticals and Analyses

Graffeo, proposed the reforms . . . . The new rule was approved on November 15."; "Though there is no figure on how many additional lawyers will sign up for pro bono, Lippman put the number in the 'thousands.' Between 5,000 and 10,000 in-house counsel are based in New York, Lippman said, and many have been unable to perform pro bono work until now."; "New York is the fourth state to lessen its restrictions on in-house counsel taking on pro bono clients, following Colorado, Virginia and Illinois.")

The District of Columbia courts followed suit a year later.

• Zoe Tillman, D.C. Expands Pro Bono Practice Rule to In-House Lawyers, Legal Times, July 17, 2014 ("Legal services lawyers in D.C. are hoping a recent change in the local practice rules will bump up pro bono involvement by corporate in-house lawyers."; "On Wednesday, the D.C. Court of Appeals adopted a rule allowing in-house lawyers in Washington who aren't members of the D.C. Bar to perform pro bono work. Those in-house lawyers must be a member in 'good standing' with the highest court of another state or territory; have no history of suspension or being disbarred; work under the supervision of an active D.C. Bar member; and be assigned or referred by a local legal- services group."; "'It allows us to bring in a new category of lawyers to help low-income people in D.C. who can't afford a lawyer,' said Jim Sandman, president of the Legal Services Corporation and chairman of the D.C. Bar Pro Bono Committee."; "There are an estimated 500 in-house lawyers working in Washington, according to proponents of the new rule. It's unknown how many of those attorneys are already members of the D.C. Bar, but Sandman said he believed many, if not a majority, are not."; "The new rule mirrors a similar pro bono exception carved out for lawyers working for the federal government who aren't members of the D.C. Bar. D.C. Court of Appeals Chief Judge Eric Washington said in an email Thursday that the purpose of the new rule was to allow local in-house counsel to join with other D.C. attorneys 'to help close the justice gap here in Washington.'"; "Lawyers understand that the practice of law is a privilege, not a right, and that with that privilege comes responsibility,' Washington said. 'Those responsibilities include working to improve access to justice for all.'"; "Sandman said getting more corporate lawyers involved in pro bono would also spur law firms to step up. Corporate legal departments 'have a leveraging effect on the private bar,' he said. 'Law firms pay attention to what clients care about.'"; "There are at least 20 states with rules explicitly allowing nonlocally barred in-house lawyers to do pro bono work, according to a survey this year by Corporate Pro Bono, a partnership between the Pro Bono Institute and Association of Corporate Counsel."; "States vary in how broad they make practice exceptions for in-house lawyers who aren't members of the local bar. The pro bono rule adopted in D.C. places more restrictions than some states that don't require supervision by a bar member or an affiliation with a legal-services organization."; "Amar Sarwal, vice president and chief legal

281

35951124_10 UPL, MDP and MJP (Defining What Lawyers Do and McGuireWoods LLP Where They Can Do It): Part II T. Spahn (1/1/19) Hypotheticals and Analyses

strategist of the Association of Corporate Counsel, said there's been a push nationwide over the past six years to expand pro bono practice rules to in- house lawyers.")

The ABA's and others' charts available on the Internet can provide useful guidance on this issue.

(e) In 2012 the ABA amended its Model Rules for Admission by Motion --

allowing in-house lawyers to count toward the required practice duration any time that

the in-house lawyer spends practicing in a state even without a full license there. Am.

Bar Ass'n Resolution 105E, Amending ABA Model Rule on Admission by Motion (Aug.

2012).

As in other areas, states take different approaches. For instance, Texas counts

such time toward its admission by motion eligibility. Texas Board of Examiners Rule

XIII. In contrast, many states do not allow in-house lawyers to automatically apply time

spent in the state (under the provision allowing in-house lawyers to practice without a

full license) toward the time required for being admitted by motion. Virginia takes this

approach. The 2011 New York State regulation contains the same limitation. 22

N.Y.C.R.R. § 522.1 (3/25/11).

Best Answer

The best answer to (a) is YES; the best answer to (b) is NO; the best answer to

(c) is NO (WITHOUT PRO HAC VICE ADMISSION); the best answer to (d) is MAYBE;

the best answer to (e) is MAYBE.

B 2/13, 3/17

282

35951124_10 UPL, MDP and MJP (Defining What Lawyers Do and McGuireWoods LLP Where They Can Do It): Part II T. Spahn (1/1/19) Hypotheticals and Analyses

Foreign Lawyers

Hypothetical 26

You have relatives in Ireland, and have always been particularly close with a cousin who became an Irish lawyer. After spending some time with you and your husband in sunny California, your cousin has been exploring the possibility of moving to America to practice law here.

(a) May your Irish cousin lawyer move to the United States as an employee of an Irish company and provide guidance on Irish law to the Irish-based client?

YES (PROBABLY)

(b) May your Irish cousin lawyer move to the United States and open an office – limiting his practice to Irish law?

YES (PROBABLY)

(c) May your Irish cousin lawyer appear pro hac in United States courts?

MAYBE

(d) May your Irish cousin lawyer take a state's bar exam with the hopes of continuously practicing in that state as a fully licensed lawyer?

MAYBE

(e) May your Irish cousin lawyer join a state's bar through admission by motion?

MAYBE

Analysis

The increasing globalization of legal issues has forced states to confront the possibility that foreign lawyers might want to relocate to the United States and assist foreign companies (and perhaps others).

283

35951124_10 UPL, MDP and MJP (Defining What Lawyers Do and McGuireWoods LLP Where They Can Do It): Part II T. Spahn (1/1/19) Hypotheticals and Analyses

United States Lawyers' Ability to Practice Law in Foreign Countries

Despite efforts by negotiators in GATS ("The General Agreement on Trade in

Services") trade talks, countries have not begun to agree on how they should address

foreign lawyers' ability to practice in a country where they are not licensed.

Every country essentially takes the same approach, and most countries are hostile to foreign lawyers practicing within their borders.

• Jessica Seah, In Vietnam, a Push for Restrictions on Foreign Firms, Asian Lawyer, Nov. 12, 2012 ("Vietnam's top law firms are asking their government to impose greater restrictions on the practices of their foreign counterparts."; "The country is currently one of the most open markets in Asia for foreign firms. They are free to hire Vietnamese lawyers, who have been permitted to advise on local law short of appearing in court or signing official documents. In many other Asian markets, including China, Korea, and India, foreign firms are explicitly barred from practicing local law."; "International firms with significant practices in Vietnam include Baker & McKenzie, Mayer Brown JSM, Freshfields Bruckhaus Deringer, Hogan Lovells, Allens, and Duane Morris. The market has also been a destination for law firms hailing from elsewhere in Asia, like Japan's Nishimura & Asahi, Korea's Yulchon, and Singapore's Rajah & Tann."; "'In my view, the legal market for foreign lawyers in Vietnam is more open and liberalized than that of the United States,' says Tung Ngo, chairman of 50-lawyer Vietnamese firm Vilaf, noting that foreign graduates of United States law schools are only allowed to take bar exams in a few states. 'So I think it is understandable and legitimate if local lawyers raise these concerns to the lawmakers to protect our interest.'"; "Vilaf was one of 18 Vietnamese firms that submitted a letter last month to the Vietnamese Ministry of Justice urging revision of the country's Lawyers Law to more strictly require foreign firms. The group argued that the 'open door' policy toward foreign lawyers had hindered 'the formation of a healthy and equal legal environment for the development of a force of Vietnamese lawyers who would be dynamic and professional, and who would possess qualifications tantamount to those of the lawyers in the region and in the world.'").

• Foreign Law Firms Cannot Establish Liaison Offices in India, High Court Says, 28 ABA/BNA Law. Manual on Prof. Conduct [Current Reports] 447 (July 18, 2012) ("Mumbai—Foreign law firms currently are not permitted to open liaison offices in India and only 'advocates' falling under the purview of the Advocates Act of 1961 are allowed to practice law in the country, the India Supreme Court declared in July 4 in an interim order (Bar Council of

284

35951124_10 UPL, MDP and MJP (Defining What Lawyers Do and McGuireWoods LLP Where They Can Do It): Part II T. Spahn (1/1/19) Hypotheticals and Analyses

India v. Balaji, India, No. 17150-17154/2012, 7/4/12, on appeal of 28 Law. Man. Prof. Conduct 133)."; "The court acted on a special leave petition filed by the Bar Council of India that appealed an order of the Madras High Court. That order stated that foreign-licensed lawyers are permitted to 'fly in and fly out' of India to advise clients on foreign law on a case-to-case basis. See 28 Law. Man. Prof. Conduct 133.").

• Ben Lewis, Foreign Lawyers 'Can Fly In, Fly Out' of India, Court Rules, The Asian Lawyer, Feb. 22, 2012 ("The Madras High court in Chennai, India ruled Tuesday that foreign law firms are able to advise on international law and take part in international arbitration while in that country."; "In its ruling, the court rejected a petition filed in March 2010 by a local lawyer, A.K. Balaji, who accused 31 foreign law firms and a legal process outsourcing company of illegally practicing law in violation of India's Advocates Act, which governs the country's legal profession."; "Foreign law firms have long been prohibited from opening offices in India, advising on Indian law, or appearing in Indian courts, but Balaji's complaint -- which he brought on behalf of a group of local lawyers called the Association of Indian Lawyers -- targeted activities that have become routine among international law firms such as meeting with clients in hotels, hosting seminars, and participating in arbitration in India."; "Tuesday's ruling by Judges M.Y. Eqbal and T.S. Sivagnanam said foreign lawyers are entitled to participate in international arbitration proceedings in that country and advise clients on foreign law on a 'fly in fly out' basis."; "In their decision, the judges noted that many Indian companies take part in transactions that require advice on foreign laws, which Indian lawyers are unable to provide. A ruling in the petitioners' favor, they said, would result in a 'manifestly absurd situation wherein only Indian citizens with Indian Law degree who are enrolled as an advocate under the Advocates Act could practice foreign law."; "The court did reiterate that foreign firms are barred from advising on Indian law under the Advocates Act as well as Indian Bar rules.").

Foreign Lawyers' Ability to Practice Law in the United States

Although many United States lawyers have been frustrated by other countries' unwelcoming approach to American lawyers continuously or even temporarily practicing overseas, the United States has not been particularly hospitable either.

The ABA Ethics 20/20 Commission explicitly focused on globalization as one of the key changes in the legal profession that justified updates in the ABA Model Rules

285

35951124_10 UPL, MDP and MJP (Defining What Lawyers Do and McGuireWoods LLP Where They Can Do It): Part II T. Spahn (1/1/19) Hypotheticals and Analyses

and related provisions. The Commission's February 2013 Introduction and Overview

discussed this focus.

One important practical effect of globalization is that clients regularly expect lawyers in firms of all sizes to handle matters that involve multiple jurisdictions, domestic and international. For example, many U.S.-based businesses operate in numerous jurisdictions and have legal problems that span several countries and continents. Similarly, many foreign-owned companies have substantial business and legal interests that involve numerous jurisdictions, including in the U.S.

Related demographic shifts have increased the number of individuals in the U.S. who have legal needs that cross national lines. In 2000, the foreign-born population in the U.S. was 31,107,899. By 2009, the total U.S. foreign-born population had risen to 36,750,000, approximately 12% of the U.S. population. These foreign-born residents have family law, estate planning, and business relationships in their countries of origin or the countries of origin of their spouses or business associates.

Lawyers can no longer assume that their practices will be contained by national borders. Lawyers in all practice settings, from solo practitioners handling divorces for foreign-born clients to law firms handling international business transactions, may encounter work that involves or relates to the law of other nations.

Am. Bar Ass'n Comm'n on Ethics 20/20, Introduction and Overview, at 3 (Feb. 2013).

(a) The ABA Model Rules allow a foreign lawyer to provide foreign law advice to a United States-based employer in the United States.

A lawyer admitted in another United States jurisdiction or in a foreign jurisdiction, and not disbarred or suspended from practice in any jurisdiction or the equivalent thereof, may provide legal services through an office or other systematic and continuous presence in this jurisdiction that: (1) are provided to the lawyer's employer or its organizational affiliates; are not services for which the forum requires pro hac vice admission; and, when performed by a foreign

286

35951124_10 UPL, MDP and MJP (Defining What Lawyers Do and McGuireWoods LLP Where They Can Do It): Part II T. Spahn (1/1/19) Hypotheticals and Analyses

lawyer and requires advice on the law of this or another jurisdiction or of the United States, such advice shall be based upon the advice of a lawyer who is duly licensed and authorized by the jurisdiction to provide such advice.

(e) For purposes or paragraph (d), the foreign lawyer must be a member in good standing of a recognized legal profession in a foreign jurisdiction, the members of which are admitted to practice as lawyers or counselors at law or the equivalent, and are subject to effective regulation and discipline by a duty constituted professional body or a public authority.

ABA Model Rule 5.5(d)(1), (e) (emphasis added). Three comments provide guidance.

[15] Paragraph (d) identifies two circumstances in which a lawyer who is admitted to practice in another United States or a foreign jurisdiction, and is not disbarred or suspended from practice in any jurisdiction, or the equivalent thereof, may establish an office or other systematic and continuous presence in this jurisdiction for the practice of law. Pursuant to paragraph (c) of this Rule, a lawyer admitted in any U.S. jurisdiction may also provide legal services in this jurisdiction on a temporary basis. See also Model Rule on Temporary Practice by Foreign Lawyers. Except as provided in paragraphs (d)(1) and (d)(2), a lawyer who is admitted to practice law in another United States or foreign jurisdiction and who establishes an office or other systematic or continuous presence in this jurisdiction must become admitted to practice law generally in this jurisdiction (emphasis added).

[16] Paragraph (d)(1) applies to a U.S. or foreign lawyer who is employed by a client to provide legal services to the client or its organizational affiliates, i.e., entities that control, are controlled by, or are under common control with the employer. This paragraph does not authorize the provision of personal legal services to the employer's officers or employees. The paragraph applies to in-house corporate lawyers, government lawyers and others who are employed to render legal services to the employer. The lawyer's ability to represent the employer outside the jurisdiction in which the lawyer is licensed generally serves the interests of the employer and does not create an unreasonable risk to the client and others because the employer is well situated to

287

35951124_10 UPL, MDP and MJP (Defining What Lawyers Do and McGuireWoods LLP Where They Can Do It): Part II T. Spahn (1/1/19) Hypotheticals and Analyses

assess the lawyer's qualifications and the quality of the lawyer's work. To further decrease any risk to the client, when advising on the domestic law of a United States jurisdiction or on the law of the United States, the foreign lawyer authorized to practice under paragraph (d)(1) of this Rule needs to base that advice on the advice of a lawyer licensed and authorized by the jurisdiction to provide it (emphasis added).

[17] If an employed lawyer establishes an office or other systematic presence in this jurisdiction for the purpose of rendering legal services to the employer, the lawyer may be subject to registration or other requirements, including assessments for client protection funds and mandatory continuing legal education. See Model Rule for Registration of In-House Counsel.

ABA Model Rule 5.5 cmt. [15], [16], [17].

Many companies and commentators have urged states to allow such limited practice by foreign lawyers located in the United States.

• Melissa Maleske, Why GCs Are Fighting To Break Barriers For Foreign Lawyers, Law360, July 28, 2015 ("Foreign in-house counsel are vital to a global legal department, but in most states, those lawyers are still unable to jet into United States offices to lend their local expertise to cross-border matters -- a rule some general counsels are fighting hard to change."; "Foreign in-house counsel have long been unable to practice law in most United States legal departments without contending with often difficult and complicated requirements, such as qualifying for and passing state bar exams. However, several states have begun to tweak rules of professional conduct to allow foreign corporate counsel to practice in United States offices, allowing stateside legal departments to benefit from a global perspective."; "By 2011, seven states had adopted amendments allowing for the cross-border movement of in-house counsel, and currently 15 states and the District of Columbia have such rules on the books. Iowa is the most recent state to do so, putting amendments into effect July 1."; "And seven more states are now actively considering whether to adopt amendments allowing the practice of foreign in-house counsel, according to Carol Needham, a Saint Louis University law professor. One is Illinois, which could soon become the seventeenth jurisdiction to adopt explicit rules allowing companies to send lawyers there in response to the legal needs and training demands of increasingly global legal operations. Utah is the only state to have rejected such amendments after studying them.").

288

35951124_10 UPL, MDP and MJP (Defining What Lawyers Do and McGuireWoods LLP Where They Can Do It): Part II T. Spahn (1/1/19) Hypotheticals and Analyses

Many states have moved in this direction, usually with state-specific regulations.

• New Jersey Supreme Court, Administrative Determinations on the Report and Recommendations of the Special Committee on Attorney Ethics and Admissions (4/14/16) ("The ABA proposed permitting foreign (licensed outside the United States) lawyers to provide legal advice on foreign law as in-house counsel. The ABA proposal further would permit foreign lawyers to advise their employer company on United States law if the foreign lawyer has consulted with a United States lawyer. The Special Committee recommended the portion of the rule permitting foreign lawyers to advise their employer on foreign law but rejected the proposal that a foreign lawyer should be permitted to advise the employer on United States law."; "The Supreme Court decided to amend the Court Rules to permit foreign lawyers to practice law as in-house counsel to an employer company from an office in New Jersey, with the restriction that the foreign lawyer can only advise his or her employer on foreign law and not on United States law.").

(b) There has been similar state movement towards allowing what normally are

called "foreign legal consultants." These are foreign lawyers not working in-house who

locate permanently to the United States and provide limited advice about foreign law.

The ABA Model Rules recognize such a role.

(d) A lawyer admitted in another United States jurisdiction or in a foreign jurisdiction, and not disbarred or suspended from practice in any jurisdiction or the equivalent thereof, may provide legal services through an office or other systematic and continuous presence in this jurisdiction that: (2) are services that the lawyer is authorized by federal or other law or rule to provide in this jurisdiction.

(e) For purposes or paragraph (d), the foreign lawyer must be a member in good standing of a recognized legal profession in a foreign jurisdiction, the members of which are admitted to practice as lawyers or counselors at law or the equivalent, and are subject to effective regulation and discipline by a duty constituted professional body or a public authority.

ABA Model Rule 5.5(d)(2), (e) (emphasis added). A comment provides some guidance, and refers to another ABA Model Rule.

289

35951124_10 UPL, MDP and MJP (Defining What Lawyers Do and McGuireWoods LLP Where They Can Do It): Part II T. Spahn (1/1/19) Hypotheticals and Analyses

[18] Paragraph (d)(2) recognizes that a U.S. or foreign lawyer may provide legal services in a jurisdiction in which the lawyer is not licensed when authorized to do so by federal or other law, which includes statute, court rule, executive regulation or judicial precedent. See, e.g., Model Rule on Practice Pending Admission.

ABA Model Rule 5.5 cmt. [18] (emphasis added).

The ABA has provided additional guidance on such foreign legal consultants.

In 2006, the ABA adopted an amended Model Rule for the Licensing and

Practice of Foreign Legal Consultants.

That Model Rule provided that states may exercise their discretion to allow the practice of law in the state by a foreign legal consultant who

(a) is, and for at least five years has been, a member in good standing of a recognized legal profession in a foreign country, the members of which are admitted to practice as lawyers or counselors at law or the equivalent and are subject to effective regulation and discipline by a duly constituted professional body or a public authority

(b) for at least five years preceding his or her application has been a member in good standing of such legal profession and has been lawfully engaged in the practice of law in the foreign country or elsewhere substantially involving or relating to the rendering of advice or the provision of legal services concerning the law of the foreign country;

(c) possesses the good moral character and general fitness requisite for a member of the bar of this jurisdiction; and

(d) intends to practice as a foreign legal consultant in this jurisdiction and to maintain an office in this jurisdiction for that purpose.

ABA Model Rule for the Licensing and Practice of Foreign Legal Consultants § 1 (Aug.

2006). 290

35951124_10 UPL, MDP and MJP (Defining What Lawyers Do and McGuireWoods LLP Where They Can Do It): Part II T. Spahn (1/1/19) Hypotheticals and Analyses

Not surprisingly, applicants for such a license had to supply various certificates

and letters of recommendation demonstrating their good standing status in their home

jurisdiction, educational and professional qualifications, etc. They also had to subject

themselves to the discipline of the state in which they were practicing, and pay an annual fee.

Under this Model Rule, such foreign legal consultants could not hold themselves out as members of a state's bar, or even consider themselves "admitted" in the jurisdiction.1 Foreign legal consultants who violated this or any other provision risked losing their status.

Significantly, such foreign legal consultants could provide only a very limited

range of legal services in a jurisdiction. Such foreign legal consultants could not:

(a) appear as a lawyer on behalf of another person in any court, or before any magistrate or other judicial officer, in this jurisdiction (except when admitted pro hac vice pursuant to [citation of applicable rule]);

(b) prepare any instrument effecting the transfer or registration of title to real estate located in the United States of America;

(c) prepare:

(i) any will or trust instrument effecting the disposition on death of any property located and owned by a resident of the United States of America, or

(ii) any instrument relating to the administration of a decedent's estate in the United States of America;

1 This ABA Model Rule included an optional provision allowing such foreign legal consultants approved for practice in another United States jurisdiction to temporarily practice in the jurisdiction adopting the ABA approach -- essentially treating them as admitted United States lawyers for that temporary practice purpose. 291

35951124_10 UPL, MDP and MJP (Defining What Lawyers Do and McGuireWoods LLP Where They Can Do It): Part II T. Spahn (1/1/19) Hypotheticals and Analyses

(d) prepare any instrument in respect of the marital or parental relations, rights or duties of a resident of the United States of America, or the custody or care of the children of such a resident;

(e) render professional legal advice on the law of this jurisdiction or of the United States of America (whether rendered incident to the preparation of legal instruments or otherwise) except on the basis of advice from a person duly qualified and entitled (other than by virtue of having been licensed under this Rule) to render professional legal advice in this jurisdiction;

Id. § 3.

States allowing foreign legal consultants have occasionally found that such consultants have improperly exceeded the limited type of practice they were permitted to engage in.

• Report of Tri-County Hearing Panel #16, at 3, 4, 5, Grievance Adm'r v. Anjorin, ADB Case No. 11-119-GA (Mich. Attorney Grievance Bd. Feb. 17, 2012), http://www.adbmich.org/coveo/boardorders/ 2012-05-23-11bo-119.pdf ("In January 2008, respondent specifically made application to the Michigan Supreme Court - Board of Law Examiners for certification as a 'Special Legal Consultant' which designation did not require that he have attended an ABA approved law school or demonstrated his legal knowledge by having taken a Bar examination in the U.S. He was advised that certification as a 'Special Legal Consultant' would permit him to use the title 'Special Legal Consultant' and render professional legal advice on the law of Nigeria (Petitioner's Exhibit A), and would permit him to seek active membership in the State Bar of Michigan pursuant to the State Bar of Michigan Rule 3. The evidence reflects that the Michigan Supreme Court - Board of Law Examiners' Rule 5(B)(d) did not permit a 'Special Legal Consultant' to appear in Michigan State or Federal Courts and limited his authority to advising clients on the law of Nigeria."; "Respondent subsequently made application for admission to the State Bar of Michigan and was assigned P number P 71514 (later changed to PL 1047)."; "On October 13, 2011, the Grievance Administrator filed Formal Compliant 11-119-GA against respondent alleging that, contrary to the limited purpose for which respondent had been admitted to the State Bar of Michigan, he had undertaken to represent a plaintiff in a negligence and breach of contract action pending in Wayne County Circuit Court . . . ."; "Respondent explained

292

35951124_10 UPL, MDP and MJP (Defining What Lawyers Do and McGuireWoods LLP Where They Can Do It): Part II T. Spahn (1/1/19) Hypotheticals and Analyses

his representation in the Anumba matter and other matters on the basis that he limited his practice of law in the State of Michigan to those matters he had involved himself in Nigeria and where the laws of Nigeria were applicable to U.S. Courts. . . . He indicated that the Anumba matter involved issues of negligence and breach of contract and the law as to each were the same or similar of that in Nigeria. However, he further indicated that he had never been given any reason to believe that he could practice law so long as the laws of Michigan and the laws of Nigeria were identical or similar."; "Respondent's testimony and other evidence reflects that respondent had engaged in the general practice of law for some time and had not limited his legal representations to matters where the law of Michigan and Nigeria were same or similar."; "That he had a website under the name of Gbenga Anjorin, PC in which he held himself out as offering legal services for traffic matters, drunk driving, criminal law practice, divorce, child support, child custody, immigration, negligence and international law."; "That he had appeared before several authorities in the State of Michigan, both in civil and criminal matters."; "Though he had not done a DUI in Nigeria inasmuch as there was no DUI in Nigeria, there was no doubt that in the District Court in the County of Wayne he was one of the leading DUI lawyers."; "As to the unauthorized practice of law, he admitted that he was doing something he had never done before, namely DUI."; "That he knew he was not authorized to practice DUI law."; "Though he admitted that DUI practice was unauthorized, it was unfortunately what he did best."; "That he had been counsel in a thousand traffic matters."; "Though in violation, he had appeared in many cases in Michigan and won a lot of the cases."; "That he was still representing clients in district and circuit courts after August of 2011."; "The panel has considered the recommended sanctions contained in the ABA Standards for Imposing Lawyer Sanctions and holds that Respondent's membership in the State Bar of Michigan should be revoked and his certification as a 'Special Legal Consultant' be revoked for reasons of the misconduct set forth herein.").

• Joe Swickard, Lawyer Had Lots of Cases, No Michigan Law License, , Apr. 10, 2012, at A1 ("He's gotten dozens of court-assigned cases, handled hundreds of DUI cases, appeared in civil suits and even monitored the federal terrorism trial of Umar Farouk Abdulmutllab, often called the Underwear Bomber, who tried to blow up an airliner headed to Detroit on Dec. 25, 2009."; "And he did it all without a Michigan law license."; "Anjorin was authorized under a special license from the State Bar of Michigan to give legal advice about Nigerian law. The limited license as a special legal consultant included what is called a 'P number,' the five-digit identifier fully licensed Michigan lawyers all have."; "Occupying a little-known niche, there have been just 14 special legal consultants in Michigan since 1988. And last year, Anjorin was one of four in the state."; "The limited license authorizes lawyers with foreign accreditation to give legal advice about their homeland to clients in Michigan. It does not make them lawyers

293

35951124_10 UPL, MDP and MJP (Defining What Lawyers Do and McGuireWoods LLP Where They Can Do It): Part II T. Spahn (1/1/19) Hypotheticals and Analyses

approved to practice in Michigan courts -- a limitation that was not part of the bar association online listing."; "Anjorin admitted he handled many criminal cases, and by his own estimation he took 'like a thousand' traffic cases."; "'I've prosecuted my duties very diligently,' he told the hearing panel. 'There is no doubt of the fact that in the district court of the Wayne County, I'm one of the leading DUI lawyers.'"; "He said he strayed from his own guidelines for taking only Michigan cases that paralleled Nigerian law because there is no DUI law in Nigeria.").

• In re Antoine, 899 N.Y.S.2d 41, 44 (N.Y. App. Div. 2010) (stripping a Haitian lawyer of the right to practice as a foreign legal consultant in New York State, because the lawyer had incorrectly described himself as licensed to practice in New York; "The record supports sustaining charges one, two and six as found by the Hearing Panel. Respondent intentionally misrepresented his license to practice as a legal consultant on various applications and forms submitted to courts by not qualifying his 'admission' status, and therefore, falsely and impermissibly held himself out as a member of the bar of this state. By engaging in such misconduct, we find that the respondent lacks the good moral character and general fitness requisite for a member of the bar of this state (22 NYCRR 521.1[a][3])."; "Accordingly, the Committee's petition seeking to confirm the findings of fact, conclusions of law and determination of the Hearing Panel is granted, and respondent's license as a legal consultant is revoked.").

(c) As in other areas, United States jurisdictions have become more welcoming to foreign lawyers who seek to be admitted pro hac vice in United States courts.

The ABA Ethics 20/20 Commission's supporting report explained the current status of such pro hac possibilities.

This proposal has ample precedent. A form of pro hac vice admission for non-U.S. lawyers is already permitted in at least fifteen states, and is allowed in the U.S. Supreme Court. Numerous federal courts also have rules or other authority that permit foreign lawyers to be specially admitted to appear before them in a particular matter. Notably, the Commission has not learned of any resulting difficulties.

Am. Bar Ass'n Comm'n on Ethics 20/20, Report to the House of Delegates, at 8 (Aug.

2012).

294

35951124_10 UPL, MDP and MJP (Defining What Lawyers Do and McGuireWoods LLP Where They Can Do It): Part II T. Spahn (1/1/19) Hypotheticals and Analyses

The Report also explained that federal courts have also permitted such pro hac admission.

Other federal courts, whether by rule or practice, have permitted foreign lawyers to be admitted pro hac vice. For example, in In re Livent, Inc., Nos. 98 Civ. 5686 (VM)(DFE) & 98 Civ. 7161 (VM)(DFE), 2004 WL 385048 (S.D.N.Y. March 2, 2004), the U.S. District Court for the Southern District of New York admitted pro hac vice two Canadian lawyers who were not admitted to any U.S. jurisdictions at the time of application. In granting admission pro hac vice, the Court stated: "Our Court's pro hac vice rule, Local Civil Rule 1.3(c), omits any mention of an attorney of a foreign country. But admission pro hac vice is a sensible exercise of discretion on the particular facts of this litigation." Obviously, there are occasions in which judges exercise their discretion to deny applications for pro hac vice admission by a foreign lawyer.

Id. at 4.

In 2013, the ABA recommended much more specific and expanded pro hac rights for foreign lawyers. The 2013 changes to the ABA Model Rule on Pro Hac

Admission allow courts or agencies to admit foreign lawyers to appear pro hac.

A court or agency of this state may, in its discretion, admit a foreign lawyer in a particular proceeding pending before such court or agency to appear pro hac vice as co-counsel or in an advisory or consultative role in that proceeding with an in-state lawyer, provided that the in-state lawyer is responsible to the client, responsible for the conduct of the proceeding, responsible for independently advising the client on the substantive law of a United States jurisdiction and procedural issues in the proceeding, and for advising the client whether the in-state lawyer's judgment differs from that of the foreign lawyer.

ABA Model Rule on Pro Hac Admission III(C). The revised rule lists a number of factors that a court or agency may consider in determining whether to admit a foreign lawyer pro hac.

295

35951124_10 UPL, MDP and MJP (Defining What Lawyers Do and McGuireWoods LLP Where They Can Do It): Part II T. Spahn (1/1/19) Hypotheticals and Analyses

In addition to the factors listed in paragraph I(D)(3) above, a court or agency in ruling on an application to admit a foreign lawyer pro hac vice, or in an advisory or consultative role, may weigh the following factors:

1. the legal training and experience of the foreign lawyer including in matters similar to the matter before the court or agency;

2. the extent to which the matter will include the application of:

a. the law of the jurisdiction in which the foreign lawyer is admitted or

b. international law or other law with which the foreign lawyer has a demonstrated expertise;

3. the foreign lawyer's familiarity with the law of a United States jurisdiction applicable to the matter before the court or agency;

4. the extent to which the foreign lawyer's relationship and familiarity with the client or with the facts and circumstances of the matter will facilitate the fair and efficient resolution of the matter;

5. the foreign lawyer's English language ability; and

6. the extent to which it is possible to define the scope of the foreign lawyer's authority in the matter as described in paragraph III (E) so as to facilitate its fair and efficient resolution, including by a limitation on the foreign lawyer's authority to advise the client on the law of a United States jurisdiction except in consultation with the in-state lawyer.

Id. at III(F).

The rule requires intensive involvement by a local lawyer.

The court or agency, in its discretion, may limit the activities of the foreign lawyer or require further action by the in-state lawyer, including but not limited to, requiring the in-state lawyer to sign all pleadings and other documents submitted to the court or to other parties, to be present at all

296

35951124_10 UPL, MDP and MJP (Defining What Lawyers Do and McGuireWoods LLP Where They Can Do It): Part II T. Spahn (1/1/19) Hypotheticals and Analyses

depositions and conferences among counsel, and to attend all proceedings before the court or agency.

Id. at III(D).

The supporting Report emphasizes the limit on what such foreign lawyers can do.

2. The foreign lawyer may only appear as a co-counsel or in an advisory or consultative role, alongside an in-state lawyer in the proceeding. Importantly, authorization to practice pro hac vice does not constitute, and should not be read to provide, full admission to the practice of law in the jurisdiction in which the foreign lawyer seeks this privilege; it is a supervised limited practice authorization for a particular matter on a temporary basis.

3. When a foreign lawyer is authorized to appear pro hac vice, the in-state lawyer has added duties. The in-state lawyer is responsible to the court and the client for the conduct of the proceeding, for independently advising the client on the substantive law of a United States jurisdiction as well as procedural issues, and for advising the client whether the in-state lawyer's judgment differs from that of the foreign lawyer. State courts have elaborated on the extent of local counsel's gatekeeping responsibilities and the extent to which local counsel will be held accountable.

Am. Bar Ass'n Report on Pro Hac Vice Admission, at 5.

Not surprisingly, states have adopted their own requirements for foreign lawyers' pro hac admission in their courts.

• Jeff Mordock, Delaware Opening Pro Hac Vice Door to Lawyers From Abroad, Del. L. Wkly., Sept. 18, 2013 ("Attorneys from abroad can now be admitted pro hac vice under a new amendment to Delaware Supreme Court rules."; "While some say the change is important to further enhance the international reputation of Delaware's Supreme Court, one practitioner questions if foreign attorneys can actually play a significant role in United States litigation."; "The new rule, adopted as an amendment to Rule 71, will permit an international lawyer, defined as a person admitted in any jurisdiction outside the United States, to practice pro hac vice before the Supreme Court. The justices will have the discretion of whether or not to

297

35951124_10 UPL, MDP and MJP (Defining What Lawyers Do and McGuireWoods LLP Where They Can Do It): Part II T. Spahn (1/1/19) Hypotheticals and Analyses

admit the foreign attorney and will be required to review several factors before granting pro hac vice status, including the attorney's legal training, familiarity with United States law, relation to the client, knowledge of the case's facts, and ability to speak English. Delaware counsel will be responsible for the foreign attorney and be required to sign all pleadings and other documents in a manner similar to United States attorneys from other jurisdictions being admitted into the state pro hac vice.").

(d) As in other areas involving foreign lawyers, the ABA and states have struggled with the question whether foreign lawyers should be allowed to take a state's bar exam.

The ABA initially considered accrediting foreign law schools, which presumably could have supported those law schools' graduates efforts to take a state bar exam.

However, in 2012 the ABA abandoned that effort.

• Karen Sloan, ABA Panel Votes "No" on Accrediting Overseas Law Schools, Nat'l Law Journal, Aug, 7, 2012 ("The American Bar Association [ABA] won't extend its seal of approval to foreign law schools anytime soon. The ABA's Council of Legal Education and Admissions to the Bar overwhelmingly voted on August 3 to withhold accreditation from law schools outside the United States."; "The vote capped four years of debate into whether it makes sense for the ABA to allow overseas law schools that follow the United States educational model to apply for accreditation. Those discussions began in 2008 after the Peking University School of Transnational Law -- founded by former University of Michigan Law School dean Jeffrey Lehman in 2007 -- indicated that it would pursue ABA accreditation."; "The ABA formed a committee to examine the issue, and in 2010 it recommended seriously considering extending accreditation to certain non-United States schools. That panel concluded that the move would help state judges and bar associations decide whether to allow foreign-trained lawyers to practice within their jurisdictions."; "But a second committee reached the opposite conclusion earlier this year, following an outpouring of opposition, much of it from law students fearful of additional competition for scarce law jobs. Committee members decided that overseas accreditation would be expensive and hard to administer."; "The council spent about 30 minutes hearing from the leaders of the second committee, law professors Martin Burke and Joan Howland, and discussing the issue, said chairman John O'Brien. He noted that the panel has reviewed the opinions of bar associations, chief judges and law school administrators."; "'I certainly think the thoroughness

298

35951124_10 UPL, MDP and MJP (Defining What Lawyers Do and McGuireWoods LLP Where They Can Do It): Part II T. Spahn (1/1/19) Hypotheticals and Analyses

of the study and the number of groups that weighed in had a big impact on the final decision,' he said. 'I also think that the fact that we're in the process of our periodic review of all our standards was a factor.'"; "Officials from the Peking University School of Transnational Law sent a letter to the council in late July making a case for overseas accreditation. Officials there could not be reached for comment regarding the vote.").

In December 2012, a Texas Bar Supreme Court Commission described the status of states' willingness to allow foreign lawyers to take the bar exam.

The National Conference of Bar Examiners ('NCBE') publishes a Comprehensive Guide to Bar Admission Requirements ('Guide'), which contains a 50-state survey on licensing requirements and data regarding foreign lawyers. The 2012 Guide reported the following information regarding the licensing requirements and bar examiners' experience: (1) Only four U.S. states (Arkansas, Connecticut, New Hampshire, and Rhode Island) impose a residency requirement. Only four states (Florida, Georgia, Ohio, and Utah) require proof of U.S. citizenship or immigration status[;] (2) 54% of U.S. states and the District of Columbia ('D.C.') permit graduates of foreign law schools to sit for the bar exam. States that do not accept a foreign law degree to establish eligibility to sit for the state's bar exam include Delaware, Georgia, and New Jersey[;] (3) 22% of U.S. states that permit a foreign law school graduate to sit for the bar exam require the applicant to have received an education in English common law[;] (4) 24% of U.S. states that permit a foreign law school graduate to sit for the bar exam require additional education at an ABA-approved law school[;] (5) 8% of U.S. states that permit a foreign law school graduate to sit for the bar exam permit the applicant to sit for the bar exam in the applicant obtains an LL.M or other graduate degree from an ABA-approved school. The states which permit applicants with LL.M degrees to sit for their bar exams are Alabama, California, New Hampshire, and New York[;] (6) 54% of U.S. states that permit a foreign law school graduate to sit for the bar exam require that the applicant have practiced law in the home jurisdiction before applying for licensure[;] (7) 36% of U.S. states that permit a foreign law school graduate to sit for the bar exam require a determination that the foreign school's education is equivalent to a U.S. school's education. Only two states, Alabama and Massachusetts, recognize with regularity the

299

35951124_10 UPL, MDP and MJP (Defining What Lawyers Do and McGuireWoods LLP Where They Can Do It): Part II T. Spahn (1/1/19) Hypotheticals and Analyses

sufficiency of a legal education received at a particular foreign law school[;] (8) The bar exam fees for attorneys in Texas are on the high side in comparison to other states, although recent increases in other states has made Texas relatively more competitive (e.g., Texas - $1,040/$1,140; California - $892.00; Florida - $1,600 - $3,000; New York $250.00/$750.00)[;] (9) 60% of states license or register foreign legal consultants.

Larry B. Pascal & Leland C. de la Garza, Int'l Law Practice in Texas, Final Report to

Texas Supreme Court, 7-9 (Dec. 2012) (footnote omitted).

The Texas Commission Report described other states' more welcoming posture, especially New York's liberal approach.

The New York Bar has a well-earned reputation for allowing foreign lawyers to sit for the bar, inspiring trust and confidence in the foreign attorney community. Nevertheless, New York has been reviewing the growth of foreign lawyers sitting for its exam, and recently took some modest steps to reform its rule. In 2011, the New York Bar increased the curricular requirements for the LL.M, but did not fundamentally change its orientation in terms of foreign lawyer eligibility to sit for the New York bar exam. As a result, we do not believe these changes will significantly weaken New York's position within the U.S. as the most attractive market to become licensed as a foreign attorney. In July 2011, 4,427 foreign-educated candidates took the New York bar exam, a significantly larger number than any other U.S. jurisdiction. California (which has a rule that is even more open than New York, but was adopted after the New York rule) was second with 764 foreign candidates. The District of Columbia, Alabama and Virginia came next with 94, 65, and 38, respectively. Texas only had 22 foreign-educated candidates take its bar examination, which was less than its much smaller neighbor, Louisiana (which had 29).

Id. at 5 (footnote omitted). The Report explained how Texas traditionally has taken a more restrictive view.

300

35951124_10 UPL, MDP and MJP (Defining What Lawyers Do and McGuireWoods LLP Where They Can Do It): Part II T. Spahn (1/1/19) Hypotheticals and Analyses

The Task Force believes that one of the main reasons these other states attract more foreign applicants than Texas is their lack of a foreign practice requirement. In contrast, Texas requires significant foreign legal experience before foreign-educated attorneys can even sit for the bar exam, which is often difficult to meet or prove due to the need to request supporting documentation from past and present employers during the applicable period. Also, Texas requires all foreign-educated attorneys to submit a certificate of good standing, issued by the licensing body of their country of origin, as part of their application. Although this is a relatively simple process in the U.S., in some countries, it is extremely difficult to obtain such a good standing certificate, which can serve as a deterrent to even applying to take the Texas bar exam. In contrast, New York does not require a good standing certificate, but rather satisfactory proof of the candidates' legal education. The New York Bar also does not require foreign-educated candidates to have any prior experience, but rather only to have successfully completed twenty-four semester hours of law school credit, including a basic introductory course in U.S. law at an approved law school in the U.S. These educational requirements are usually completed while pursuing an LL.M degree.

Id. at 5-6 (footnotes omitted).

The Report recommended a dramatic change in the Texas rules governing the ability of foreign lawyers to take the Texas Bar exam.

The Task Force unanimously recommends that Texas reform its international practice rules in order to adjust to the changing market realities facing the state and thereby offer law schools, law firms, and clients based in Texas greater access to international resources. The most important areas to address are: (1) the ability of foreign lawyers to sit for the Texas Bar under rules comparable to those used by New York and California (typically with an LL.M degree from a U.S. law school), (2) greater curricular definition and requirements for LL.M graduates, as generally contemplated in the draft ABA Model Rule, and as subsequently largely adopted by the New York Bar in 2011, (3) the recognition of the ability to achieve admission to the Texas Bar based on a Non-Resident F-1 student visa allowing for optional practical

301

35951124_10 UPL, MDP and MJP (Defining What Lawyers Do and McGuireWoods LLP Where They Can Do It): Part II T. Spahn (1/1/19) Hypotheticals and Analyses

training ('Student Visa'), as is common in New York, California, and other major states, (4) refinements to the Texas foreign legal consultant rule to promote greater use of this rule, ensure appropriate access to privileges and immunities, particularly by in-house counsel, and facilitate renewal for these foreign attorneys, and (5) application of the Texas pro hac vice rule to foreign attorneys.")

Id. at 3.

The ABA and various other sources have prepared useful compilations of various states' requirements for foreign lawyers to be admitted pro hac in those jurisdictions' courts.

(e) Starting in 2012, states began to liberalize their rules governing foreign lawyers' attempts to be admitted by motion rather than after taking a bar exam.

• Leigh Jones, Minnesota Makes History in Admitting Foreign Law Graduate, Nat'l L.J., Sept. 26, 2012 ("For the first time in state history, the Minnesota Supreme Court has admitted to practice a graduate from a foreign law school without requiring her to pass the state's bar examination."; "The court ruled on Sept. 26 that JaneAnne Murray possessed the education and professional experience necessary to exempt her from the usual requirements for bar admission, and that she would suffer undue hardship if refused."; "Murray, a native of Ireland, received a bachelor's degree from University College Cork in Ireland in 1989 and a law degree from University of Cambridge in England. Minnesota law in general requires licensed practitioners to have graduated from law schools accredited by the American Bar Association [ABA]. The ABA does not accredit foreign schools."; "The Minnesota bar only once before, in 1986, admitted a graduate from a law school not accredited by the ABA without requiring passage of the bar exam, but that applicant was not from a foreign law school. That attorney graduated from an unaccredited law school in Louisiana that closed in 1964."; "Murray moved to New York after graduating from law school, passed the bar there and worked for Paul, Weiss, Rifkind, Wharton & Garrison. She also worked as counsel at O'Melveny & Myers, for a legal aid provider and as a federal public defender in New York. In 2005, she became a solo practitioner in New York, focusing on criminal defense."; "She moved to Minnesota last year when her husband took a job at the University of Minnesota. She was appointed as a practitioner-in-residence at the University of

302

35951124_10 UPL, MDP and MJP (Defining What Lawyers Do and McGuireWoods LLP Where They Can Do It): Part II T. Spahn (1/1/19) Hypotheticals and Analyses

Minnesota Law School. At the time she filed her petition for bar admission, she maintained her New York practice.").

Not surprisingly, some foreign lawyers satisfy jurisdictions' requirements and some do not.

• Chong v. Tenn. Bd. of Law Exam'rs, 481 S.W.3d 609, 613 (Tenn. 2015) (holding that Tennessee properly rejected a Korean lawyer's motion to be admitted to the Tennessee Bar, because a lawyer's additional education at a U.S. law school was not in-person; "In conclusion, we hold that: (1) the words '[i]n addition,' as used in section 7.01, mean that the minimum credit hours required by that section must be earned over and above the credit hours required for obtaining the law degree from the applicant's foreign law school; and (2) the words 'in residence,' as used in section 7.01, mean 'physically in residence' at an ABA-approved law school. Accordingly, we affirm the Board's decision to deny Mr. Chong's application. Costs of this appeal are taxed to Daniel Sungkook Chong and his surety, for which execution may issue if necessary.").

• In re O'Siochain, 842 N.W.2d 763, 765. 765-66, 768, 770 (Neb. 2014) (allowing an Irish lawyer who had admitted to the New York Bar to also be admitted to the Nebraska Bar; waiving the normal requirement that a lawyer seeking admission to the Nebraska Bar must have graduated from a Nebraska-approved law school; "O'Siochan graduated from University College Dublin (UCD) in Ireland in 2004 with a bachelor of business and legal studies degree. He enrolled at UCD after graduation from high school, as is customary in Ireland, and completed a 4-year law and business program."; "Upon graduating from UCD in 2004, O'Siochain took a 'Barbri' course in Ireland to prepare for the New York bar examination in February 2005. Barbri is a franchise that offers bar examination preparation courses and includes video lectures and course materials corresponding with the relevant state bar examination. O'Siochain took Barbri courses in New York practice, professional responsibility, trusts and estates, federal jurisdiction and procedure, and domestic relations, among others."; "The New York State Board of Law Examiners allowed O'Siochain to sit for the New York bar examination because his legal education satisfied the durational and substantive equivalency requirements contained in the Rules of the Court of Appeals of the State of New York. O'Siochain passed the New York bar examination and the Multistate Professional Responsibility Examination (MPRE), on which he scored 104. The minimum score required in Nebraska is 85. See Neb. Ct. R. § 3-116(A). He was admitted to the New York bar in March 2006 and has been a member in good standing since that time."; "O'Siochain earned his law degree from UCD in Ireland, a school that is not ABA-approved. Therefore, O'Siochain's degree does not satisfy the

303

35951124_10 UPL, MDP and MJP (Defining What Lawyers Do and McGuireWoods LLP Where They Can Do It): Part II T. Spahn (1/1/19) Hypotheticals and Analyses

educational requirement of § 3-105(A)(1)(b), and we must determine whether to waive this requirement."; "O'Siochain studied U.S. law in preparation for the New York bar examination, and his studies included trust and estates, family law, and civil procedure. O'Siochain was deemed qualified to sit for the New York bar examination, having shown, inter alia, that the course study he successfully completed was the substantial equivalent of the legal education provided by an ABA-approved law school. He was tested by the New York bar examination in all fundamental areas of U.S. law, including trusts and estates, family law, and civil procedure. He passed the New York bar examination and is a licensed attorney in good standing with the New York bar."; "[W]e conclude that O'Siochain has met his burden of proving his law school education and experience were functionally equivalent to the education received at an ABA-approved law school and that as a result, a waiver of the educational qualifications requirement of § 3-105(A)(1)(b) is appropriate. We waive this requirement as it applies to O'Siochain and will allow him to be admitted to the Nebraska bar.").

• In re Tulshyan, No. 13-0072, 2013 W. Va. LEXIS 1235, at *2, *11-12 (W. Va. Nov. 6, 2013) ("The petitioner is a graduate of Bishnu Ram Medhi Government Law college in Guwahati, India. After moving to the United States, she sought admission to the New York State Bar. New York's rules for eligibility for admission to the practice of law require that an applicant of a foreign law school complete twenty credit hours in a program of study at an approved law school in the United States. In order to meet this requirement, the petitioner enrolled in the Masters of Law Program at Fordham University School of Law and received her LLM degree in May 2004 with 24 credit hours. Having satisfied the New York educational requirements as an applicant of a foreign law school, the petitioner applied for admission to the New York Bar through examination. She passed the July 2004 New York bar examination and was admitted to the practice of law in New York on May 18, 2005."; "[T]he petitioner is required to satisfy the equivalency provisions of Rule 3.0(b). Specifically, Rule 3.0(b)(4)(c) requires that as a graduate of such foreign law school, she has to successfully complete study at an ABA- accredited law school, with a minimum of 30 credit hours of basic courses selected from certain listed areas of law. The petitioner has successfully completed only 24 credit hours at Fordham University School of Law, an ABA-accredited law school, which she did to satisfy the educational requirements of eligibility to practice law in New York. She does not meet the 30 credit hour requirement of this rule as referenced in the general requirements for admission eligibility set forth in Rule 2.0. The standards for admission to practice law in New York at the time of petitioner's admission to practice law in New York were not substantially equivalent to the requirements for admission to practice law in West Virginia. Petitioner has not met the educational requirements for eligibility to practice law in West

304

35951124_10 UPL, MDP and MJP (Defining What Lawyers Do and McGuireWoods LLP Where They Can Do It): Part II T. Spahn (1/1/19) Hypotheticals and Analyses

Virginia as a graduate of a foreign law school. Therefore, she is not currently eligible for admission to practice law in West Virginia without examination.").

The ABA and various other sources have prepared useful compilations of various

states' requirements for foreign lawyers to be admission by motion.

Best Answer

The best answer to (a) is YES (PROBABLY); the best answer to (b) is YES

(PROBABLY); the best answer to (c) is MAYBE; the best answer to (d) is MAYBE; the best answer to (e) is MAYBE.

B 2/13

305

35951124_10